Go Math Grade 8 Answer Key Chapter 14 Scatter Plots

go-math-grade-8-chapter-14-scatter-plots-answer-key

Students can grab the complete knowledge on Divide Fractions on Go Math Grade 8 Answer Key Chapter 14 Scatter Plots. This article consists of the solutions to practice problems, review tests along with answers and explanations for students to have more practice. So, the students who are in search of Go Math 8th Grade Answer Key Chapter 14 Scatter Plots can Download pdf from here. It is difficult for parents and teachers to deal with the students & give explanations for their questions. Therefore, start practicing the maths with the help of Go Math Grade 8 Answer Key.

Go Math Grade 8 Answer Key Chapter 14 Scatter Plots

We the team of ccss math answers offer the pdf of Go Math Grade 8 Answer Key Chapter 14 Scatter Plots. Get step by step solution for Go Math Grade 8 Answer Key Chapter 14 Scatter Plots here. With the help of the HMH Go Math 8th Grade Solution Key you can score good marks in the exams. If you learn the concepts you can make the question on your own and test your knowledge. Hence make use of the below links and practice the given problems from now.

Lesson 1: Scatter Plots and Association

Lesson 2: Trend Lines and Predictions

Model Quiz

Mixed Review

Guided Practice – Scatter Plots and Association – Page No. 436

Bob recorded his height at different ages. The table below shows his data.
Go Math Grade 8 Answer Key Chapter 14 Scatter Plots Lesson 1: Scatter Plots and Association img 1

Question 1.
Make a scatter plot of Bob’s data.
Go Math Grade 8 Answer Key Chapter 14 Scatter Plots Lesson 1: Scatter Plots and Association img 2
Type below:
_____________

Answer:
grade 8 chapter 14 image 1

Explanation:
As Bob gets older, his height increases along with the straight line on the
graph. So, the association is positive and linear.

Question 2.
Describe the association between Bob’s age and his height. Explain the association.
Type below:
_____________

Answer:
The association is positive and linear. Bob’s height increases as he gets older. We would see that Bob’s height eventually stops increasing if the data continued.

Question 3.
The scatter plot shows the basketball shooting results for 14 players. Describe any clusters you see in the scatter plot. Identify any outliers.
Go Math Grade 8 Answer Key Chapter 14 Scatter Plots Lesson 1: Scatter Plots and Association img 3
Type below:
_____________

Answer:
There is an outlier at (35,18)

Explanation:
There is a cluster in the “20 – 25” shots attempted range and a smaller cluster in the “5 – 14” shots attempted range.
There is an outlier at (35,18)

ESSENTIAL QUESTION CHECK-IN

Question 4.
Explain how you can make a scatter plot from a set of bivariate data.
Type below:
_____________

Answer:
Bivariate data – data that has two variables per observation,
An x variable and y variable.
Scatterplot – The graph displaying categorical data, with an x and y-axis.
Response Variable – the variable that is explained by the other.
Explanatory Variable – the variable which explains the other.

14.1 Independent Practice – Scatter Plots and Association – Page No. 437

Sports Use the scatter plot for 5–8.

Olympic Men’s Long Jump Winning Distances
Go Math Grade 8 Answer Key Chapter 14 Scatter Plots Lesson 1: Scatter Plots and Association img 4

Question 5.
Describe the association between the year and the distance jumped for the years 1960 to 1988.
Type below:
_____________

Answer:
The data shows a positive linear association. If the year increases, the winning distance increases.

Question 6.
Describe the association between the year and the distance jumped for the years after 1988.
Type below:
_____________

Answer:
Between 1996 and 2004, there was a slight increase in distance over time. The data from 1988 to 2012 will show a negative association.

Question 7.
For the entire scatter plot, is the association between the year and the distance jumped linear or nonlinear?
_____________

Answer:
The data show a rise between 1960 and 1988. The data also show a fall between 1988 and 2012. Therefore, overall, there is no linear pattern.

Question 8.
Identify the outlier and interpret its meaning.
Type below:
_____________

Answer:
The outlier is at (1968, 8.9). It represents a long jump of 8.9 meters in 1968 that exceeds the other jumps made in the surrounding years.

Question 9.
Communicate Mathematical Ideas Compare a scatter plot that shows no association to one that shows negative association.
Type below:
_____________

Answer:
Randomly scattered data points with no apparent pattern define a scatter plot with no association. Data points that fall from left to right and has data set values that increases as the other decreases define a scatter plot with a negative association.

Scatter Plots and Association – Page No. 438

For 10–11, describe a set of real-world bivariate data that the given scatter plot could represent. Define the variable represented on each axis.

Question 10.
Go Math Grade 8 Answer Key Chapter 14 Scatter Plots Lesson 1: Scatter Plots and Association img 5
_____________

Answer:
The x-axis represents the number of containers. The y-&is represents the price per container.

Question 11.
Go Math Grade 8 Answer Key Chapter 14 Scatter Plots Lesson 1: Scatter Plots and Association img 6
_____________

Answer:
The x-axis represents the number of hours spent watching tv. The y-axis represents the number of TVs owned.

FOCUS ON HIGHER ORDER THINKING

Question 12.
Multiple Representations Describe what you might see in a table of bivariate data that would lead you to conclude that the scatter plot of the data would show a cluster.
Type below:
_____________

Answer:
A cluster in a scatter plot is when there are a lot of points all grouped around the same location.
Look for points that have the same input and output values. If there are a lot of points together, you must have a cluster in your scatter plot.

Question 13.
Justify Reasoning Is it possible for a scatter plot to have a positive or negative association that is not linear? Explain.
Type below:
_____________

Answer:
Yes

Explanation:
Yes; it is possible for a scatter plot to have a positive or negative association that is not linear. The data points may have a falling or rising curve that will exhibit a nonlinear association.

Question 14.
Critical Thinking To try to increase profits, a theater owner increases the price of a ticket by $25 every month. Describe what a scatter plot might look like if x represents the number of months and y represents the profits. Explain your reasoning.
Type below:
_____________

Answer:
Initially, the number of tickets sold might decline a little, but the price increase would offset the loss in sales. That means that profits would increase, showing a positive association.
When the price would get too high, ticket sales would decline rapidly, so profits would fall giving a negative association.

Guided Practice – Trend Lines and Predictions – Page No. 442

Angela recorded the price of different weights of several bulk grains. She made a scatter plot of her data. Use the scatter plot for 1–4.

Question 1.
Draw a trend line for the scatter plot.
Go Math Grade 8 Answer Key Chapter 14 Scatter Plots Lesson 2: Trend Lines and Predictions img 7
Type below:
_____________

Answer:
grade 8 chapter 14 image 2

Question 2.
How do you know whether your trend line is a good fit for the data?
Type below:
_____________

Answer:
Most of the data points are close to the trend line. The trend line has about the same number of points above and below it.

Question 3.
Write an equation for your trend line.
Type below:
_____________

Answer:
y = 0.09x

Explanation:
The trend line passes through (0, 0) and (19, 1.80).
Find the slope by using the slope formula.
slope = m = (y2 – y1)/(x2 – x1) = 1.80/19 = 0.09
The line passes through the origin. So, the y-intercept is 0.
From an equation for the trend line by substituting the slope value for m and the value of the y-intercept b in the slope-intercept formula.
y = mx + b
y = 0.09x + 0
y = 0.09x

Question 4.
Use the equation for your trend line to interpolate the price of 7 ounces and extrapolate the price of 50 ounces.
Type below:
_____________

Answer:
The price for 7 and 50 ounces is $0.63 and $4.50

Explanation:
Use the equation for the trend line (y = 0.09x) to interpolate the price of 7 ounces by substituting 7 for x (y= 0.09 • 7) and solving for y.
Use the equation for the trend line (y = 0.09x) to interpolate the price of 50 ounces by substituting 50 for x (y= 0.09 • 50) and solving for y.

ESSENTIAL QUESTION CHECK-IN

Question 5.
A trend line passes through two points on a scatter plot. How can you use the trend line to make a prediction between or outside the given data points?
Type below:
_____________

Answer:
Use two points on the line. rind the slope and y-intercept. Substitute the values of the slope (m) and y-intercept (b) to form an equation using y = mx + b. Substitute the value of x for which you want to make a prediction and solve for y OR substitute your prediction for y and solve to find its value.

14.2 Independent Practice – Trend Lines and Predictions – Page No. 443

Use the data in the table for Exercises 6–10.
Go Math Grade 8 Answer Key Chapter 14 Scatter Plots Lesson 2: Trend Lines and Predictions img 8

Question 6.
Make a scatter plot of the data and draw a trend line.
Go Math Grade 8 Answer Key Chapter 14 Scatter Plots Lesson 2: Trend Lines and Predictions img 9
Type below:
_____________

Answer:
grade 8 chapter 14 image 3

Question 7.
What type of association does the trend line show?
Type below:
_____________

Answer:
Negative Association

Explanation:
One data set increases – Wind Speed and the other – Wind Chill decreases. So, the trend line shows a Negative Association.

Question 8.
Write an equation for your trend line.
Type below:
_____________

Answer:
y = -0.25x + 2.5

Explanation:
Find the slope using the Slope Formula
m = (y2 – y1)/(x2 – x1) = ((-10) – 5)/(50 – 30) = -5/20 = -0.25
Find the y-intercept using the Slope-Intercept Formula
y = mx + b
-5 = -0.25(30) + b
-5 = -7.5 + b
2.5 = b
Substitute the value of m and b into the Slope-Intercept Formula to form an equation for the trend line.
y = -0.25x + 2.5

Question 9.
Make a Prediction Use the trend line to predict the wind chill at these wind speeds.
a. 36 mi/h
_________ °F

Answer:
-6.5°F

Explanation:
Use the trend line to predict the wind chill at 36mi/h
y = -0.25x + 2.5
y = -0.25(36) + 2.5
y = -9 + 2.5
y = -6.5
The wind chill at 36mi/h is -6.5ºF

Question 9.
b. 100 mi/h
_________ °F

Answer:
-22.5°F

Explanation:
Use the trend line to predict the wind chill at 100mi/h
y = -0.25x + 2.5
y = -0.25(100) + 2.5
y = -25 + 2.5
y = -22.5
The wind chill at 100mi/h is -22.5ºF

Question 10.
What is the meaning of the slope of the line?
Type below:
_____________

Answer:
The slope means that the wind chill falls about 1°F for every 4 mph increase in wind speed.

Use the data in the table for Exercises 11–14.
Go Math Grade 8 Answer Key Chapter 14 Scatter Plots Lesson 2: Trend Lines and Predictions img 10

Question 11.
Make a scatter plot of the data and draw a trend line.
Go Math Grade 8 Answer Key Chapter 14 Scatter Plots Lesson 2: Trend Lines and Predictions img 11
Type below:
_____________

Answer:
grade 8 chapter 14 image 4

Question 12.
Write an equation for your trend line.
Type below:
_____________

Answer:
y = -(2/15)x + 64

Explanation:
Find the slope using the Slope Formula
m = (y2 – y1)/(x2 – x1) = (72 – 64)/(60 – 0) = 8/60 = -2/15
Find the y-intercept using the Slope-Intercept Formula at (0, 64)
y = mx + b
b = 64
Substitute the value of m and b into the Slope-Intercept Formula to form an equation for the trend line.
y = -2/15x + 64

Question 13.
Make a Prediction Use the trend line to predict the apparent temperature at 70% humidity.
Type below:
_____________

Answer:
73.3º F

Explanation:
Use the equation of the trend line. Substitute 70(for 70%) into the equation for x.
y = -(2/15)x + 64
y = -(2/15)(70) + 64
y = -140/15 + 64
y = -9.3 + 64
y = 73.3
The apparent temperature is 73.3º F

Question 14.
What is the meaning of the y-intercept of the line?
Type below:
_____________

Answer:
The y-intercept explains that at 0% humidity, the apparent temperature is 64ºF

FOCUS ON HIGHER ORDER THINKING – Trend Lines and Predictions – Page No. 444

Question 15.
Communicate Mathematical Ideas Is it possible to draw a trend line on a scatter plot that shows no association? Explain.
_____________

Answer:
No

Explanation:
It is not possible to draw a trend line on a scatter plot that shows no association. If the scatter plot shows no association, the data points have no relationships with one another. You can draw a trend line if a linear association is available.

Question 16.
Critique Reasoning Sam drew a trend line that had about the same number of data points above it as below it, but did not pass through any data points. He then picked two data points to write the equation for the line. Is this a correct way to write the equation? Explain.
_____________

Answer:
No

Explanation:
Sam did not use the correct way to write an equation.
Sam may have drawn a correct trend line but using the data points that are not on the trend line may have an incorrect equation for the line. He should use two points on that trend line to write the equation.

Question 17.
Marlene wanted to find a relationship between the areas and populations of counties in Texas. She plotted x (area in square miles) and y (population) for two counties on a scatter plot:
Kent County (903, 808)                                Edwards County (2118, 2002)
She concluded that the population of Texas counties is approximately equal to their area in square miles and drew a trend line through her points.
a. Critique Reasoning Do you agree with Marlene’s method of creating a scatter plot and a trend line? Explain why or why not.
_____________

Answer:
I do not agree with Marlene’s method of creating a scatter plot and a trend line. She did not have enough data. Marlene should have collected and plotted data for many more counties.

Question 17.
b. Counterexamples Harris County has an area of 1778 square miles and a population of about 4.3 million people. Dallas County has an area of 908 square miles and a population of about 2.5 million people. What does this data show about Marlene’s conjecture that the population of Texas counties is approximately equal to their area?
Type below:
_____________

Answer:
The data collected are only of two counties whose populations are nearly equal to their area. The fact that the populations of Harris and Dallas counties are in the millions, Marlene’s conjecture about the population of Texas counties being equivalent to their area is invalid.

Ready to Go On? – Model Quiz – Page No. 445

14.1 Scatter Plots and Association

An auto store is having a sale on motor oil. The chart shows the price per quart as the number of quarts purchased increases. Use the data for Exs. 1–2.
Go Math Grade 8 Answer Key Chapter 14 Scatter Plots Model Quiz img 12

Question 1.
Use the given data to make a scatter plot.
Go Math Grade 8 Answer Key Chapter 14 Scatter Plots Model Quiz img 13
Type below:
_____________

Answer:
grade 8 chapter 14 image 5

Question 2.
Describe the association you see between the number of quarts purchased and the price per quart. Explain.
Type below:
_____________

Answer:
Negative nonlinear association

Explanation:
The association seen between the number of quarts purchased and the price per quart is negative and nonlinear. As the number of quarts rises, the price per quart decreases but you can see a data curve.

14.2 Trend Lines and Predictions

The scatter plot below shows data comparing wind speed and wind chill for an air temperature of 20 °F. Use the scatter plot for Exs. 3–5.

Question 3.
Draw a trend line for the scatter plot.
Go Math Grade 8 Answer Key Chapter 14 Scatter Plots Model Quiz img 14
Type below:
_____________

Answer:
grade 8 chapter 14 image 6

Question 4.
Write an equation for your trend line.
Type below:
_____________

Answer:
y = -0.35x + 12.25

Explanation:
The line passes through (10, 8.75) and (35, 0) so we can use these points to find the slope.
The slope of the line is :
Slope = m = (y2 – y1)/(x2 – x1) = (0 – 8.75)/(35 – 10) = -8.75/25 = -0.35
Find the y-intercept using the slope-intercept formula :
y = mx + b
0 = -0.35 . 35 + b
0 = -12.25 + b
b = 12.25
Substitute the slope m and the y-intercept b in the slope-intercept formula.
The equation for the trend line is :
y = mx + b
y = -0.35x + 12.25

Question 5.
Use your equation to predict the wind chill to the nearest degree for a wind speed of 60 mi/h.
________ °F

Answer:
9°F

Explanation:
y = −0.35x + 12.25
y = -0.35(60) + 12.25
y = -21 + 12.25
y = -8.75
The wind chill to the nearest degree for a wind speed of 60 mi/h is 9°F.

ESSENTIAL QUESTION

Question 6.
How can you use scatter plots to solve real-world problems?
Type below:
_____________

Answer:
Using a scatter plot, you can see positive and negative trends such as prices over time. You can also make predictions such as height at a certain age.

Selected Response – Mixed Review – Page No. 446

Question 1.
Which scatter plot could have a trend line whose equation is y = 3x + 10?
Go Math Grade 8 Answer Key Chapter 14 Scatter Plots Mixed Review img 15
Options:
a. A
b. B
c. C
d. D

Answer:
b. B

Question 2.
What type of association would you expect between a person’s age and hair length?
Options:
a. linear
b. negative
c. none
d. positive

Answer:
c. none

Explanation:
The length of their hair reduces. This is because the length of hair changes with the growth phase of the hair follicles. When one is young, the cells of the papilla divide more rapidly, and hence the length of the hair to be long before reaching the transitional phase and then shed off in the telogen phase. The older one gets, the papilla cells do not divide as rapidly and the length of the hair shortens with age.

Question 3.
Which is not shown on the scatter plot?
Go Math Grade 8 Answer Key Chapter 14 Scatter Plots Mixed Review img 16
Options:
a. cluster
b. negative association
c. outlier
d. positive association

Answer:
d. positive association

Explanation:
The scatter plot shows a cluster, some outliers, and a negative association.
It does not show a positive association.

Question 4.
A restaurant claims to have served 352,000,000 hamburgers. What is this number in scientific notation?
Options:
a. 3.52 × 106
b. 3.52 × 108
c. 35.2 × 107
d. 352 × 106

Answer:
b. 3.52 × 108

Explanation:
100,000,000
So, 3.52 × 108

Question 5.
Which equation describes the relationship between x and y in the table?
Go Math Grade 8 Answer Key Chapter 14 Scatter Plots Mixed Review img 17
Options:
a. y = −4x
b. y = −\(\frac{1}{4}\)x
c. y = 4x
d. y = \(\frac{1}{4}\)x

Answer:
b. y = −\(\frac{1}{4}\)x

Explanation:
In order to find out the relationship between x and y, we have to use the values in the question and substitute them into the solution options.
So, y = -1/4x

Mini-Task

Question 6.
Use the data in the table.
Go Math Grade 8 Answer Key Chapter 14 Scatter Plots Mixed Review img 18
a. Make a scatterplot of the data.
Go Math Grade 8 Answer Key Chapter 14 Scatter Plots Mixed Review img 19
Type below:
______________

Answer:
grade 8 chapter 14 image 7

Question 6.
b. Which data point is an outlier?
Type below:
______________

Answer:
The outlier is the point (92, 135).

Question 6.
c. Predict the number of visitors on a day when the high temperature is 102 °F.
Type below:
______________

Answer:
Based on the cluster around 100°F, I would expect that on a day with a temperature of 102 °F, the pool would have between 350 and 400 visitors.

Conclusion:

Hope our Go Math Grade 8 Answer Key Chapter 14 Scatter Plots helped you a lot. Stick to our page to get chapter-wise Answer Keys of Go Math Grade 8. You can get step by step explanations for all the questions in an easy manner.

Go Math Grade 3 Answer Key Chapter 11 Perimeter and Area Extra Practice

go-math-grade-3-chapter-11-perimeter-and-area-extra-practice-answer-key

Go Math Grade 3 Answer Key Chapter 11 Perimeter and Area Extra Practice will make you familiar with the area and perimeter of different shapes. By following the Go Math Grade 3 Answer Key, you can get acquainted with the Perimeter and Area of the different shapes like rectangle, square, and etc. 3rd Grade Go Math Answer Key has detailed solutions to all the problems and makes it easy for you to grasp the concepts behind them.

Go Math Grade 3 Chapter 11 Perimeter and Area Answer Key Extra Practice

Go Math Grade 3 Chapter 11 Perimeter and Area Answer Key covers Questions from Chapter Tests, Practice Tests, Performance Tests, etc. You can gain Complete Knowledge on Perimeter and Area topic by using the Go Math 3rd Grade Chapter 11 Perimeter and Area Answer Key Extra Practice. Solve as many times as possible and attempt the exam with confidence and score better grades in the exams.

Common Core – Page No. 235000

Chapter 11 Extra Practice

Lessons 11.1, 11.3

Question 1.
Find the perimeter of the shape. Each unit is 1 centimeter.
Go Math Grade 3 Answer Key Chapter 11 Perimeter and Area Extra Practice Common Core img 1
______ centimeters

Answer:
18 centimeters

Explanation:
chapter 11 - common core - image 1 - 235000
Each square in the grid is a 1 by 1-centimeter square. So, we have to do is add up the
lengths of the dark segments right over the figure. Start the count from the box where 1
is placed. This parameter is 1, 2, 3, 4, 5, 6, 7, 8, 9, 10, 11, 12, 13, 14, 15, 16, 17, 18 centimeters long. So, it is 18 centimeters.

Question 2.
The square has a perimeter of 28 inches. What is the length of each side of the square?
Go Math Grade 3 Answer Key Chapter 11 Perimeter and Area Extra Practice Common Core img 2
______ inches

Answer:
7 inches

Explanation:
From the given data
Perimeter of a square is = 28 inch
Length of each side of the square is = a inch
Perimeter of a square = a + a + a + a = 4a = 28 inch
4a= 28
Then, a = 28/4= 7
Therefore, length of each side of the square is = a = 7 inch

Lesson 11.2

Use a centimeter ruler to find the perimeter.

Question 3.
Go Math Grade 3 Answer Key Chapter 11 Perimeter and Area Extra Practice Common Core img 3
______ cm

Answer:
11 cm

Explanation:
chapter 11 - common core - image 7 - 235000
1 + 4 + 1 + 2 + 3 = 11 cm

Question 4.
Go Math Grade 3 Answer Key Chapter 11 Perimeter and Area Extra Practice Common Core img 4
______ cm

Answer:
15 cm

Explanation:
chapter 11 - common core - image 8 - 235000
5 + 4 + 3 + 3 = 15 cm

Lessons 11.4–11.6

Find the area of the shape.
Each unit square is 1 square inch.

Question 5.
Go Math Grade 3 Answer Key Chapter 11 Perimeter and Area Extra Practice Common Core img 5
Area = ______ square inches

Answer:
Area = 15 square inches

Explanation:
chapter 11 - common core - image 2 - 235000
As per the given data,
Each unit square is 1 square inch
Then, the area of the shape is = 15 square inches

Question 6.
Go Math Grade 3 Answer Key Chapter 11 Perimeter and Area Extra Practice Common Core img 6
Area = ______ square inches

Answer:
Area = 20 Square inches

Explanation:
chapter 11 - common core - image 3 - 235000
From the given data,
Each unit square is 1 square inch
Then, the area of the shape = 5 x 4 = 20 Square inches

Common Core – Page No. 236000

Lesson 11.7

Use the rectangles for 1–2.
Go Math Grade 3 Answer Key Chapter 11 Perimeter and Area Extra Practice Common Core img 7

Question 1.
How do the length and width change from Rectangle A to Rectangle B?
Type below:
__________

Answer:
22 centimeters

Explanation:
Rectangle A: Length = 2 ft; Width = 2 ft.
Rectangle B: Length = 3 ft; Width = 2 ft.
The Width from Rectangle A to Rectangle B is the same.
The Length is increased 1 ft from Rectangle A to Rectangle B.

Question 2.
How do the areas change from Rectangle A to Rectangle B to Rectangle C?
Rectangle A: ______ sq. ft.
Rectangle B: ______ sq. ft.
Rectangle C: ______ sq. ft.

Answer:
22 centimeters

Explanation:
Area of Rectangle A: 2 x 2 = 4 sq. ft
Area of Rectangle B: 2 x 3 = 6 sq. ft
Area of Rectangle C: 2 x 4 = 8 sq. ft
Then, areas change from Rectangle A to Rectangle B to Rectangle C is 2 sq. ft

Lesson 11.8

Draw a line to break apart the shape into rectangles.
Find the area of the shape.

Question 3.
Go Math Grade 3 Answer Key Chapter 11 Perimeter and Area Extra Practice Common Core img 8
______ square units

Answer:
23 square units

Explanation:
chapter 11 - common core - image 4 - 236000
2 x 7= 14 Sq. units; 3 x 3= 9 Sq. units
14 + 9 = 23 Sq. units
Area of the shape is = 23 Sq. units

Question 4.
Go Math Grade 3 Answer Key Chapter 11 Perimeter and Area Extra Practice Common Core img 9
______ square units

Answer:
26 square units

Explanation:
chapter 11 - common core - image 5 - 236000
4 x 5 = 20 Sq. units; 2 x 3 = 6 Sq. units
20 + 6 = 26 Sq. units
Then, the area of the shape is = 26 Sq. units

Lessons 11.9–11.10

Find the perimeter and area of each rectangle.
Use your results to answer questions 5–6.
Go Math Grade 3 Answer Key Chapter 11 Perimeter and Area Extra Practice Common Core img 10

chapter 11 - common core - image 6 - 236000

Question 5.
Which two rectangles have the same perimeter?
Rectangles ______ and ______

Answer:
Rectangles A and C

Explanation:
Rectangle A: Permiter = 4 + 4 + 4 + 4 = 16 Units.
Rectangle B: Permiter = 3 + 4 + 3 + 4 = 14 Units.
Rectangle C: Permiter = 6 + 2 + 6 + 2 = 16 Units.
Rectangle A and Rectangle C have the same perimeter

Question 6.
Which two rectangles have the same area?
Rectangles ______ and ______

Answer:
Rectangles B and C

Explanation:
Rectangle A: Area = 4 x 4 = 16 Sq. Units
Rectangle B: Area = 4 x 3 = 12 Sq. Units
Rectangle C: Area = 6 x 2 = 12 Sq. Units
Rectangle B and Rectangle C have the same area.

Conclusion

We have explained Go Math 3rd Grade Answer Key Extra Practice taking the help of the Perimeter and Area of different Shapes by using images, indicating images with arrows, and numbers. Know how to find the Area and Perimeter of different Shapes by accessing our Go Math Grade 3 Answer Key Chapter 11 Perimeter and Area and resolve your doubts.

Go Math Grade 5 Answer Key Chapter 3 Add and Subtract Decimals

go-math-grade-5-chapter-3-add-and-subtract-decimals-answer-key

Large Collection of Go Math Grade 5 Answer Key Chapter 3 Add and Subtract Decimals is provided here.  Get acquainted with the topics of Grade 5 Ch 3 Add and Subtract Decimals through quick links available. Learning will be much simple using the 3rd Grade Go Math Solutions Key Chapter 3 Add and Subtract Decimals. We have listed detailed explanations for all the Problems in Go Math Grade 5 Chapter 3 Add and Subtract Decimals Solution Key. You can get the Go math Grade 5 Answer Key free of cost here to kick start your preparation.

Go Math Grade 5 Chapter 3 Add and Subtract Decimals Answer Key

HMH Go Math Grade 5 Answer Key PDF is written in a simple and easy to understand language. Solutions provided covers the Cumulative Practice, Practice Test, Chapter Test of Go Math 5th Grade Ch 3 Add and Subtract Decimals. Score better grades in exams taking the help of the Go math grade 5 Chapter 3 Answer Key and we don’t charge any amount. 5th Grade Go Math Answer Key Chapter 3 Add and Subtract Decimals Topics are given using the quick links. All you need to do is simply tap on them and get a grip on them.

Lesson 1: Investigate • Thousandths

Lesson 2: Place Value of Decimals

Lesson 3: Compare and Order Decimals

Lesson 4: Round Decimals

Lesson 5: Investigate • Decimal Addition

Lesson 6: Investigate • Decimal Subtraction

Mid-Chapter Checkpoint

Lesson 7: Estimate Decimal Sums and Differences

Lesson 8: Add Decimals

Lesson 9: Subtract Decimals

Lesson 10: Algebra • Patterns with Decimals

Lesson 11: Problem Solving • Add and Subtract Money

Lesson 12: Choose a Method

Review/Test

Share and Show – Page No. 111

Write the decimal shown by the shaded parts of each model.

Question 1.
Go Math Grade 5 Answer Key Chapter 3 Add and Subtract Decimals img 1
______

Answer:
0.665

Explanation:
The given picture shows
6 hundredths, 6 tenths, and 5 thousandths are shaded
665/1000 = 0.665

Question 2.
Go Math Grade 5 Answer Key Chapter 3 Add and Subtract Decimals img 2
______

Answer:
0.398

Explanation:
The given picture shows
3 hundredths, 9 tenths, and 8 thousandths are shaded
398/1000 = 0.398

Question 3.
Go Math Grade 5 Answer Key Chapter 3 Add and Subtract Decimals img 3
______

Answer:
0.181

Explanation:
The given picture shows
1 hundredth, 8 tenths, and 1 thousandth are shaded
181/1000 = 0.181

Question 4.
Go Math Grade 5 Answer Key Chapter 3 Add and Subtract Decimals img 4
______

Answer:
0.990

Explanation:
The given picture shows
9 hundredth, 9 tenths, and 0 thousandths are shaded
990/1000 = 0.990

Complete the sentence.

Question 5.
0.6 is 10 times as much as ______ .
______

Answer:
\(\frac{6}{100}\) = 0.06

Explanation:
Let the unknown number is S
0.6 = 10S
S = 0.6/10 = \(\frac{6}{10}\) x \(\frac{1}{10}\)
S = \(\frac{6}{100}\) = 0.06

Question 6.
0.007 is \(\frac{1}{10}\) of _______ .
______

Answer:
0.07

Explanation:
Let the unknown number is S
0.007 = \(\frac{1}{10}\)S
S = 0.007 x 10 = 0.07

Question 7.
0.008 is \(\frac{1}{10}\) of ________ .
______

Answer:
0.08

Explanation:
Let the unknown number is S
0.008 = \(\frac{1}{10}\)S
S = 0.008 x 10 = 0.08

Question 8.
0.5 is 10 times as much as ______ .
______

Answer:
0.05

Explanation:
Let the unknown number is S
0.5 = 10S
S = 0.5/10 = \(\frac{5}{10}\) x \(\frac{1}{10}\)
S = \(\frac{5}{100}\) = 0.05

Use place-value patterns to complete the table.

Question 9.
Go Math Grade 5 Answer Key Chapter 3 Add and Subtract Decimals img 5
Type below:
_________

Answer:
grade 5 chapter 3 Add and Subtract Decimals image 1

Explanation:
0.2 is 10 times as much as
Let the unknown number is S
0.2 = 10S
S = 0.2/10 = 0.02
0.2 is 1/10 of
0.2 = S/10
S = 0.2 x 10 = 2
0.07 is 10 times as much as
Let the unknown number is S
0.07 = 10S
S = 0.07/10 = 0.007
0.07 is 1/10 of
0.07 = S/10
S = 0.07 x 10 = 0.7
0.05 is 10 times as much as
Let the unknown number is S
0.05 = 10S
S = 0.05/10 = 0.005
0.05 is 1/10 of
0.05 = S/10
S = 0.05 x 10 = 0.5
0.4 is 10 times as much as
Let the unknown number is S
0.4 = 10S
S = 0.4/10 = 0.04
0.4 is 1/10 of
0.4 = S/10
S = 0.4 x 10 = 4

Question 10.
Go Math Grade 5 Answer Key Chapter 3 Add and Subtract Decimals img 6
Type below:
_________

Answer:
grade 5 chapter 3 Add and Subtract Decimals image 2

Explanation:
0.06 is 10 times as much as
Let the unknown number is S
0.06 = 10S
S = 0.06/10 = 0.006
0.06 is 1/10 of
0.06 = S/10
S = 0.06 x 10 = 0.6
0.9 is 10 times as much as
Let the unknown number is S
0.9 = 10S
S = 0.9/10 = 0.09
0.9 is 1/10 of
0.9 = S/10
S = 0.9 x 10 = 9
0.3 is 10 times as much as
Let the unknown number is S
0.3 = 10S
S = 0.3/10 = 0.03
0.3 is 1/10 of
0.3 = S/10
S = 0.3 x 10 = 3
0.08 is 10 times as much as
Let the unknown number is S
0.08 = 10S
S = 0.08/10 = 0.006
0.08 is 1/10 of
0.08 = S/10
S = 0.08 x 10 = 0.8

Problem Solving Applications – Page No. 112

Use the table for 17 and 20.
Go Math Grade 5 Answer Key Chapter 3 Add and Subtract Decimals img 7

Question 17.
A science teacher showed an image of a carpenter bee on a wall. The image is 10 times as large as the actual bee. Then he showed another image of the bee that is 10 times as large as the first image. What is the length of the bee in the second image?
______ meters

Answer:
2.5 meters

Explanation:
A science teacher showed an image of a carpenter bee on a wall. The image is 10 times as large as the actual bee.
carpenter bee = 0.025
The first image = 0.025 x 10 = 0.25
The second image = 10 times as large as the first image = 0.25 x 10 = 2.5

Question 18.
Math Explain how you can use place value to describe how 0.05 and 0.005 compare.
Type below:
_________

Answer:
Both numbers have 0 ones. So, we cannot compare these two numbers.
Look at the tenths. Both numbers have 0 tenths. So, we cannot compare these numbers.
Look at the hundredths.
The first number has 5 hundredths. The second number has 0 hundredths.
So, 0.05 > 0.005

Question 19.
Use Repeated Reasoning Terry, Sasha, and Harry each chose a number. Terry’s number is ten times as much as Sasha’s. Harry’s number is \(\frac{1}{10}\) of Sasha’s. Sasha’s number is 0.4. What number did each person choose?
Terry’s number: ______
Harry’s number: ______

Answer:
Terry’s number: 4
Harry’s number: 0.04

Explanation:
Sasha’s number is 0.4
Terry’s number is ten times as much as Sasha’s.
Terry’s number = 10 x 0.4 = 10 x \(\frac{4}{10}\) = 4
Harry’s number is \(\frac{1}{10}\) of Sasha’s.
Harry’s number = \(\frac{1}{10}\) x 0.4 = \(\frac{1}{10}\) x \(\frac{4}{10}\) = \(\frac{4}{100}\) = 0.04
Sasha’s number is 0.4
Terry’s number is 4
Harry’s number is 0.04

Question 20.
An atlas beetle is about 0.14 of a meter long. How does the length of the atlas beetle compare to the length of a leafcutting bee?
Type below:
_________

Answer:
An atlas beetle is about 0.14 of a meter long.
length of a leafcutting bee = 0.014
1 tenth is greater than 0 tenths.
So, 0.14 > 0.014
So, atlas beetle length is greater than the length of a leafcutting bee

Question 21.
Choose the numbers that make the statement true.
0.65 is 10 times as much as Go Math Grade 5 Answer Key Chapter 3 Add and Subtract Decimals img 8 and \(\frac{1}{10}\) of Go Math Grade 5 Answer Key Chapter 3 Add and Subtract Decimals img 9
Type below:
_________

Answer:
0.65 is 10 times as much as 0.065
0.65 is \(\frac{1}{10}\) of 6.5

Explanation:
0.65 is 10 times as much as
0.65 = 10S
S = 0.65/10 = 0.065
0.65 is \(\frac{1}{10}\) of
0.65 x 10 = 6.5
So, 0.65 is 10 times as much as 0.065
0.65 is \(\frac{1}{10}\) of 6.5

Share and Show – Page No. 115

Question 1.
Complete the place-value chart to find the value of each digit.
Go Math Grade 5 Answer Key Chapter 3 Add and Subtract Decimals img 10
Type below:
_________

Answer:
grade 5 chapter 3 Add and Subtract Decimals 115image 1

Explanation:
3 x 1 = 3
5 Tenths = 5 x 1/10 = 0.5
2 hundredths = 2 x 1/100 = 0.02
3 thousandths = 3 x 1/1000 = 0.003

Write the value of the underlined digit.

Question 2.
0.543
Type below:
_________

Answer:
0.04

Explanation:
(0 x 1) + (5 x \(\frac{1}{10}\)) + (4 x \(\frac{1}{100}\)) + (3 x \(\frac{1}{1000}\))
4 x \(\frac{1}{100}\) = 4 hundredths = 0.04

Question 3.
6.234
Type below:
_________

Answer:
0.2

Explanation:
(6 x 1) + (2 x \(\frac{1}{10}\)) + (3 x \(\frac{1}{100}\)) + (4 x \(\frac{1}{1000}\))
2 x \(\frac{1}{10}\) = 2 tenths = 0.2

Question 4.
3.954
Type below:
_________

Answer:
0.004

Explanation:
(3 x 1) + (9 x \(\frac{1}{10}\)) + (5 x \(\frac{1}{100}\)) + (4 x \(\frac{1}{1000}\))
4 x \(\frac{1}{1000}\) = 4 thousandths = 0.004

Write the number in two other forms.

Question 5.
0.253
Type below:
_________

Answer:
Word Form: two hundred fifty-three thousandths
Expanded Form: (0 x 1) + (2 x \(\frac{1}{10}\)) + (5 x \(\frac{1}{100}\)) + (3 x \(\frac{1}{1000}\))

Question 6.
7.632
Type below:
_________

Answer:
Word Form: seven and six hundred thirty-two thousandths
Expanded Form: (7 x 1) + (6 x \(\frac{1}{10}\)) + (3 x \(\frac{1}{100}\)) + (2 x \(\frac{1}{1000}\))

On Your Own

Write the value of the underlined digit.

Question 7.
0.496
Type below:
_________

Answer:
0.09

Explanation:
(0 x 1) + (4 x \(\frac{1}{10}\)) + (9 x \(\frac{1}{100}\)) + (6 x \(\frac{1}{1000}\))
9 x \(\frac{1}{100}\) = 9 hundredths = 0.09

Question 8.
2.726
Type below:
_________

Answer:
0.7

Explanation:
(2 x 1) + (7 x \(\frac{1}{10}\)) + (2 x \(\frac{1}{100}\)) + (6 x \(\frac{1}{1000}\))
7 x \(\frac{1}{10}\) = 0.7

Question 9.
1.066
Type below:
_________

Answer:
0.006

Explanation:
(1 x 1) + (0 x \(\frac{1}{10}\)) + (6 x \(\frac{1}{100}\)) + (6 x \(\frac{1}{1000}\))
6 x \(\frac{1}{1000}\) = 0.006

Question 10.
6.399
Type below:
_________

Answer:
0.3

Explanation:
(6 x 1) + (3 x \(\frac{1}{10}\)) + (9 x \(\frac{1}{100}\)) + (9 x \(\frac{1}{1000}\))
3 x \(\frac{1}{10}\) = 0.3

Question 11.
0.002
Type below:
_________

Answer:
0.002

Explanation:
(0 x 1) + (0 x \(\frac{1}{10}\)) + (0 x \(\frac{1}{100}\)) + (2 x \(\frac{1}{1000}\))
2 x \(\frac{1}{1000}\) = 0.002

Question 12.
4.371
Type below:
_________

Answer:
0.001

Explanation:
(4 x 1) + (3 x \(\frac{1}{10}\)) + (7 x \(\frac{1}{100}\)) + (1 x \(\frac{1}{1000}\))
1 x \(\frac{1}{1000}\) = 0.001

Write the number in two other forms.

Question 13.
0.489
Type below:
_________

Answer:
Word Form: four hundred eighty-nine thousandths
Expanded Form: (0 x 1) + (4 x \(\frac{1}{10}\)) + (8 x \(\frac{1}{100}\)) + (9 x \(\frac{1}{1000}\))

Question 14.
5.916
Type below:
_________

Answer:
Word Form: five and nine hundred sixteen thousandths
Expanded Form: (5 x 1) + (9 x \(\frac{1}{10}\)) + (1 x \(\frac{1}{100}\)) + (6 x \(\frac{1}{1000}\))

Problem Solving Applications – Page No. 116

Use the table for 15–16.
Go Math Grade 5 Answer Key Chapter 3 Add and Subtract Decimals img 11

Question 15.
What is the value of the digit 7 in New Mexico’s average annual rainfall?
Type below:
_________

Answer:
0.07

Explanation:
New Mexico’s average annual rainfall = 0.372
(0 x 1) + (3 x \(\frac{1}{10}\)) + (7 x \(\frac{1}{100}\)) + (2 x \(\frac{1}{1000}\))
7 x \(\frac{1}{100}\) = 0.07

Question 16.
Which of the states has an average annual rainfall with the least number in the thousandths place? What is another way to write the total annual rainfall in this state?
_________

Answer:
Wisconsin
(0 x 1) + (8 x \(\frac{1}{10}\)) + (2 x \(\frac{1}{100}\)) + (0 x \(\frac{1}{1000}\))

Explanation:
California = 0.564
New Mexico = 0.372
New York = 1.041
Wisconsin = 0.820
Maine = 1.074
The state that has an average annual rainfall with the least number in the thousandths place
0 < 1 < 2 < 4. So, the state is Wisconsin.
Another way to write the total annual rainfall in Wisconsin state is (0 x 1) + (8 x \(\frac{1}{10}\)) + (2 x \(\frac{1}{100}\)) + (0 x \(\frac{1}{1000}\))

Question 17.
Verify the Reasoning of Others Damian wrote the number four and twenty-three thousandths as 4.23. Describe and correct his error.
Type below:
_________

Answer:
four and twenty-three thousandths = 4 ones and 0 tenths, 2 hundredths, three thousandths = 4.023.
He has written 4.23 which is wrong.

Question 18.
Dan used a meter stick to measure some seedlings in his garden. One day, a corn stalk was 0.85 m tall. A tomato plant was 0.850 m. A carrot top was 0.085 m. Which plant was shortest?
_________

Answer:
the carrot top is the shortest plant

Explanation:
Dan used a meter stick to measure some seedlings in his garden. One day, a corn stalk was 0.85 m tall. A tomato plant was 0.850 m. A carrot top was 0.085 m. 0 tenths are less than the 8 tenths. So, 0.085 is less than 0.85 or 0.850. So, the carrot top is the shortest plant.

Question 19.
Math Explain how you know that the digit 6 does not have the same value in the numbers 3.675 and 3.756.
Type below:
_________

Answer:
In 3.675, the digit of 6 is in the tenths place. So, its value is 6 x 1/10 or 0.6.
In 3.756, the digit of 6 is in the thousandths place, so its value is 6 x 1/1000 or 0.006.

Question 20.
What is the value of the underlined digit? Mark all that apply.
0.589
Options:
a. 0.8
b. 0.08
c. eight tenths
d. eight hundredths
e. 8 × (\(\frac{1}{10}\))

Answer:
b. 0.08
d. eight hundredths

Explanation:
(0 x 1) + (5 x \(\frac{1}{10}\)) + (8 x \(\frac{1}{100}\)) + (9 x \(\frac{1}{1000}\))
8 x \(\frac{1}{100}\) = 8 hundredths = 0.08

Share and Show – Page No. 119

Question 1.
Use the place-value chart to compare the two numbers. What is the greatest place-value position where the digits differ?
Go Math Grade 5 Answer Key Chapter 3 Add and Subtract Decimals img 12
Type below:
_________

Answer:
3.472 > 3.445
They differ in hundredths position

Explanation:
Compare the ones; 3 = 3
Compare the tenths; 4 = 4
Compare the hundredths; 7 > 4
So, 3.472 > 3.445

Compare. Write <, >, or =.

Question 2.
4.563 ______ 4.536

Answer:
4.563 > 4.536

Explanation:
Compare the ones; 4 = 4
Compare the tenths; 5 = 5
Compare the hundredths; 6 > 3
So, 4.563 > 4.536

Question 3.
5.640 ______ 5.64

Answer:
5.640 = 5.64

Explanation:
Compare the ones; 5 = 5
Compare the tenths; 6 = 6
Compare the hundredths; 4 = 4
Compare the thousandths; 0 = 0
So, 5.640 = 5.64

Question 4.
8.673 ______ 8.637

Answer:
8.673 > 8.637

Explanation:
Compare the ones; 8 = 8
Compare the tenths; 6 = 6
Compare the hundredths; 7 > 3
So, 8.673 > 8.637

Name the greatest place-value position where the digits differ.

Name the greater number.

Question 5.
3.579; 3.564
______

Answer:
3.579 > 3.564
The greatest place-value position where the digits differ are hundredths

Explanation:
Compare the ones; 3 = 3
Compare the tenths; 5 = 5
Compare the hundredths; 7 > 6
So, 3.579 > 3.564
The greatest place-value position where the digits differ are hundredths

Question 6.
9.572; 9.637
______

Answer:
9.572 < 9.637
The greatest place-value position where the digits differ are tenths

Explanation:
Compare the ones; 9 = 9
Compare the tenths; 5 < 6
So, 9.572 < 9.637
The greatest place-value position where the digits differ are tenths

Question 7.
4.159; 4.152
______

Answer:
4.159 > 4.152
The greatest place-value position where the digits differ are thousandths

Explanation:
Compare the ones; 4 = 4
Compare the tenths; 1 = 1
Compare the hundredths; 5 = 5
Compare the thousandths; 9 > 2
So, 4.159 > 4.152
The greatest place-value position where the digits differ are thousandths

Order from least to greatest.

Question 8.
4.08; 4.3; 4.803; 4.038

Answer:
4.038, 4.08, 4.3, 4.803

Explanation:
Compare the ones; All are equal
Compare the tenths; 0 < 3 < 8.
So, 4.08, 4.038, 4.3, 4.803
Compare the hundredths of 4.08 and 4.038; 8 > 3
So, 4.038, 4.08, 4.3, 4.803

Question 9.
1.703; 1.037; 1.37; 1.073

Answer:
1.037, 1.073, 1.37, 1.703

Explanation:
Compare the ones; All are equal
Compare the tenths; 0 < 3 < 7.
So, 1.037; 1.073; 1.37; 1.703
Compare the hundredths of 1.037 and 1.073; 3 < 7
So, 1.037, 1.073, 1.37, 1.703

On Your Own

Compare. Write <, >, or =.

Question 10.
8.72 ______ 8.720

Answer:
8.72 = 8.720

Explanation:
Compare the ones; 8 = 8
Compare the tenths; 7 = 7
Compare the hundredths; 2 = 2
Compare the thousands; 0 = 0
So, 8.72 = 8.720

Question 11.
5.4 ______ 5.243

Answer:
5.4 > 5.243

Explanation:
Compare the ones; 5 = 5
Compare the tenths; 4 > 2
So, 5.4 > 5.243

Question 12.
1.036 ______ 1.306

Answer:
1.036 < 1.306

Explanation:
Compare the ones; 1 = 1
Compare the tenths; 0 < 3
So, 1.036 < 1.306

Question 13.
2.573 ______ 2.753

Answer:
2.573 < 2.753

Explanation:
Compare the ones; 2 = 2
Compare the tenths; 5 < 7
So, 2.573 < 2.753

Question 14.
9.300 ______ 9.3

Answer:
9.300 = 9.3

Explanation:
Compare the ones; 9 = 9
Compare the tenths; 3 = 3
Compare the hundredths; 0 = 0
Compare the thousands; 0 = 0
So, 9.300 = 9.3

Question 15.
6.76 ______ 6.759

Answer:
6.76 > 6.759

Explanation:
Compare the ones; 6 = 6
Compare the tenths; 7 = 7
Compare the hundredths; 6 > 5
So, 6.76 > 6.759

Order from greatest to least.

Question 16.
2.007; 2.714; 2.09; 2.97
______ ; ______ ; ______ ; ______

Answer:
2.97; 2.714; 2.09; 2.007

Explanation:
Compare the ones; All are equal
Compare the tenths; 0 < 7 < 9.
So, 2.007; 2.09; 2.714; 2.97
Compare the hundredths of 2.007 and 2.09; 0 < 9
So, 2.007; 2.09; 2.714; 2.97
Order from greatest to least = 2.97; 2.714; 2.09; 2.007

Question 17.
0.386; 0.3; 0.683; 0.836
______ ; ______ ; ______ ; ______

Answer:
0.836; 0.683; 0.386; 0.3

Explanation:
Compare the ones; All are equal
Compare the tenths; 0 < 3 < 6 < 8.
So, 0.386; 0.3; 0.683; 0.836
Compare the hundredths of 0.386 and 0.3; 8 > 0
So, 0.3; 0.386; 0.683; 0.836
Order from greatest to least = 0.836; 0.683; 0.386; 0.3

Question 18.
5.249; 5.43; 5.340; 5.209
______ ; ______ ; ______ ; ______

Answer:
5.43; 5.340; 5.249; 5.209

Explanation:
Compare the ones; All are equal
Compare the tenths; 2 < 3 < 4.
So, 5.249; 5.209; 5.340; 5.43
Compare the hundredths of 5.249 and 5.209; 4 > 0
So, 5.209; 5.249; 5.340; 5.43
Order from greatest to least = 5.43; 5.340; 5.249; 5.209

Question 19.
0.678; 1.678; 0.587; 0.687
______ ; ______ ; ______ ; ______

Answer:
1.678; 0.687; 0.678; 0.587

Explanation:
Compare the ones; 0 < 1
So, 0.678; 0.587; 0.687; 1.678
Compare the tenths of 0.678; 0.587; 0.687; 5 < 6.
So, 0.587; 0.678; 0.687; 1.678
Compare the hundredths of 0.678 and 0.687; 7 < 8
So, 0.587; 0.678; 0.687; 1.678
Order from greatest to least = 1.678; 0.687; 0.678; 0.587

Use Reasoning Algebra Find the unknown digit to make each statement true.

Question 20.
3.59 > 3.5 ______ 1 > 3.572

Answer:
3.59 > 3.581 > 3.572

Explanation:
The possible values are
3.573; 3.574; 3.575; 3.578; 3.579; 3.580; 3.581; 3.582; 3.583; 3.584; 3.585; 3.586; 3.587; 3.588; 3.589;
The digit that ends with 1 is 3.581.
So, the unknown digit is 3.581

Question 21.
6.837 > 6.83 ______ > 6.835

Answer:
6.837 > 6.836 > 6.835

Explanation:
The value must be 6.836. Because 6 is the only digit between 5 and 7.
So, the unknown digit is 6.836

Question 22.
2.45 < 2 ______ 6 < 2.461

Answer:
2.45 < 2.456 < 2.461

Explanation:
2.451; 2.452; 2.453; 2.454; 2.455; 2.456; 2.457; 2.458; 2.459; 2.460; 2.461
The digit that ends with 6 is 2.456.
So, the unknown digit is 2.456

Problem Solving Applications – Page No. 120

Use the table for 23–26.
Go Math Grade 5 Answer Key Chapter 3 Add and Subtract Decimals img 13

Question 23.
In comparing the height of the mountains, which is the greatest place value where the digits differ?
_________

Answer:
The greatest place value where the digits differ is hundredths

Explanation:
3.104; 3.134; 3.152
0 hundredths < 3 hundredths < 5 hundredths
3.152; Mount Steele, Yukon is the greatest mountain.
The greatest place value where the digits differ is hundredths.

Question 24.
Use Math Vocabulary How does the height of Mount Steele compare to the height of Mount Blackburn? Compare the heights using words.
Type below:
_________

Answer:
The Height of Mount Steele is greater than Height of Mount Blackburn.

Explanation:
Height of Mount Steele = 3.152
Height of Mount Blackburn = 3.104
3.152 > 3.104
The Height of Mount Steele is greater than Height of Mount Blackburn.

Question 25.
Explain how to order the heights of the mountains from greatest to least.
Type below:
_________

Answer:
3.152 > 3.134 > 3.104

Explanation:
3.104; 3.134; 3.152
0 hundredths < 3 hundredths < 5 hundredths
3.152 > 3.134 > 3.104

Question 26.
What if the height of Mount Blackburn were 0.05 mile greater? Would it then be the mountain with the greatest height? Explain.
______

Answer:
Height of Mount Blackburn = 3.104 + 0.05 = 3.154
3.154 > 3.152 > 3.134.
Yes, Mount Blackburn would have the greatest height if it has 0.05 mile greater.

Question 27.
Orlando kept a record of the total rainfall each month for 5 months.
Go Math Grade 5 Answer Key Chapter 3 Add and Subtract Decimals img 14
Order the months from the least amount of rainfall to the greatest amount of rainfall.
Least ______ ______ ______ ______ ______ Greatest

Answer:
Least: 3.09; 3.75; 4.04; 4.09; 4.42 Greatest

Explanation:
3.75; 4.42; 4.09; 3.09; 4.04
3 < 4
3.75; 3.09; 4.42; 4.09; 4.04
Compare tenths of 3.75 and 3.09; 0 < 7
3.09; 3.75; 4.42; 4.09; 4.04
Compare tenths of 4.42; 4.09; 4.04; 0 <4
3.09; 3.75; 4.09; 4.04; 4.42
Compare hundredths of 4.09 and 4.04; 4 < 9
So, 3.09; 3.75; 4.04; 4.09; 4.42

Share and Show – Page No. 123

Write the place value of the underlined digit. Round each number to the place of the underlined digit.

Question 1.
0.673
Place value: ________
Round: ________

Answer:
Place value: 7 hundredths = 0.07
Round: 0.670

Explanation:
0.673
(0 x 1) + (6 x \(\frac{1}{10}\)) + (7 x \(\frac{1}{100}\)) + (3 x \(\frac{1}{1000}\))
Place Value: 7 x \(\frac{1}{100}\) = 7 hundredths = 0.07
0.673
3 < 5
0.670

Question 2.
4.282
Place value: ________
Round: ________

Answer:
Place value: 2 tenths = 0.2
Round: 4.300

Explanation:
4.282
(4 x 1) + (2 x \(\frac{1}{10}\)) + (8 x \(\frac{1}{100}\)) + (2 x \(\frac{1}{1000}\))
Place Value: 2 x \(\frac{1}{10}\) = 2 tenths = 0.2
4.282
8 > 5
4.300

Question 3.
12.917
Place value: ________
Round: ________

Answer:
Place value: 2 ones = 2
Round: 13

Explanation:
12.917
(1 x 10) + (2 x 1) + (9 x \(\frac{1}{10}\)) + (1 x \(\frac{1}{100}\)) + (7 x \(\frac{1}{1000}\))
Place Value: 2 x 1 = 2 ones = 2
12.917
9 > 5
13

Name the place value to which each number was rounded.

Question 4.
0.982 to 0.98
________

Answer:
The hundredths

Explanation:
As 2 < 5, We round 0.982 to 0.98.
The place value of the digit 8 is hundredths.
The hundredths

Question 5.
3.695 to 4
________

Answer:
The ones

Explanation:
As 6 > 5, We round 3.695 to 4.
The place value of the digit 3 is ones.
The ones

Question 6.
7.486 to 7.5
________

Answer:
The tenths

Explanation:
As 8 > 5, We round 7.486 to 7.5.
The place value of the digit 4 is tenths.
The tenths

On Your Own

Write the place value of the underlined digit. Round each number to the place of the underlined digit.

Question 7.
0.592
Place value: ________
Round: ________

Answer:
Place value: 5 tenths = 0.5
Round: 0.6

Explanation:
0.592
(0 x 1) + (5 x \(\frac{1}{10}\)) + (9 x \(\frac{1}{100}\)) + (2 x \(\frac{1}{1000}\))
Place Value: 5 x \(\frac{1}{10}\) = 5 tenths = 0.5
0.592
9 > 5
0.6

Question 8.
6.518
Place value: ________
Round: ________

Answer:
Place value: 6 ones = 6
Round: 7

Explanation:
6.518
(6 x 1) + (5 x \(\frac{1}{10}\)) + (1 x \(\frac{1}{100}\)) + (8 x \(\frac{1}{1000}\))
Place Value: 6 x 1 = 6 ones = 6
6.518
5 = 5
7

Question 9.
0.809
Place value: ________
Round: ________

Answer:
Place value: 0 hundredths = 0
Round: 0.8

Explanation:
0.809
(0 x 1) + (8 x \(\frac{1}{10}\)) + (0 x \(\frac{1}{100}\)) + (9 x \(\frac{1}{1000}\))
Place Value: 0 x \(\frac{1}{100}\) = 0 hundredths = 0
0.809
0 < 5
0.800

Question 10.
3.334
Place value: ________
Round: ________

Answer:
Place value: 7 tenths = 0.7
Round: 3

Explanation:
3.334
(3 x 1) + (3 x \(\frac{1}{10}\)) + (3 x \(\frac{1}{100}\)) + (4 x \(\frac{1}{1000}\))
Place Value: 3 x \(\frac{1}{10}\) = 7 tenths = 0.7
3.334
3 < 5
3.000

Question 11.
12.074
Place value: ________
Round: ________

Answer:
Place value: 0 tenths = 0
Round: 12.1

Explanation:
12.074
(1 x 10) + (2 x 1) + (0 x \(\frac{1}{10}\)) + (7 x \(\frac{1}{100}\)) + (4 x \(\frac{1}{1000}\))
Place Value: 0 x \(\frac{1}{10}\) = 0 tenths = 0
12.074
7 > 5
12.1

Question 12.
4.494
Place value: ________
Round: ________

Answer:
Place value: 9 hundredths = 0.09
Round: 4.49

Explanation:
4.494
(4 x 1) + (4 x \(\frac{1}{10}\)) + (9 x \(\frac{1}{100}\)) + (4 x \(\frac{1}{1000}\))
Place Value: 9 x \(\frac{1}{100}\) = 9 hundredths = 0.09
4.494
4 < 5
4.49

Name the place value to which each number was rounded.

Question 13.
0.328 to 0.33
________

Answer:
The hundredths

Explanation:
As 8 > 5, We round 0.328 to 0.33.
The place value of the digit 2 is hundredths.
The hundredths

Question 14.
2.607 to 2.61
________

Answer:
The hundredths

Explanation:
As 7 > 5, We round 2.607 to 2.61.
The place value of the digit 0 is hundredths.
The hundredths

Question 15.
12.583 to 13
________

Answer:
The ones

Explanation:
As 5 = 5, We round 12.583 to 13.
The place value of the digit 2 is ones.
The ones

Round 16.748 to the place named.

Question 16.
tenths: ______

Answer:
16.7

Explanation:
Round 16.748 to the nearest tenths
The tenth digit is 7. So, 4 < 5
16.7

Question 17.
hundredths: ______

Answer:
16.75

Explanation:
Round 16.748 to the nearest hundredths
The hundredth digit is 4. So, 8 > 5
16.75

Question 18.
ones: ______

Answer:
17

Explanation:
Round 16.748 to the nearest ones
The ones digit is 6. So, 7 > 5
17

Question 19.
Explain what happens when you round 4.999 to the nearest tenth.
Type below:
_________

Answer:
5

Explanation:
round 4.999 to the nearest tenth
The tenth digit is 9. So, 9 > 5
5

Problem Solving Applications – Page No. 124

Use the table for 20–22.
Go Math Grade 5 Answer Key Chapter 3 Add and Subtract Decimals img 15

Question 20.
The speeds of two insects when rounded to the nearest whole number are the same. Which two insects are they?
_________
_________

Answer:
Bumblebee
Honeybee

Explanation:
Dragonfly = 6.974 meters; nearest whole number = 7
Horsefly = 3.934 meters; nearest whole number = 4
Bumblebee = 2.861 meters; nearest whole number = 3
Honeybee = 2.548 meters; nearest whole number = 3
Housefly = 1.967 meters; nearest whole number = 2
Bumblebee and Honeybee speeds are the same if their rounded to the nearest whole number.

Question 21.
What is the speed of the housefly rounded to the nearest hundredth?
______ meters per second

Answer:
3.93 meters per second

Explanation:
Horsefly = 3.934 meters rounded to the nearest hundredth
The hundredth digit is 3. So, 4 < 5
3.93

Question 22.
What’s the Error? Mark said that the speed of a dragonfly rounded to the nearest tenth was 6.9 meters per second. Is he correct? If not, what is his error?
Type below:
_________

Answer:
Dragonfly = 6.974 meters rounded to the nearest tenth.
The tenth digit is 9. So, 7 > 5
7.
So, Mark said is wrong.

Question 23.
A rounded number for the speed of an insect is 5.67 meters per second. What are the fastest and slowest speeds to the thousandths that could round to 5.67 meters per second? Explain.
Type below:
_________

Answer:
The slowest speed to the thousandths that could round to 5.67 meters per second is 5.671
The fastest speed to the thousandths that could round to 5.67 meters per second is 5.674

Explanation:
To find the slowest speed to the thousandths that could round to 5.67 meters per second we need to find the lowest digit which will not affect the digit in the hundredths place, and that is 1. So, the slowest speed to the thousandths that could round to 5.67 meters per second is 5.671.
To find the fastest speed to the thousandths that could round to 5.67 meters per second we need to find the greatest digit which will not affect the digit in the hundredths place, and that is 4. So, the fastest speed to the thousandths that could round to 5.67 meters per second is 5.674.

Question 24.
The price of a certain box of cereal at the grocery store is $0.258 per ounce. For numbers 24a–24c, select True or False for each statement.
a. Rounded to the nearest whole number, the price is $1 per ounce.
i. yes
ii. no

Answer:
ii. no

Explanation:
$0.258
2 < 5.
So, if we rounded to the nearest whole number, the value becomes 0.

Question 24.
b. Rounded to the nearest tenth, the price is $0.3 per ounce.
i. yes
ii. no

Answer:
i. yes

Explanation:
$0.258
5 = 5
So, $3 is the answer.

Question 24.
c. Rounded to the nearest hundredth, the price is $0.26 per ounce.
i. yes
ii. no

Answer:
i. yes

Explanation:
$0.258
8 > 5
$0.26

Share and Show – Page No. 127

Complete the quick picture

Question 1.
1.37 + 1.85 =
Go Math Grade 5 Answer Key Chapter 3 Add and Subtract Decimals img 16
______

Answer:
grade 5 chapter 3 Add and Subtract Decimals 127 image 1

Explanation:
1.37 + 1.85 = 3. 22
Add hundredths; 7 + 5 = 12; Regroup
Add tenths; 3 + 8 + 1 = 12; Regroup
Add tens; 1 + 1 + 1 = 3

Add. Draw a quick picture.

Question 2.
0.9 + 0.7 =
______

Answer:
0.9 + 0.7 = 1.6
grade 5 chapter 3 Add and Subtract Decimals 127 image 2

Explanation:
0.9 + 0.7 =
Add tenths 9 + 7 = 16; Regroup
Add ones 0 + 0 + 1 = 1
0.9 + 0.7 = 1.6

Question 3.
0.65 + 0.73 =
______

Answer:
0.65 + 0.73 = 1.38
grade 5 chapter 3 Add and Subtract Decimals 127 image 3

Explanation:
0.65 + 0.73 = 1.38
Add hundredths 5 + 3 = 8;
Add tenths 6 + 7 = 13; Regroup
Add ones 0 + 0 + 1 = 1
0.65 + 0.73 = 1.38

Question 4.
1.3 + 0.7 =
______

Answer:
1.3 + 0.7 = = 2
grade 5 chapter 3 Add and Subtract Decimals 127 image 4

Explanation:
Add tenths 3 + 7 = 10; Regroup
Add ones 1 + 1 = 2
1.3 + 0.7 = = 2

Question 5.
2.72 + 0.51 =
______

Answer:
2.72 + 0.51 = 3.23
grade 5 chapter 3 Add and Subtract Decimals 127 image 5

Explanation:
Add hundredths 2 + 1 = 3
Add tenths 5 + 7 = 12; Regroup
Add ones 2 + 0 + 1 = 3
2.72 + 0.51 = 3.23

Problem Solving Applications

Question 6.
Carissa bought 2.35 pounds of chicken and 2.7 pounds of turkey for lunches this week. She used a quick picture to and the amount of lunch meat. Does Carissa’s work make sense? Explain.
Go Math Grade 5 Answer Key Chapter 3 Add and Subtract Decimals img 17
______

Answer:
Yes; Because she bought 2.35 pounds of chicken and 2.7 pounds
2.35 + 2.7 = 5.05 pounds.
there is 5 ones and 5 hundredths.

Sense or Nonsense? – Page No. 128

Question 7.
Robyn and Jim used quick pictures to model 1.85 + 2.73.
Robyn’s Work
Go Math Grade 5 Answer Key Chapter 3 Add and Subtract Decimals img 18
1.85 + 2.73 = 3.158
Does Robyn’s work make sense?
Explain your reasoning.
Type below:
_________

Answer:
Robyn’s work doesn’t make sense. Because 7 + 8 = 15. So, he needs to regroup and then add 1 to the one’s digits.
1 + 2 + 1 = 4
1.85 + 2.73 = 4.58 is the correct answer.

Jim’s Work
Go Math Grade 5 Answer Key Chapter 3 Add and Subtract Decimals img 19
1.85 + 2.73 = 4.58
Does Jim’s work make sense?
Explain your reasoning.

Answer:
Jim’s work makes sense.
Add: 1.85 + 2.73 = 4.58.
4 ones, 5 tenths, and 8 hundredths.

Question 8.
Make Arguments Explain how you would help Robyn understand that regrouping is important when adding decimals.
Type below:
_________

Answer:
Regrouping is important when adding decimals. When you add two digits, if their addition is more than 10 then we need to regroup the values to find the correct answer.

Question 9.
Write a decimal addition problem that requires regrouping the hundredths. Explain how you know you will need to regroup.
Type below:
_________

Answer:
Let’s add 1.47 and 1.35 As we have more than 9 hundredths we have to regroup and mid the tenths.
So, now we have 8 tenths and two-hundredths left.
Also, as we have less than 9 tenths we do not have to regroup and add the ones.
The answer is 2.82.
As we have more than 9 hundredths we have to regroup and mid the tenths.

Share and Show – Page No. 131

Complete the quick picture to find the difference.

Question 1.
0.62 − 0.18 =
Go Math Grade 5 Answer Key Chapter 3 Add and Subtract Decimals img 20
______

Answer:
grade 5 chapter 3 Add and Subtract Decimals 131 image 1

Explanation:
0.62 − 0.18
Subtract hundredths:  2 – 8;
There are not enough hundredths. So, regroup
12 – 8 = 4
Subtract tenths: 5 – 1 = 4
Subtract ones: 0 – 0 = 0
So, 0.62 − 0.18 = 0.44

Subtract. Draw a quick picture.

Question 2.
3.41 − 1.74 =
______

Answer:
grade 5 chapter 3 Add and Subtract Decimals 131 image 2

Explanation:
3.41 − 1.74
Subtract hundredths:  1 – 4;
There are not enough hundredths. So, regroup
11 – 4 = 7
Subtract tenths: 3 – 7
There are not enough tenths. So, regroup
13 – 7 = 6
Subtract ones: 2 – 1 = 1
So, 3.41 − 1.74 = 1.67

Question 3.
0.84 − 0.57 =
______

Answer:
grade 5 chapter 3 Add and Subtract Decimals 131 image 3

Explanation:
0.84 − 0.57
Subtract hundredths:  4 – 7;
There are not enough hundredths. So, regroup
14 – 7 = 7
Subtract tenths: 7 – 5 = 2
Subtract ones: 0 – 0 = 0
So, 0.84 − 0.57 = 0.27

Question 4.
4.05 − 1.61 =
______

Answer:
grade 5 chapter 3 Add and Subtract Decimals 131 image 4

Explanation:
4.05 − 1.61
Subtract hundredths:  5 – 1 = 4;
Subtract tenths: 0 – 6
There are not enough hundredths. So, regroup
10 – 6 = 4
Subtract ones: 3 – 1 = 2
So, 4.05 − 1.61 = 2.44

Problem Solving Applications

Question 6.
Write a decimal subtraction equation that requires regrouping from the tenths. Explain how you know you will need to regroup.
Type below:
__________

Answer:
Subtract 0.32 and 0.05
Subtract hundredths. As there are not enough hundredths we have to regroup. So, we have 10 more hundredths and one-tenth I.
Subtract tenths. As there are enough tenths we do not have to regroup.
The answer: 0.27

Pose a Problem – Page No. 132

Question 7.
Antonio left his MathBoard on his desk during lunch. The quick picture below shows the problem he was working on when he left.
Go Math Grade 5 Answer Key Chapter 3 Add and Subtract Decimals img 21
Write a word problem that can be solved using the quick picture above.
Pose a problem.          Solve your problem.
Type below:
__________

Answer:
From the given picture, he has drawn eight-hundredths and crosses two-hundredths. Then, he has drawn six tenths and crossed four-tenths. When comes to ones. he has taken three ones and crossed two out of them.
there are 7 – 2 = 5 hundredths
6 – 4 = 2 tenths
3 – 2 = 1 ones
1.25

Question 7.
Use Reasoning Describe how you can change the problem by changing the quick picture.
Type below:
__________

Answer:
By changing the quick picture, the value of place value is changed.

Question 8.
The price of a box of markers at a retail store is $4.65. he price of a box of markers at the school bookstore is $3.90. How much more do the markers cost at the retail store? Explain how you can use a quick picture to solve the problem.
$ ______

Answer:
The price of a box of markers at a retail store is $4.65. he price of a box of markers at the school bookstore is $3.90.
$4.65 – $3.90 = $0.75
grade 5 chapter 3 Add and Subtract Decimals 131 image 5

Concepts and Skills – Page No. 133

Question 1.
Explain how you can use base-ten blocks to find 1.54 + 2.37.
Type below:
__________

Answer:
1.54 + 2.37
Add hundredths 4 + 7 = 11; Regroup
Add tenths 5 + 3 + 1 = 9;
Add ones 2 + 1 = 3
1.54 + 2.37 = 3.91
We have to use three square boxes to show three ones, 9 lines to show 9 tenths, and 1 dot to show one hundredth

Complete the sentence.

Question 2.
0.04 is \(\frac{1}{10}\) of

Answer:
0.04 is \(\frac{1}{10}\) of 0.4

Explanation:
Let the unknown number is S
0.04 = \(\frac{1}{10}\)S
S = 0.04 x 10 = 0.4

Question 3.
0.06 is 10 times as much as

Answer:
\(\frac{6}{1000}\) = 0.006

Explanation:
Let the unknown number is S
0.06 = 10S
S = 0.06/10
S = \(\frac{6}{100}\) x \(\frac{1}{10}\)
S = \(\frac{6}{1000}\) = 0.006

Write the value of the underlined digit.

Question 4.
6.54
Type below:
__________

Answer:
4 hundredths = 0.04

Explanation:
(6 x 1) + (5 x \(\frac{1}{10}\)) + (4 x \(\frac{1}{100}\))
4 x \(\frac{1}{100}\) = 4 hundredths = 0.04

Question 5.
0.837
Type below:
__________

Answer:
8 tenths = 0.8

Explanation:
(0 x 1) + (8 x \(\frac{1}{10}\)) + (3 x \(\frac{1}{100}\)) + (7 x \(\frac{1}{1000}\))
8 x \(\frac{1}{10}\) = 8 tenths = 0.8

Question 6.
8.702
Type below:
__________

Answer:
2 thousandths = 0.002

Explanation:
(8 x 1) + (7 x \(\frac{1}{10}\)) + (0 x \(\frac{1}{100}\)) + (2 x \(\frac{1}{1000}\))
2 x \(\frac{1}{1000}\) = 2 thousandths = 0.002

Question 7.
9.173
Type below:
__________

Answer:
9 ones = 9

Explanation:
(9 x 1) + (1 x \(\frac{1}{10}\)) + (7 x \(\frac{1}{100}\)) + (3 x \(\frac{1}{1000}\))
9 x 1 = 9 ones = 9

Compare. Write <, >, or =.

Question 8.
6.52 _____ 6.520

Answer:
6.52 = 6.520

Explanation:
Compare the ones; 6 = 6
Compare the tenths; 5 = 5
Compare the hundredths; 2 = 2
Compare the thousandths; 0 = 0
So, 6.52 = 6.520

Question 9.
3.589 _____ 3.598

Answer:
3.589 < 3.598

Explanation:
Compare the ones; 3 = 3
Compare the tenths; 5 = 5
Compare the hundredths; 8 < 9
So, 3.589 < 3.598

Question 10.
8.483 _____ 8.463

Answer:
8.483 > 8.463

Explanation:
Compare the ones; 8 = 8
Compare the tenths; 4 = 4
Compare the hundredths; 8 > 6
So, 8.483 > 8.463

Write the place value of the underlined digit. Round each number to the place of the underlined digit.

Question 11.
0.724
Place value: __________
Round: __________

Answer:
Place value: 7 tenths = 0.7
Round: 0.7

Explanation:
0.724
(0 x 1) + (7 x \(\frac{1}{10}\)) + (2 x \(\frac{1}{100}\)) + (4 x \(\frac{1}{1000}\))
Place Value: 7 x \(\frac{1}{10}\) = 7 tenths = 0.7
0.724
2 < 5
0.7

Question 12.
2.576
Place value: __________
Round: __________

Answer:
Place value: 2 ones = 2
Round: 3

Explanation:
2.576
(2 x 1) + (5 x \(\frac{1}{10}\)) + (7 x \(\frac{1}{100}\)) + (6 x \(\frac{1}{1000}\))
Place Value: 2 x 1 = 2 ones = 2
2.576
5 = 5
3

Question 13.
4.769
Place value: __________
Round: __________

Answer:
Place value: 6 hundredths = 0.06
Round: 4.77

Explanation:
4.769
(4 x 1) + (7 x \(\frac{1}{10}\)) + (6 x \(\frac{1}{100}\)) + (9 x \(\frac{1}{1000}\))
Place Value: 6 x \(\frac{1}{100}\)) = 6 hundredths = 0.06
4.769
9 > 5
4.77

Draw a quick picture to find the sum or difference.

Question 14.
2.46 + 0.78 =

Answer:
grade 5 chapter 3 Add and Subtract Decimals 133 image 1

Explanation:
2.46 + 0.78 = 3.24

Question 15.
3.27 − 1.84 =

Answer:
grade 5 chapter 3 Add and Subtract Decimals 133 image 2

Explanation:
3.27 − 1.84 = 1.43

Page No. 134

Question 16.
Marco read that a honeybee can fly up to 2.548 meters per second. He rounded the number to 2.55. To which place value did Marco round the speed of a honeybee?
__________

Answer:
Marco read that a honeybee can fly up to 2.548 meters per second. He rounded the number to 2.55.
The speed of a honeybee is 2.548.
Marco has to round this number to the nearest hundredth to get 2.55.
The digit in the hundredths places increases by 1.
The 8 > 5
So, the rounded number is 2.55.

Question 17.
What is the relationship between 0.04 and 0.004?
Type below:
__________

Answer:
Comapre ones; 0 = 0
Compare tenths; 0 = 0
Compare hundredths; 4 > 0
So, 0.04 > 0.004

Question 18.
Jodi drew a quick picture to model the answer for 3.14 − 1.75. Draw what her picture might look like.
Type below:
__________

Answer:
grade 5 chapter 3 Add and Subtract Decimals 133 image 3

Explanation:
Jodi drew a quick picture to model the answer for 3.14 − 1.75
3.14 – 1.75 = 1.39

Question 19.
The average annual rainfall in California is 0.564 of a meter per year. What is the value of the digit 4 in that number?
Type below:
__________

Answer:
The average annual rainfall in California is 0.564 of a meter per year.
(0 x 1) + (5 x \(\frac{1}{10}\)) + (6 x \(\frac{1}{100}\)) + (4 x \(\frac{1}{1000}\))
4 x \(\frac{1}{1000}\) = 4 thousandths = 0.004

Question 20.
Jan ran 1.256 miles on Monday, 1.265 miles on Wednesday, and 1.268 miles on Friday. What were her distances from greatest to least?
_____ mi; _____ mi; _____ mi

Answer:
1.268 mi; 1.265 mi; 1.256 mi

Explanation:
Jan ran 1.256 miles on Monday, 1.265 miles on Wednesday, and 1.268 miles on Friday.
Compare hundredths: 6 > 5
So, 1.265; 1.268; 1.256
Compare thousandths in 1.265 and 1.268
8 > 5
1.268 mi; 1.265 mi; 1.256 mi

Share and Show – Page No. 137

Use rounding to estimate.

Question 1.
2.3 4
1.9
+5.2 3
————
Estimate: _____

Answer:
Estimate: About 9

Explanation:
2.34; 3 < 5; 2
1.9; 9 > 5; 2
5.23; 2 < 5; 5
Add: 2 + 2 + 5 = 9

Question 2.
10.3 9
-4.2 8
————
Estimate: _____

Answer:
Estimate: About 6

Explanation:
10.39; 3 < 5; 10
4.28; 2 < 5; 4
Subtract: 10 – 4 = 6

Question 3.
$ 19.7 5
+$3.9 8
————
Estimate: $ _____

Answer:
Estimate: About $24

Explanation:
19.7 5; 7 > 5; 20
3.98; 9 > 5; 4
20 + 4 = 24

Use benchmarks to estimate.

Question 4.
0.3 4
0.1
+0.2 5
————
Estimate: _____

Answer:
Estimate: About 0.55

Explanation:
0.3 4 is closer to 0.35
0.1 is closer to 0
0.25
0.35 + 0 + 0.25 = 0.55

Question 5.
10.3 9
-4.2 8
————
Estimate: _____

Answer:
Estimate: About 6

Explanation:
10.3 9 is closer to 10
4.2 8 is closer to 4
10 – 4 = 6

On Your Own

Use rounding to estimate.

Question 6.
0.9 3
+0.1 8
————
Estimate: _____

Answer:
Estimate: About 1

Explanation:
0.93; 9 >5; 1
0.18; 1 < 5; 0
1 + 0 = 1

Question 7.
7.4 1
-3.8 8
————
Estimate: _____

Answer:
Estimate: About 3

Explanation:
7.41; 4 < 5; 7
3.88; 8 > 5; 4
7 – 4 = 3

Question 8.
14.6 8
-3.9 3
————
Estimate: _____

Answer:
Estimate: About 11

Explanation:
14.68; 6 > 5; 15
3.93; 9 > 5; 4
15 – 4 = 11

Use benchmarks to estimate.

Question 9.
12.4 1
-6.4 7
————
Estimate: _____

Answer:
Estimate: About 6

Explanation:
12.41 is closer to 12
6.47 is closer to 6
12 – 6 = 6

Question 10.
8.1 2
-5.5 2
————
Estimate: _____

Answer:
Estimate: About 2

Explanation:
8.12 is closer to 8
5.52 is closer to 6
8 – 6 = 2

Question 11.
9.7 5
-3.4 7
————
Estimate: _____

Answer:
Estimate: About 7

Explanation:
9.75 is closer to 10
3.47 is closer to 3
10 – 3 = 7

Practice: Copy and Solve Use rounding or benchmarks to estimate.

Question 12.
12.83 + 16.24
Estimate: _____

Answer:
Estimate: About 29

Explanation:
12.83; 8 > 5; 13
16.24; 2 <5; 16
13 + 16 = 29

Question 13.
$26.92 − $11.13
Estimate: $ _____

Answer:
Estimate: About $16

Explanation:
26.92; 9 > 5; 27
11.13; 1 < 5; 11
27 – 11 = 16

Question 14.
9.41 + 3.82
Estimate: _____

Answer:
Estimate: About 13

Explanation:
9.41; 4 < 5; 9
3.82; 8 > 5; 4
9 + 4 = 13

Use Reasoning Estimate to compare. Write < or >.

Question 15.
2.74 + 4.22 _____ 3.13 + 1.87

Answer:
2.74 + 4.22 > 3.13 + 1.87

Explanation:
2.74; 7 > 5; 3
4.22; 2 < 5 ; 4
3 + 4 = 7
3.13; 1 < 5; 3
1.87; 8 > 5; 2
3 + 2 = 5
So, 7 > 5
2.74 + 4.22 > 3.13 + 1.87

Question 16.
6.25 – 2.39 _____ 9.79 – 3.84

Answer:
6.25 – 2.39 < 9.79 – 3.84

Explanation:
6.25; 2 < 5; 6
2.39; 3 < 5; 2
6 – 2 = 4
9.79; 7 > 5; 10
3.84; 8 >5; 4
10 – 4 = 6
4 < 6
6.25 – 2.39 < 9.79 – 3.84

Problem Solving Applications – Page No. 138

Use the table to solve 17–18. Show your work.
Go Math Grade 5 Answer Key Chapter 3 Add and Subtract Decimals img 22

Question 17.
For the week of April 4, 1964, the Beatles had the top four songs. About how long would it take to listen to these four songs?
about _____ minutes

Answer:
about 10 minutes

Explanation:
Can’t Buy Me Love = 2.30 min
She Loves You = 2.50 min
I Want to Hold You Hand = 2.75 min
Please Please Me = 2.00 min
2.30; 3 < 5; 2
2.50; 5 = 5; 3
2.75; 7 > 5; 3
2.00; 2 < 5; 2
2 + 3 + 3 + 2 = 10 min

Question 18.
What’s the Error? Isabelle says she can listen to the first three songs in the table in 6 minutes.
Type below:
_________

Answer:
Can’t Buy Me Love = 2.30 min
She Loves You = 2.50 min
I Want to Hold You Hand = 2.75 min
2.30; 3 < 5; 2
2.50; 5 = 5; 3
2.75; 7 > 5; 3
2 + 3 + 3 = 8 minutes
About 8 minutes

Question 19.
Tracy ran a lap around the school track in 74.2 seconds. Malcolm ran a lap in 65.92 seconds. Estimate the difference in the times in which the students completed the lap.
about _____ seconds

Answer:
about 8 seconds

Explanation:
Tracy ran a lap around the school track in 74.2 seconds.
74.2; 2 < 5; 74
Malcolm ran a lap in 65.92 seconds.
65.92; 9 > 5; 66
74 – 66 = 8
about 8 seconds

Nutrition

Your body needs protein to build and repair cells. You should get a new supply of protein each day. The average 10-year-old needs 35 grams of protein daily. You can find protein in foods like meat, vegetables, and dairy products.
Go Math Grade 5 Answer Key Chapter 3 Add and Subtract Decimals img 23
Use estimation to solve.

Question 20.
Gina had a scrambled egg and a cup of low-fat milk for breakfast. She had an oat bran muffin for a morning snack. About how many more grams of protein did Gina have for breakfast than for a snack?
about _____ grams

Answer:
about 17 grams

Explanation:
Gina had a scrambled egg and a cup of low-fat milk for breakfast. She had an oat bran muffin for a morning snack.
1 scrambled egg = 6.75 grams
1 cup shredded whear cereal = 5.56 grams
1 oat bran muffin = 3.99 grams
1 cup low-fat milk = 8.22 grams
6.75; 7 > 5; 7
8.22; 2 < 5; 8
3.99; 9 > 5; 4
7 + 2 + 9 = 18
The average 10-year-old needs 35 grams of protein daily.
So, 35 – 18 = 17
Gina have 17 more grams of protein for breakfast than for a snack.

Question 21.
Pablo had a cup of shredded wheat cereal, a cup of low-fat milk, and one other item for breakfast. He had about 21 grams of protein. What was the third item Pablo had for breakfast?
_________

Answer:
6 grams

Explanation:
1 cup shredded whear cereal = 5.56 grams
1 cup low-fat milk = 8.22 grams
5.56; 5 = 5; 6
8.22; 2 < 5; 9
6 + 9 = 15
15 + S = 21 grams
S = 21 – 15 = 6 grams
The third item Pablo had 6 grams for breakfast

Share and Show – Page No. 140

Estimate. Then find the sum.

Question 1.
2.5
+4.6
Estimate: _____
Sum: _____

Answer:
Estimate: 8
Sum: 7.1

Explanation:
2.5 nearest whole number is 3
4.6 nearest whole number is 5
Estimate: 3 + 5 = 8
Sum: 2.5 + 4.6 = 7.1

Question 2.
8.7 5
+6.4 3
Estimate: _____
Sum: _____

Answer:
Estimate: 15
Sum: 15.18

Explanation:
8.75 nearest whole number is 9
6.43 nearest whole number is 6
Estimate: 9 + 6 = 15
Sum: 8.75 + 6.43 = 15.18

Question 3.
2.0 3
+7.8 9
Estimate: _____
Sum: _____

Answer:
Estimate: 10
Sum: 9.92

Explanation:
2.03 nearest whole number is 2
7.89 nearest whole number is 8
Estimate: 2 + 8 = 10
Sum: 2.03 + 7.89 = 9.92

Question 4.
6.34 + 3.8 =
Estimate: _____
Sum: _____

Answer:
Estimate: 10
Sum: 10.14

Explanation:
6.34 nearest whole number is 6
3.8 nearest whole number is 4
Estimate: 6 + 4 = 10
Sum: 6.34 + 3.8 = 10.14

Question 5.
5.63 + 2.6 =
Estimate: _____
Sum: _____

Answer:
Estimate: 9
Sum: 8.23

Explanation:
5.63 nearest whole number is 6
2.6 nearest whole number is 3
Estimate: 6 + 3 = 9
Sum: 5.63 + 2.6 = 8.23

On Your Own – Page No. 141

Connect Symbols and Words Find the sum.

Question 6.
seven and twenty-five hundredths added to nine and four tenths
Type below:
________

Answer:
7.25 + 9.4 = 16.65

Explanation:
seven and twenty-five hundredths = 7.25
nine and four tenths = 9.4
7.25 + 9.4 = 16.65

Question 7.
twelve and eight hundredths added to four and thirty-five hundredths
Type below:
________

Answer:
12.08 + 4.35 = 16.43

Explanation:
twelve and eight hundredths = 12.08
four and thirty-five hundredths = 4.35
12.08 + 4.35 = 16.43

Question 8.
nineteen and seven tenths added to four and ninety-two hundredths
Type below:
________

Answer:
19.7 + 4.92 = 24.62

Explanation:
nineteen and seven tenths  = 19.7
four and ninety-two hundredths = 4.92
19.7 + 4.92 = 24.62

Question 9.
one and eighty-two hundredths added to fifteen and eight tenths
Type below:
________

Answer:
1.82 + 15.8 = 17.62

Explanation:
one and eighty-two hundredths = 1.82
fifteen and eight tenths = 15.8
1.82 + 15.8 = 17.62

Practice: Copy and Solve Find the sum.

Question 10.
7.99 + 8.34
_____

Answer:
16.33

Explanation:
7.99 + 8.34
Add hundredths; 9 + 4 = 13; regroup
Add tenths; 9 + 3 + 1 = 13; regroup
Add tens; 7 + 8  + 1 = 16
16.33

Question 11.
15.76 + 8.2
_____

Answer:
23.96

Explanation:
15.76 + 8.2
Add hundredths; 6 + 0 = 6;
Add tenths; 7 + 2 = 9;
Add tens; 5 + 8  = 13; regroup
Add hundreds; 1 + 1 = 2
23.96

Question 12.
9.6 + 5.49
_____

Answer:
15.09

Explanation:
9.6 + 5.49
Add hundredths; 0 + 9 = 9;
Add tenths; 6 + 4 = 10; regroup;
Add tens; 9 + 5 +  1 = 15; regroup
15.09

Question 13.
33.5 + 16.4
_____

Answer:
49.9

Explanation:
33.5 + 16.4
Add tenths; 5 + 4 = 9;
Add tens; 3 + 6 = 9;
Add hundreds; 3 + 1 = 4
49.9

Question 14.
9.84 + 21.52
_____

Answer:
31.36

Explanation:
9.84 + 21.52
Add hundredths; 4 + 2 = 6;
Add tenths; 8 + 5 = 13; regroup
Add tens; 9 + 1 + 1  = 11; regroup
Add hundreds; 0 + 2 + 1 = 3
31.36

Question 15.
3.89 + 4.6
_____

Answer:
8.49

Explanation:
3.89 + 4.6
Add hundredths; 9 + 0 = 9;
Add tenths; 8 + 6 = 14;
Add tens; 3 + 4 + 1 = 8;
8.49

Question 16.
42.19 + 8.8
_____

Answer:
50.99

Explanation:
42.19 + 8.8
Add hundredths; 0 + 9 = 9;
Add tenths; 1 + 8 = 9;
Add tens; 2 + 8  = 10; regroup
Add hundreds; 4 + 1 = 5
50.99

Question 17.
16.74 + 5.34
_____

Answer:
22.08

Explanation:
16.74 + 5.34
Add hundredths; 4 + 4 = 8;
Add tenths; 7 + 3 = 10; regroup
Add tens; 6 + 5 + 1 = 12; regroup
Add hundreds; 1 + 1 = 2
22.08

Question 18.
27.58 + 83.9
_____

Answer:
111.48

Explanation:
27.58 + 83.9
Add hundredths; 8 + 0 = 8;
Add tenths; 5 + 9 = 14; regroup
Add tens; 7 + 3 + 1  = 11; regroup
Add hundreds; 2 + 8 + 1 = 11
111.48

Question 19.
Tania measured the growth of her plant each week. The first week, the plant’s height measured 2.65 decimeters. During the second week, Tania’s plant grew 0.7 decimeter. How tall was Tania’s plant at the end of the second week?
Describe the steps you took to solve the problem.
_____ decimeters

Answer:
3.35 decimeters

Explanation:
Tania measured the growth of her plant each week. The first week, the plant’s height measured 2.65 decimeters. During the second week, Tania’s plant grew 0.7 decimeter.
2.65 + 0.7 = 3.35

Question 20.
Maggie had $35.13. Then her mom gave her $7.50 for watching her younger brother. She was paid $10.35 for her old roller skates. How much money does Maggie have now?
$ _____

Answer:
$52.98

Explanation:
Maggie had $35.13. Then her mom gave her $7.50 for watching her younger brother. She was paid $10.35 for her old roller skates.
35.13 + 7.50 + 10.35 = 52.98

Unlock the Problem – Page No. 142

Go Math Grade 5 Answer Key Chapter 3 Add and Subtract Decimals img 24

Question 21.
A city receives an average rainfall of 16.99 centimeters in August. One year, during the month of August, it rained 8.33 centimeters by August 15th. Then it rained another 4.65 centimeters through the end of the month. What was the total rainfall in centimeters for the month?
a. What do you need to find?
Type below:
________

Answer:
We need to find out what was the total rainfall in centimeters for the month, so we have to find the sum 8.33+ 4.65.

Explanation:
A city receives an average rainfall of 16.99 centimeters in August. One year, during the month of August, it rained 8.33 centimeters by August 15th. Then it rained another 4.65 centimeters through the end of the month. We need to find out what was the total rainfall in centimeters for the month, so we have to find the sum 8.33+ 4.65.

Question 21.
b. What information are you given?
Type below:
________

Answer:
We know that one year during the month Aug., it rained 8.33 centimeters by Aug. 15th. Then it rained another 4.65 centimeters through the end of the month.

Question 21.
c. How will you use addition to find the total number of centimeters of rain that fell?
Type below:
________

Answer:
We have to add the hundredths first, then the tenths and in the end the ones.

Question 21.
d. Show how you solved the problem.
Type below:
________

Answer:
sum 8.33+ 4.65.
Add the hundredths first. 3 hundredths + 5 hundredths = 8 hundredths.
Add the tenths. 3 tenths + 6 tenths = 9 tenths.
Add the ones. 8 + 4 = 12 tens
Therefore, the sum is 8.33+ 4.65 = 12.98.

Question 21.
e. Complete the sentence. It rained _________ centimeters for the month.
______ centimeters

Answer:
12.98 centimeters

Explanation:
It rained 12.98 centimeters for the month.

Question 22.
Horatio caught a fish that weighed 1.25 pounds. Later he caught another fish that weighed 1.92 pounds. What was the combined weight of both fish? Use the digits on the tiles to solve the problem. Digits may be used more than once or not at all.
Go Math Grade 5 Answer Key Chapter 3 Add and Subtract Decimals img 25
______ pounds

Answer:
3.17 pounds

Explanation:
Horatio caught a fish that weighed 1.25 pounds. Later he caught another fish that weighed 1.92 pounds.
1.25 + 1.92 = 3.17 pounds
grade 5 chapter 3 Add and Subtract Decimals 143 image 1

Share and Show – Page No. 144

Estimate. Then find the difference.

Question 1.
5.8 3
−2.1 8
———-
Estimate: ______
Difference: ______

Answer:
Estimate: 4
Difference: 3.65

Explanation:
5.83 is closer to 6
2.18 is closer to 2
6 – 2 = 4
5.83 – 2.18 = 3.65

Question 2.
4.4 5
−1.8 6
———–
Estimate: ______
Difference: ______

Answer:
Estimate: 2
Difference: 2.59

Explanation:
4.45 is closer to 4
1.86 is closer to 2
4 – 2 = 2
4.45 – 1.86 = 2.59

Question 3.
4.0 3
−2.2 5
———-
Estimate: ______
Difference: ______

Answer:
Estimate: 2
Difference: 1.78

Explanation:
4.03 is closer to 4
2.25 is closer to 2
4 – 2 = 2
4.03 – 2.25 = 1.78

Find the difference. Check your answer.

Question 4.
0.7 0
−0.4 3
———-
______

Answer:
0.27

Explanation:
0.70 − 0.43
Subtract hundredths: 0 – 3;
There are not enough hundredths. So, regroup
10 – 3 = 7
Subtract tenths: 6 – 4 = 2
Subtract ones: 0 – 0 = 0
0.27
Check: 0.70 − 0.43 = 0.27
0.27 = 0.27

Question 5.
13.2
−8.0 4
———-
______

Answer:
5.16

Explanation:
13.2 − 8.04
Subtract hundredths: 0 – 4;
There are not enough hundredths. So, regroup
10 – 4 = 6
Subtract tenths: 1 – 0 = 1
Subtract ones: 3 – 8;
There are not enough tens. So, regroup
13 – 8 = 5
Subtract hundreds: 0 – 0 = 0;
5.16
Check: 13.2 − 8.04 = 5.16
5.16 = 5.16

Question 6.
15.8
−9.6 7
———-
______

Answer:
6.13

Explanation:
15.8 − 9.67
Subtract hundredths: 0 – 7;
There are not enough hundredths. So, regroup
10 – 7 = 3
Subtract tenths: 7 – 6 = 1
Subtract ones: 5 – 9;
There are not enough ones. So, regroup
15 – 9 = 6
Subtract hundreds: 0 – 0 = 0;
6.13
Check: 15.8 − 9.67 = 6.13
6.13 = 6.13

On Your Own – Page No. 145

Connect Symbols and Words Find the difference.

Question 7.
three and seventy-two hundredths subtracted from five and eighty-one hundredths
______

Answer:
three and seventy-two hundredths = 3.72
five and eighty-one hundredths = 5.81
5.81 – 3.72 = 2.09

Question 8.
one and six-hundredths subtracted from eight and thirty-two hundredths
______

Answer:
one and six-hundredths = 1.06
eight and thirty-two hundredths = 8.23
8.23 – 1.06 = 7.17

Use Reasoning Algebra Write the unknown number for n.

Question 9.
5.28 − 3.4 = n
n = ______

Answer:
n = 1.88

Explanation:
5.28 − 3.4 = 1.88

Question 10.
n − 6.47 = 4.32
n = ______

Answer:
n = 10.79

Explanation:
n − 6.47 = 4.32
n = 4.32 + 6.47
n = 10.79

Question 11.
11.57 − n = 7.51
n = ______

Answer:
n = 4.06

Explanation:
11.57 − n = 7.51
11.57 = 7.51 + n
n = 11.57 – 7.51
n = 4.06

Practice: Copy and Solve Find the difference.

Question 12.
8.42 − 5.14 = ______

Answer:
3.28

Explanation:
8.42 − 5.14
Subtract hundredths: 2 – 4;
There are not enough hundredths. So, regroup
12 – 4 = 8
Subtract tenths: 3 – 1 = 2
Subtract ones: 8 – 5 = 3
3.28

Question 13.
16.46 − 13.87 = ______

Answer:
2.59

Explanation:
16.46 − 13.87
Subtract hundredths: 6 – 7;
There are not enough hundredths. So, regroup
16 – 7 = 9
Subtract tenths: 3 – 8
There are not enough tenths. So, regroup
13 – 8 = 5
Subtract ones: 5 – 3 = 2;
Subtract hundreds: 1 – 1 = 0;
2.59

Question 14.
34.27 − 17.51 = ______

Answer:
16.76

Explanation:
34.27 − 17.51
Subtract hundredths: 7 – 1 = 6;
Subtract tenths: 2 – 5
There are not enough tenths. So, regroup
12 – 5 = 7;
Subtract ones: 3 – 7
There are not enough ones. So, regroup
13 – 7 = 6
Subtract hundreds: 2 – 1 = 1;
16.76

Question 15.
15.83 − 11.45 = ______

Answer:
4.38

Explanation:
15.83 − 11.45
Subtract hundredths: 3 – 5;
There are not enough hundredths. So, regroup
13 – 5 = 8
Subtract tenths: 7 – 4 = 3
Subtract ones: 5 – 1 = 4;
Subtract hundreds: 1 – 1 = 0;
4.38

Question 16.
12.74 − 10.54 = ______

Answer:
2.2

Explanation:
12.74 − 10.54
Subtract hundredths: 4 – 4 = 0;
Subtract tenths: 7 – 5 = 2
Subtract ones: 2 – 0 = 2;
Subtract hundreds: 1 – 1 = 0;
2.20

Question 17.
48.21 − 13.65 = ______

Answer:
34.56

Explanation:
48.21 − 13.65
Subtract hundredths: 1 – 5;
There are not enough hundredths. So, regroup
11 – 5 = 6
Subtract tenths: 1 – 6
There are not enough tenths. So, regroup
11 – 6 = 5
Subtract ones: 7 – 3 = 4;
Subtract hundreds: 4 – 1 = 3;
34.56

Question 18.
Beth finished a race in 3.35 minutes. Ana finished the race in 0.8 minute less than Beth. Fran finished the race in 1.02 minutes less than Ana. What was Fran’s time to finish the race in minutes?
______ minutes

Answer:
1.53 minutes

Explanation:
Beth finished a race in 3.35 minutes. Ana finished the race in 0.8 minute less than Beth.
3.35 – 0.8 = 2.55
Fran finished the race in 1.02 minutes less than Ana.
2.55 – 1.02 = 1.53

Question 19.
Fatima planted sunflower seeds in a flower patch. The tallest sunflower grew 2.65 meters tall. The height of the shortest sunflower was 0.34 meter less than the tallest sunflower. What was the height, in meters, of the shortest sunflower?
______ meters

Answer:
2.31 meters

Explanation:
Fatima planted sunflower seeds in a flower patch. The tallest sunflower grew 2.65 meters tall. The height of the shortest sunflower was 0.34 meter less than the tallest sunflower.
2.65 – 0.34 = 2.31

Unlock the Problem – Page No. 146

Question 20.
In peanut butter, how many more grams of protein are there than grams of carbohydrates? Use the label below.
Go Math Grade 5 Answer Key Chapter 3 Add and Subtract Decimals img 26
a. What do you need to know?
Type below:
_________

Answer:
We need to find how many more grams of protein are there than grams of carbohydrates?

Question 20.
b. How will you use subtraction to find how many more grams of protein there are than grams of carbohydrates?
Type below:
_________

Answer:
Grams of protein = 8.1 g
grams of carbohydrates = 6.2g
8.1 – 6.2 = 1.9 grams

Question 20.
c. Show how you solved the problem.
Type below:
_________

Answer:
8.1 – 6.2
Subtract tenths: 1 – 2
There are not enough tenths. So, regroup
11 – 2 = 9
Subtract ones:
7 – 6 = 1
1.9 grams

Question 20.
d. Complete each sentence.
The peanut butter has ______ grams of protein.
The peanut butter has ______ grams of carbohydrates.
There are ______ more grams of protein than grams of carbohydrates in the peanut butter.
Type below:
_________

Answer:
The peanut butter has 8.1 grams of protein.
The peanut butter has 6.2 grams of carbohydrates.
There are 1.9 more grams of protein than grams of carbohydrates in the peanut butter.

Question 21.
Kyle is building a block tower. Right now the tower stands 0.89 meter tall. How much higher does the tower need to be to reach a height of 1.74 meters?
______ meters

Answer:
0.85 meters

Explanation:
Kyle is building a block tower. Right now the tower stands 0.89 meter tall.
To reach a height of 1.74, 1.74 – 0.89 = 0.85

Question 22.
Dialyn scored 2.5 points higher than Gina at a gymnastics event. Select the values that could represent each student’s gymnastics score. Mark all that apply.
Options:
a. Dialyn: 18.4 points, Gina: 16.9 points
b. Dialyn: 15.4 points, Gina: 13.35 points
c. Dialyn: 16.2 points, Gina: 13.7 points
d. Dialyn: 19.25 points, Gina: 16.75 points

Answer:
c. Dialyn: 16.2 points, Gina: 13.7 points
d. Dialyn: 19.25 points, Gina: 16.75 points

Explanation:
Dialyn scored 2.5 points higher than Gina at a gymnastics event.
a. 18.4 – 16.9 = 1.5
b. 15.4 – 13.35 = 2.05
c. 16.2 – 13.7 = 2.5
d. 19.25 – 16.75 = 2.5

Share and Show – Page No. 149

Write a rule for the sequence.

Question 1.
0.5, 1.8, 3.1, 4.4, …
Think: Is the sequence increasing or decreasing?
Rule: _________

Answer:
Add 1.3 to the previous term in the sequence to get the next one.

Explanation:
Compare 0.5, 1.8; 0.5 < 1.8
The sequence is increasing as the second term is greater than the first term.
The operation will use addition.
0.5 + x = 1.8
x = 1.8 – 0.5 = 1.3
1.8 + 1.3 = 3.1
3.1 + 1.3 = 4.4
Add 1.3 to the previous term in the sequence to get the next one.

Question 2.
23.2, 22.1, 21, 19.9, …
Rule: _________

Answer:
Subtract 1.1 to the previous term in the sequence to get the next one.

Explanation:
Compare 23.2 and 22.1; 23.2 > 22.1
The sequence is decreasing as the second term is lesser than the first term.
The operation will use subtraction.
23.2 – 22.1 = 1.1
22.1 – 21 = 1.1
21 – 19.9 = 1.1
Subtract 1.1 to the previous term in the sequence to get the next one.

Write a rule for the sequence. Then find the unknown term.

Question 3.
0.3, 1.5, ____ , 3.9, 5.1
Missing value: ______
Rule: ______

Answer:
Missing value: 2.7
Rule: Add 1.2 to the previous term in the sequence to get the next one.

Explanation:
Compare 0.3 and 1.5; 0.3 < 1.5
The sequence is increasing as the second term is greater than the first term.
The operation will use addition.
1.5 – 0.3 = 1.2
0.3 + 1.2 = 1.5
1.5 + 1.2 = 2.7
2.7 + 1.2 = 3.9
3.9 + 1.2 = 5.1
Add 1.2 to the previous term in the sequence to get the next one.

Question 4.
19.5, 18.8, 18.1, 17.4, ______
Missing value: ______
Rule: ______

Answer:
Missing value: 16.7
Rule: Subtract 0.7 to the previous term in the sequence to get the next one.

Explanation:
Compare 19.5 and 18.8; 19.5 > 18.8
The sequence is decreasing as the second term is lesser than the first term.
The operation will use subtraction.
19.5 – 18.8 = 0.7
18.8 – 18.1 = 0.7
18.1 – 17.4 = 0.7
17.4 – 0.7 = 16.7
Subtract 0.7 to the previous term in the sequence to get the next one.

On Your Own

Write the first four terms of the sequence.

Question 5.
Rule: start at 10.64, subtract 1.45
______ ; ______ ; ______ ; ______

Answer:
9.19; 7.74; 6.29; 4.84

Explanation:
10.64 – 1.45 = 9.19
9.19 – 1.45 = 7.74
7.74 – 1.45 = 6.29
6.29 – 1.45 = 4.84
9.19; 7.74; 6.29; 4.84

Question 6.
Rule: start at 0.87, add 2.15
______ ; ______ ; ______ ; ______

Answer:
3.02; 5.17; 7.32; 9.47

Explanation:
0.87 + 2.15 = 3.02
3.02 + 2.15 = 5.17
5.17 + 2.15 = 7.32
7.32 + 2.15 = 9.47
3.02; 5.17; 7.32; 9.47

Question 7.
Rule: start at 19.3, add 1.8
______ ; ______ ; ______ ; ______

Answer:
21.1; 22.9; 24.7; 26.5

Explanation:
19.3 + 1.8 = 21.1
21.1 + 1.8 = 22.9
22.9 + 1.8 = 24.7
24.7 + 1.8 = 26.5
21.1; 22.9; 24.7; 26.5

Question 8.
Rule: start at 29.7, subtract 0.4
______ ; ______ ; ______ ; ______

Answer:
29.3; 28.9; 28.5; 28.1

Explanation:
29.7 – 0.4 = 29.3
29.3 – 0.4 = 28.9
28.9 – 0.4 = 28.5
28.5 – 0.4 = 28.1
29.3; 28.9; 28.5; 28.1

Question 9.
Marta put $4.87 in her coin bank. Each day she added 1 quarter, 1 nickel, and 3 pennies. How much money was in her coin bank after 6 days? Describe the pattern you used to solve.
$ ______

Answer:
$10.52
Add 1.13 to the previous term in the sequence to get the next one.

Explanation:
Marta put $4.87 in her coin bank. Each day she added 1 quarter, 1 nickel, and 3 pennies.
She added 1.13 each day.
4.87 + 1.13 = 6.00
6.00 + 1.13 = 7.13
7.13 + 1.13 = 8.26
8.26 + 1.13 = 9.39
9.39 + 1.13 = 10.52
Add 1.13 to the previous term in the sequence to get the next one.

Question 10.
Identify Relationships Look at the list below. Do the numbers show a pattern? Explain how you know.
11.23, 10.75, 10.3, 9.82, 9.37, 8.89
Type below:
_________

Answer:
Compare 11.23 and 10.75; 11.23 > 10.75
The sequence is decreasing as the second term is greater than the first term.
The operation will use subtraction.
11.23 – 10.75 = 0.48
10.75 – 10.3 = 0.45
10.3 – 9.82 = 0.48
9.82 – 9.37 = 0.45
9.37 – 8.89 = 0.48
First two terms difference is 0.48
Second and third term difference is 0.45
third and fourth term difference is 0.48
fourth and fifth term difference is 0.45
fifth and sixth term difference is 0.48

Problem Solving Applications – Page No. 150

Pose a Problem

Question 11.
Bren has a deck of cards. As shown below, each card is labeled with a rule describing a pattern in a sequence. Select a card and decide on a starting number. Use the rule to write the first five terms in your sequence.
Go Math Grade 5 Answer Key Chapter 3 Add and Subtract Decimals img 27
Sequence: _____ , _____ , _____ , _____ , _____
Write a problem that relates to your sequence and requires the sequence be extended to solve.
Pose a Problem         Solve your problem.
Type below:
_________

Answer:
1.6 + 0.33 = 1.93
1.93 + 0.33 = 2.26
2.26 + 0.33 = 2.59
2.59 + 0.33 = 2.92
2.92 + 0.33 = 3.25
Start at 1.6 and write the first five terms of the sequence?
Add 0.3 to the previous term in the sequence to get the next one.

Question 12.
Colleen and Tom are playing a number pattern game. Tom wrote the following sequence.
33.5, 34.6, 35.7, ________, 37.9
What is the unknown term in the sequence?
_____

Answer:
36.8

Explanation:
33.5 < 34.6
34.6 – 33.5 = 1.1
33.5 + 1.1 = 34.6
34.6 + 1.1 = 35.7
35.7 + 1.1 = 36.8
36.8 + 1.1 = 37.9

Share and Show – Page No. 153

Question 1.
Sara wants to buy a bottle of apple juice from a vending machine. She needs exactly $2.30. She has the following bills and coins:
Go Math Grade 5 Answer Key Chapter 3 Add and Subtract Decimals img 28
Make and complete a table to find all the ways Sara could pay for the juice. First, draw a table with a column for each type of bill or coin. Next, fill in your table with each row showing a different way Sara can make exactly $2.30.
Type below:
_________

Answer:
Sara wants to buy a bottle of apple juice from a vending machine. She needs exactly $2.30.
grade 5 chapter 3 Add and Subtract Decimals 153 image 1

Question 2.
What if Sara decides to buy a bottle of water that costs $1.85? What are all the different ways she can make exactly $1.85 with the bills and coins she has? Which coin must Sara use?
Type below:
_________

Answer:
If Sara decides to buy a bottle of water that costs $1.85, then
1 bill, 3 quarters, 1 dime; 1 bill, 3 quarters, 2 nickels; quarter

Question 3.
At the end of August, Mr. Diaz had a balance of $441.62. Since then, he has written two checks for $157.34 and $19.74 and made a deposit of $575.00. Mr. Diaz says his balance is $739.54. Find Mr. Diaz’s correct balance.
$ _____

Answer:
At the end of August, Mr. Diaz had a balance of $441.62.
Since then, he has written two checks for $157.34 and $19.74 and made a deposit of $575.00.
Subtract the checks from the initial amount, and add the deposit.
441.85 – (157.34 + 19.74) + 575 = 839.77
So, $839.77

On Your Own – Page No. 154

Use the following information to solve 4–6.

At Open Skate Night, admission is $3.75 with a membership card and $5.00 without a membership card. Skate rentals are $3.00.

Question 4.
Aidan paid the admission for himself and two friends at Open Skate Night. Aidan had a membership card, but his friends did not. Aidan paid with a $20 bill. How much change should Aidan receive?
$ _____

Answer:
$6.25

Explanation:
Aidan had a membership card, but his friends did not.
$3.75 + $5.00 + $5.00 = $13.75
Aidan paid with a $20 bill.
$20 – $13.75 = $6.25

Question 5.
The Moores paid $6 more for skate rentals than the Cotters did. Together, the two families paid $30 for skate rentals. How many pairs of skates did the Moores rent?
_____ pairs of skates

Answer:
6 pairs of skates

Question 6.
Analyze Jennie and 5 of her friends are going to Open Skate Night. Jennie does not have a membership card. Only some of her friends have membership cards. What is the total amount that Jennie and her friends might pay for admission?
Type below:
_________

Answer:
They will pay $27.50 if only 2 of her friends have membership cards.

Question 7.
Marisol bought 5 movie tickets for a show. Each ticket cost $6.25. Complete the table to show the price of 2, 3, 4, and 5 tickets.
Go Math Grade 5 Answer Key Chapter 3 Add and Subtract Decimals img 29
Type below:
_________

Answer:
grade 5 chapter 3 Add and Subtract Decimals 153 image 2

Share and Show – Page No. 156

Find the sum or difference.

Question 1.
4.19 + 0.58
_____

Answer:
4.77

Explanation:
4.19 + 0.58 = 1.38
Add hundredths 9 + 8 = 17; Regroup;
Add tenths 1 + 5 + 1 = 7;
Add ones 4 + 0 = 4
4.19 + 0.58 = 4.77

Question 2.
9.99 − 4.1
_____

Answer:
5.89

Explanation:
9.99 − 4.1
Subtract hundredths: 9 – 0 = 9;
Subtract tenths: 9 – 1 = 8
Subtract ones: 9 – 4 = 5
So, 9.99 − 4.1 = 5.89

Question 3.
5.7 + 2.25 + 1.3
_____

Answer:
9.25

Explanation:
5.7 + 2.25 + 1.3
Add hundredths 0 + 5 + 0 = 5;
Add tenths 7 + 2 + 3 = 12; Regroup
Add ones 5 + 2 + 1 + 1 = 9
5.7 + 2.25 + 1.3 = 9.25

Question 4.
28.6 − 9.84
_____

Answer:
18.76

Explanation:
28.6 − 9.84
Subtract hundredths: 0 – 4;
There are not enough hundredths. So, regroup
10 – 4 = 6.
Subtract tenths: 5 – 8;
There are not enough tenths. So, regroup
15 – 8 = 7
Subtract ones: 7 – 9;
There are not enough ones. So, regroup
17 – 9 = 8
Subtract hundreds: 1 – 0 = 1;
So, 28.6 − 9.84 = 18.76

Question 5.
$15.79 + $32.81
$ _____

Answer:
$48.6

Explanation:
$15.79 + $32.81
Add hundredths 9 + 1 = 10; Regroup
Add tenths 7 + 8 + 1 = 16; Regroup
Add ones 5 + 2 + 1  = 8
Add hundreds 1 + 3 = 4
$15.79 + $32.81 = $48.60

Question 6.
38.44 − 25.86
_____

Answer:
12.58

Explanation:
38.44 − 25.86
Subtract hundredths: 4 – 6;
There are not enough hundredths. So, regroup
14 – 6 = 8
Subtract tenths: 3 – 8;
There are not enough tenths. So, regroup
13 – 8 = 5
Subtract ones: 7 – 5 = 2;
Subtract hundreds: 3 – 2 = 1;
So, 38.44 − 25.86 = 12.58

On Your Own – Page No. 157

Find the sum or difference.

Question 7.
$ 18.39
+$7.56
————
$ _____

Answer:
$25.95

Explanation:
$ 18.39 + $7.56
Add hundredths 9 + 6 = 15; Regroup
Add tenths 5 + 3 + 1 = 9;
Add ones 8 + 7  = 15; Regroup
Add hundreds 1 + 0 + 1 = 2
$ 18.39 + $7.56 = $25.95

Question 8.
8.22 − 4.39
_____

Answer:

Explanation:
8.22 − 4.39
Subtract hundredths: 2 – 9;
There are not enough hundredths. So, regroup
12 – 9 = 3
Subtract tenths: 1 – 3;
There are not enough tenths. So, regroup
11 – 3 = 8
Subtract ones: 7 – 4 = 3;
So, 8.22 − 4.39 = 3.83

Question 9.
93.6 − 79.84
_____

Answer:
13.76

Explanation:
93.6 − 79.84
Subtract hundredths: 0 – 4;
There are not enough hundredths. So, regroup
10 – 4 = 6
Subtract tenths: 5 – 8;
There are not enough tenths. So, regroup
15 – 8 = 7
Subtract ones: 2 – 9;
There are not enough ones. So, regroup
12 – 9 = 3
Subtract hundreds: 8 – 7 = 1;
So, 93.6 − 79.84 = 13.76

Question 10.
1.82
2.28
+2.18
————
_____

Answer:
6.28

Explanation:
1.82 + 2.28 + 2.18
Add hundredths 2 + 8 + 8 = 18; Regroup
Add tenths 8 + 2 + 1 + 1 = 12;  Regroup
Add ones 1 + 2 + 2 + 1  = 6;
1.82 + 2.28 + 2.18 = 6.28

Practice: Copy and Solve Find the sum or difference.

Question 11.
6.3 + 2.98 + 7.7
_____

Answer:
16.98

Explanation:
6.3 + 2.98 + 7.7
Add hundredths 0 + 8 + 0 = 8;
Add tenths 3 + 9 + 7 = 19;  Regroup
Add ones 6 + 2 + 7 + 1  = 16;
6.3 + 2.98 + 7.7 = 16.98

Question 12.
27.96 − 16.2
_____

Answer:
11.76

Explanation:
27.96 − 16.2
Subtract hundredths: 6 – 0 = 6;
Subtract tenths: 9 – 2 = 7;
Subtract ones: 7 – 6 = 1;
Subtract hundreds: 2 – 1 = 1;
So, 27.96 − 16.2 = 11.76

Question 13.
12.63 + 15.04
_____

Answer:
27.67

Explanation:
12.63 + 15.04
Add hundredths 3 + 4 = 7;
Add tenths 6 + 0 = 6;
Add ones 2 + 5 = 7;
Add hundreds 1 + 1 = 2
12.63 + 15.04 = 27.67

Question 14.
9.24 − 2.68
_____

Answer:
6.56

Explanation:
9.24 − 2.68
Subtract hundredths: 4 – 8;
There are not enough hundredths. So, regroup
14 – 8 = 6
Subtract tenths: 1 – 6;
There are not enough tenths. So, regroup
11 – 6 = 5
Subtract ones: 8 – 2 = 6;
So, 9.24 − 2.68 = 6.56

Question 15.
$18 − $3.55
$ _____

Answer:
$14.45

Explanation:
$18 − $3.55
Subtract hundredths: 0 – 5;
There are not enough hundredths. So, regroup
10 – 5 = 5
Subtract tenths;
There are not enough tenths. So, regroup
9 – 5 = 4
Subtract ones: 7 – 3 = 4;
Subtract hundreds: 1 – 0 = 0
So, $18 − $3.55 = $14.45

Question 16.
9.73 − 2.52
_____

Answer:
7.21

Explanation:
9.73 − 2.52
Subtract hundredths: 3 – 2 = 1;
Subtract tenths; 7 – 5 = 2
Subtract ones: 9 – 2 = 7;
So, 9.73 − 2.52 = 7.21

Question 17.
$54.78 + $43.62
$ _____

Answer:
$98.4

Explanation:
$54.78 + $43.62
Add hundredths 8 + 2 = 10; Regroup
Add tenths 7 + 6 + 1 = 14;  Regroup
Add ones 4 + 3 + 1 = 8;
Add hundreds 5 + 4 = 9
$54.78 + $43.62 = $98.40

Question 18.
7.25 + 0.25 + 1.5
_____

Answer:
9

Explanation:
7.25 + 0.25 + 1.5
Add hundredths 5 + 5 + 0 = 10; Regroup
Add tenths 2 + 2 + 5 + 1 = 10;  Regroup
Add ones 7 + 0 + 1 + 1 = 9;
7.25 + 0.25 + 1.5 = 9.00

Use Reasoning Algebra Find the missing number.

Question 19.
n − 9.02 = 3.85
n = _____

Answer:
n = 12.87

Explanation:
n − 9.02 = 3.85
n = 3.85 + 9.02
n = 12.87

Question 20.
n + 31.53 = 62.4
n = _____

Answer:
n = 30.87

Explanation:
n + 31.53 = 62.4
n = 62.4 – 31.53 = 30.87
n = 30.87

Question 21.
9.2 + n + 8.4 = 20.8
n = _____

Answer:
n = 3.2

Explanation:
9.2 + n + 8.4 = 20.8
n + 17.6 = 20.8
n = 20.8 – 17.6
n = 3.2

Problem Solving Applications

Question 22.
Jake needs 7.58 meters of wood to complete a school project. He buys a 2.25-meter plank of wood and a 3.12-meter plank of wood. How many more meters of wood does Jake need to buy?
_____ meters

Answer:
2.21 meters

Explanation:
Jake needs 7.58 meters of wood to complete a school project. He buys a 2.25-meter plank of wood and a 3.12-meter plank of wood.
2.25 + 3.12 = 5.37
7.58 – 5.37 = 2.21

Question 23.
Lori needs a length of twine 8.5 meters long to mark a row in her garden. Andrew needs a length of twine 7.25 meters long for his row. They have one length of twine that measures 16.27 meters. After they each take the lengths they need, how much twine will be left?
_____ meters

Answer:
0.52 meters

Explanation:
Lori needs a length of twine 8.5 meters long to mark a row in her garden. Andrew needs a length of twine 7.25 meters long for his row. They have one length of twine that measures 16.27 meters.
8.5 + 7.25 = 15.75
16.27 – 15.75 = 0.52

Page No. 158

Use the table to solve 24–26.
Go Math Grade 5 Answer Key Chapter 3 Add and Subtract Decimals img 30

Question 24.
How much farther did the gold medal winner jump than the silver medal winner?
_____ meters

Answer:
0.1 meters

Explanation:
Gold medal = 8.34 meters
Silver medal = 8.24 meters.
8.34 – 8.24 = 0.10 meters
gold medal winner jump 0.1 meters than the silver medal winner

Question 25.
The fourth-place competitor’s jump measured 8.19 meters. If his jump had been 0.10 meter greater, what medal would he have received? Explain how you solved the problem.
_________

Answer:

Explanation:
The fourth-place competitor’s jump measured 8.19 meters. If his jump had been 0.10 meter greater
8.19 + 0.1 = 8.29
He may receive a silver medal. 8.29 is in between 8.24 and 8.34

Question 26.
In the 2004 Olympics, the gold medalist for the men’s long jump had a jump of 8.59 meters. How much farther did the 2004 gold medalist jump compared to the 2008 gold medalist?
_____ meters

Answer:
0.25 meters

Explanation:
In the 2004 Olympics, the gold medalist for the men’s long jump had a jump of 8.59 meters.
In 2008, 8.34 meters
8.59 – 8.34 = 0.25 meters

Question 27.
Alexander and Holly are solving the following word problem.
At the supermarket Carla buys 2.25 pounds of hamburger. She also buys 3.85 pounds of chicken. How many pounds of hamburger and chicken did Carla buy?
Alexander set up his problem as 2.25 + 3.85.
Holly set up her problem as 3.85 + 2.25.
Who is correct? Explain your answer and solve the problem.

Answer:
Alexander and Holly are solving the following word problem.
At the supermarket, Carla buys 2.25 pounds of hamburger. She also buys 3.85 pounds of chicken. She buys 2.25 + 3.85 = 6.10 pounds.
From the commutative property, 2.25 + 3.85 = 3.85 + 2.25
So, both answers are correct

Chapter Review/Test – Page No. 159

Question 1.
Chaz kept a record of how many gallons of gas he purchased each day last week.
Go Math Grade 5 Answer Key Chapter 3 Add and Subtract Decimals Chapter Review/Test img 31
Order the days from least amount of gas Chaz purchased to greatest amount of gas Chaz purchased.
Go Math Grade 5 Answer Key Chapter 3 Add and Subtract Decimals Chapter Review/Test img 32
Least: _____ ; _____ ; _____ ; _____ ; _____ Greatest

Answer:
grade 5 chapter 3 Add and Subtract Decimals 153 image 3
Least: 3.75; 3.9; 4.256; 4.258; 4.5 Greatest

Explanation:
Monday = 4.5 gallons
Tuesday = 3.9 gallons
Wednesday = 4.258 gallons
Thursday = 3.75 gallons
Friday = 4.256 gallons
The days from least amount of gas Chaz purchased to the greatest amount of gas Chaz purchased
4.5; 3.9; 4.258; 3.75; 4.256
3 < 4
3.9; 3.75; 4.5; 4.258; 4.256
9 > 7. So, 3.9; 3.75
5 > 2; 4.5; 4.258; 4.256
8 > 6; 4.258; 4.256
4.5; 4.258; 4.256; 3.9; 3.75
3.75; 3.9; 4.256; 4.258; 4.5

For 2a–2c, select True or False for each statement

Question 2.
2a. 16.437 rounded to the nearest whole number is 16.
i. TRUE
ii. FALSE

Answer:
i. TRUE

Explanation:
16.437; 4 < 5.
So, the nearest whole number is 16

Question 2.
2b. 16.437 rounded to the nearest tenth is 16.4.
i. TRUE
ii. FALSE

Answer:
i. TRUE

Explanation:
16.437 rounded to the nearest tenth
3 < 5
16.4

Question 2.
2c. 16.437 rounded to the nearest hundredth is 16.43.
i. TRUE
ii. FALSE

Answer:
ii. FALSE

Explanation:
16.437 rounded to the nearest hundredth is
7 > 5
16.44

Question 3.
Students are selling muffins at a school bake sale. One muffin costs $0.25, 2 muffins cost $0.37, 3 muffins cost $0.49, and 4 muffins cost $0.61. If this pattern continues, how much will 7 muffins cost? Explain how you found your answer.
$ _____

Answer:
$0.97

Explanation:
Students are selling muffins at a school bake sale. One muffin costs $0.25, 2 muffins cost $0.37, 3 muffins cost $0.49, and 4 muffins cost $0.61.
0.37 – 0.25 = 0.12
0.49 – 0.37 = 0.12
0.61 – 0.49 = 0.12
For 5 muffins 0.61 + 0.12 = 0.73
For 6 muffins 0.73 + 0.12 = 0.85
For 7 muffins 0.85 + 0.12 = 0.97
Every muffin cost increases with 0.12.

Chapter Review/Test – Page No. 160

Question 4.
What is the value of the underlined digit? Mark all that apply. 0.679
Options:
a. 0.6
b. 0.06
c. six tenths
d. six hundredths
e. 6 × \(\frac{1}{10}\)

Answer:
a. 0.6
c. six tenths
e. 6 × \(\frac{1}{10}\)

Explanation:
0.679
(0 x 1) + (6 x \(\frac{1}{10}\)) + (7 x \(\frac{1}{100}\)) + (9 x \(\frac{1}{1000}\))
6 x \(\frac{1}{10}\) = 0.6 = 6 tenths

Question 5.
Rowanda jogged 2.14 kilometers farther than Terrance. Select the values that could represent how far each student jogged. Mark all that apply.
Options:
a. Rowanda: 6.5 km, Terrance: 4.36 km
b. Rowanda: 4.8 km, Terrance: 2.76 km
c. Rowanda: 3.51 km, Terrance: 5.65 km
d. Rowanda: 7.24 km, Terrance: 5.1 km

Answer:
a. Rowanda: 6.5 km, Terrance: 4.36 km
d. Rowanda: 7.24 km, Terrance: 5.1 km

Explanation:
Rowanda jogged 2.14 kilometers farther than Terrance.
a. Rowanda: 6.5 km, Terrance: 4.36 km
6.5 – 4.36 = 2.14
b. Rowanda: 4.8 km, Terrance: 2.76 km
4.8 – 2.76 = 2.04
c. Rowanda: 3.51 km, Terrance: 5.65 km
5.65 – 3.51 = 2.14
d. Rowanda: 7.24 km, Terrance: 5.1 km
7.24 – 5.1 = 2.14
The first and fourth values can represent how far each student jogged.

Question 6.
Shade the model to show the decimal 0.542.
Go Math Grade 5 Answer Key Chapter 3 Add and Subtract Decimals Chapter Review/Test img 33
Type below:
_________

Answer:
grade 5 chapter 3 Add and Subtract Decimals 160 image 1

Explanation:
0.542 = 542/1000
5 hundredths, 4 tenths, 2 thousandths

Question 7.
Benjamin rode his bicycle 3.6 miles on Saturday and 4.85 miles on Sunday. How many miles did he ride Saturday and Sunday combined?
Use the digits on the tiles to solve the problem. Digits may be used more than once or not at all.
Go Math Grade 5 Answer Key Chapter 3 Add and Subtract Decimals Chapter Review/Test img 34
_________ miles

Answer:
8.45 miles
grade 5 chapter 3 Add and Subtract Decimals 153 image 4

Explanation:
Benjamin rode his bicycle 3.6 miles on Saturday and 4.85 miles on Sunday.
3.6 + 4.85 = 8.45

Chapter Review/Test – Page No. 161

Question 8.
The school is 3.65 miles from Tonya’s house and 1.28 miles from Jamal’s house. How much farther from school is Tonya’s house than Jamal’s house? Explain how you can use a quick picture to solve the problem.
_____ miles

Answer:
grade 5 chapter 3 Add and Subtract Decimals 161 image 2
2.37 miles

Explanation:
The school is 3.65 miles from Tonya’s house and 1.28 miles from Jamal’s house.
3.65 – 1.28 = 2.37

Question 9.
A vet measured the mass of two birds. The mass of the robin was 76.64 grams. The mass of the blue jay was 81.54 grams. Estimate the difference in the masses of the birds.
≈ _____ grams

Answer:
5 grams

Explanation:
A vet measured the mass of two birds. The mass of the robin was 76.64 grams. The mass of the blue jay was 81.54 grams.
76.64 grams is closer to 77
81.54 grams is closer to 82
82 – 77 = 5
The estimated difference in the masses of the birds is 5 grams.

Question 10.
Rick bought 5 yogurt bars at a snack shop. Each yogurt bar cost $1.75. Complete the table to show the price of 2, 3, 4, and 5 yogurt bars.
Go Math Grade 5 Answer Key Chapter 3 Add and Subtract Decimals Chapter Review/Test img 35
Type below:
_________

Answer:
grade 5 chapter 3 Add and Subtract Decimals 161 image 1

Explanation:

Question 11.
Clayton Road is 2.25 miles long. Wood Pike Road is 1.8 miles long. Kisha used a quick picture to find the combined length of Clayton Road and Wood Pike Road. Does Kisha’s work make sense? Explain why or why not
Go Math Grade 5 Answer Key Chapter 3 Add and Subtract Decimals Chapter Review/Test img 36
i. Yes
ii. No

Answer:
i. Yes

Explanation:
Clayton Road is 2.25 miles long. Wood Pike Road is 1.8 miles long.
2.25 + 1.8 = 4.05
4 tens, 0 tenths, 5 hundredths

Chapter Review/Test – Page No. 162

Question 12.
Bob and Ling are playing a number pattern game. Bob wrote the following sequence.
28.9, 26.8, 24.7, __, 20.5
What is the unknown term in the sequence?
_____

Answer:
26.8

Explanation:
Bob and Ling are playing a number pattern game. Bob wrote the following sequence.
28.9, 26.8, 24.7, __, 20.5
28.9 – 26.8 = 2.1
26.8 – 24.7 = 2.1
Every number is increased by 2.1
So, the unknown number is 24.7 + 2.1 = 26.8

Rafael bought 2.15 pounds of potato salad and 4.2 pounds of macaroni salad to bring to a picnic. For 13a–13c, select Yes or No to indicate whether each statement is true.

Question 13.
13a. Rounded to the nearest whole number, Rafael bought 2 pounds of potato salad.
i. Yes
ii. No

Answer:
i. Yes

Explanation:
2.15 pounds of potato salad
1 < 5 ;
So, Rounded to the nearest whole number is 2

Question 13.
13b. Rounded to the nearest whole number, Rafael bought 4 pounds of macaroni salad.
i. Yes
ii. No

Answer:
i. Yes

Explanation:
4.2 pounds of macaroni salad
2 < 5
So, Rounded to the nearest whole number is 4

Question 13.
13c. Rounded to the nearest tenth, Rafael bought 2.1 pounds of potato salad.
i. Yes
ii. No

Answer:
ii. No

Explanation:
2.15 pounds of potato salad
5 = 5 ;
So, Rounded to the nearest whole number is 2.2

Question 14.
The four highest scores on the floor exercise at a gymnastics meet were 9.675, 9.25, 9.325, and 9.5 points. Choose the numbers that make the statement true.
Go Math Grade 5 Answer Key Chapter 3 Add and Subtract Decimals Chapter Review/Test img 37
The lowest: _________
The highest: _________

Answer:
The lowest: 9.25
The highest: 9.75

Explanation:
Compare ones; All ones are the same.
Compare tenths; 9.75 has the highest number of tenths and 9.25 has the lowest number of tenths.
The lowest of these four scores was 9.25 points. The highest of these four scores was 9.75 points.

Chapter Review/Test – Page No. 163

Question 15.
Michelle records the value of one euro in U.S. dollars each day for her social studies project. The table shows the data she has recorded so far.
Go Math Grade 5 Answer Key Chapter 3 Add and Subtract Decimals Chapter Review/Test img 38
On which two days was the value of 1 euro the same when rounded to the nearest hundredth of a dollar?
Options:
a. Monday
b. Tuesday
c. Wednesday
d. Thursday

Answer:
a. Monday
c. Wednesday

Explanation:
Monday = 1.448
The digit in the hundredths place is 4. 8 > 5; So, the rounded number is 1.45
Tuesday = 1.443
The digit in the hundredths place is 4. 3 < 5; So, the rounded number is 1.44
Wednesday = 1.452
The digit in the hundredths place is 5. 2 < 5; So, the rounded number is 1.45
Thursday = 1.458
The digit in the hundredths place is 5. 8 > 5; So, the rounded number is 1.46

Question 16.
Miguel has $20. He spends $7.25 on a movie ticket, $3.95 for snacks, and $1.75 for bus fare each way. How much money does Miguel have left?
$ _____

Answer:
$7.05

Explanation:
Miguel has $20. He spends $7.25 on a movie ticket, $3.95 for snacks, and $1.75 for bus fare each way.
$7.25 + $3.95 + $1.75 = $12.95
$20 – $12.95 = $7.05

Question 17.
Yolanda’s sunflower plant was 64.34 centimeters tall in July. During August, the plant grew 18.2 centimeters.
Part A
Estimate the height of Yolanda’s plant at the end of August by rounding each value to the nearest whole number. Will your estimate be less than or greater than the actual height? Explain your reasoning.
_____ cm

Answer:
First, we want to round the number 64.34 to the nearest whole number.
1. We have to round this number to the molest tenth. To round the number to the nearest tenth we need to look at the digit in the hundredths place. So, as 4 < 5, the rounded number is 64.3.
2. We now have to round this number to the nearest one. lb round the number to the nearest one we need to look at the digit in the tenths place. So, as 3 < 5, the rounded number is 64.
Now, we have to round the number 18.2 to the nearest whole number.
1. We have to round this number to the nearest one. To round the number to the nearest one we need to look at the digit in the tenths place. So, as 2 <5, the rounded number is 18.
So, we now have to find the sum of these rounded values: 64 + 18 = 82. Therefore, the estimated height of Volanda’s plant at the and of August is: 82 centimeters.
The estimate is less than the actual height because rounded values are less than the actual values.

Question 17.
Part B
What was the exact height of the plant at the end of August? Was the estimate less than or greater than the exact value?
_____ cm

Answer:
The exact height of the plant is: 64.34 + 18.2
Add the hundredths first.
4 hundre.dths + 0 hundredths = 4 hundredths
Add the tenths.
3 tenths + 2 tenths = 5 tenths Add the ones. Regroup as nee.ded
Add the tens.
6 tens + 1 ten + 1 regrouped ten = 8 tens.
Therefore, the exact height is 64.34+ 18.2 = 82.54.
The estimate is less than the actual height.

Chapter Review/Test – Page No. 164

Question 18.
Oscar ran the 100-yard dash in 12.41 seconds. Jesiah ran the 100-yard dash in 11.85 seconds. How many seconds faster was Jesiah’s time than Oscar’s time?
_____ second(s)

Answer:
0.56 seconds

Explanation:
Oscar ran the 100-yard dash in 12.41 seconds. Jesiah ran the 100-yard dash in 11.85 seconds.
12.41 – 11.85 = 0.56 seconds.
Jesiah’s time is 0.56 seconds faster than Oscar’s time.

Question 19.
Choose the value that makes the statement true.
Go Math Grade 5 Answer Key Chapter 3 Add and Subtract Decimals Chapter Review/Test img 39
Type below:
_________

Answer:
2 hundredths and 5 thousandths

Explanation:
1.025
(1 x 1) + (0 x \(\frac{1}{10}\)) + (2 x \(\frac{1}{100}\)) + (5 x \(\frac{1}{1000}\))
2 x \(\frac{1}{100}\) = 2 hundredths
5 x \(\frac{1}{1000}\) = 5 thousandths
In the number 1.025, the value of the digit 2 is 2 hundredths, and the value of the digit 5 is 5 thousandths.

Question 20.
Troy and Lazetta are solving the following word problem. Rosalie’s cat weights 9.8 pounds. Her dog weighs 25.4 pounds. What is the weight of both animals combined. Troy sets up his problem as 9.8 + 25.4. Lazetta sets up her problem as 25.4 + 9.8. Who is correct? Explain your answer and solve the problem.
_________

Answer:
Troy and Lazetta are solving the following word problem. Rosalie’s cat weighs 9.8 pounds. Her dog weighs 25.4 pounds.
9.8 + 25.4
Add tenths 8 + 4 = 12; regroup
Add ones 9 + 5 + 1 regrouped one = 15 ones; regroup
Add tens 0 + 2 + 1 regrouped ten = 3 tens.
35.2
Lazetta: 25.4 + 9.8 = 35.2
Therefore, the answer is 25.4 + 9.8 = 35.2
The weight of both animals combined is 35.2 pounds. So, both were right.

Question 21.
Go Math Grade 5 Answer Key Chapter 3 Add and Subtract Decimals Chapter Review/Test img 40
Type below:
_________

Answer:
0.084 and 8.4

Explanation:
0.84 is 10 times as much as
0.84 = 10S
S = 0.84/10 = 0.084
0.84 is 1/10 of
0.84 = 1/10 x S
S = 0.84 x 10 = 8.4
So, from the given answers, 0.84 is 10 times as much as 0.084, and 0.84 is 1/10 of 8.4

Conclusion

Hoping that Go Math Grade 5 Answer Key Chapter 3 Add and Subtract Decimals has helped you clear your queries. Access the Go Math Grade 5 Answer Key for free of cost prepared by subject experts. 5th Grade Go Math Answer Key Ch 3 Add and Subtract Decimals is prepared keeping in mind the Students Level of Understanding.

Go Math Grade 6 Answer Key Chapter 6 Convert Units of Length

go-math-grade-6-chapter-6-convert-units-of-length-answer-key

In order to solve real-life mathematical problems, students must understand how the information is related, and how to convert the units. You can learn the concepts only when you start from the basics. Download Free Pdf of Go Math Grade 6 Answer Key Chapter 6 Convert Units of length to practice the exercise and homework problems. We have provided the solutions for all the questions in the HMH Go Math Grade 6 Answer Key Chapter 6 Convert Units of length.

Go Math Grade 6 Chapter 6 Convert Units of Length Answer Key

The topics covered in this chapter are Convert units of length, capacity, convert units of weight and mass, transform units, distance, rate and time formulas. This is the easiest and important among all the chapters in the 6th standard. You can score the maximum marks in the exams with the help of our HMH Go Math Grade 6 Answer Key Chapter 6 Convert Units of Length. Tap the links which are provided according to the topics and kickstart your preparation.

Lesson 1: Convert Units of Length

Lesson 2: Convert Units of Capacity

Lesson 3: Convert Units of Weight and Mass

Mid-Chapter Checkpoint

Lesson 4: Transform Units

Lesson 5: Problem Solving • Distance, Rate, and Time Formulas

Chapter 6 Review/Test

Share and Show – Page No. 317

Convert to the given unit.

Question 1.
3 miles = ? yards
_______ yd

Answer:
5280 yd

Explanation:
3 miles = ? yards
1 yard = 3 feet
1 mile = 5280 feet
So, 3 miles = 3 x 5280 feet
= 15,840 feet
3 feet = 1 yard
Then, 15,840 feet = 15,840 ÷ 3
= 5280 yards
So, 3 miles = 5280 yards

Question 2.
43 dm = ? hm
_______ hm

Answer:
0.043 hm

Explanation:
43 dm= ?hm
10 decimeters = 1 meter
1 hectometer = 100 meters
1 meter = 10 decimeter
100 meters = 10×100 decimeters = 1000 decimeters
So 1 hectometer = 1000 decimeters
Then, 43 decimeters = 43/1000 = 0.043 hectometers
So, 43 dm = 0.043 hm

Question 3.
9 yd = ? in.
_______ inches

Answer:
324 inches

Explanation:
9 yd= ? in.
1 yard = 36 inches
So 9 yards = 9×36 = 324 inches
9 yards = 324 inches

Question 4.
72 ft = 24 yd
_______ yd

Answer:
24 yd

Explanation:
72 ft = 24 yd
1 yard = 3 feet
So, 1 feet = 1/3 yard
Then, 72 feet = 72/3 yard
So, 72 feet = 24 yards

Question 5.
7,500 mm = ? dm
_______ dm

Answer:
75 dm

Explanation:
7,500 mm = ?dm
1000 millimeters = 1 meter
10 decimeters = 1 meter
So, 1000 millimeters = 10 decimeters
Then 1 millimeter = 10/1000 decimeter = 1/100 decimeters
So 7500 millimeters = 7500/100 decimeters
Then 7500 mm = 75 dm

On Your Own

Question 6.
Rohan used 9 yards of ribbon to wrap gifts. How many inches of ribbon did he use?
_______ inches

Answer:
324 inches

Explanation:
As per the given data,
Rohan used 9 yards of ribbon to wrap gifts
1 yard = 36 inches
So, 9 yards = 9×36 = 324 inches
So, Rohan used 324 inches ribbon to wrap gifts

Question 7.
One species of frog can grow to a maximum length of 12.4 millimeters. What is the maximum length of this frog species in centimeters?
_______ cm

Answer:
1.24 cm

Explanation:
One species of frog can grow to a maximum length of 12.4 millimeters.
From the given information
One species of frog can grow to a maximum length of 12.4 millimeters
1000 millimeters = 1 meter
100 centimeters = 1 meter
So, 1000 millimeters = 100 centimeters
1 millimeter = 100/1000 centimeters = 1/10 centimeters
So, 12.4 millimeters = 12.4/10 centimeters = 1.24 centimeters
12.4 millimeters = 1.24 centimeters

Question 8.
The height of the Empire State Building measured to the top of the lightning rod is approximately 443.1 meters. What is this height in hectometers?
_______ hectometers

Answer:
4.431 hectometers

Explanation:
The height of the Empire State Building measured to the top of the lightning rod is approximately 443.1 meters.
443.1 meters in hectometers
1 hectometer = 100 meters
Then, 1 meter = 1/100 hectometers
So, 443.1 meters = 443.1/100 hectometers
443.1 meters = 4.431 hectometers

Question 9.
A snail moves at a speed of 2.5 feet per minute. How many yards will the snail have moved in half of an hour?
_______ yards

Answer:
25 yards

Explanation:
From the given information
A snail moves at a speed of 2.5 feet per minute
1 hour = 60 minutes
1 minute = 2.5 feet speed
60 minutes = 60×2.5 feet = 150 feet
1 yard = 3 feet
So 1 feet = 1/3 yards
Then, 150 feet = 150/3 yards = 50 yards per hour
For half of an hour, a snail moves 25 yards

Practice: Copy and Solve Compare. Write <, >, or =.

Question 10.
32 feet _______ 11 yards

Answer:
32 feet < 11 yards

Explanation:
32 feet _______ 11 yards
1 yard = 3 feet
So, 11 yards = 11×3 = 33 feet
So, 32 feet < 11 yards

Question 11.
537 cm _______ 5.37 m

Answer:
537 cm = 5.37 m

Explanation:
537 cm _______ 5.37 m
100 centimeters = 1 meter
1 centimeter = 0.01 meter
So, 537 centimeters = 537×0.01 meters
That is 537 centimeters = 5.37 meters

Question 12.
75 inches _______ 6 feet

Answer:
75 inches > 6 feet

Explanation:
75 inches _______ 6 feet
1 foot = 12 inches
6 feet = 6×12 = 72 inches
So, 75 inches > 6 feet

Problem Solving + Applications – Page No. 318

What’s the Error?

Question 13.
The Redwood National Park is home to some of the largest trees in the world. Hyperion is the tallest tree in the park, with a height of approximately 379 feet. Tom wants to find the height of the tree in yards.
Tom converted the height this way :
3 feet = 1 yard
conversion factor: \(\frac{3 \mathrm{ft}}{1 \mathrm{yd}}\)
\(\frac{379 \mathrm{ft}}{1} \times \frac{3 \mathrm{ft}}{1 \mathrm{yd}}\) = 1,137 yd
Find and describe Tom’s error.
Show how to correctly convert from 379 feet to yards.
Explain how you knew Tom’s answer was incorrect.
Type below:
____________

Answer:
conversion factor: 3ft1yd
379ft1 × 3ft1yd = 1,137 yd
We need to divide the 379 feet with 3 to get the height of the Hyperion tree, but tom multiplies the 379 with 3 and that is the error part
1 yard = 3 feet
1 feet = 1/3 yards
So, 379 feet = 379/3 yards = 126.3 yards
So, the height of the Hyperion tree is 126.3 yards

Question 14.
Choose <, >, or =.
14a. 12 yards Ο 432 inches
14b. 321 cm Ο 32.1 m
12 yards _______ 432 inches
321 cm _______ 32.1 m

Answer:
14a. 12 yards Ο 432 inches
14b. 321 cm Ο 32.1 m
12 yards = 432 inches
321 cm < 32.1 m

Explanation:
14a. 12 yards Ο 432 inches
1 yard = 36 inches
12 yards = 12×36 = 432 inches
So, 12 yards = 432 inches
14b. 321 cm Ο 32.1 m
100 centimeters = 1 meter
1 centimeter = 0.01 meter
321 centimeters = 321×0.01 meters = 3.21 meters
3.21 < 32.1
So, 321 centimeters < 32.1 meters

Convert Units of Length – Page No. 319

Convert to the given unit.

Question 1.
42 ft = ? yd
_______ yd

Answer:
14yd

Explanation:
42 ft= ?yd
3 feet = 1 yard
1 feet = 1/3 yard
So, 42 feet = 42/3 = 14 yard
So, 42 feet = 14 yards

Question 2.
2,350 m = ? km
_______ km

Answer:
2.350 km

Explanation:
2,350 m = ? km
1 kilometer = 1000 meters
1 meter = 1/1000 kilometers
Then, 2350 meters = 2350/1000 kilometers
2350 meters = 2.350 kilometers

Question 3.
18 ft = ? in.
_______ inches

Answer:
216 inches

Explanation:
18 ft= ? in
1 foot = 12 inches
18 feet = 12×18 = 216 inches
18 feet = 216 inches

Question 4.
289 m = ? dm
_______ dm

Answer:
2890 dm

Explanation:
289 m = ?dm
10 decimeters = 1 meter
289 meters = 289×10 decimeters
So, 289 meters = 2890 decimeters

Question 5.
5 mi = ? yd
_______ yd

Answer:
8,800 yd

Explanation:
1. 5 mi = ? yd
1 mile = 1760 yards
5 miles = 5×1760 = 8800 yards
5 mi = 8,800 yards

Question 6.
35 mm = ? cm
_______ cm

Answer:
3.5 cm

Explanation:
35 mm = ? cm
1000 millimeters = 1 meter
100 centimeters = 1 meter
So, 1000 millimeters = 100 centimeters
1 millimeter = 100/1000 centimeters
Then, 35 millimeters = 35×100/1000 centimeters = 3.5 centimeters
35 millimeters = 3.5 centimeters

Compare. Write <, >, or =.

Question 7.
1.9 dm _______ 1,900 mm

Answer:
1.9 dm < 1,900 mm

Explanation:
1.9 dm _______ 1,900 mm
10 decimeters = 1 meter
1000 millimeters = 1 meter
So, 10 decimeters = 1000 millimeters
1 decimeter = 100 millimeters
1.9 decimeters = 1.9 x 100 = 190 millimeters
So, 1.9 decimeters = 190 millimeters
So, 1.9 dm < 1900 mm

Question 8.
12 ft _______ 4 yd

Answer:
12 ft  = 4 yd

Explanation:
12 ft _______ 4 yd
3 feet = 1 yard
3×4 feet = 12 feet = 1×4 = 4 yard
So, 12 feet = 4 yards

Question 9.
56 cm _______ 56,000 km

Answer:
56 cm < 56,000 km

Explanation:
56 cm _______ 56,000 km
100 centimeters = 1 meter
1 kilometer = 1000 meters
0.01 kilometer = 1 meter
So, 100 centimeters = 0.01 kilometers
1 centimeter = 0.01/100 kilometers
56 centimeters = 56 x 0.01/100 kilometers =0.0056 kilometers
So, 56 cm < 56,000 km

Question 10.
98 in. _______ 8 ft

Answer:
98 in. > 8 ft

Explanation:
98 in. _______ 8 ft
1 foot = 12 inches
8 feet = 8×12 = 96 inches
So, 98 in > 8 feet

Question 11.
64 cm _______ 630 mm

Answer:
64 cm  > 630 mm

Explanation:
64 cm _______ 630 mm
1000 millimeters = 1 meter
100 centimeters = 1 meter
So, 100 centimeters = 1000 millimeters
1 centimeter = 10 millimeters
so, 64 centimeters = 64×10 millimeters = 640 millimeters
then, 64 cm > 630 mm

Question 12.
2 mi _______ 10,560 ft

Answer:
2 mi  = 10,560 ft

Explanation:
1 mi _______ 10,560 ft
1 mile = 5280 feet
so, 2 miles = 2×5280 = 10560 feet
then, 2 miles = 10,560 feet

Question 13.
The giant swallowtail is the largest butterfly in the United States. Its wingspan can be as large as 16 centimeters. What is the maximum wingspan in millimeters?
_______ mm

Answer:
160 mm

Explanation:
The giant swallowtail is the largest butterfly in the United States. Its wingspan can be as large as 16 centimeters.
100 centimeters = 1 meter
1000 millimeters = 1 meter
So, 100 centimeters = 1000 millimeters
1 centimeters = 10 millimeters
then 16 centimeters = 16×10 millimeters = 160 millimeters
So, giant swallowtail wingspan is 160 millimeters large

Question 14.
The 102nd floor of the Sears Tower in Chicago is the highest occupied floor. It is 1,431 feet above the ground. How many yards above the ground is the 102nd floor?
_______ yd

Answer:
477 yd

Explanation:
The 102nd floor of the Sears Tower in Chicago is the highest occupied floor. It is 1,431 feet above the ground.
3 feet = 1 yard
1 feet = 1/3 yard
Then, 1431 feet = 1431/3 yard = 477 yards
So, the height of the 102nd floor from the ground = 477 yards

Question 15.
Explain why units can be simplified first when measurements are multiplied.
Type below:
____________

Answer:
Units can be simplified first, because if (60 min)/(1 hr) = 1, then I can multiply any measurement by that fraction and not change its value.

Lesson Check – Page No. 320

Question 1.
Justin rides his bicycle 2.5 kilometers to school. Luke walks 1,950 meters to school. How much farther does Justin ride to school than Luke walks to school?
_______ meters

Answer:
550 meters

Explanation:
Justin rides his bicycle 2.5 kilometers to school. Luke walks 1,950 meters to school.
1 kilometer = 1000 meters
Then, 2.5 kilometers = 2.5 x 1000 = 2500 meters
So, Justin rides his bicycle 2500 meters and Luke walks 1950 meters
2500 – 1950 = 550 meters
So, Justin rides more 550 meters than Luke to school

Question 2.
The length of a room is 10 \(\frac{1}{2}\) feet. What is the length of the room in inches?
_______ inches

Answer:
126 inches

Explanation:
1 feet = 12 inches
10 1/2 feet = ?
10 1/2 = 21/2
21/2 × 12 = 21 × 6 = 126
126 inches

Spiral Review

Question 3.
Each unit on the map represents 1 mile. What is the distance between the campground and the waterfall?
Go Math Grade 6 Answer Key Chapter 6 Convert Units of Length img 1
_______ miles

Answer:
4 miles

Explanation:
Each unit on the map represents 1 mile
The distance between the campground and the waterfall is 4 units that is 4 miles

Question 4.
On a field trip, 2 vans can carry 32 students. How many students can go on a field trip when there are 6 vans?
_______ students

Answer:
96 students

Explanation:
On a field trip, 2 vans can carry 32 students
So, 1 van can carry the students = 32/2 = 16 students
Then, students can go in 6 vans = 6×16 = 96 students

Question 5.
According to a 2008 survey, \(\frac{29}{50}\) of all teens have sent at least one text message in their lives. What percent of teens have sent a text message?
_______ %

Answer:
58%

Explanation:
From the given information
According to a 2008 survey
29/50 of all teens have sent at least one text message in their lives
Percent of teens have sent a text message = 29/50 x 100 = 58%
So, 58% of teens have sent text messages

Question 6.
Of the students in Ms. Danver’s class, 6 walk to school. This represents 30% of her students. How many students are in Ms. Danver’s class?
_______ students

Answer:
20 students

Explanation:
Of the students in Ms. Danver’s class, 6 walk to school
It represents 30% of her students
That is 30% = 6 students
Then 100% = (100×6)/30 = 20
Total number of students in Ms. Danver’s class = 20 students

Share and Show – Page No. 323

Convert to the given unit.

Question 1.
5 quarts = ? cups
_______ cups

Answer:
20 cups

Explanation:
5 quarts = ? cups
4cups = 1 quart
So, 5 quarts = 5×4 = 20 cups
5 quarts = 20 cups

Question 2.
6.7 liters = ? hectoliters
_______ hectoliters

Answer:
0.067 hectoliters

Explanation:
1.7 liters = ? hectoliters
1 hectoliter= 100 liters
1 liter = 1/100 hectoliters
6.7 liters = 6.7/100 hectoliters = 0.067 hectoliters

Question 3.
5.3 kL = ? L
_______ L

Answer:
5300 L

Explanation:
5.3 kL= ? L
1 Kiloliter = 1000 liters
Then, 5.3 kiloliters = 5.3 x 1000 = 5300 liters
So, 5.3 kL = 5300 L

Question 4.
36 qt = ? gal
_______ gal

Answer:
9 gal

Explanation:
36 qt = ? gal
4 quarts = 1 gallon
So, 36 qts = 9×4 quarts = 9×1 gallons
So, 36 qt = 9 gallons

Question 5.
5,000 mL = ? cL
_______ cL

Answer:
500 cL

Explanation:
5,000 mL = ?cL
1000 milliliters = 1 liter
100 centiliters = 1 liter
So, 1000 milliliters = 100 centiliters
Then, 5000 milliliters = 5×100 centiliters = 500 centiliters
5000 milliliters = 500 centiliters

On Your Own

Question 6.
It takes 41 gallons of water for a washing machine to wash a load of laundry. How many quarts of water does it take to wash one load?
_______ quarts

Answer:
164 quarts

Explanation:
It takes 41 gallons of water for a washing machine to wash a load of laundry.
41 gallons of water is required for a washing machine to wash a load of laundry
1 gallon = 4 quarts
Then, 41 gallons = 41×4 quarts = 164 quarts
164 quarts of water us required for a washing machine to wash a load of laundry

Question 7.
Sam squeezed 237 milliliters of juice from 4 oranges. How many liters of juice did Sam squeeze?
_______ L

Answer:
0.237 L

Explanation:
Sam squeezed 237 milliliters of juice from 4 oranges
1000 liliters = 1 liter
1 milliliter = 1/1000 liter
237 milliliters = 237/1000 liters
237 milliliters = 0.237 liters

Question 8.
Reason Quantitatively A bottle contains 3.78 liters of water. Without calculating, determine whether there are more or less than 3.78 deciliters of water in the bottle. Explain your reasoning
Type below:
____________

Answer:
Reason Quantitatively A bottle contains 3.78 liters of water
1 liter = 10 deciliters
Then 3.78 liters = 3.78×10 = 37.8 deciliters
So, bottle contains more than 3.78 deciliters of water

Question 9.
Tonya has a 1-quart, a 2-quart, and a 3-quart bowl. A recipe asks for 16 ounces of milk. If Tonya is going to triple the recipe, what is the smallest bowl that will hold the milk?
The _______ bowl

Answer:
The 3 quarts bowl

Explanation:
Tonya has a 1-quart, a 2-quart, and a 3-quart bowl
A recipe asks for 16 ounces of milk
If Tonya triples the recipe, then 1 quart = 3, 2 quart = 6, 3 quart = 9
The smallest bowl is 3 quarts

Practice: Copy and Solve Compare. Write <, >, or =.

Question 10.
700,000 L _______ 70 kL

Answer:
700,000 L > 70 kL

Explanation:
700,000 L _______ 70 kL
1 kiloliter = 1000 liters
Then, 70 kiloliters = 70×1000 liters = 70,000 liters
So, 700,000 liters > 70 kiloliters

Question 11.
6 gal _______ 30 qt

Answer:
6 gal < 30 qt

Explanation:
6 gal _______ 30 qt
4 quarts = 1 gallon
So, 6 gallons = 6×4 = 24 quarts
So, 6 gallons < 30 quarts

Question 12.
54 kL _______ 540,000 dL

Answer:
54 kL  = 540,000 dL

Explanation:
54 kL _______ 540,000 dL
1 kiloliter = 1000 liters
1 liter = 10 deciliters
Then, 1000 liters = 10×1000 = 10,000 deciliters
So, 1 kiloliter = 10,000 deciliters
Then, 54 kiloliters = 54×10,000 = 540,000 deciliters
So, 54 kL = 540,000 dL

Question 13.
10 pt _______ 5 qt

Answer:
10 pt  = 5 qt

Explanation:
10 pt _______ 5 qt
1 pints = 1 quart
then, 10 pints = 2×5 pints = 1×5 quart = 5 quarts
So, 10 pints = 5 quarts

Question 14.
500 mL _______ 50 L

Answer:
500 mL  < 50 L

Explanation:
500 mL _______ 50 L
1000 milliliters = 1 liter
Then, 1000/2 milliliters = 500 milliliters = ½ liters= 0.5 liters
So, 500 mL < 50 L

Question 15.
14 c _______ 4 qt

Answer:
14 c  < 4 qt

Explanation:

14 c _______ 4 qt
4 cups = 1 quart
1 cup = ¼ quart
Then, 14 cups = 14/4 quarts = 3.5 quarts
So, 14 cups < 4 quarts

Unlock the Problem – Page No. 324

Question 16.
Jeffrey is loading cases of bottled water onto a freight elevator. There are 24 one-pint bottles in each case. The maximum weight that the elevator can carry is 1,000 pounds. If 1 gallon of water weighs 8.35 pounds, what is the maximum number of full cases Jeffrey can load onto the elevator?
a. What do you need to find?
Type below:
____________

Answer:
the maximum number of full cases Jeffrey can load onto the elevator

Question 16.
b. How can you find the weight of 1 case of bottled water? What is the weight?
Type below:
____________

Answer:
Using one-pint bottles and 1 gallon of water weighs 8.35 pounds information

Explanation:

Question 16.
c. How can you find the number of cases that Jeffrey can load onto the elevator?
Type below:
____________

Answer:
1 US liquid pint is equivalent to 0.125 US liquid gallons.
So, 24 one-pint bottles is equivalent to (24 × 0.125) =3 gallons.
Therefore, one full case of bottled water is equal to 3 gallons.
Now, 1 gallon is equal to 8.35 pounds
And hence, 3 gallons is equal to (8.35 × 3) = 25.05 pounds.

Question 16.
d. What is the maximum number of full cases Jeffrey can load onto the elevator?
_______ cases

Answer:
39 cases

Explanation:
1 US liquid pint is equivalent to 0.125 US liquid gallons.
So, 24 one-pint bottles is equivalent to (24 × 0.125) =3 gallons.
Therefore, one full case of bottled water is equal to 3 gallons.
Now, 1 gallon is equal to 8.35 pounds
And hence, 3 gallons is equal to (8.35 × 3) = 25.05 pounds.
If the maximum weight that the elevator can carry is 1000 pounds, then the maximum number of cases of bottled water that the elevator can carry is ≈ 39
We can not take the number as 40, because then the total weight will become more than 1000 pounds which is not allowed.

Question 17.
Monica put 1 liter, 1 deciliter, 1 centiliter, and 1 milliliter of water into a bowl. How many milliliters of water did she put in the bowl?
_______ milliliters

Answer:
1111 milliliters

Explanation:
Monica put 1 liter, 1 deciliter, 1 centiliter, and 1 milliliter of water into a bowl
1 liter = 1000 milliliters
1 liter = 10 deciliters
so, 10 deciliters = 1000 milliliters
then, 1 deciliter = 100 milliliters
1 liter = 100 centiliters
So, 100 centiliters = 1000 milliliters
Then, 1 centiliter = 10 milliliters
1 liter + 1 deciliter + 1 centiliter + 1 milliliter
= 1000 milliliters + 100 milliliters + 10 milliliters + 1 milliliter
= 1111 milliliters
Monica filled the bowl with 1111 milliliters of water

Question 18.
Select the conversions that are equivalent to 235 liters. Mark all that apply.
Options:
a. 235,000 milliliters
b. 0.235 milliliters
c. 235,000 kiloliters
d. 0.235 kiloliters

Answer:
a. 235,000 milliliters

Explanation:
a. 235,000 milliliters
1000 milliliters = 1 liter
Then, 235×1000 milliliters = 1×235 liters = 235 liters
So, 235,000 milliliters are equivalent to 235 liters

Convert Units of Capacity – Page No. 325

Convert to the given unit.

Question 1.
7 gallons = ? quarts
_______ quarts

Answer:
28 quarts

Explanation:
6 gallons = ? quarts
4 quarts = 1 gallon
then, 7 gallons = 4×7 = 28 quarts

Question 2.
5.1 liters = ? kiloliters
_______ kiloliters

Answer:
0.0051 kiloliters

Explanation:
5.1 liters = ? kiloliters
1 kiloliter = 1000 liters
So, 1 liter = 1/1000 kiloliter
Then, 5.1 liters = 5.1/1000 kiloliters
5.1 liters = 0.0051 kiloliters

Question 3.
20 qt = ? gal
_______ gal

Answer:
5 gal

Explanation:
20 t = ? gal
4 quarts = 1 gallon
Then, 4×5 quarts = 1×5 gallons
That is 20 quarts = 5 gallons

Question 4.
40 L = ? mL
_______ mL

Answer:
40,000 mL

Explanation:
40 L = ? mL
1000 milliliters = 1 liter
Then, 40 liters = 40×1000 milliliters = 40,000 milliliters
40 L = 40,000 mL

Question 5.
33 pt = ? qt ? pt
_______ qt _______ pt

Answer:
33/2 quarts = 16.5 quarts

Explanation:
33 pt= ?qt ? pt
1 pints = 1 quart
1 pint = ½ quart
then, 33 pint = 33/2 quarts = 16.5 quarts

Question 6.
29 cL = ? daL
_______ daL

Answer:
0.029 daL

Explanation:
29 cL = ? daL
100 centiliters = 1 liter
1 dekaliter = 10 liters
So, 1 liter = 1/10 dekaliters
Then, 100 centiliters = 1/10 dekaliters
1 centiliter = 1/1000 dekaliters
then, 29 centiliters = 29/1000 dekaliters = 0.029 dekaliters
29 cL = 0.029 daL

Question 7.
7.7 kL = ? cL
_______ cL

Answer:
7,70,000 cL

Explanation:
6.7 kL = ? cL
1 kiloliter = 1000 liters
100 centiliters = 1 liter
So, 1000 liters = 100×1000 centiliters = 1,00,000 centiliters
Then, 1 kiloliter = 1,00,000 centiliters
Then, 7.7 kiloliters = 7.7 x 1,00,000 centiliters = 7,70,000 centiliters

Question 8.
24 fl oz = ? pt ? c
_______ pt _______ c

Answer:
3/2 pt and 3 cups

Explanation:
24 floz= ?pt ? c
6 fluids ounces = 1 cup
then, 24 fluid ounces = 8×3 = 1×3 cups = 3 cups
1 cups = 1 pint
then, 1 cup = ½ pint
then, 3 cups = 3/2 pint
so, 24 fluids ounces = 3/2 pint and 3 cups

Problem Solving

Question 9.
A bottle contains 3.5 liters of water. A second bottle contains 3,750 milliliters of water. How many more milliliters are in the larger bottle than in the smaller bottle?
_______ mL

Answer:
250 mL

Explanation:
A bottle contains 3.5 liters of water. A second bottle contains 3,750 milliliters of water.
A bottle contains 3.5 liters of water
A second bottle contains 3,750 milliliters of water
1000 milliliters = 1 liter
Then, 3.5 liters = 3.5×1000 = 3500 milliliters
So, 3750 – 3500 = 250 milliliters
250 milliliters of water is more than in the larger bottle than the smaller bottle

Question 10.
Arnie’s car used 100 cups of gasoline during a drive. He paid $3.12 per gallon for gas. How much did the gas cost?
$ _______

Answer:
$19.5

Explanation:
Arnie’s car used 100 cups of gasoline during a drive. He paid $3.12 per gallon for gas.
Arnie’s car used 100 cups of gasoline during a drive
He paid $3.12 per gallon for gas
1 gallon = 4 quarts
1 quart = 4 cups
then, 4 quarts = 4×4 cups = 16 cups
So, 1 gallon = 16 cups
Then, 1 cup = 1/16 gallons
Then, 100 cups = 100/16 gallons = 6.25 gallons
Total gas cost = $3.12 x 6.25 = $19.5

Question 11.
Explain how units of length and capacity are similar in the metric system.
Type below:
____________

Answer:
In the metric system, The unit of length is a meter (m) and the unit of capacity is the liter (L)

Lesson Check – Page No. 326

Question 1.
Gina filled a tub with 25 quarts of water. What is this amount in gallons and quarts?
_______ gallons _______ quart

Answer:
6 gallons and 1 quart

Explanation:
Gina filled a tub with 25 quarts of water
4quarts = 1 gallon
1 quart = ¼ gallon
25 quarts = 25/4 gallon = 6 gallons and 1 quart
Gina filled a tub with 6 gallons and 1 quart

Question 2.
Four horses are pulling a wagon. Each horse drinks 45,000 milliliters of water each day. How many liters of water will the horses drink in 5 days?
_______ liters

Answer:
900 liters

Explanation:
Four horses are pulling a wagon
Each horse drinks 45,000 milliliters of water each day
Then, four horses drinks 4×45,000 milliliters = 1,80,000
1000 milliliters = 1 liter
Then, 180×1000 = 1,80,000 milliliters = 180 liters
180 x 5 = 900 liters
Horses drink 900 liters of water in 5 days

Spiral Review

Question 3.
The map shows Henry’s town. Each unit represents 1 kilometer. After school, Henry walks to the library. How far does he walk?
Go Math Grade 6 Answer Key Chapter 6 Convert Units of Length img 2
_______ kilometers

Answer:
7 kilometers

Explanation:
The map shows Henry’s town. Each unit represents 1 kilometer. After school, Henry walks to the library.
Each unit represents 1 kilometer
After school, Henry walks to the library
Distance between school and library = 7 kilometers
So, henry walks 7 kilometers from school to library

Question 4.
An elevator travels 117 feet in 6.5 seconds. What is the elevator’s speed as a unit rate?
_______ feet per second

Answer:
18 feet per second

Explanation:
An elevator travels 117 feet in 6.5 seconds.
The elevator’s speed as a unit rate = 117/6.5 = 18 feet per second

Question 5.
Julie’s MP3 player contains 860 songs. If 20% of the songs are rap songs and 15% of the songs are R&B songs, how many of the songs are other types of songs?
_______ songs

Answer:
559 songs

Explanation:
Julie’s MP3 player contains 860 songs
20% of the songs are rap songs = 860×20/100 = 172
15% of the songs are R & B songs = 860×15/100 = 129
Other types of songs = 860 – 172-129 = 559

Question 6.
How many kilometers are equivalent to 3,570 meters?
_______ kilometers

Answer:
3.57 kilometers

Explanation:
1 kilometer = 1000 meters
then,1 meter = 1/1000 kilometer
So, 3570 meters = 3570/1000 kilometer
3570 meters = 3.57 kilometers

Share and Show – Page No. 329

Convert to the given unit.

Question 1.
9 pounds = ? ounces
_______ ounces

Answer:
144 ounces

Explanation:
6 pounds = ? ounces
1 pound = 16 ounces
then, 9 pounds = 9×16 ounces = 144 ounces

Question 2.
3.77 grams = ? dekagram
_______ dekagram

Answer:
0.377 dekagram

Explanation:
3.77 grams = ? dekagram
1 dekagram = 10 grams
1 gram = 1/10 dekagram
Then, 3.77 grams = 3.77/10 dekagram = 0.377 dekagram
So, 3.77 grams = 0.377 dekagram

Question 3.
Amanda’s computer weighs 56 ounces. How many pounds does it weigh?
_______ pounds

Answer:
3.5 pounds

Explanation:
Amanda’s computer weighs 56 ounces
1 pound = 16 ounces
then, 1 ounce = 1/16 pound
So, 56 ounces = 56/16 pounds = 3.5 pounds

Question 4.
A honeybee can carry 40 mg of nectar. How many grams of nectar can a honeybee carry?
_______ grams

Answer:
0.04 grams

Explanation:
A honeybee can carry 40 mg of nectar.
1000 milligrams = 1 gram
1 milligram = 1/1000 grams
Then, 40 milligrams = 40/1000 grams = 0.04 grams
So, the honeybee can carry 0.04 grams of nectar

On Your Own

Convert to the given unit.

Question 5.
4 lb = ? oz
_______ oz

Answer:
64 oz

Explanation:
4lb = ?oz
1 pound (lb) = 16 ounces
then, 4 pounds = 4×16 ounces = 64 ounces

Question 6.
7.13 g = ? cg
_______ cg

Answer:
713 cg

Explanation:
7.13g = ? cg
100 centigrams = 1 gram
Then, 7.13 grams = 100×7.13 = 713 centigrams
So, 7.13 grams = 713 centigrams

Question 7.
3 T = ? lb
_______ lb

Answer:
6000 lb

Explanation:
3T = ?lb
1 ton = 2000 pounds (lb)
then, 3 tons = 3×2000 = 6000 pounds (lb)

Question 8.
The African Goliath frog can weigh up to 7 pounds. How many ounces can the Goliath frog weigh?
_______ ounces

Answer:
112 ounces

Explanation:
The African Goliath frog can weigh up to 7 pounds.
1 pound = 16 ounces
7 pounds = 7×16 = 112 pounds
So, the goliath frog can weigh up to 112 pounds

Question 9.
The mass of a standard hockey puck must be at least 156 grams. What is the minimum mass of 8 hockey pucks in kilograms?
_______ kg

Answer:
1.248 kg

Explanation:
The mass of a standard hockey puck must be at least 156 grams.
1 kilogram = 1000 grams
1 gram = 1/1000 kilogram
then, 156 grams = 156/1000 kilograms = 0.156 kilograms
mass of a hockey puck is 0.156 kilograms
then, the mass of 8 hockey pucks is 8×0.156 = 1.248 kilograms

Practice: Copy and Solve Compare. Write <, >, or =.

Question 10.
250 lb _______ 0.25 T

Answer:
250 lb < 0.25 T

Explanation:
250 lb_______ 0.25 T
1 ton = 2000 pounds(lb)
then, 0.25 tons =0.25×2000 = 500 pounds = 500lb
So, 250 lb < 0.25 T

Question 11.
65.3 hg _______ 653 dag

Answer:
65.3 hg = 653 dag

Explanation:
65.3 hg _______ 653 dag
1 hectogram = 100 grams
Then, 65.3 hectograms = 65.3×100 = 6530 grams
1 dekagram = 10 grams
then, 653 dekagram = 6530 grams
So, 65.3 hectogram = 653 dekagram

Question 12.
5 T _______ 5,000 lb

Answer:
5 T  > 5,000 lb

Explanation:
5 T _______ 5,000 lb
1 ton = 2000 pounds (lb)
5 tons = 5×2000 lb = 10,000 lb
Then, 5 T > 5000 lb

Question 13.
Masses of precious stones are measured in carats, where 1 carat = 200 milligrams. What is the mass of a 50-dg diamond in carats?
_______ carats

Answer:
25 carats

Explanation:
Masses of precious stones are measured in carats, where 1 carat = 200 milligrams.
1 carat = 200 milligrams
6 decigrams = 1 gram
1000 milligrams = 1 gram
So, 10 decigrams = 1000 milligrams
Then, 1 decigram = 100 milligram
2 decigrams = 200 milligrams = 1 carat
then, 50 decigrams = 2×25 decigrams = 25×200 milligrams = 25 carats

Problem Solving + Applications – Page No. 330

Use the table for 14–17.
Go Math Grade 6 Answer Key Chapter 6 Convert Units of Length img 3

Question 14.
Express the weight range for bowling balls in pounds.
_______ lb

Answer:
16 lb

Explanation:
Weight range for bowling balls = 160 to 256 ounces
1 pound = 16 ounces
So, 1 ounce = 1/16 pounds
Then, 160 ounces = 160/16 pounds = 10 pounds
256 ounces = 256/16 pounds = 16 pounds
So, the weight range for bowling balls is 10 to 16 pounds

Question 15.
How many more pounds does the heaviest soccer ball weigh than the heaviest baseball? Round your answer to the nearest hundredth.
_______ lb

Answer:
0.68 lb

Explanation:
Heaviest soccer ball weight = 16 ounces
1 pound = 16 ounces
Heaviest baseball weight = 5.25 ounces
1 pound = 16 ounces
1 ounce = 1/16 pounds
then, 5.25 ounces = 5.25/16 = 0.32 pounds
difference between soccer ball and baseball weight = 1 – 0.32 = 0.68 pounds
So, the soccer ball weight is 0.68 pounds more than the weight of the baseball.

Question 16.
A manufacturer produces 3 tons of baseballs per day and packs them in cartons of 24 baseballs each. If all of the balls are the minimum allowable weight, how many cartons of balls does the company produce each day?
_______ cartons

Answer:
800 cartons

Explanation:
3 tons = 6000 lbs.
Base ball = 5 ounces
16 ounces in 1 pound
6000 × 16 = 96,000
96,000/5 = 19,200
19,200/24 = 800

Question 17.
Communicate Explain how you could use mental math to estimate the number of soccer balls it would take to produce a total weight of 1 ton.
Type below:
____________

Answer:
Soccer balls range 14 to 16 ounces
1 ton = 2000 pounds
then, 1 pound = 1/2000 tons
1 pound = 16 ounces
So, 16 ounces = 1/2000 tons = 0.0005 tons
1 ounce = 1/32000 tons
then, 14 ounces = 14/32000 tons =0.0004375 tons
So, the range of soccer balls is 0.0005 to 0.0004375 tons

Question 18.
The Wilson family’s newborn baby weighs 84 ounces. Choose the numbers to show the baby’s weight in pounds and ounces.
_______ pounds and _______ ounces

Answer:
5 pounds and 4 ounces

Explanation:
The Wilson family’s newborn baby weighs 84 ounces
1 pound = 16 ounces
then, 1 ounce = 1/16 pounds
So, 84 ounces = 84/16 pounds = 5 pounds and 4 ounces

Convert Units of Weight and Mass – Page No. 331

Convert to the given unit.

Question 1.
5 pounds = ? ounces
_______ ounces

Answer:
80 ounces

Explanation:
5 pounds = ? ounces
1 pound = 16 ounces
Then, 5 pounds = 5×16 = 80 ounces
So, 5 pounds = 80 ounces

Question 2.
2.36 grams = ? hectograms
_______ hectograms

Answer:
0.0236 hectograms

Explanation:
1.36 grams = ? hectograms
1 hectogram = 100 grams
1 gram = 1/100 hectograms
then, 2.36 grams = 2.36/100 hectograms = 0.0236 hectograms
So, 2.36 grams = 0.0236 hectograms

Question 3.
30 g = ? dg
_______ dg

Answer:
300 dg

Explanation:
29 g = ? dg
10 decigrams = 1 gram
then, 30 grams = 30×10 decigrams = 300 decigrams
30 grams = 300 decigrams

Question 4.
17.2 hg = ? g
_______ g

Answer:
1720 g

Explanation:
17.2 hg = ? g
1 hectogram = 100 grams
Then, 17.2 hectograms = 17.2×100 = 1720 grams
So, 17.2 hectograms = 1720 grams

Question 5.
400 lb = ? T
_______ T

Answer:
0.2 T

Explanation:
1. 400 lb = ? T
1 ton = 2000 pounds (lb)
400 lb = 2000/5 pounds (lb) = 1/5 tons
So, 400 lb = 0.2 tons

Question 6.
38,600 mg = ? dag
_______ dag

Answer:
3.86 dag

Explanation:
38,600 mg = ? dag
1000 milligrams = 1 gram
1 dekagram = 10 grams
So, 1 gram = 1/10 dekagram
Then, 1000 milligrams = 1/10 dekagrams
1 milligram = 1/10,000 dekagrams
So, 38,600 milligrams = 38,600/10,000 = 3.86 dekagrams
38,600 milligrams = 3.86 dekagrams

Question 7.
87 oz = ? lb ? oz
_______ pounds _______ ounces

Answer:
5 pounds and 7 ounces

Explanation:
87 oz = ? lb ? oz
1 pound = 16 ounces
1 ounce = 1/16 pounds
then, 87 ounces = 87/16 pounds
87 ounces = 5 pounds and 7 ounces

Question 8.
0.65 T = ? lb
_______ lb

Answer:
1300 lb

Explanation:
0.65 T = ?lb
1 ton = 2000 pounds
Then, 0.65 tons = 0.65×2000 = 1300 pounds
0.65 T = 1300 lb

Problem Solving

Question 9.
Maggie bought 52 ounces of swordfish selling for $6.92 per pound. What was the total cost?
$ _______

Answer:
$22.49

Explanation:
Maggie bought 52 ounces of swordfish selling for $6.92 per pound.
Maggie bought 52 ounces of swordfish selling for $6.92 per pound
1 pound = 16 ounces
1 ounce = 1/16 pounds
then, 52 ounces = 52/16 pounds = 3.25 pounds
1 pound cost = $6.92
then, 3.25 pounds cost = $6.92 x 3.25 = $22.49
So, the cost for swordfish is $22.49

Question 10.
Three bunches of grapes have masses of 1,000 centigrams, 1,000 decigrams, and 1,000 grams, respectively. What is the total combined mass of the grapes in kilograms?
_______ kg

Answer:
1.11 kg

Explanation:
Three bunches of grapes have masses of 1,000 centigrams, 1,000 decigrams, and 1,000 grams, respectively.
Three bunches of grapes have masses of 1,000 centigrams, 1,000 decigrams, and 1,000 grams
100 centigrams = 1 gram
then, 1000 centigrams = 10×100 centigrams = 10 grams
1 kilogram = 1000 grams
So, 1 gram = 1/1000 kilograms
Then, 10 grams = 10/1000 = 1/100 kilograms = 0.01 kilograms
10 decigrams = 1 gram
then, 100×10 decigrams = 100×1 gram = 100 grams
1000 grams = 1 kilogram
Then, 100 grams = 1/10 kilograms = 0.1 kilograms
1000 grams = 1 kilogram
Total weight of the grapes = 1 + 0.1 + 0.01 = 1.11 kilograms

Question 11.
Explain how you would find the number of ounces in 0.25T.
Type below:
____________

Answer:
number of ounces in 0.25T
1 ton = 2000 pounds
then, 1 pound = 1/2000 tons
1 pound = 16 ounces
so, 16 ounces = 1/2000 tons
then, 1 ton = 16×2000 ounces = 32000 ounces
So, 0.25 tons = 0.25×32000 ounces = 8000 ounces
8000 ounces = 0.25 T

Lesson Check – Page No. 332

Question 1.
The mass of Denise’s rock sample is 684 grams. The mass of Pauline’s rock sample is 29,510 centigrams. How much greater is the mass of Denise’s sample than Pauline’s sample?
_______ centigrams

Answer:
38900 centigrams

Explanation:
The mass of Denise’s rock sample is 684 grams
The mass of Pauline’s rock sample is 29,510 centigrams
100 centigrams = 1 gram
1 centigram = 1/100 gram
then, 29,510 centigrams = 29,510/100 grams = 295.1 grams
So, the mass of Pauline’s rock sample is 295.1 grams
By comparing Denise’s rock sample with Pauline’s rock sample
684 – 295 = 389
The mass of Denise’s rock sample is 389 grams more than the mass of Pauline’s rock sample
389 grams = 38900 centigrams

Question 2.
A sign at the entrance to a bridge reads: Maximum allowable weight 2.25 tons. Jason’s truck weighs 2,150 pounds. How much additional weight can he carry?
_______ pounds

Answer:
2,350 pounds

Explanation:
A sign at the entrance to a bridge reads: Maximum allowable weight 2.25 tons
Jason’s truck weighs 2,150 pounds
1 ton = 2000 pounds
then, 2.25 tons = 2.25×2000 = 4500 pounds
So, maximum allowable weight = 4500 pounds
4500 – 2150 = 2350
So, Jason can carry an additionally 2350 pounds’ weight

Spiral Review

Question 3.
There are 23 students in a math class. Twelve of them are boys. What is the ratio of girls to total number of students?
Type below:
____________

Answer:
11 : 23

Explanation:
There are 23 students in a math class. Twelve of them are boys.
Number of students in a math class = 23
Number of boys in a class = 12
Number of girls in a class = 23-12 = 11
Then, the ratio of girls to the total number of students = 11/23

Question 4.
Miguel hiked 3 miles in 54 minutes. At this rate, how long will it take him to hike 5 miles?
_______ minutes

Answer:
90 minutes

Explanation:
Miguel hiked 3 miles in 54 minutes.
Then, time for 5 miles = 5×54/3 = 90 minutes
So, Miguel hikes 5 miles in 90 minutes

Question 5.
Marco borrowed $150 from his brother. He has paid back 30% so far. How much money does Marco still owe his brother?
$ _______

Answer:
$60

Explanation:
Marco borrowed $150 from his brother
He has paid back 30% of amount = 30/100 (150) = $45
Remaining amount = 150 -45 = 60
So, still $60 amount Marco need to give his brother

Question 6.
How many milliliters are equivalent to 2.7 liters?
_______ milliliters

Answer:
2,700 milliliters

Explanation:
2.7 liters
1000 milliliters = 1 liter
Then, 2.7 liters = 2.7 x 1000 = 2700 milliliters
So, 2,700 milliliters are equivalent to 2.7 liters

Mid-Chapter Checkpoint – Vocabulary – Page No. 333

Choose the best term from the box to complete the sentence.
Go Math Grade 6 Answer Key Chapter 6 Convert Units of Length img 4

Question 1.
A _____ is a rate in which the two quantities are equal, but use different units.
Type below:
____________

Answer:
Conversion factor

Question 2.
_____ is the amount a container can hold.
Type below:
____________

Answer:
Capacity

Concepts and Skills

Convert units to solve.

Question 3.
A professional football field is 160 feet wide. What is the width of the field in yards?
_____ \(\frac{□}{□}\) yd

Answer:
53\(\frac{1}{3}\) yd

Explanation:
A professional football field is 160 feet wide
3feet = 1 yard
Then, 160 feet = 160/3 = 53.33
So, the width of football field is 53.33 yards
160/3 = 53 1/3

Question 4.
Julia drinks 8 cups of water per day. How many quarts of water does she drink per day?
_____ quarts

Answer:
2 quarts

Explanation:
Julia drinks 8 cups of water per day.
4 cups = 1 quart
Then, 8 cups = 8/4 = 2 quarts
So, Julia drinks 2 quarts of water per day

Question 5.
The mass of Hinto’s math book is 4,458 grams. What is the mass of 4 math books in kilograms?
_____ kilograms

Answer:
17.832 kilograms

Explanation:
The mass of Hinto’s math book is 4,458 grams
1kilogram = 1000 grams
Then, 4,458 grams = 4,458/1000 = 4.458 kilograms
Then, the mass of 4 math books = 4×4.458 = 17.832 kilograms
The mass of 4 math books is 17.832 kilograms

Question 6.
Turning off the water while brushing your teeth saves 379 centiliters of water. How many liters of water can you save if you turn off the water the next 3 times you brush your teeth?
_____ liters

Answer:
11.37 liters

Explanation:
Turning off the water while brushing your teeth saves 379 centiliters of water
100centiliters = 1 liter
Then, 379 centiliters = 379/100 = 3.79 liters
if you turn off the water the next 3 times = 3×3.79 liters = 11.37 liters
So, you can save 11.37 liters of water when you turn off the water for 3 times

Convert to the given unit.

Question 7.
34.2 mm = ? cm
_____ cm

Answer:
3.42 cm

Explanation:
34.2 mm = ? cm
1000 millimeters = 1 meter
100centimeters = 1 meter
so, 1000 millimeters = 100 centimeters
then, 10 millimeters = 1 centimeter
then, 34.2 millimeters = 34.2/10 = 3.42 centimeters
So, 34.2 mm = 3.42 cm

Question 8.
42 in. = ? ft
_____ \(\frac{□}{□}\) ft

Answer:
3\(\frac{1}{2}\) ft

Explanation:
41 in. = ? ft
12 inches = 1 foot
then, 42 inches = 42/12 = 3.5 feet
So, 42 in = 3.5 ft
42/12 = 3 1/2

Question 9.
1.4 km = ? hm
_____ hm

Answer:
140 hm

Explanation:
1.4 km = ? hm
1 kilometer = 1000 meters
1 hectometer = 100 meters
So, 1 meter = 0.001 kilometers
1 meter = 0.01 hectometers
Now, 0.001 kilometer = 0.01 hectometer
That is 0.1 kilometer = 1 hectometer
Then, 1.4 kilometer = 1.4/0.1 = 140 hectometers
So, 1.4 km = 140 hm

Question 10.
4 gal = ? qt
_____ qt

Answer:
16 qt

Explanation:
4gal = ?qt
1gallon = 4 quarts
Then, 4 gallons = 4×4 = 16 quarts
So, 4 gal = 16 qt

Question 11.
53 dL = ? daL
_____ daL

Answer:
0.53 daL

Explanation:
53 dL = ? daL
10deciliters = 1 liter
1 dekaliter = 10 liters that is 0.1 dekaliters = 1 liter
So, 10 dL = 0.1 daL
Then, 53 dL = 53×0.1/10 =0.53 daL
So, 53 dL = 0.53 daL

Question 12.
28 c = ? pt
_____ pt

Answer:
14 pt

Explanation:
28 c = ?pt
1 cups = 1pint
then, 28 cups = 28/2 = 14 pints
So, 28 c = 14 pt

Page No. 334

Question 13.
Trenton’s laptop is 32 centimeters wide. What is the width of the laptop in decimeters?
_____ dm

Answer:
3.2 dm

Explanation:
Trenton’s laptop is 32 centimeters wide.
100 centimeters = 1 meter
10decimeters = 1 meter
So, 100 centimeters = 10 decimeters
Then, 32 centimeters = 32×10/100 = 3.2 decimeters
So, the width of the laptop is 3.2 decimeters

Question 14.
A truck is carrying 8 cars weighing an average of 4,500 pounds each. What is the total weight in tons of the cars on the truck?
_____ tons

Answer:
18 tons

Explanation:
A truck is carrying 8 cars weighing an average of 4,500 pounds each.
So, total weight = 8 x 4500 pounds = 36,000 pounds
2000 pounds = 1 ton
Then, 36,000 pounds = 36,000 / 2000 = 18 tons
So, total weight of the cars in truck is 18 tons

Question 15.
Ben’s living room is a rectangle measuring 10 yards by 168 inches. By how many feet does the length of the room exceed the width?
_____ feet

Answer:
16 feet

Explanation:
Ben’s living room is a rectangle measuring 10 yards by 168 inches.
12inches = 1 foot
Then, 168 inches = 168/12 = 14 feet
1 yard = 3 feet
then, 10 yards = 10×3 = 30 feet
30-14 = 16 feet
So, the length of the room exceeds 16 feet in width

Question 16.
Jessie served 13 pints of orange juice at her party. How many quarts of orange juice did she serve?
_____ quarts

Answer:
6.5 quarts

Explanation:
Jessie served 13 pints of orange juice at her party
1 pints = 1 quart
then, 13 pints = 13/2 = 6.5 quarts
So, Jessie served 6.5 quarts of orange juice at her party

Question 17.
Kaylah’s cell phone has a mass of 50,000 centigrams. What is the mass of her phone in grams?
_____ grams

Answer:
500 grams

Explanation:
Kaylah’s cell phone has a mass of 50,000 centigrams
100 centigrams = 1 gram
then, 50,000 centigrams = 50,000/100 = 500 grams
So, the mass of Kaylah’s phone is 500 grams

Share and Show – Page No. 337

Question 1.
A dripping faucet leaks 12 gallons of water per day. How many gallons does the faucet leak in 6 days?
_____ gallons

Answer:
72 gallons

Explanation:
A dripping faucet leaks 12 gallons of water per day
Then, faucet leaks how many gallons of water per 6 days = 12 x 6 = 72 gallons

Question 2.
Bananas sell for $0.44 per pound. How much will 7 pounds of bananas cost?
$ _____

Answer:
$3.08

Explanation:
Bananas sell for $0.44 per pound
1 pound banana cost is $0.44
then, 7 pounds bananas cost is = 7 x 0.44 = $3.08

Question 3.
Grizzly Park is a rectangular park with an area of 24 square miles. The park is 3 miles wide. What is its length in miles?
_____ miles

Answer:
8 miles

Explanation:
Grizzly Park is a rectangular park with an area of 24 square miles
The park is 3 miles wide
Rectangular park area = length x breadth
That is 24 = 3 x b
So, breadth = 8 miles
The rectangular park length is 8 miles

On Your Own

Multiply or divide the quantities.

Question 4.
\(\frac{24 \mathrm{kg}}{1 \mathrm{min}}\) × 15 min
_____ kg

Answer:
6 kg

Explanation:
24kg1min × 15 min
24 kg / 1min x 15 min
60 min = 1 hour
Then, 15 min = 15/60 = ¼ hours
24 kg x 1/ 4 = 6 kg

Question 5.
216 sq cm÷8 cm
_____ cm

Answer:
27 cm

Explanation:
216 sq cm ÷ 8 cm
216 sq cm/ 8 cm = 27 cm

Question 6.
\(\frac{17 \mathrm{L}}{1 \mathrm{hr}}\) × 9 hr
_____ L

Answer:
153 L

Explanation:
17L1hr x 9 hr
17L/1hr x 9 hr = 153 L

Question 7.
The rectangular rug in Marcia’s living room measures 12 feet by 108 inches. What is the rug’s area in square feet?
_____ square feet

Answer:
108 square feet

Explanation:
The rectangular rug in Marcia’s living room measures 12 feet by 108 inches
1 foot = 12 inches
108 inches = 108/12 = 9 feet
12 x 9 = 108 square feet
Are of rug is 108 square feet

Question 8.
Make Sense of Problems A box-making machine makes cardboard boxes at a rate of 72 boxes per minute. How many minutes does it take to make 360 boxes?
_____ minutes

Answer:
5 minutes

Explanation:
A box-making machine makes cardboard boxes at a rate of 72 boxes per minute
Then, time for 360 boxes = 360/72 = 5 minutes
So, it takes 5 minutes’ time to make 360 boxes

Question 9.
The area of an Olympic-size swimming pool is 1,250 square meters. The length of the pool is 5,000 centimeters. Select True or False for each statement.
9a. The length of the pool is 50 meters.
9b. The width of the pool is 25 meters.
9c. The area of the pool is 1.25 square kilometers
9a. ____________
9b. ____________
9c. ____________

Answer:
9a. True
9b. True
9c. True

Explanation:
The area of an Olympic-size swimming pool is 1,250 square meters
The length of the pool is 5,000 centimeters
100centimeters = 1meter
Then, 5000 centimeters = 5000/100 = 50 meters
Areas of the swimming pool = length x width
1250 square meters = 50 length x 25 width
Then, width = 25 meters
1000 meters = 1 kilometer
then, 1250 square meters = 1250/1000 = 1.25 square meters

Make Predictions – Page No. 338

A prediction is a guess about something in the future. A prediction is more likely to be accurate if it is based on facts and logical reasoning.

The Hoover Dam is one of America’s largest producers of hydroelectric power. Up to 300,000 gallons of water can move through the dam’s generators every second. Predict the amount of water that moves through the generators in half of an hour.
Go Math Grade 6 Answer Key Chapter 6 Convert Units of Length img 5
Use what you know about transforming units to make a prediction.
You know the rate of the water through the generators, and you are given an amount of time. Rate of flow:
\(\frac{300,000 \text { gallons }}{1 \text { sec }}\); time: \(\frac{1}{2}\) hr
You want to find the amount of water. Amount of water : ? gallons
Convert the amount of time to seconds to match the units in the rate. \(\frac{1}{2}\) hr=30 min
Multiply the rate by the amount of time to find the amount of water. \(\frac{300,000 \text { gallons }}{1 \mathrm{sec}} \times \frac{1,800 \mathrm{sec}}{1}\) = 540,000,000 gal
So, a good prediction of the amount of water that moves through the generators in half of an hour is 540,000,000 gallons.
Transform units to solve.

Question 10.
An average of 19,230 people tour the Hoover Dam each week. Predict the number of people touring the dam in a year.
_____ people

Answer:
999,960 people

Explanation:
An average of 19,230 people tour the Hoover Dam each week
Number of weeks per year = 52
Then, total number of people tour the hoover dam in the year = 52 x 19, 230 = 999,960
So, 999,960 people touring the hoover dam per year

Question 11.
The Hoover Dam generates an average of about 11,506,000 kilowatt-hours of electricity per day. Predict the number of kilowatt-hours generated in 7 weeks.
_____ kilowatt-hours

Answer:
563,794 kilowatt-hours

Explanation:
The Hoover Dam generates an average of about 11,506,000 kilowatt-hours of electricity per day
1 week = 7 days
7weeks = 7 × 7 = 49 days
Then, Hoover Dam generated electricity per 7 weeks = 49 × 11,506,000 = 563,794,000
So, the total number of kilowatt-hours generated in 7 weeks by the Hoover Dam is 563,794,000

Transform Units – Page No. 339

Multiply or divide the quantities.

Question 1.
\(\frac{62 \mathrm{g}}{1 \mathrm{day}}\) × 4 days
_____ g

Answer:
248 g

Explanation:
62g1day × 4 days
62 g÷1 day × 4 days
Then, 62 g × 4 = 248 g

Question 2.
322 sq yd ÷ 23 yd
_____ yd

Answer:
14 yd

Explanation:
322 sqyd ÷ 23 yd
322 sqyd / 23 yd = 14 sq

Question 3.
\(\frac{128 \mathrm{kg}}{1 \mathrm{hr}}\) × 10 hr
_____ kg

Answer:
1,280 kg

Explanation:
128kg1hr × 10 hr
128 kg/1hr * 10hr
So, 1,280 kg

Question 4.
136 sq km ÷ 8 km
_____ km

Answer:
17 km

Explanation:
136 sq km ÷ 8 km
136 sq km / 8 km
136 sq / 8 = 17

Question 5.
\(\frac{88 \mathrm{lb}}{1 \mathrm{day}}\) × 12 days
_____ lb

Answer:
1,056 lb

Explanation:
88lb1day × 12 days
88lb / 1 day × 12days
That is 88lb × 12 = 1,056 lb

Question 6.
154 sq mm ÷ 11 mm
_____ mm

Answer:
14  mm

Explanation:
154 sq mm ÷ 11 mm
154 sq / 11 = 14

Question 7.
\(\frac{\$ 150}{1 \mathrm{sq} \mathrm{ft}}\) × 20 sq ft
$ _____

Answer:
$30,020 sqft

Explanation:
$1501sqft × 20 sqft
Multiplication of 1501 and 20 is
30,020
That is $1501sqft x 20 sqft = $30,020 sqft

Question 8.
234 sq ft÷18 ft
_____ ft

Answer:
13 ft

Explanation:
234 sq ft÷18 ft
234 sq / 18 = 13

Problem Solving

Question 9.
Green grapes are on sale for $2.50 a pound. How much will 9 pounds cost?
$ _____

Answer:
$22.5

Explanation:
Green grapes are on sale for $2.50 a pound
1 pound = $2.50
then, 9 pounds cost = 9*$2.50 = $22.5
green grapes cost for 9 pounds is $22.5

Question 10.
A car travels 32 miles for each gallon of gas. How many gallons of gas does it need to travel 192 miles?
_____ gallons

Answer:
6 gallons

Explanation:
A car travels 32 miles for each gallon of gas
Then, 192 miles is = 192/ 32 = 6 gallons of gas
So, total 6 gallons of gas is required to travel 192 miles

Question 11.
Write and solve a problem in which you have to transform units. Use the rate 45 people per hour in your question.
Type below:
____________

Answer:
A fast-food restaurant is trying to find out how many customers they had in the last 3 hours, and they know they get 45 people per hour. How many customers were served in the last 3 hours? The answer is 135 people.

Lesson Check – Page No. 340

Question 1.
A rectangular parking lot has an area of 682 square yards. The lot is 22 yards wide. What is the length of the parking lot?
_____ yards

Answer:
31 yards

Explanation:
A rectangular parking lot has an area of 682 square yards
Width of the parking lot = 22 yards wide
Area = length *width
682 square yards= length * 22 yards wide
So, length = 682 square yards / 22 yards = 31 yards
Then, length of the parking lot = 31 yards

Question 2.
A machine assembles 44 key chains per hour. How many key chains does the machine assemble in 11 hours?
_____ key chains

Answer:
484 key chains

Explanation:
A machine assembles 44 key chains per hour
Then, the machine assembles key chains per 11 hours = 11*44 = 484 key chains
So, the machine assembles totally 484 key chains in 11 hours

Spiral Review

Question 3.
Three of these ratios are equivalent to \(\frac{8}{20}\). Which one is NOT equivalent?
\(\frac{2}{5} \quad \frac{12}{24} \quad \frac{16}{40} \quad \frac{40}{100}\)
\(\frac{□}{□}\)

Answer:
\(\frac{8}{20}\)

Explanation:
The below mentioned ratios are equivalent to 8/20
i. 2/5
Multiply the numerator and denominator with 4
That is (2*4)/(5*4) = 8/20
ii. 12/24
Divide the numerator and denominator with 6
That is (12÷6)/(24÷6) = 2/4
Now, multiply the numerator and denominator with 4
That is (2*4)/(4*4) = 8/16
So, 12/14 is not equal to 8/20
iii. 16/40
Divide the numerator and denominator with 2
That is, (16÷2)/(40÷2) = 8/20
iv. 40/100
Divide the numerator and denominator with 5
That is (40÷5)/(100÷5) = 8/20

Question 4.
The graph shows the money that Marco earns for different numbers of days worked. How much money does he earn per day?
Go Math Grade 6 Answer Key Chapter 6 Convert Units of Length img 6
$ _____

Answer:
$80

Explanation:
Total number of days worked = 7
Total earned money = 560 dollars
560 / 7 = 80 dollars per day

Question 5.
Megan answered 18 questions correctly on a test. That is 75% of the total number of questions. How many questions were on the test?
_____ questions

Answer:
24 questions

Explanation:
Megan answered 18 questions correctly
That is 75% of the total number of questions = 18
Then, 100% of the questions = 18*100/75 = 24
So, the total number of questions on the test = 24 questions

Share and Show – Page No. 343

Question 1.
Mariana runs at a rate of 180 meters per minute. How far does she run in 5 minutes?
_____ meters

Answer:
900 meters

Explanation:
Mariana runs at a rate of 180 meters per minute
Then, Mariana runs per 5 minutes = 5×180 = 900 meters
So, Mariana runs 900 meters per 5 minutes

Question 2.
What if Mariana runs for 20 minutes at the same speed? How many kilometers will she run?
_____ kilometers

Answer:
3.6 kilometers

Explanation:
From the given information
Marians runs at a rate of 180 meters per minute
Then the speed of Mariana = 180/1 = 180 meters per minute
If Mariana runs 20 minutes then the covered distance = 20×180 = 3600 meters
1000 meters = 1 kilometer
Then, 3600 meters = 3600/1000 = 3.6 kilometers
So, Mariana runs 3.6 kilometers in 20 minutes

Question 3.
A car traveled 130 miles in 2 hours. How fast did the car travel?
_____ miles per hour

Answer:
65 miles per hour

Explanation:
A car travelled 130 miles in 2 hours
Then the speed of the car = Distance/Time
That is, Speed of the car = 130 miles/ 2 hours = 65 miles per hour
So, the car travels 65 miles per hour

Question 4.
A subway car travels at a rate of 32 feet per second. How far does it travel in 16 seconds?
_____ feet

Answer:
512 feet

Explanation:
A subway car travels at a rate of 32 feet per second
1 second = 32 feet
then, 16 seconds = 16 x 32/1 = 512 feet
So, a subway car travels 512 feet per 16 seconds

Question 5.
A garden snail travels at a rate of 2.6 feet per minute. At this rate, how long will it take for the snail to travel 65 feet?
_____ minutes

Answer:
25 minutes

Explanation:
A garden snail travels at a rate of 2.6 feet per minute
So, 2.6 feet = 1 minute
Then, 65 feet = 65/2.6 = 650/26 = 25 minutes
So, the snail travels 65 feet in 25 minutes

Question 6.
A squirrel can run at a maximum speed of 12 miles per hour. At this rate, how many seconds will it take the squirrel to run 3 miles?
_____ seconds

Answer:
900 seconds

Explanation:
A squirrel can run at a maximum speed of 12 miles per hour
1 hour = 3600 seconds
So, the squirrel can run 12 miles in 3600 seconds
Then, the squirrel can run 3 miles in 3×3600/12 = 900 seconds
So, the squirrel can take 900 seconds of time to run 3 miles

Question 7.
A cyclist rides 8 miles in 32 minutes. What is the speed of the cyclist in miles per hour?
_____ miles per hour

Answer:
15 miles per hour

Explanation:
A cyclist rides 8 miles in 32 minutes
32minutes = 8 miles
Then, 60 minutes = 60×8/32 = 15 miles
So, a cyclist rides 15 miles in 60 minutes that is one hour
So, the speed of the cyclist per hour = 15 miles/ 1 = 15 miles per hour

Share and Show – Page No. 344

On Your Own

Question 8.
A pilot flies 441 kilometers in 31.5 minutes. What is the speed of the airplane?
_____ kilometers per minute

Answer:
14 kilometers per minute

Explanation:
From the given information
A pilot flies 441 kilometers in 31.5 minutes
Speed = Distance / Time
Here, distance = 441 kilometers
Time = 31.5 minutes
Speed of the airplane = 441/31.5 = 4410/315 = 14 kilometers per minute

Question 9.
Chris spent half of his money on a pair of headphones. Then he spent half of his remaining money on CDs. Finally, he spent his remaining $12.75 on a book. How much money did Chris have to begin with?
$ _____

Answer:
$51

Explanation:
Total money with the Chris= x amount
Chris spent half of his money on a pair of headphones = x/2
Then he spent half of his remaining money on CDs = x/4
Finally, he spent his remaining $12.75 on a book
So, total amount x = x/2+x/4+$12.75
$12.75 = (x-x/2-x/4)
= (4x-2x-x)/4
$12.75 = x/4
Then, x = $12.75×4 = $51
So, Chris have to begin with $51

Question 10.
André and Yazmeen leave at the same time and travel 75 miles to a fair. André drives 11 miles in 12 minutes. Yazmeen drives 26 miles in 24 minutes. If they continue at the same rates, who will arrive at the fair first? Explain.
____________

Answer:
André and Yazmeen leave at the same time and travel 75 miles to a fair
André drives 11 miles in 12 minutes
So, Andre can reach 75 miles in = 75×12/11
That is, Andre can travel 75 miles in 81 minutes
Yazmeen drives 26 miles in 24 minutes
So, Yazmeen can reach 75 miles in = 75×24/26 = 69 minutes
That means, Yazmeen can reach 75 miles in 69 minutes
So, Yazmeen can reach the fair first

Question 11.
Make Arguments Bonnie says that if she drives at an average rate of 40 miles per hour, it will take her about 2 hours to drive 20 miles across town. Does Bonnie’s statement make sense? Explain.
____________

Answer:
Make Arguments Bonnie says that if she drives at an average rate of 40 miles per hour, it will take her about 2 hours to drive 20 miles across town
Speed of the Bonnie = 40 miles per hour
Then, Bonnie can cover the distance in 2 hours = 2×40 = 80 miles
So, Bonnie statement is wrong

Question 12.
Claire says that if she runs at an average rate of 6 miles per hour, it will take her about 2 hours to run 18 miles. Do you agree or disagree with Claire? Use numbers and words to support your answer.
____________

Answer:
Claire says that if she runs at an average rate of 6 miles per hour, it will take her about 2 hours to run 18 miles
Claire runs in 1 hour = 6 miles
Then, Claire runs in 2 hours = 2×6 = 12 miles
So, the Claire statement is wrong

Problem Solving Distance, Rate, and Time Formulas – Page No. 345

Read each problem and solve.

Question 1.
A downhill skier is traveling at a rate of 0.5 mile per minute. How far will the skier travel in 18 minutes?
_____ miles

Answer:
9 miles

Explanation:
A downhill skier is traveling at a rate of 0.5 miles per minute
1 minute = 0.5 mile
then, 18 minutes = 18×0.5 = 9 miles
So, the skier travel 9 miles in 18 minutes

Question 2.
How long will it take a seal swimming at a speed of 8 miles per hour to travel 52 miles?
_____ hours

Answer:
6.5 hours

Explanation:
A seal swimming at a speed of 8 miles per hour
Then,52 miles = 52/8 = 6.5 hours
So, A seal swimming can travel 52 miles in 6.5 hours

Question 3.
A dragonfly traveled at a rate of 35 miles per hour for 2.5 hours. What distance did the dragonfly travel?
_____ miles

Answer:
87.5 miles

Explanation:
A dragonfly traveled at a rate of 35 miles per hour for 2.5 hours
That means, 1 hour = 35 miles
Then, 2.5 hours = 2.5×35 = 87.5 miles
So, a dragonfly travels 87.5 miles in 2.5 hours

Question 4.
A race car travels 1,212 kilometers in 4 hours. What is the car’s rate of speed?
_____ kilometers per hour

Answer:
303 kilometers per hour

Explanation:
A race car travels 1,212 kilometers in 4 hours
Speed = Distance/ Time
Here, distance = 1212 kilometers
Time = 4 hours
Then, Speed of the race car = 1212/4 = 303 kilometers per hour

Question 5.
Kim and Jay leave at the same time to travel 25 miles to the beach. Kim drives 9 miles in 12 minutes. Jay drives 10 miles in 15 minutes. If they both continue at the same rate, who will arrive at the beach first?
____________

Answer:
Kim reaches the beach first

Explanation:
Kim and Jay leave at the same time to travel 25 miles to the beach
Kim drives 9 miles in 12 minutes
Then, Kim travels 25 miles in = 25×12/9 = 33 minutes
Jay drives 10 miles in 15 minutes
Then, Jay travels 25 miles in = 25×15/10 = 37.5 minutes
So, Kim reaches the beach first

Question 6.
Describe the location of the variable d in the formulas involving rate, time, and distance.
Type below:
____________

Answer:
Formula Distance = Rate x Time
Distance (d) = Rate x Time

Lesson Check – Page No. 346

Question 1.
Mark cycled 25 miles at a rate of 10 miles per hour. How long did it take Mark to cycle 25 miles?
_____ hours

Answer:
2.5 hours

Explanation:
Mark cycled 25 miles at a rate of 10 miles per hour
That means, 10 miles = 1 hour
Then, 25 miles = 25/10 =2.5 hours
So, Mark take 2.5 hours to cycle 25 miles

Question 2.
Joy ran 13 miles in 3 \(\frac{1}{4}\) hours. What was her average rate?
_____ miles per hour

Answer:
4 miles per hour

Explanation:
Joy ran 13 miles in 3 ¼ hours
3 ¼ = 13/4 = 3.25 hours
Then, the average rate of the Joy = 13/3.25 hours = 4 miles per hour

Spiral Review

Question 3.
Write two ratios that are equivalent to \(\frac{9}{12}\).
Type below:
____________

Answer:
3/4 and 18/24

Explanation:
Equivalent ratios of 9/12 is 3/4 and 18/24
Multiply the numerator and denominator of ¾ with 3
That is 3×3/4×3 = 9/12
Divide the numerator and denominator of 18/24 with 2
That is (18/2)/(24/2) = 9/12

Question 4.
In the Chang family’s budget, 0.6% of the expenses are for internet service. What fraction of the family’s expenses is for internet service? Write the fraction in simplest form.
\(\frac{□}{□}\)

Answer:
\(\frac{3}{500}\)

Explanation:
In the Chang family’s budget, 0.6% of the expenses are for internet service
That is 0.6% = 0.6/100 = 6/1000 = 3/500
So, 3/500 part of the family’s expenses is used for internet service

Question 5.
How many meters are equivalent to 357 centimeters?
_____ meters

Answer:
3.57 meters

Explanation:
357 ntimeters
100centimeters = 1 meter
Then, 357 centimeters = 357/100 = 3.57 meters
3.57 meters is equivalent to 357 centimeters

Question 6.
What is the product of the two quantities shown below?
\(\frac{60 \mathrm{mi}}{1 \mathrm{hr}}\) × 12 hr
_____ miles

Answer:
720 miles

Explanation:
60 mi/1hr x 12 hr
That is, 60 milesx12 = 720 miles
So, the equivalent quantity of 60mi/1hr x 12hr is 720 miles

Chapter 6 Review/Test – Page No. 347

Question 1.
A construction crew needs to remove 2.5 tons of river rock during the construction of new office buildings
The weight of the rocks is ____________ pounds

Answer:
The weight of the rocks is 5,000 pounds

Explanation:
A construction crew needs to remove 2.5 tons of river rock during the construction of new office buildings
1 Ton = 2000 pounds
Then, 2.5 Tons = 2.5×2000
= 25×200 = 5000 pounds
So, the weight of the rocks is 5000 pounds

Question 2.
Select the conversions that are equivalent to 10 yards. Mark all that apply
Options:
a. 20 feet
b. 240 inches
c. 30 feet
d. 360 inches

Answer:
c. 30 feet
d. 360 inches

Explanation:
a. 20 feet
3feet = 1 yard
Then, 20 feet = 20/3 yard
b. 240 inches
36 inches = 1 yard
Then, 240 inches = 240/36 = 6 yards
c. 30 feet
3feet = 1 yard
Then, 30 feet =30/3 = 10 yards
d. 360inches
36 inches = 1 yard
Then, 360 inches = 360/36 = 10 yards
So, 30 feet and 360 inches are equivalent to 10 yards

Question 3.
Meredith runs at a rate of 190 meters per minute. Use the formula d=r×t to find how far she runs in 6 minutes.
_____ meters

Answer:
1,140 meters

Explanation:
Meredith runs at a rate of 190 meters per minute
Formula d = r x t
Here, d= 190 meters, t = 1 minute
Then, r = 190/1 = 190 meters per minute
Now, t = 6 minutes and r = 190 meters per minute
Then d = 190 x 6 = 1,140 meters

Question 4.
The table shows data for 4 cyclists during one day of training. Complete the table by finding the speed for each cyclist. Use the formula r = d ÷ t.
Go Math Grade 6 Answer Key Chapter 6 Convert Units of Length img 7
Type below:
____________

Answer:
D = RxT
Alisha
36 = Rx3
Then, Rate of Alisha =36/3 =12 miles per hour
Jose
39 = Rx3
Then, Rate of Jose = 39/3 = 13 miles per hour
Raul
40 = Rx4
Then, Rate of Raul = 40/4 = 10 miles per hour
Ruthie
22= Rx2
Then, Rate of Ruthie = 22/2 = 11 miles per hour

Page No. 348

Question 5.
For numbers 5a–5c, choose <, >, or =.
5a. 5 kilometers Ο 5,000 meters
5b. 254 centiliters Ο 25.4 liters
5c. 6 kilogram Ο 600 gram
5 kilometers _____ 5,000 meters
254 centiliters _____ 25.4 liters
6 kilogram _____ 600 gram

Answer:
5a. 5 kilometers Ο 5,000 meters
5b. 254 centiliters Ο 25.4 liters
5c. 6 kilogram Ο 600 gram
5 kilometers = 5,000 meters
254 centiliters < 25.4 liters
6 kilogram > 600 gram

Explanation:
a. 5 kilometers —— 5000 meters
1 kilometer = 1000 meters
then,5 kilometers 5×1000 = 5000 meters
So, 5 kilometers = 5000 meters
b. 254 centiliters ——25.4 liters
100centiliters = 1 liter
Then, 254 centiliters = 254/100 = 2.54 liters
So, 254 centiliters < 25.4 liters
c. 6 kilograms —– 600 grams
1kilogram = 1000 grams
Then, 6 kilograms = 6000 grams
So, 6 kilograms > 600 grams

Question 6.
A recipe calls for 16 fluid ounces of light whipping cream. If Anthony has 1 pint of whipping cream in his refrigerator, does he have enough for the recipe? Explain your answer using numbers and words.
____________

Answer:
A recipe calls for 16 fluid ounces of light whipping cream
8 fluid ounces = 1 cup
So, 16 fluid ounces = 2 cups = 1 pint
If Anthony has 1 pint of whipping cream in his refrigerator, then it is enough for the recipe

Question 7.
For numbers 7a–7d, choose <, >, or =.
7a. 43 feet Ο 15 yards
7b. 10 pints Ο 5 quarts
7c. 5 tons Ο 5,000 pounds
7d. 6 miles Ο 600 yards
43 feet _____ 15 yards
10 pints _____ 5 quarts
5 tons _____ 5,000 pounds
6 miles _____ 600 yards

Answer:
43 feet < 15 yards
10 pints = 5 quarts
5 tons > 5,000 pounds
6 miles > 600 yards

Explanation:
a. 43 feet —- 15 yards
3feet = 1 yard
Then, 43 feet = 43/3 = 14.3 yards
So, 43 feet < 15 yards
b. 10 pints —- 5 quarts
1 pints = 1 quart
then, 10 pints = 10/2 = 5 quarts
So, 10 pints = 5 quarts
c. 5 tons —– 5000 pounds
1 ton = 2000 pounds
then, 5 tons = 5×2000 = 10,000 pounds
So, 5 tons > 5000 pounds
d. 6 miles —- 600 yards
1 mile =1760 yards
then, 6 miles = 6×1760 = 10,560yards
So, 6 miles > 600 yards

Question 8.
The distance from Caleb’s house to the school is 1.5 miles, and the distance from Ashlee’s house to the school is 3,520 feet. Who lives closer to the school, Caleb or Ashlee? Use numbers and words to support your answer.
____________

Answer:
There are 5280 feet in one mile.
So, you need to change the miles to feet.
1.5 x 5280 = 7920.
7920 > 3520
So, Ashley lives closer.

Page No. 349

Question 9.
Write the mass measurements in order from least to greatest.
Go Math Grade 6 Answer Key Chapter 6 Convert Units of Length img 8
Type below:
____________

Answer:
7.4 kilograms, 7.4 centigrams, 7.4 decigrams

Question 10.
An elephant’s heart beats 28 times per minute. Complete the product to find how many times its heart beats in 30 minutes
Go Math Grade 6 Answer Key Chapter 6 Convert Units of Length img 9
Type below:
____________

Answer:
840 beats

Explanation:
An elephant’s heart beats 28 times per minute
Then, elephant’s heart beats in 30 minutes = 28 x 30 = 840
So, an elephant’s heartbeat is 840 times in 30 minutes

Question 11.
The length of a rectangular football field, including both end zones, is 120 yards. The area of the field is 57,600 square feet. For numbers 11a–11d, select True or False for each statement.
11a. The width of the field is 480 yards.
11b. The length of the field is 360 feet.
11c. The width of the field is 160 feet.
11d. The area of the field is 6,400 square yards.
11a. ____________
11b. ____________
11c. ____________
11d. ____________

Answer:
11a. True
11b. True
11c. False
11d. False

Explanation:
The length of a rectangular football field, including both end zones, is 120 yards
The area of the field is 57,600 square feet
That is length x width = 57,600 square feet
Here, length = 120 yards
Then, width = 57,600/120 = 480 yards
11a. True
11b. 1 yard = 3 feet
Then, 120 yards = 120×3 = 360 feet
True
11c. 480 yards = 480×3 = 1440
False
11d. 6400 square yards
3 feet = 1 yard
then, 57,600 square feet = 57,600/3 = 19,200 square yards
False

Question 12.
Harry received a package for his birthday. The package weighed 357,000 centigrams. Select the conversions that are equivalent to 357,000 centigrams. Mark all that apply.
Options:
a. 3.57 kilograms
b. 357 dekagrams
c. 3,570 grams
d. 3,570,000 decigrams

Answer:
a. 3.57 kilograms
b. 357 dekagrams
c. 3,570 grams

Explanation:
Harry received a package for his birthday
The package weighed 357,000 centigrams
100centigrams = 1 gram
Then, 357,000 centigrams = 357,000/100 = 3570 grams
1000 grams = 1 kilogram
Then, 3570 grams = 3570/1000 = 3.57 kilograms
10grams = 1 dekagram
Then, 3570 grams = 3570/10 = 357 dekagrams
1gram = 10 decigrams
Then, 3570 grams = 35700 decigrams
Options a, b and c are true

Page No. 350

Question 13.
Mr. Martin wrote the following problem on the board.
Juanita’s car has a gas mileage of 21 miles per gallon. How many miles can Juanita travel on 7 gallons of gas?
Alex used the expression \(\frac{21 \text { miles }}{\text { 1 gallon }} \times \frac{1}{7 \text { gallons }}\) to find the answer. Explain Alex’s mistake.
Type below:
____________

Answer:
Juanita’s car has a gas mileage of 21 miles per gallon
Juanita traveled miles on 7 gallons of gas = 21×7 = 147 miles
But, Alex used the expression 21 miles 1 gallon ×17 gallons
In the place of 7 gallons, Alex used 17 gallons

Question 14.
Mr. Chen filled his son’s wading pool with 20 gallons of water.
20 gallons is equivalent to ____________ quarts.

Answer:
80 quarts

Explanation:
Mr. Chen filled his son’s wading pool with 20 gallons of water
1gallon = 4 quarts
Then, 20 gallons = 20×4 = 80 quarts
So, 20 gallons is equivalent to 80 quarts

Question 15.
Nadia has a can of vegetables with a mass of 411 grams. Write equivalent conversions in the correct boxes.
Go Math Grade 6 Answer Key Chapter 6 Convert Units of Length img 10
Type below:
____________

Answer:
0.411, 41.1, 4.11

Explanation:
Nadia has a can of vegetables with a mass of 411 grams
1000 grams = 1 kilogram
Then, 411 grams = 411/1000 = 0.411 kilograms
100grams = 1 hectogram
Then, 411 grams = 411/100 = 4.11 hectograms
10grams = 1 dekagram
Then, 411 grams = 411/10 = 41.1 dekagram

Question 16.
Steve is driving 440 miles to visit the Grand Canyon. He drives at an average rate of 55 miles per hour. Explain how you can find the amount of time it will take Steve to get to the Grand Canyon.
Type below:
____________

Answer:
Steve is driving 440 miles to visit the Grand Canyon
He drives at an average rate of 55 miles per hour
Then, 440 miles = 440/55 = 8 hours
So, Steve can take 8 hours of time to visit the Grand Canyon

Page No. 351

Question 17.
Lucy walks one time around the lake. She walks for 1.5 hours at an average rate of 3 miles per hour. What is the distance, in miles, around the lake?
_____ miles

Answer:
4.5 miles

Explanation:
Lucy walks one time around the lake
She walks for 1.5 hours at an average rate of 3 miles per hour
1 hour = 3 miles
Then, 1.5 hours = 1.5×3 = 4.5 miles
So, Luke walks 4.5 miles around the lake

Question 18.
The parking lot at a store has a width of 20 yards 2 feet and a length of 30 yards.
Go Math Grade 6 Answer Key Chapter 6 Convert Units of Length img 11
Part A
Derrick says that the width could also be written as 22 feet. Explain whether you agree or disagree with Derrick.
Type below:
____________

Answer:
Derrick says that the width could also be written as 22 feet
1yard = 3 feet
20 yards = 60 feet
So, we cannot write 20 yards 2 feet as 22 feet

Question 19.
Part B
The cost to repave the parking lot is $2 per square foot. Explain how much it would cost to repave the parking lot.
Type below:
____________

Answer:
The cost to repave the parking lot is $2 per square foot
Parking lot area =20 yards 2 feet x 30 yards
1yard = 3 feet
Then, 20 yards = 20×3 = 60 feet
30 yards = 30×3 = 90 feet
so, Parking lot area = 62 feet x 90 feet = 5580 feet
1 square foot cost = $2
then, 5580 feet cost = 2×5580 = $11,160

Page No. 352

Question 19.
Jake is using a horse trailer to take his horses to his new ranch.
Part A
Complete the table by finding the weight, in pounds, of Jake’s horse trailer and each horse.
Go Math Grade 6 Answer Key Chapter 6 Convert Units of Length img 12
Type below:
____________

Answer:
Horse weight in Tons = 0.5 T
Trailer weight in Tons = 1.25 T
1 ton = 2000 pounds
then, 0.5 T = 0.5×2000 = 1000 pounds
then, 1.25 T = 1.25×2000 = 2,500 pounds

Question 19.
Part B
Jake’s truck can tow a maximum weight of 5,000 pounds. What is the maximum number of horses he can take in his trailer at one time without going over the maximum weight his truck can tow? Use numbers and words to support your answer.
Type below:
____________

Answer:
Max. No of Horses = (Max weight truck can tow)/(Average weight of one horse)
The weight of a horse is not given in the question .Thus, we assume the average weight of one horse, to be equal to 1000 pounds,
Max. No of Horses = 5000 pounds/ 1000 pounds
Max. No of Horses = 5

Question 20.
A rectangular room measures 13 feet by 132 inches. Tonya said the area of the room is 1,716 square feet. Explain her mistake, then find the area in square feet.
Type below:
____________

Answer:
A rectangular room measures 13 feet by 132 inches =13 feetx132 inches
Tonya said the area of the room is 1,716 square feet
Area of the rectangular room = 13 feet x 132 inches
12inches = 1 foot
Then, 132 inches = 132/12 = 11 feet
So, the area of the rectangular room = 13 feet x 12 feet = 156 feet
So, Tonya’s answer is wrong

Conclusion:

Get free access to Download Go Math Grade 6 Answer Key Chapter 6 Convert Units of Length from here. I wish the information prevailed in Go Math Grade 6 Answer Key is beneficial for all the students. We have given the step by step explanation for each and every problem here. So refer to our Go Math 6th Grade Solution Key Chapter 6 Convert Units of Length. If you have any doubts regarding the tops you can ask your doubts in the below mentioned comment section.

Go Math Grade 5 Answer Key Chapter 4 Multiply Decimals

go-math-grade-5-chapter-4-multiply-decimals-answer-key

Go Math Grade 5 Answer Key Chapter 4 Multiply Decimals comes in handy while preparing for the exams. Go Math Grade 5 Chapter 4 Solution Key is given by the subject experts. We listed Go Math Grade 5 Answer Key covering the questions from Practice Test, Chapter Test, Cumulative Practice. Know how to solve various models of 5th Grade Go Math Ch 4 Multiply Decimals by referring to the Solutions Key Provided. The Best Part about or 5th Grade Ch 4 Multiply Decimals Answer Key is that we have given an elaborate explanation for all the Problems.

Go Math Grade 5 Chapter 4 Multiply Decimals Answer Key

Go Math Grade 5 Chapter 4 Multiply Decimals Solution Key covers Questions from basic to advanced level of difficulty. All the Problems are explained by subject experts and create an Interest in the Subject among the Students. If they have any difficulty in solving the Chapter 4 Lessons Problems they can cross check the detailed explanation provided below to resolve their doubts. Students can feel confident with the concepts by practicing from the 5th Grade Go Math Multiply Decimals Answer Key.

Lesson 1: Algebra • Multiplication Patterns with Decimals

Lesson 2: Investigate • Multiply Decimals and Whole Numbers

Lesson 3: Multiplication with Decimals and Whole Numbers

Lesson 4: Multiply Using Expanded Form

Lesson 5: Problem Solving • Multiply Money

Mid-Chapter Checkpoint

Lesson 6: Investigate • Decimal Multiplication

Lesson 7: Multiply Decimals

Lesson 8: Zeros in the Product

Review/Test

Share and Show – Page No. 165

Complete the pattern.

Question 1.
100 × 17.04 = 17.04
101 × 17.04 = 17.04
102 × 17.04 = 17.04
103 × 17.04 = 17.04
_____

Answer:
100 × 17.04 = 17.04
101 × 17.04 = 170.4
102 × 17.04 = 1,704
103 × 17.04 =17,040

Explanation:
As you multiply by increasing powers of 10, then the position of the decimal point moves towards the right side.
100 × 17.04 = 1 x 17.04 = 17.04
101 × 17.04 = 10 x 17.04 = 170.4
102 × 17.04 = 100 x 17.04 = 1,704
103 × 17.04 = 1000 x 17.04 = 17,040

Complete the pattern.

Question 2.
1 × 3.19 = _____
10 × 3.19 = _____
100 × 3.19 = _____
1,000 × 3.19 = _____

Answer:
1 × 3.19 = 3.19
10 × 3.19 = 31.9
100 × 3.19 = 319
1,000 × 3.19 = 3,190
As you multiply by increasing powers of 10, then the position of the decimal point moves towards the right side.

Question 3.
45.6 × 100 = _____
45.6 × 101 = _____
45.6 × 102 = _____
45.6 × 103 = _____

Answer:
45.6 × 100 = 45.6
45.6 × 101 = 456
45.6 × 102 = 4,560
45.6 × 103 = 45,600

Explanation:
As you multiply by increasing powers of 10, then the position of the decimal point moves towards the right side.
45.6 × 100 = 45.6 x 1 = 45.6
45.6 × 101 = 45.6 x 10 = 456
45.6 × 102 = 45.6 x 100 = 4,560
45.6 × 103 = 45.6 x 1000 = 45,600

Question 4.
1 × 6,391 = _____
0.1 × 6,391 = _____
0.01 × 6,391 = _____

Answer:
1 × 6,391 = 6,391
0.1 × 6,391 = 639.1
0.01 × 6,391 = 63.91
As you multiply by decreasing powers of 10, the position of the decimal point moves towards the left side

On Your Own

Complete the pattern.

Question 5.
1.06 × 1 = _____
1.06 × 10 = _____
1.06 × 100 = _____
1.06 × 1,000 = _____

Answer:
1.06 × 1 = 1.06
1.06 × 10 = 10.6
1.06 × 100 = 106
1.06 × 1,000 = 1,060
As you multiply by increasing powers of 10, then the position of the decimal point moves towards the right side.

Question 6.
1 × 90 = _____
0.1 × 90 = _____
0.01 × 90 = _____

Answer:
1 × 90 = 90
0.1 × 90 = 9.0 = 9
0.01 × 90 = 0.9

Explanation:
As you multiply by decreasing powers of 10, the position of the decimal point moves towards the left side
1 × 90 = 90
0.1 × 90 = 9.0
0.01 × 90 = 0.90

Question 7.
100 × $0.19 = $ _____
101 × $0.19 = $ _____
102 × $0.19 = $ _____
103 × $0.19 = $ _____

Answer:
100 × $0.19 = $ 0.19
101 × $0.19 = $ 1.9
102 × $0.19 = $ 19
103 × $0.19 = $ 190

Explanation:
As you multiply by increasing powers of 10, then the position of the decimal point moves towards the right side.
100 × $0.19 = $ 0.19
101 × $0.19 = $ 1.9
102 × $0.19 = $ 19
103 × $0.19 = $ 190

Question 8.
580 × 1 = _____
580 × 0.1 = _____
580 × 0.01 = _____

Answer:
580 × 1 = 580
580 × 0.1 = 58
580 × 0.01 = 5.8

Explanation:
As you multiply by increasing powers of 10, then the position of the decimal point moves towards the right side.
580 × 1 = 580
580 × 0.1 = 58.0 = 58
580 × 0.01 = 5.8

Question 9.
100 × 80.72 = _____
101 × 80.72 = _____
102 × 80.72 = _____
103 × 80.72 = _____

Answer:
100 × 80.72 = 80.72
101 × 80.72 = 807.2
102 × 80.72 = 8,072
103 × 80.72 = 80,720

Explanation:
As you multiply by decreasing powers of 10, the position of the decimal point moves towards the left side
100 × 80.72 = 80.72
101 × 80.72 = 807.2
102 × 80.72 = 8,072
103 × 80.72 = 80,720

Question 10.
1 × 7,230 = _____
0.1 × 7,230 = _____
0.01 × 7,230 = _____

Answer:
1 × 7,230 = 7,230
0.1 × 7,230 = 723
0.01 × 7,230 = 72.3

Explanation:
As you multiply by increasing powers of 10, then the position of the decimal point moves towards the right side.
1 × 7,230 = 7,230
0.1 × 7,230 = 723.0 = 723
0.01 × 7,230 = 72.3

Algebra Find the value

of n.

Question 11.
n × $3.25 = $325.00
n = _____

Answer:
n = 100

Explanation:
n × $3.25 = $325.00
n × $3.25 = $325.00
n x $325 x $0.01 = $325.00
n x $325 x $1/100 = $325.00
n =  $325.00/$325 x 100
n = 1 x 100 = 100

Question 12.
0.1 × n = 89.5
n = _____

Answer:
n = 895

Explanation:
0.1 × n = 89.5
1/10 x n = 895 x 0.1
n = 895 x 0.1 x 10
n = 895

Question 13.
103 × n = 630
n = _____

Answer:
n = 0.63

Explanation:
103 × n = 630
1000 x n = 630
n = 630 x 1/1000
n = 630 x 0.001
n = 0.63

Problem Solving – Page No. 166

What’s the Error?

Question 14.
Kirsten is making lanyards for a convention. She needs to make 1,000 lanyards and knows that 1 lanyard uses 1.75 feet of cord. How much cord will Kirsten need?
Go Math Grade 5 Answer Key Chapter 4 Multiply Decimals img 1
Kirsten’s work is shown below.
1 × 1.75 = 1.75
10 × 1.75 = 10.75
100 × 1.75 = 100.75
1,000 × 1.75 = 1,000.75

Find and describe Kirsten’s error. Solve the problem using the correct pattern.
As you can see from the given pattern, by multiplying 1.75 by different multiplicands, she just replaced the whole number, the number before the decimal point (in our use number 1), with belonging.
But this is not the way we multiply decimal numbers with different powers of number 10.
1 x 1.75= 1.75
10 x 1.75= 17.5
100 x 1.75= 175
1,000 x 1.75= 1,750

So, Kirsten needs ______ feet of cord to make 1,000 lanyards.
Describe how Kirsten could have solved the problem without writing out the pattern needed.
Type below:
________

Answer:
Kirsten needs 1,750 feet of cord to make 1,000 lanyards.
that decimal point moves one Noce M the right for each increasing power of 10. So, the answer is 1,750 feet.

Share and Show – Page No. 167

Use the decimal model to find the product.

Question 1.
5 × 0.06 =
Go Math Grade 5 Answer Key Chapter 4 Multiply Decimals img 2
_____

Answer:
5 × 0.06 = 0.3
Go Math Grade 5 Answer Key Chapter 4 Multiply Decimals img 2

Explanation:
The picture shows that 5 groups of 6 hundredths.
0.06 = 6 hundredths
Each square box shows 1/ 100.
So, shade 6 boxes 5 times to get the product.
Count the number of boxes shaded. There are 30 hundredths are shaded = 0.30 = 0.3
5 × 0.06 = 0.3

Question 2.
2 × 0.38 =
_____

Answer:
2 × 0.38 = 0.76
grade 5 chapter 4 Multiply Decimals 167 image 1

Explanation:
The picture shows that 2 groups of 38 hundredths.
0.38 = 38 hundredths
Each square box shows 1/ 100.
So, shade 38 boxes 2 times to get the product. 38 hundredths + 38 hundredths = 76 hundredths = 0.76.

Question 3.
4 × 0.24 =
_____

Answer:
4 × 0.24 = 0.96
grade 5 chapter 4 Multiply Decimals 167 image 2

Explanation:
4 groups of 24 hundredths
Each square box shows 1/ 100.
So, shade 24 boxes 4 times to get the product. 24 hundredths + 24 hundredths + 24 hundredths + 24 hundredths = 96 hundredths = 0.96.

Find the product. Draw a quick picture.

Question 4.
4 × 0.6 =
_____

Answer:
4 × 0.6 = 2.4
grade 5 chapter 4 Multiply Decimals 168 image 1

Explanation:
4 × 0.6
4 groups of 6 tenths
0.6 + 0.6 + 0.6 + 0.6 = 2.4
4 × 0.6 = 2.4

Question 5.
2 × 0.67 =
_____

Answer:
2 × 0.67 = 1.34
grade 5 chapter 4 Multiply Decimals 168 image 2

Explanation:
2 × 0.67
2 groups of 67 hundredths
0.67 + 0.67 = 1.34
2 × 0.67 = 1.34

Question 6.
3 × 0.62 =
_____

Answer:
3 × 0.62 = 1.86
grade 5 chapter 4 Multiply Decimals 168 image 3

Explanation:
3 × 0.62
3 groups of 62 hundredths
0.62 + 0.62 + 0.62 = 1.86
3 × 0.62 = 1.86

Question 7.
4 × 0.32 =
_____

Answer:
4 × 0.32 = 1.28
grade 5 chapter 4 Multiply Decimals 168 image 4

Explanation:
4 × 0.32
4 groups of 32 hundredths
0.32 + 0.32 + 0.32 + 0.32 = 1.28
4 × 0.32 = 1.28

Question 8.
Describe how you solved Exercise 7 using place value and renaming.
Type below:
________

Answer:
4 × 0.32
4 groups of 32 hundredths
There are 32 hundredths.
32 hundredths there are 30 tenths and 2 hundredths.
Combine the tenths and rename.
2 + 2 + 2 + 2 = 8
Combine the tenths and rename.
There are 3 tenths.
3 + 3 + 3 + 3 = 12; 2 tenths and 1 tens
Cross out the tenths you renamed.
Combine the ones and rename them.
0 + 0 + 0 + 0 + 1 = 1
1.28
4 × 0.32 = 1.28

Problem Solving – Page No. 168

Use the table for 9–11.
Go Math Grade 5 Answer Key Chapter 4 Multiply Decimals img 3

Question 9.
Each day a bobcat drinks about 3 times as much water as a Canada goose drinks. How much water can a bobcat drink in one day?
_____ liter

Answer:
0.72 liters

Explanation:
Each day a bobcat drinks about 3 times as much water as a Canada goose drinks.
Canada goose = 0.24 liters
bobcat drinks = 3 x 0.24
3 x 0.24 = 0.72 liters

Question 10.
River otters drink about 5 times as much water as a bald eagle drinks in a day. How much water can a river otter drink in one day?
_____ liter

Answer:
0.8 liter

Explanation:
River otters drink about 5 times as much water as a bald eagle drinks in a day.
bald eagle drinks 0.16 liters
5 times as 0.16 liters = 5 x 0.16 = 0.8 liter

Question 11.
Explain how you could use a quick picture to find the amount of water that a cat drinks in 5 days.
Type below:
________

Answer:
grade 5 chapter 4 Multiply Decimals 168 image 5

Explanation:
Cat drinks 0.15 liters of water in a day.
In 5 days, 5 x 0.15 = 0.75

Question 12.
Test Prep Jared has a parakeet that weighs 1.44 ounces. Susie has a Senegal parrot that weighs 3 times as much as Jared’s parakeet. How many ounces does Susie’s parrot weigh?
Options:
a. 0.32 ounce
b. 0.43 ounce
c. 4.32 ounces
d. 43.2 ounces

Answer:
c. 4.32 ounces

Explanation:
Jared has a parakeet that weighs 1.44 ounces. Susie has a Senegal parrot that weighs 3 times as much as Jared’s parakeet.
Susie’s parrot weigh 3 x 1.44 ounces = 4.32 ounces

Share and Show – Page No. 171

Place the decimal point in the product.

Question 1.
6.81
×   7
———-
4767
Think: The place value of the decimal factor is hundredths.

Answer:
6.81 x 7 = 47.67

Explanation:
6.81 x 7 = 7 x 6.81
7 x (6 + 0.81) = (7 x 6) + (7 x 0.81) = 42 + 5.67 = 47.67

Question 2.
3.7
× 2
———-
74
_____

Answer:
7.4

Explanation:
3.7 x 2
3.7 x 10 = 37
37 x 2 = 74
37 x 0.1 = 3.7
74 x 0.1 = 7.4

Question 3.
19.34
×    5
———-
9670
_____

Answer:
96.7

Explanation:
19.34 x 100 = 1934
1934 x 5 = 9670
1934 x 0.01 = 19.34
9670 x 0.01 = 96.7

Find the product.

Question 4.
6.32
×  3
———-
_____

Answer:
18.96

Explanation:
6.32 x 100 = 632
632 x 3 = 1896
632 x 0.01 = 6.32
1896 x 0.01 = 18.96

Question 5.
4.5
× 8
———-
_____

Answer:
36

Explanation:
4.5 x 10 = 45
45 x 8 = 360
45 x 0.1 = 4.5
360 x 0.1 = 36.0

Question 6.
40.7
×  5
———-
_____

Answer:
203.5

Explanation:
40.7 x 10 = 407
407 x 5 = 2035
407 x 0.1 = 40.7
2035 x 0.1 = 203.5

On Your Own

Find the product.

Question 7.
4.93
×   7
———-
_____

Answer:
34.51

Explanation:
7 x 3 = 21 hundredths; 2 tenths and 1 hundredths
7 x 9 = 63 tenths; 63 + 2 tenths = 65 tenths; 6 ones and 5 tenths
4 x 7 = 28; 28 + 6 = 34 ones;
34.51

Question 8.
8.2
× 6
———-
_____

Answer:
49.2

Explanation:
6 x 2 = 12 tenths; 1 ones and 2 tenths
6 x 8 = 48; 48 + 1 = 49 ones
49.2

Question 9.
0.49
×   4
———-
_____

Answer:
1.96

Explanation:
9 x 4 = 36 hundredths; 3 tenths and 6 hundredths
4 x 4 = 16 tenths; 16 + 3 tenths = 19 tenths; 1 ones and 9 tenths
4 x 0 = 0; 0 + 1 = 1ones;
1.96

Question 10.
9.08
×   9
———-
_____

Answer:
81.72

Explanation:
9 x 8 = 72 hundredths; 7 tenths and 2 hundredths
9 x 0 = 0 tenths; 0 + 7 tenths = 7 tenths; 7 tenths
9 x 9 = 81; 81
81.72

Question 11.
7.55
×  8
———-
_____

Answer:
60.4

Explanation:
8 x 5 = 40 hundredths; 4 tenths and 0 hundredths
8 x 5 = 40 tenths; 40 + 4 tenths = 44 tenths; 4 ones and 4 tenths
8 x 7 = 56 ones; 56 + 4 = 60 ones
60.40 = 60.4

Question 12.
15.37
×    5
———-
_____

Answer:
76.85

Explanation:
5 x 7 = 35 hundredths; 3 tenths and 5 hundredths
5 x 3 = 15 tenths; 15 + 3 tenths = 18 tenths; 1 ones and 8 tenths
5 x 5 = 25 ones; 25 + 1 = 26 ones; 2 hundreds and 6 ones
5 x 1 = 5 hundreds; 5 + 2 = 7 hundreds
76.85

Practice: Copy and Solve Find the product.

Question 13.
8 × 7.2 = _____

Answer:
8 × 7.2 = 57.6

Explanation:
8 × 7.2 = 8 x (7 + 0.2) = (8 x 7) + (8 x 0.2) = 56 + 1.6 = 57.6

Question 14.
3 × 1.45 = _____

Answer:
3 × 1.45 = 4.35

Explanation:
3 x 1.45 = 3 x (1 + 0.45) = (3 x 1) + (3 x 0.45) = 3 + 1.35 = 4.35

Question 15.
9 × 8.6 = _____

Answer:
9 × 8.6 = 77.4

Explanation:
9 × 8.6 = 9 x (8 + 0.6) = (9 x 8) + (9 x 0.6) = 72 + 5.4 = 77.4

Question 16.
6 × 0.79 = _____

Answer:
6 × 0.79 = 4.74

Explanation:
6 x 0.79 = 4.74

Question 17.
4 × 9.3 = _____

Answer:
4 × 9.3 = 37.2

Explanation:
4 × 9.3 = 4 x (9 + 0.3) = (4 x 9) + (4 x 0.3) = 36 + 1.2 = 37.2

Question 18.
7 × 0.81 = _____

Answer:
7 × 0.81 = 5.67

Explanation:
7 × 0.81 = 5.67

Question 19.
6 × 2.08 = _____

Answer:
6 × 2.08 = 12.48

Explanation:
6 × 2.08 = 6 x (2 + 0.08) = (6 x 2) + (6 x 0.08) = 12 + 0.48 = 12.48

Question 20.
5 × 23.66 = _____

Answer:
5 × 23.66 = 118.3

Explanation:
5 × 23.66 = 5 x (23 + 0.66) = (5 x 23) + (5 x 0.66) = 115 + 3.3 = 118.3

Problem Solving – Page No. 172

Use the table for 21–23.
Go Math Grade 5 Answer Key Chapter 4 Multiply Decimals img 4

Question 21.
Sari has a bag containing 6 half dollars. What is the weight of the half dollars in Sari’s bag?
_____ grams

Answer:
68.04 grams

Explanation:
Sari has a bag containing 6 half dollars.
Half dollars = 11.34 grams
6 x 11.34 = 68.04 grams
The weight of the half dollars in Sari’s bag is 68.04 grams.

Question 22.
Felicia is running a game booth at a carnival. One of the games requires participants to guess the weight, in grams, of a bag of 9 dimes. What is the actual weight of the dimes in the bag?
_____ grams

Answer:
20.43 grams

Explanation:
Felicia is running a game booth at a carnival. One of the games requires participants to guess the weight, in grams, of a bag of 9 dimes.
9 x 2.27 grams = 20.43 grams

Question 23.
Chance has $2 in quarters. Blake has $5 in dollar coins. Whose coins have the greatest weight? Explain.
_________

Answer:
Dollar coins has the greatest weight than quarters.

Explanation:
$2 means 4 quarters = 4 x 5.67 = 22.68
$5 in dollar coins = 5 x 8.1 = 40.5
Dollar coins has the greatest weight than quarters.

Question 24.
Julie multiplies 6.27 by 7 and claims the product is 438.9. Explain without multiplying how you know Julie’s answer is not correct. Find the correct answer.
Type below:
_________

Answer:
6.27 has two decimal digits
438.9 has one decimal digit
Therefore, Julie’s answer is not correct.
6.27 x 7 = 43.89

Question 25.
Test Prep Every day on his way to and from school, Milo walks a total of 3.65 miles. If he walks to school 5 days, how many miles will Milo have walked?
_____ miles

Answer:
18.25 miles

Explanation:
Milo walks a total of 3.65 miles.
If he walks to school 5 days, 5 x 3.65 = 18.25 miles

Share and Show – Page No. 175

Draw a model to find the product.

Question 1.
19 × 0.75 =
Go Math Grade 5 Answer Key Chapter 4 Multiply Decimals img 5
_____

Answer:
grade 5 chapter 4 Multiply Decimals 175 image 1
14.25

Explanation:
19 × 0.75
19 = 10 + 9
0.75 = 0.7 + 0.05
10 x 0.7 = 7
10 x 0.05 = 0.5
9 x 0.7 = 6.3
9 x 0.05 = 0.45
7 + 0.5 + 6.3 + 0.45 = 14.25
19 × 0.75 = 14.25

Question 2.
27 × 8.3 =
_____

Answer:
grade 5 chapter 4 Multiply Decimals 175 image 2
224.1

Explanation:
27 × 8.3 = 224.1
27 = 20 + 7
8.3 = 8 + 0.3
20 x 8 = 160
20 x 0.3 = 6
7 x 8 = 56
7 x 0.3 = 2.1
160 + 6 + 56 + 2.1 = 224.1

Find the product.

Question 3.
18 × 8.7 = _____

Answer:
18 × 8.7 = 156.6

Explanation:
8.7 x 10 = 87
18 x 87 = 1566
87 x 0.1 = 8.7
1566 x 0.1 = 156.6

Question 4.
23 × 56.1 = _____

Answer:
1290.3

Explanation:
56.1 x 10 = 561
561 x 23 = 12,903
561 x 0.1 = 56.1
12,903 x 0.1 = 1290.3

Question 5.
47 × 5.92 = _____

Answer:
278.24

Explanation:
5.92 x 100 = 592
592 x 47 = 27,824
592 x 0.01 = 5.92
27,824 x 0.01 = 278.24

On Your Own

Draw a model to find the product.

Question 6.
71 × 8.3 =
_____

Answer:
grade 5 chapter 4 Multiply Decimals 175 image 3
589.3

Explanation:
71 = 70 + 1
8.3 = 8 + 0.3
70 x 8 = 560
70 x 0.3 = 21
1 x 8 = 8
1 x 0.3 = 0.3
560 + 21 + 8 + 0.3 = 589.3

Question 7.
28 × 0.91 =
_____

Answer:
grade 5 chapter 4 Multiply Decimals 175 image 4
25.48

Explanation:
28 = 20 + 8
0.91 = 0.90 + 0.01
20 x 0.90 = 18
20 x 0.01 = 0.2
8 x 0.90 = 7.2
8 x 0.01 = 0.08
18 + 0.2 + 7.2 + 0.08 = 25.48

Find the product.

Question 8.
19 × 0.65 = _____

Answer:
19 × 0.65 = 12.35

Explanation:
0.65 x 100 = 65
65 x 19 = 1235
65 x 0.01 = 0.65
1235 x 0.01 = 12.35

Question 9.
34 × 98.3 = _____

Answer:
34 × 98.3 = 3342.2

Explanation:
98.3 x 10 = 983
983 x 34 = 33,422
983 x 0.1 = 98.3
33,422 x 0.1 = 3342.2

Question 10.
26 × 16.28 = _____

Answer:
26 × 16.28 = 423.28

Explanation:
16.28 x 100 = 1628
1628 x 26 = 42,328
1628 x 0.01 = 16.28
42,328 x 0.01 = 423.28

UNLOCK the Problem – Page No. 176

Question 11.
While researching facts on the planet Earth, Kate learned that a true Earth day is about 23.93 hours long. How many hours are in 2 weeks on Earth?
Go Math Grade 5 Answer Key Chapter 4 Multiply Decimals img 6
a. What are you being asked to find?
Type below:
_________

Answer:
We need to find How many hours are in 2 weeks on Earth? 2 weeks x 23.93 hours per day?

Question 11.
b. What information do you need to know to solve the problem?
Type below:
_________

Answer:
Number of days in a week
Hours per day

Question 11.
c. Write an expression to represent the problem to be solved.
Type below:
_________

Answer:
2 weeks = 14 days
14 x 23.93 hours

Question 11.
d. Show the steps you used to solve the problem.
Type below:
_________

Answer:
335.02 hours

Explanation:
23.93 = 23.93 x 100 = 2393
2393 x 14 = 33,502
2393 x 0.01 = 23.93
33502 x 0.01 = 335.02

Question 11.
e. Complete the sentences.
On Earth, there are about _____ hours in a day, _____ days in 1 week, and _____ days in two weeks.
Since _____ × _____ = _____ , there are about _____ hours in 2 weeks on Earth.
Type below:
_________

Answer:
On Earth, there are about 23.93 hours in a day,  7 days in 1 week, and 14 days in two weeks.
Since 23.93 × 14 = 335.02, there are about 335.02 hours in 2 weeks on Earth.

Question 12.
Michael’s favorite song is 3.19 minutes long. If he listens to the song 15 times on repeat, how long will he have listened to the same song?
_____ minutes

Answer:
47.85 minutes

Explanation:
Michael’s favorite song is 3.19 minutes long.
If he listens to the song 15 times, 15 x 3.19 = 47.85 minutes

Question 13.
Test Prep A car travels 56.7 miles in an hour. If it continues at the same speed, how far will the car travel in 12 hours?
Options:
a. 68.004 miles
b. 680.04 miles
c. 680.4 miles
d. 6,804 miles

Answer:
c. 680.4 miles

Explanation:
A car travels 56.7 miles in an hour.
In 12 hours, 12 x 56.7 = 680.4 hours

Share and Show – Page No. 179

Question 1.
Manuel collects $45.18 for a fundraiser. Gerome collects $18.07 more than Manuel. Cindy collects 2 times as much as Gerome. How much money does Cindy collect for the fundraiser?
First, draw a diagram to show the amount Manuel collects.
Then, draw a diagram to show the amount Gerome collects.
Next, draw a diagram to show the amount Cindy collects.
Finally, find the amount each person collects.
Cindy collects ______ for the fundraiser.
Type below:
_________

Answer:
Manuel collects $45.18 for a fundraiser. Gerome collects $18.07 more than Manuel. Cindy collects 2 times as much as Gerome.
grade 5 chapter 4 Multiply Decimals 179 image 1
Manuel: $45.18
Gerome: $45.18 + $18.07 = $63.25
Cindy: 2 x $63.25 = $126.5

Question 2.
What if Gerome collects $9.23 more than Manuel? If Cindy still collects 2 times as much as Gerome, how much money would Cindy collect?
Type below:
_________

Answer:
Gerome collects $9.23 more than Manuel
Manuel: $45.18
Gerome: $45.18 + $9.23 = $54.41
Cindy: 2 x $54.41 = $108.82

Question 3.
It costs $5.15 to rent a kayak for 1 hour at a local state park. The price per hour stays the same for up to 5 hours of rental. After 5 hours, the cost is decreased to $3.75 per hour. How much would it cost to rent a kayak for 6 hours?
$ ______

Answer:
$29.5

Explanation:
It costs $5.15 to rent a kayak for 1 hour at a local state park. The price per hour stays the same for up to 5 hours of rental. After 5 hours, the cost is decreased to $3.75 per hour.
For first 5 hours, $5.15
Next hour after 5 hours, it decreased to $3.75
For 6 hours, 5 x $5.15 + 1 x $3.75
5 x $5.15 = $25.75
1 x $3.75 = $3.75
$25.75 + $3.75 = $29.5

Question 4.
Jenn buys a pair of jeans for $24.99. Her friend Karen spends $3.50 more for the same pair of jeans. Vicki paid the same price as Karen for the jeans but bought 2 pairs. How much did Vicki spend?
$ ______

Answer:
$56.98

Explanation:
Jenn buys a pair of jeans for $24.99.
Karen: $24.99 + $3.50 = $28.49
Vicky: 2 x $28.49 = $56.98

On Your Own – Page No. 180

Use the sign for 5–8.
Go Math Grade 5 Answer Key Chapter 4 Multiply Decimals img 7

Question 5.
Austin shops at Surfer Joe’s Surf Shop before going to the beach. He buys 2 T-shirts, a pair of board shorts, and a towel. If he gives the cashier $60, how much change will Austin get back?
$ ______

Answer:
$2.86

Explanation:
T-Shirt = $12.75
Board Shorts = $25.99
Sandals = $8.95
Towel = $5.65
Sunglasses = $15.50
Austin shops at Surfer Joe’s Surf Shop before going to the beach. He buys 2 T-shirts, a pair of board shorts, and a towel.
(2 x $12.75) + ($25.99) + $5.65 = $25.5 + $31.64 = $57.14
$60 – $57.14 = $2.86

Question 6.
Maria buys 3 T-shirts and 2 pairs of sandals at Surfer Joe’s Surf Shop. How much does Maria spend?
$ ______

Answer:
$56.15

Explanation:
Maria buys 3 T-shirts and 2 pairs of sandals at Surfer Joe’s Surf Shop.
3 x $12.75 = $38.25
2 x $8.95 = $17.9
$38.25 + $17.9 = $56.15

Question 7.
Nathan receives a coupon in the mail for $10 off of a purchase of $100 or more. If he buys 3 pairs of board shorts, 2 towels, and a pair of sunglasses, will he spend enough to use the coupon? How much will his purchase cost?
Type below:
_________

Answer:
$94.77

Explanation:
3 pairs of board shorts, 2 towels, and a pair of sunglasses
3 x $25.99 = $77.97
2 x $5.65 = $11.3
Sunglasses = $15.50
$77.97 + $11.3 + $15.50 = $104.77
$10 off of a purchase of $100 or more
$104.77 – $10 = $94.77

Question 8.
Moya spends $33.90 on 3 different items. If she did not buy board shorts, which three items did Moya buy?
Type below:
_________

Answer:
T-Shirt, Towel, and Sunglasses

Explanation:
Moya spends $33.90 on 3 different items. If she did not buy board shorts,
T-Shirt = $12.75
Towel = $5.65
Sunglasses = $15.50

Question 9.
Test Prep At a donut shop in town, each donut costs $0.79. If Mr. Thomas buys a box of 8 donuts, how much will he pay for the donuts?
Options:
a. $6.32
b. $8.79
c. $63.20
d. $87.90

Answer:
a. $6.32

Explanation:
At a donut shop in town, each donut costs $0.79. If Mr. Thomas buys a box of 8 donuts, 8 x $0.79 = $6.32

Mid-Chapter Checkpoint – Page No. 181

Concepts and Skills

Question 1.
Explain how you can use a quick picture to find 3 × 2.7.
Type below:
________

Answer:
3 × 2.7 = 8.1;
As there are 8 ones and 1 tenths, we can draw eight square boxes and 1 line to represent 1 tenth.

Complete the pattern.

Question 2.
1 × 3.6 = _______
10 × 3.6 = _______
100 × 3.6 = _______
1000 × 3.6 = _______

Answer:
1 × 3.6 = 3.6
10 × 3.6 = 36
100 × 3.6 = 360
1000 × 3.6 = 3,600

Question 3.
100 × 17.55 = _______
101 × 17.55 = _______
102 × 17.55 = _______
103 × 17.55 = _______

Answer:
100 × 17.55 = 17.55
101 × 17.55 = 175.5
102 × 17.55 = 1755
103 × 17.55 = 17,550

Explanation:
100 × 17.55 = 1 x 17.55 = 17.55
101 × 17.55 = 10 x 17.55 = 175.5
102 × 17.55 = 100 x 17.55 = 1755
103 × 17.55 = 1000 x 17.55 = 17,550

Question 4.
1 × 29 = _______
0.1 × 29 = _______
0.01 × 29 = _______

Answer:
1 × 29 = 29
0.1 × 29 = 2.9
0.01 × 29 = 0.29

Find the product.

Question 5.
3.14
×   8
———–
_____

Answer:
25.12

Explanation:
8 x (3.14) = 8 x (3 + 0.14) = (8 x 3) + (8 x 0.14) = 24 + 1.12 = 25.12

Question 6.
17 × 0.67 = _____

Answer:
11.39

Explanation:
0.67 x 100 = 67
67 x 17 = 1139
67 x 0.01 = 0.67
1139 x 0.01 = 11.39

Question 7.
29 × 7.3 = _____

Answer:
211.7

Explanation:
29 × 7.3 = 29 x (7 + 0.3) = (29 x 7) + (29 x 0.3) = 203 + 8.7 = 211.7

Draw a diagram to solve.

Question 8.
Julie spends $5.62 at the store. Micah spends 5 times as much as Julie. Jeremy spends $6.72 more than Micah. How much money does each person spend?
Julie: $ _______
Micah: $ _______
Jeremy: $ _______

Answer:
grade 5 chapter 4 Multiply Decimals 181 image 1
Julie: $ 5.62
Micah spends 5 times as much as Julie = 5 x $5.62 = $28.1
Jeremy spends $6.72 more than Micah = $28.1 + $6.72 = $34.82

Mid-Chapter Checkpoint – Page No. 182

Question 9.
Sarah is cutting ribbons for a pep rally. The length of each ribbon needs to be 3.68 inches. If she needs 1,000 ribbons, what is the length of ribbon Sarah needs?
_____ inches

Answer:
3680 inches

Explanation:
Sarah is cutting ribbons for a pep rally. The length of each ribbon needs to be 3.68 inches.
If she needs 1,000 ribbons, 3.68 x 1,000 = 3680 inches

Question 10.
Adam is carrying books to the classroom for his teacher. Each books weighs 3.85 pounds. If he carries 4 books, how many pounds is Adam carrying?
_____ pounds

Answer:
15.4 pounds

Explanation:
Adam is carrying books to the classroom for his teacher. Each books weighs 3.85 pounds. If he carries 4 books, 4 x 3.85 = 15.4 pounds.

Question 11.
A car travels 54.9 miles in an hour. If the car continues at the same speed for 12 hours, how many miles will it travel?
_____ miles

Answer:
658.8 miles

Explanation:
A car travels 54.9 miles in an hour. If the car continues at the same speed for 12 hours, 12 x 54.9 = 658.8 miles

Question 12.
Charlie saves $21.45 each month for 6 months. In the seventh month, he only saves $10.60. How much money will Charlie have saved after 7 months?
$ __________

Answer:
$139.3

Explanation:
Charlie saves $21.45 each month for 6 months. In the seventh month, he only saves $10.60.
6 x $21.45 + $10.60 = $128.7 + $10.60 = $139.3

Share and Show – Page No. 185

Multiply. Use the decimal model.

Question 1.
0.8 × 0.4 =
Go Math Grade 5 Answer Key Chapter 4 Multiply Decimals img 8

Answer:
0.8 × 0.4 = 0.32
grade 5 chapter 4 Multiply Decimals 183 image 1

Explanation:
The shaded and crossed parts represent the product.
32 hundredths = 0.32

Question 2.
0.1 × 0.7 =
Go Math Grade 5 Answer Key Chapter 4 Multiply Decimals img 9
_____

Answer:
grade 5 chapter 4 Multiply Decimals 183 image 2
0.1 × 0.7 = 0.7

Explanation:
Count the number of overlapped boxes to find the product. 7 tenths = 0.7

Question 3.
0.4 × 1.6 =
Go Math Grade 5 Answer Key Chapter 4 Multiply Decimals img 10
_____

Answer:
0.4 × 1.6 = 0.64
grade 5 chapter 4 Multiply Decimals 185 image 1

Explanation:
Count the red line crossed boxes to get the product.
4 x 16 = 64
0.1 x 0.1 = 0.01
64 x 0.01 = 0.64

Question 4.
0.3 × 0.4 =
Go Math Grade 5 Answer Key Chapter 4 Multiply Decimals img 11
_____

Answer:
0.3 × 0.4 = 0.12
grade 5 chapter 4 Multiply Decimals 190 image 2

Explanation:
3 x 4 = 12
0.1 x 0.1 = 0.01
12 x 0.01 = 0.12

Question 5.
0.9 × 0.6 =
Go Math Grade 5 Answer Key Chapter 4 Multiply Decimals img 12
_____

Answer:
0.9 x 0.6 = 0.54
grade 5 chapter 4 Multiply Decimals 190 image 4

Explanation:
9 x 6 = 54
0.1 x 0.1 = 0.01
54 x 0.01 = 0.54

Question 6.
0.5 × 1.2 =
Go Math Grade 5 Answer Key Chapter 4 Multiply Decimals img 13
_____

Answer:
0.5 × 1.2 = 0.60
grade 5 chapter 4 Multiply Decimals 185 image 2

Explanation:
Count the red line crossed boxes to get the product.
5 x 12 = 60
0.1 x 0.1 = 0.01
60 x 0.01 = 0.60

Question 7.
0.8 × 0.9 =
Go Math Grade 5 Answer Key Chapter 4 Multiply Decimals img 14
_____

Answer:
0.8 × 0.9 = 0.72
grade 5 chapter 4 Multiply Decimals 190 image 3

Explanation:
8 x 9 = 72
0.1 x 0.1 = 0.01
72 x 0.01 = 0.72

Question 8.
0.5 × 0.3 =
Go Math Grade 5 Answer Key Chapter 4 Multiply Decimals img 15
_____

Answer:
0.5 × 0.3 = 0.15
grade 5 chapter 4 Multiply Decimals 190 image 1

Explanation:
5 x 3 = 15
0.1 x 0.1 = 0.01
15 x 0.01 = 0.15

Question 9.
0.5 × 1.5 =
Go Math Grade 5 Answer Key Chapter 4 Multiply Decimals img 16
_____

Answer:
0.5 × 1.5 = 0.75
grade 5 chapter 4 Multiply Decimals 185 image 3

Explanation:
Count the red line crossed boxes to get the product.
5 x 15 = 75
0.1 x 0.1 = 0.01
75 x 0.01 = 0.75

Question 10.
Explain why when you multiply and find one tenth of one tenth, it is equal to one hundredth.
Type below:
_________

Answer:
When you do one-tenth of one-tenth, it is one-tenth over 10 —-> (1/10) /10
So, you can consider it as (1/10) / (10/1). This is only for simplicity.
Now, you have to multiply the denominator of the fraction in the numerator with the numerator of fraction in denominator i.e., 10 with 10 and this comes in denominator only.
and numerator of fraction in the numerator with the denominator of the fraction in denominator i.e., 1 with 1.
So, you get, (1*1) / (10*10) = 1/100
This is again the 10th part of one-tenth OR 100th part of 1 = one hundredth

Problem Solving – Page No. 186

Sense or Nonsense?

Question 11.
Randy and Stacy used models to find 0.3 of 0.5. Both Randy’s and Stacy’s models are shown below. Whose model makes sense? Whose model is nonsense? Explain your reasoning below each model. Then record the correct answer.
Randy’s Model
Go Math Grade 5 Answer Key Chapter 4 Multiply Decimals img 17

Stacy’s Model
Go Math Grade 5 Answer Key Chapter 4 Multiply Decimals img 18
0.3 × 0.5 =
• For the answer that is nonsense, describe the error the student made.
_________ model is correct

Answer:
Randy’s Model is correct. Stacy’s Model makes nonsense.
Because Stacy’s Model is showing 0.10 x 0.8 which is not equal to 0.3 x 0.5

Explanation:
Randy and Stacy used models to find 0.3 of 0.5
0.3 x 0.5 = 0.15

Share and Show – Page No. 188

Place the decimal point in the product.

Question 1.
3.62
× 1.4
———-

5068
Think: A hundredth is being multiplied by a tenth. Use the pattern 0.01 × 0.1.
___

Answer:
5.068

Explanation:
3.62 x 100 = 362 = 362 x 0.01
1.4 x 10 = 14 = 14 x 0.1
362 x 14 = 5068
0.01 x 0.1 = 0.001
5068 x 0.001 = 5.068

Question 2.
6.8
×1.2
———-
816
_____

Answer:
8.16

Explanation:
6.8 x 10 = 68 = 68 x 0.1
1.2 x 10 = 12 = 12 x 0.1
68 x 12 = 816
0.1 x 0.1 = 0.01
816 x 0.01 = 8.16

Find the product.

Question 3.
0.9
× 0.8
———-
_____

Answer:
0.72

Explanation:
0.9 x 10 = 9 = 9 x 0.1
0.8 x 10 = 8 = 8 x 0.1
9 x 8 = 72
0.1 x 0.1 = 0.01
72 x 0.01 = 0.72

Question 4.
84.5
×  5.5
———-
_____

Answer:
464.75

Explanation:
84.5 x 10 = 845 = 845 x 0.1
5.5 x 10 = 55 = 55 x 0.1
845 x 55 = 46475
0.1 x 0.1 = 0.01
46475 x 0.01 = 464.75

Question 5.
2.39
×2.7
———-
_____

Answer:
6.453

Explanation:
2.39 x 100 = 239 = 239 x 0.01
2.7 x 10 = 27 = 27 x 0.1
239 x 27 = 6453
0.01 x 0.1 = 0.001
6453 x 0.001 = 6.453

On Your Own – Page No. 189

Find the product.

Question 6.
7.9
× 3.4
———-
_____

Answer:
26.86

Explanation:
7.9 x 10 = 79 = 79 x 0.1
3.4 x 10 = 34 = 34 x 0.1
79 x 34 = 2686
0.1 x 0.1 = 0.01
2686 x 0.01 = 26.86

Question 7.
9.2
×5.6
———-
_____

Answer:
51.52

Explanation:
9.2 x 10 = 92 = 92 x 0.1
5.6 x 10 = 56 = 56 x 0.1
92 x 56 = 5152
0.1 x 0.1 = 0.01
5152 x 0.01 = 51.52

Question 8.
3.45
× 9.7
———-
_____

Answer:
33.465

Explanation:
3.45 x 100 = 345 = 345 x 0.01
9.7 x 10 = 97 = 97 x 0.1
345 x 97 = 33465
0.01 x 0.1 = 0.001
33465 x 0.001 = 33.465

Question 9.
45.3
× 0.8
———-
_____

Answer:
36.24

Explanation:
45.3 x 10 = 453 = 453 x 0.1
0.8 x 10 = 8 = 8 x 0.1
453 x 8 = 3624
0.1 x 0.1 = 0.01
3624 x 0.01 = 36.24

Question 10.
6.98
× 2.5
———-
_____

Answer:
17.450

Explanation:
6.98 x 100 = 698 = 698 x 0.01
2.5 x 10 = 25 = 25 x 0.1
698 x 25 = 17,450
0.01 x 0.1 = 0.001
17450 x 0.001 = 17.450

Question 11.
7.02
×3.4
———-
_____

Answer:
23.868

Explanation:
7.02 x 100 = 702 = 702 x 0.01
3.4 x 10 = 34 = 34 x 0.1
702 x 34 = 23868
0.01 x 0.1 = 0.001
23868 x 0.001 = 23.868

Question 12.
14.9
×0.35
———-
_____

Answer:
5.215

Explanation:
14.9 x 10 = 149 = 149 x 0.1
0.35 x 100 = 35 = 35 x 0.01
149 x 35 = 5215
0.1 x 0.01 = 0.001
5215 x 0.001 = 5.215

Question 13.
50.99
×  3.7
———-
_____

Answer:
188.663

Explanation:
50.99 x 100 = 5099 = 5099 x 0.01
3.7 x 10 = 37 = 37 x 0.1
5099 x 37 = 188663
0.01 x 0.1 = 0.001
188663 x 0.001 = 188.663

Question 14.
18.43
×  1.9
———-
_____

Answer:
35.017

Explanation:
18.43 x 100 = 1843 = 1843 x 0.01
1.9 x 10 = 19 = 19 x 0.1
1843 x 19 = 35017
0.01 x 0.1 = 0.001
35017 x 0.001 = 35.017

Practice: Copy and Solve Find the product.

Question 15.
3.4 × 5.2 = _____

Answer:
17.68

Explanation:
3.4 × 5.2
34 x 52 = 1768
0.1 x 0.1 = 0.01
1768 x 0.01 = 17.68

Question 16.
0.9 × 2.46 = _____

Answer:
2.214

Explanation:
9 x 246 = 2214
0.1 x 0.01 = 0.001
2214 x 0.001 = 2.214

Question 17.
9.1 × 5.7 = ____

Answer:
51.87

Explanation:
91 x 57 = 5187
0.1 x 0.1 = 0.01
5187 x 0.01 = 51.87

Question 18.
4.8 × 6.01 = _____

Answer:
28.848

Explanation:
48 x 601 = 28848
0.1 x 0.01 = 0.001
28848 x 0.001 = 28.848

Question 20.
7.6 × 18.7 = _____

Answer:
142.12

Explanation:
76 x 187 = 14212
0.1 x 0.1 = 0.01
14212 x 0.01 = 142.12

Question 21.
0.77 × 14.9 = _____

Answer:
114.73

Explanation:
77 x 149 = 11473
0.01 x 0.1 = 0.01
11473 x 0.01 = 114.73

Question 22.
3.3 × 58.14 = _____

Answer:
191.862

Explanation:
33 x 5814 = 191862
0.1 x 0.01 = 0.001
191862 x 0.001 = 191.862

Problem Solving – Page No. 190

Go Math Grade 5 Answer Key Chapter 4 Multiply Decimals img 19

Question 23.
Charlie has an adult Netherlands dwarf rabbit that weighs 1.2 kilograms. Cliff’s adult Angora rabbit weighs 2.9 times as much as Charlie’s rabbit. How much does Cliff’s rabbit weigh?
_____ kilograms

Answer:
3.48 kilograms

Explanation:
Charlie has an adult Netherlands dwarf rabbit that weighs 1.2 kilograms. Cliff’s adult Angora rabbit weighs 2.9 times as much as Charlie’s rabbit.
1.2 x 2.9 = 3.48 kilograms

Question 24.
John has pet rabbits in an enclosure that has an area of 30.72 square feet. The enclosure Taylor is planning to build for his rabbits will be 2.2 times as large as John’s. What will be the area of the enclosure Taylor is planning to build?
_____ square feet

Answer:
67.584 square feet

Explanation:
John has pet rabbits in an enclosure that has an area of 30.72 square feet. The enclosure Taylor is planning to build for his rabbits will be 2.2 times as large as John’s.
30.72 x 2.2 = 67.584 square feet

Question 25.
A zoo is planning a new building for the penguin exhibit. First, they made a model that was 1.3 meters tall. Then, they made a more detailed model that was 1.5 times as tall as the first model. The building will be 2.5 times as tall as the height of the detailed model. What will be the height of the building?
_____ meters

Answer:
4.875 meters

Explanation:
A zoo is planning a new building for the penguin exhibit. First, they made a model that was 1.3 meters tall. Then, they made a more detailed model that was 1.5 times as tall as the first model.
1.3 x 1.5 = 1.95
The building will be 2.5 times as tall as the height of the detailed model.
2.5 x 1.95 = 4.875 meters

Question 26.
Leslie and Paul both solve the multiplication problem 5.5 x 4.6. Leslie says the answer is 25.30. Paul says the answer is 25.3. Whose answer is correct? Explain your reasoning.
Type below:
_________

Answer:
Both answers are correct. Because 25.30 = 25.3. The zeros have no value after the decimal point of a number.

Explanation:
5.5 x 4.6
55 x 46 = 2530
0.1 x 0.1 = 0.01
2530 x 0.01 = 25.30 = 25.3

Question 27.
Test Prep A vine in Mr. Jackson’s garden is 3.6 feet long. When it is measured again, it is 2.1 times as long. How long is the vine?
Options:
a. 5.7 feet
b. 6.6 feet
c. 7.5 feet
d. 7.56 feet

Answer:
a. 5.7 feet

Explanation:
A vine in Mr. Jackson’s garden is 3.6 feet long. When it is measured again, it is 2.1 times as long.
3.6 + 2.1 = 5.7 feet

Share and Show – Page No. 193

Write zeros in the product.

Question 1.
0.05
× 0.7
———-

Answer:

Explanation:

□35
Think: Hundredths are multiplied by tenths. What should be the place value of the product?
_____

Answer:
0.035

Explanation:
5 x 7 = 35
0.01 x 0.1 = 0.001
35 x 0.001 = 0.035

Question 2.
0.2
× 0.3
———-
_____

Answer:
0.06

Explanation:
2 x 3 = 6
0.1 x 0.1 = 0.01
6 x 0.01 = 0.06

Question 3.
0.02
× 0.2
———-
_____

Answer:
0.004

Explanation:
2 x 2 = 4
0.01 x 0.1 = 0.001
4 x 0.001 = 0.004

Find the product.

Question 4.
$0.05
× 0.8
———-
$ _____

Answer:
$0.04

Explanation:
5 x 8 = 40
0.01 x 0.1 = 0.001
40 x 0.001 = 0.040 = 0.04

Question 5.
0.09
× 0.7
———-
_____

Answer:
0.063

Explanation:
9 x 7 = 63
0.01 x 0.1 = 0.001
63 x 0.001 = 0.063

Question 6.
0.2
× 0.1
———-
_____

Answer:
0.02

Explanation:
2 x 1 = 2
0.1 x 0.1 = 0.01
2 x 0.01 = 0.02

On Your Own

Find the product.

Question 7.
0.3
× 0.3
———-
_____

Answer:
0.09

Explanation:
3 x 3 = 9
0.1 x 0.1 = 0.01
9 x 0.01 = 0.09

Question 8.
0.05
× 0.3
———-
_____

Answer:
0.015

Explanation:
5 x 3 = 15
0.01 x 0.1 = 0.001
15 x 0.001 = 0.015

Question 9.
0.02
× 0.4
———-
_____

Answer:
0.008

Explanation:
2 x 4 = 8
0.01 x 0.1 = 0.001
8 x 0.001 = 0.008

Question 10.
$0.40
× 0.1
———-
$ _____

Answer:
$0.04

Explanation:
40 x 1 = 40
0.10 x 0.1 = 0.010
40 x 0.010 = 0.04

Question 11.
0.09
× 0.2
———-
_____

Answer:
0.018

Explanation:
9 x 2 = 18
0.01 x 0.1 = 0.001
18 x 0.001 = 0.018

Question 12.
$ 0.05
× 0.6
———-
_____

Answer:
$0.3

Explanation:
5 x 6 = 30
0.01 x 0.1 = 0.001
30 x 0.001 = 0.30 = 0.3

Question 13.
0.04
× 0.5
———-
_____

Answer:
0.020

Explanation:
4 x 5 = 20
0.01 x 0.1 = 0.001
20 x 0.001 = 0.020

Question 14.
0.06
× 0.8
———-
_____

Answer:
0.048

Explanation:
6 x 8 = 48
0.01 x 0.1 = 0.001
48 x 0.001 = 0.048

Algebra Find the value of n.

Question 15.
0.03 × 0.6 = n
n = _____

Answer:
n = 0.018

Explanation:
0.03 × 0.6 = n
0.018 = n
n = 0.018

Question 16.
n × 0.2 = 0.08
n = _____

Answer:
n = 0.4

Explanation:
n × 0.2 = 0.08
n = 0.08/0.2
n = 0.4

Question 17.
0.09 × n = 0.063
n = _____

Answer:
n = 0.7

Explanation:
0.09 × n = 0.063
n = 0.063/0.09
n = 0.7

Page No. 194

Question 18.
On an average day, a garden snail can travel about 0.05 mile. If a snail travels 0.2 times as far as the average distance in a day, how far can it travel?
Go Math Grade 5 Answer Key Chapter 4 Multiply Decimals img 20
a. What are you being asked to find?
Type below:
_________

Answer:
We need to find how far snail travels on 0.2 times as far as the average distance in a day?

Question 18.
b. What information will you use to solve the problem?
Type below:
_________

Answer:
On an average day, a garden snail can travel about 0.05 miles.
0.2 times as far as the average distance in a day

Question 18.
c. How will you use multiplication and place value to solve the problem?
Type below:
_________

Answer:
0.2 x 0.05

Question 18.
d. Show how you will solve the problem.
Type below:
_________

Answer:
2 x 5 = 10
0.1 x 0.01 = 0.001
10 x 0.001 = 0.010 = 0.01

Question 18.
e. Fill in the bubble for the correct answer choice above.
Options:
a. 0.7 mile
b. 0.25 mile
c. 0.1 mile
d. 0.01 mile

Answer:
d. 0.01 mile

Question 19.
In a science experiment, Tania uses 0.8 ounce of water to create a reaction. She wants the next reaction to be 0.1 times the size of the previous reaction. How much water should she use?
Options:
a. 0.08 ounce
b. 0.09 ounce
c. 0.8 ounce
d. 0.9 ounce

Answer:
a. 0.08 ounce

Explanation:
In a science experiment, Tania uses 0.8 ounce of water to create a reaction. She wants the next reaction to be 0.1 times the size of the previous reaction.
0.8 x 0.1 = 0.08 ounce

Question 20.
Michael multiplies 0.2 by a number. He records the product as 0.008. What number did Michael use?
Options:
a. 0.016
b. 0.04
c. 0.28
d. 0.4

Answer:
b. 0.04

Explanation:
Michael multiplies 0.2 by a number. He records the product as 0.008.
0.2 x n = 0.008
n = 0.008/0.2
n = 0.04
Michael use 0.04

Chapter Review/Test – Page No. 195

Check Concepts

Question 1.
Explain how estimation helps you to place the decimal point when multiplying 3.9 × 5.3.
Type below:
_________

Answer:
3.9 × 5.3
39 x 53 = 2067
0.1 x 0.1 = 0.01
2067 x 0.01 = 20.67

Complete the pattern.

Question 2.
1 × 7.45 = _______
10 × 7.45 = _______
100 × 7.45 = _______
1,000 × 7.45 = _______

Answer:
1 × 7.45 = 7.45
10 × 7.45 = 74.5
100 × 7.45 = 745
1,000 × 7.45 = 7,450

Question 3.
100 × 376.2 = _______
101 × 376.2 = _______
102 × 376.2 = _______
103 × 376.2 = _______

Answer:
100 × 376.2 = 376.2
101 × 376.2 = 3,762
102 × 376.2 = 37,620
103 × 376.2 = 376,200

Explanation:
100 × 376.2 = 1 x 376.2 = 376.2
101 × 376.2 = 10 x 376.2 = 3,762
102 × 376.2 = 100 x 376.2 =  37,620
103 × 376.2 = 1000 x 376.2 = 376,200

Question 4.
1 × 191 = _______
0.1 × 191 = _______
0.01 × 191 = _______

Answer:
1 × 191 = 191
0.1 × 191 = 19.1
0.01 × 191 = 1.91_

Find the product.

Question 5.
5 × 0.89 = _____

Answer:
4.45

Explanation:
5 × 0.89
5 x 9 = 45 hundredths; 4 tenths and 5 hundredths
5 x 8 = 40 tenths; 40 + 4 tenths = 44 tenths; 4 ones and 4 tenths
5 x 0 = 0; 0 + 4 = 4 ones
4.45

Question 6.
9 × 2.35 = _____

Answer:
21.15

Explanation:
9 × 2.35
9 x 5 = 45 hundredths; 4 tenths and 5 hundredths
9 x 3 = 27 tenths; 27 + 4 tenths = 31 tenths; 3 ones and 1 tenth
9 x 2 = 18; 18 + 3 = 21 ones
21.15

Question 7.
23 × 8.6 = _____

Answer:
197.8

Explanation:
23 x 8.6
23 x 6 = 138 tenths; 13 ones and 8 tenths
23 x 8 = 184 ones; 184 + 13 = 197 ones
197.8

Question 8.
7.3 × 0.6 = _____

Answer:
4.38

Explanation:
73 x 6 = 438
0.1 x 0.1 = 0.01
438 x 0.01 = 4.38

Question 9.
0.09 × 0.7 = _____

Answer:
0.063

Explanation:
9 x 7 = 63
0.01 x 0.1 = 0.001
63 x 0.001 = 0.063

Question 10.
0.8 × $0.40 = $ _____

Answer:
$0.32

Explanation:
8 x 4 = 32
0.1 x 0.1 = 0.01
32 x 0.01 = 0.32

Draw a diagram to solve.

Question 11.
In January, Dawn earns $9.25 allowance. She earns 3 times as much in February. If during March, she earns $5.75 more than she did in February, how much allowance does Dawn earn in March?
$ _________

Answer:
$33.5

Explanation:
In January, Dawn earns $9.25 allowance.
February: 3 x $9.25 = $27.75
March: $27.75 + $5.75 = $33.5

Chapter Review/Test – Page No. 196

Fill in the bubble completely to show your answer.

Question 12.
Janet hikes a trail at a local forest each day. The trail is 3.6 miles long, and she has hiked 5 days in the past week. How many miles has Janet hiked in the past week?
Options:
A. 18 miles
B. 15.3 miles
C. 11 miles
D. 8.6 miles

Answer:
A. 18 miles

Explanation:
Janet hikes a trail at a local forest each day. The trail is 3.6 miles long, and she has hiked 5 days in the past week.
3.6 x 5 = 18 miles

Question 13.
To earn money for his vacation, Grayson works at a local shop on weekends. His job is to cut bricks of fudge into 0.25 pound squares. If he cuts 36 equal-sized squares on Saturday, how many pounds of fudge has Grayson cut?
Options:
A. 7.25 pounds
B. 9 pounds
C. 90 pounds
D. 72.5 pounds

Answer:
B. 9 pounds

Explanation:
To earn money for his vacation, Grayson works at a local shop on weekends. His job is to cut bricks of fudge into 0.25 pound squares. If he cuts 36 equal-sized squares on Saturday,
0.25 x 36 = 9 pounds

Question 14.
James is making a scale model of his bedroom. The model is 0.6 feet wide. If the actual room is 17.5 times as wide as the model, what is the width of James’s room?
Options:
A. 18.1 feet
B. 17.11 feet
C. 16.9 feet
D. 10.5 feet

Answer:
D. 10.5 feet

Explanation:
James is making a scale model of his bedroom. The model is 0.6 feet wide. If the actual room is 17.5 times as wide as the model,
0.6 x 17.5 = 10.5 feet

Question 15.
The cost of admission to the matinee showing at a movie theater is $6.75. If 7 friends want to see the matinee showing of their favorite movie, how much will it cost?
Options:
A. $11.25
B. $14.75
C. $42.75
D. $47.25

Answer:
D. $47.25

Explanation:
The cost of admission to the matinee showing at a movie theater is $6.75. If 7 friends want to see the matinee showing of their favorite movie,
7 x $6.75 = $47.25

Chapter Review/Test – Page No. 197

Fill in the bubble completely to show your answer.

Question 16.
On Friday, Gail talks for 38.4 minutes on her cell phone. On Saturday, she uses 5.5 times as many minutes as she did on Friday. How long does Gail talk on her cell phone on Saturday?
Options:
A. 2.112 minutes
B. 21.12 minutes
C. 211.2 minutes
D. 2,112 minutes

Answer:
C. 211.2 minutes

Explanation:
On Friday, Gail talks for 38.4 minutes on her cell phone. On Saturday, she uses 5.5 times as many minutes as she did on Friday.
38.4 x 5.5 = 211.2 minutes

Question 17.
Harry walks to a produce market to buy bananas. If a pound of bananas costs $0.49, how much will Harry pay for 3 pounds of bananas?
Options:
A. $1.47
B. $3.49
C. $5.49
D. $10.47

Answer:
A. $1.47

Explanation:
Harry walks to a produce market to buy bananas. If a pound of bananas costs $0.49,
For 3 pound, 3 x $0.49 = $1.47

Question 18.
At Anne’s Fabric Emporium, a yard of chiffon fabric costs $7.85. Lee plans to purchase 0.8 yard for a craft project. How much money will Lee spend on chiffon fabric?
Options:
A. $0.63
B. $6.28
C. $7.05
D. $8.65

Answer:
B. $6.28

Explanation:
At Anne’s Fabric Emporium, a yard of chiffon fabric costs $7.85. Lee plans to purchase 0.8 yard for a craft project.
0.8 x $7.85 = $6.28

Question 19.
Mitchell has $18.79 in his savings account. Jeremy has 3 times as much as Mitchell. Maritza has $4.57 more than Jeremy. How much money does Maritza have in her savings account?
Options:
A. $13.71
B. $32.50
C. $56.37
D. $60.94

Answer:
D. $60.94

Explanation:
Mitchell: $18.79
Jeremy: 3 x $18.79 = $56.37
Maritza: $56.37 + $4.57 = $60.94

Chapter Review/Test – Page No. 198

Constructed Response

Question 20.
A river otter eats about 0.15 times its weight in food each day. At the Baytown Zoo, the male river otter weighs 5 pounds. About how much food will the otter at the zoo consume each day? Explain how you found your answer.
_____ pounds

Answer:
0.75 pounds

Explanation:
A river otter eats about 0.15 times its weight in food each day. At the Baytown Zoo, the male river otter weighs 5 pounds.
0.15 x 5 = 0.75 pounds

Performance Task

Question 21.
The cost of admission to the Baytown Zoo is shown below. Use the table to answer the questions.
Go Math Grade 5 Answer Key Chapter 4 Multiply Decimals Chapter Review/Test img 21
A. A family of 2 adults and 1 child plans to spend the day at the Baytown Zoo. How much does admission for the family cost? Explain how you found your answer.
$ _____

Answer:
$39.75

Explanation:
Senior Citizen = $10.50
Adult = $15.75
Child = $8.25
A family of 2 adults and 1 child plans to spend the day at the Baytown Zoo.
(2 x $15.75) + $8.25
$31.5 + $8.25 = $39.75

Question 21.
B. Describe another way you could solve the problem.
Type below:
________

Answer:
(2 x $15.75) + $8.25
$15.75 + $15.75 + $8.25 = $39.75

Question 21.
C. What if 2 more tickets for admission are purchased? If the two additional tickets cost $16.50, determine what type of tickets the family purchases.
Explain how you can determine the answer without calculating.
Options:
a. Senior tickets
b. Adult tickets
c. Child tickets

Answer:
c. Child tickets

Explanation:
if 2 more tickets for admission are purchased? If the two additional tickets cost $16.50,
$39.75 + $16.50 = $56.25
Two additional children’s tickets are purchased. Since senior citizen tickets cost about $10 each, then 2 tickets would cost about $20, which is too much. Adult tickets cost about $16 each, so 2 adult tickets would cost about $32, which is too much. Children’s tickets cost about $8, and 2 tickets would be about $16 which is correct.

Conclusion

Download Go Math Grade 5 Answer Key Chapter 4 Multiply Decimals for free of cost and take your preparation to the next level. Every Step included while solving the Problems of Multiply Decimals makes it easy for you to grasp the concepts. Keep in touch with our site to avail the latest updates on Class Specific Go Math Answer Keys in no time.

Go Math Grade 8 Answer Key Chapter 10 Transformations and Similarity

go-math-grade-8-chapter-10-transformations-and-similarity-answer-key

Download Go Math Grade 8 Answer Key Chapter 10 Transformations and Similarity pdf for free of cost. The HMH Go Math 8th Grade Chapter 10 Transformations and Similarity Solution Key includes properties of dilations, Algebraic Representations of Dilations, and similar figures. The explanations seen in Go Math Grade 8 Answer Key Chapter 10 are prepared by the concerned subject experts. Students can overcome their difficulties with the help of the Go Math Grade 8 Chapter 10 Transformations and Similarity Answer Key.

Go Math Grade 8 Chapter 10 Transformations and Similarity Answer Key

Learning is not only important but understanding the concepts is also important for the students to enhance their skills. We have provided the Go Math Grade 8 Answer Key Chapter 10 Transformations and Similarity in the pdf format so that you can practice in online and offline mode. Thus Download Go Math Grade 8 Chapter 10 Transformations and Similarity Answer Key PDF and start your practice. The students of 8th standard can easily learn maths with the Go Math Solution Key on ccssmathanswers.com

Lesson 1: Properties of Dilations

Lesson 2: Algebraic Representations of Dilations

Lesson 3: Similar Figures

Model Quiz

Mixed Review

Guided Practice – Properties of Dilations – Page No. 318

Use triangles ABC and A′B′C ′ for 1–5.
Go Math Grade 8 Answer Key Chapter 10 Transformations and Similarity Lesson 1: Properties of Dilations img 1

Question 1.
For each pair of corresponding vertices, find the ratio of the x-coordinates and the ratio of the y-coordinates.
ratio of x-coordinates = _______
ratio of y-coordinates = _______
ratio of x-coordinates = ____________
ratio of y-coordinates = ____________

Answer:
ratio of x-coordinates = 2
ratio of y-coordinates = 2

Explanation:
A’ = (-4, 4), A = (-2, 2);
ratio of x-coordinates = -4/-2 = 2
ratio of y-coordinates = 4/2 = 2
B’ = (4, 2), B = (2, 1);
ratio of x-coordinates = 4/2 = 2
ratio of y-coordinates = 2/1 = 2
C’ = (-2, -4), C = (-1, -2);
ratio of x-coordinates = -2/-1 = 2
ratio of y-coordinates = -4/-2 = 2

Question 2.
I know that triangle A′B′C ′ is a dilation of triangle ABC because the ratios of the corresponding x-coordinates are _______ and the ratios of the corresponding y-coordinates are _______.
Type below:
_____________

Answer:
I know that triangle A′B′C ′ is a dilation of triangle ABC because the ratios of the corresponding x-coordinates are equal and the ratios of the corresponding y-coordinates are equal.

Question 3.
The ratio of the lengths of the corresponding sides of triangle A′B′C ′ and triangle ABC equals _______.
________

Answer:
The ratio of the lengths of the corresponding sides of triangle A′B′C ′ and triangle ABC equals 2.

Question 4.
The corresponding angles of triangle ABC and triangle A′B′C ′ are _______.
Type below:
_____________

Answer:
The corresponding angles of triangle ABC and triangle A′B′C ′ are congruent.

Question 5.
The scale factor of the dilation is _______.
________

Answer:
The scale factor of the dilation is 2.

ESSENTIAL QUESTION CHECK-IN

Question 6.
How can you find the scale factor of a dilation?
Type below:
_____________

Answer:
Divide a side length of the dilated figure by the corresponding side length of the original figure.

10.1 Independent Practice – Properties of Dilations – Page No. 319

For 7–11, tell whether one figure is a dilation of the other or not. Explain your reasoning.

Question 7.
Quadrilateral MNPQ has side lengths of 15 mm, 24 mm, 21 mm, and 18 mm. Quadrilateral M′N′P′Q′ has side lengths of 5 mm, 8 mm, 7 mm, and 4 mm.
_____________

Answer:
MNPQ is not a dilation of M′N′P′Q′

Explanation:
15/5 = 3 mm
24/8 = 3 mm
21/7 = 3 mm
18/4 = 4.5 mm
The ratios of the lengths of the corresponding sides are not equal.
Therefore, MNPQ is not a dilation of M′N′P′Q′

Question 8.
Triangle RST has angles measuring 38° and 75°. Triangle R′S′T ′ has angles measuring 67° and 38°. The sides are proportional.
_____________

Answer:
Yes

Explanation:
Both Triangle S have Angle S of measures 38°, 67° and 75°. So, the corresponding ∠S are congruent.

Question 9.
Two triangles, Triangle 1 and Triangle 2, are similar.
_____________

Answer:
Yes

Explanation:
a dilation produces an image similar to the original figure

Question 10.
Quadrilateral MNPQ is the same shape but a different size than quadrilateral M′N′P′Q.
_____________

Answer:
Yes

Explanation:
The figures are similar is they are the same shape but different size SO one is a dilation of the other

Question 11.
On a coordinate plane, triangle UVW has coordinates U(20, −12), V(8, 6), and W(−24, -4). Triangle U′V′W′ has coordinates U′(15, −9), V′(6, 4.5), and W′(−18, -3).
_____________

Answer:
Yes

Explanation:
Each coordinate of Triangle U′V′W′ is 3/4 times the corresponding coordinate of Triangle UVW.
So, the scale factor of the dilation is 3/4.

Complete the table by writing “same” or “changed” to compare the image with the original figure in the given transformation.

Question 12.
Go Math Grade 8 Answer Key Chapter 10 Transformations and Similarity Lesson 1: Properties of Dilations img 2
Type below:
_____________

Answer:
grade 8 chapter 10 image 1

Question 16.
Describe the image of a dilation with a scale factor of 1.
_____________

Answer:
The image is congruent to the original figure

Properties of Dilations – Page No. 320

Identify the scale factor used in each dilation.

Question 17.
Go Math Grade 8 Answer Key Chapter 10 Transformations and Similarity Lesson 1: Properties of Dilations img 3
________

Answer:
3

Explanation:
A’B’/AB = 6/2 = 3
B’D’/BD = 6/2 = 3
scale factor = 3

Question 18.
Go Math Grade 8 Answer Key Chapter 10 Transformations and Similarity Lesson 1: Properties of Dilations img 4
\(\frac{□}{□}\)

Answer:
\(\frac{1}{2}\)

Explanation:
A’B’/AB = 2/4 = 1/2
scale factor = 1/2

FOCUS ON HIGHER ORDER THINKING

Question 19.
Critical Thinking Explain how you can find the center of dilation of a triangle and its dilation.
Type below:
_____________

Answer:
If you draw a line connecting each pair of corresponding vertices, the lines will intersect at the center of dilation

Question 20.
Make a Conjecture
a. A square on the coordinate plane has vertices at (−2, 2), (2, 2), (2, −2), and (−2, −2). A dilation of the square has vertices at (−4, 4), (4, 4), (4, −4), and (−4, −4). Find the scale factor and the perimeter of each square.
Scale factor: _________
Original perimeter: _________
Image perimeter: _________

Answer:
Scale factor: 2
Original perimeter: 16
Image perimeter: 32

Explanation:
-4/-2 =2; 4/2 = 2
Scale factor = 2
perimeter of the original square = 4 + 4 + 4 + 4 = 16
perimeter of the image = 8 + 8 + 8 + 8 = 32

Question 20.
b. A square on the coordinate plane has vertices at (−3, 3), (3, 3), (3, −3), and (−3, −3). A dilation of the square has vertices at (−6, 6), (6, 6), (6, −6), and (−6, −6). Find the scale factor and the perimeter of each square.
Scale factor: _________
Original perimeter: _________
Image perimeter: _________

Answer:
Scale factor: 2
Original perimeter: 24
Image perimeter: 48

Explanation:
-6/-3 =2; 6/3 = 2
Scale factor = 2
perimeter of the original square = 6 + 6 + 6 + 6 = 24
perimeter of the image = 12 + 12 + 12 + 12 = 48

Question 20.
c. Make a conjecture about the relationship of the scale factor to the perimeter of a square and its image.
Type below:
_____________

Answer:
The perimeter of the image is the perimeter of the original figure times the scale factor

Guided Practice – Algebraic Representations of Dilations – Page No. 324

Question 1.
The grid shows a diamond-shaped preimage. Write the coordinates of the vertices of the preimage in the first column of the table. Then apply the dilation (x, y) → (\(\frac{3}{2}\)x, \(\frac{3}{2}\)y) and write the coordinates of the vertices of the image in the second column. Sketch the image of the figure after the dilation.
Go Math Grade 8 Answer Key Chapter 10 Transformations and Similarity Lesson 2: Algebraic Representations of Dilations img 5
Type below:
_____________

Answer:
grade 8 chapter 10 image 2

Graph the image of each figure after a dilation with the origin as its center and the given scale factor. Then write an algebraic rule to describe the dilation.

Question 2.
scale factor of 1.5
Go Math Grade 8 Answer Key Chapter 10 Transformations and Similarity Lesson 2: Algebraic Representations of Dilations img 6
Type below:
_____________

Answer:
(x, y) -> (1.5x, 1.5y)

Explanation:
After dilation
F’ (3, 3)
G’ (7.5, 3)
H’ (7.5, 6)
I’ (3, 6)
algebraic rule: (x, y) -> (1.5x, 1.5y)

Question 3.
scale factor of \(\frac{1}{3}\)
Go Math Grade 8 Answer Key Chapter 10 Transformations and Similarity Lesson 2: Algebraic Representations of Dilations img 7
Type below:
_____________

Answer:
(x, y) -> (1/3x, 1/3y)

Explanation:
After dilation
A’ (3, 3)
B’ (7.5, 3)
C’ (7.5, 6)
algebraic rule: (x, y) -> (1/3x, 1/3y)

ESSENTIAL QUESTION CHECK-IN

Question 4.
A dilation of (x, y) → (kx, ky) when 0 < k < 1 has what effect on the figure? What is the effect on the figure when k > 1?
Type below:
_____________

Answer:
When k is between 0 and 1, the dilation is a reduction by the scale factor k.
When k is greater than 1, the dilation is an enlargement by the scale factor k.

10.2 Independent Practice – Algebraic Representations of Dilations – Page No. 325

Question 5.
The blue square is the preimage. Write two algebraic representations, one for the dilation to the green square and one for the dilation to the purple square.
Go Math Grade 8 Answer Key Chapter 10 Transformations and Similarity Lesson 2: Algebraic Representations of Dilations img 8
Type below:
_____________

Answer:
Green square -> (x, y) -> (2x, 2y)
Purple square -> (x, y) -> (1/2x, 1/2y)

Question 6.
Critical Thinking A triangle has vertices A(-5, -4), B(2, 6), and C(4, -3). The center of dilation is the origin and (x, y) → (3x, 3y). What are the vertices of the dilated image?
Type below:
_____________

Answer:
A'(-15, -12)
B'(6, 18)
C'(12, -9)

Explanation:
A'((3.-15), (3.-12)) -> A'(-15, -12)
B'((3. 2), (3.6)) -> B'(6, 18)
C'((3. 4), (3.-3)) -> C'(12, -9)

Question 7.
Critical Thinking M′N′O′P′ has vertices at M′(3, 4), N′(6, 4), O′(6, 7), and P′(3, 7). The center of dilation is the origin. MNOP has vertices at M(4.5, 6), N(9, 6), O′(9, 10.5), and P′(4.5, 10.5). What is the algebraic representation of this dilation?
Type below:
_____________

Answer:
(x, y) -> (2/3x, 2/3y)

Explanation:
M’N’/MN = 3/4.5 = 2/3
algebraic rule: (x, y) -> (2/3x, 2/3y)

Question 8.
Critical Thinking A dilation with center (0,0) and scale factor k is applied to a polygon. What dilation can you apply to the image to return it to the original preimage?
Type below:
_____________

Answer:
A dilation with scale factor 1/k

Question 9.
Represent Real-World Problems The blueprints for a new house are scaled so that \(\frac{1}{4}\) inch equals 1 foot. The blueprint is the preimage and the house is the dilated image. The blueprints are plotted on a coordinate plane.
a. What is the scale factor in terms of inches to inches?
Scale factor: ________

Answer:
Scale factor: 48

Explanation:
scale factor = 48

Question 9.
b. One inch on the blueprint represents how many inches in the actual house? How many feet?
________ inches
________ feet

Answer:
48 inches
4 feet

Explanation:
48 inches or 4 feet

Question 9.
c. Write the algebraic representation of the dilation from the blueprint to the house.
Type below:
_____________

Answer:
(x, y) -> (48x, 48y)

Question 9.
d. A rectangular room has coordinates Q(2, 2), R(7, 2), S(7, 5), and T(2, 5) on the blueprint. The homeowner wants this room to be 25% larger. What are the coordinates of the new room?
Type below:
_____________

Answer:
Q'(2.5, 2.5),
R'(8.75, 2.5),
S'(8.75, 6.25),
T'(2.5, 6.25)

Question 9.
e. What are the dimensions of the new room, in inches, on the blueprint? What will the dimensions of the new room be, in feet, in the new house?
Type below:
_____________

Answer:
Blueprint dimensions: 6.25 in. by 3.75 in.
House dimensions: 25ft by 15ft

Algebraic Representations of Dilations – Page No. 326

Question 10.
Write the algebraic representation of the dilation shown.
Go Math Grade 8 Answer Key Chapter 10 Transformations and Similarity Lesson 2: Algebraic Representations of Dilations img 9
Type below:
_____________

Answer:
(x, y) -> (1/4x, 1/4y)

Explanation:
algebraic rule of the dilation: (x, y) -> (1/4x, 1/4y)

FOCUS ON HIGHER ORDER THINKING

Question 11.
Critique Reasoning The set for a school play needs a replica of a historic building painted on a backdrop that is 20 feet long and 16 feet high. The actual building measures 400 feet long and 320 feet high. A stage crewmember writes (x, y) → (\(\frac{1}{12}\)x, \(\frac{1}{12}\)y) to represent the dilation. Is the crewmember’s calculation correct if the painted replica is to cover the entire backdrop? Explain.
_____________

Answer:
The stage crewmember’s calculation is incorrect.
The backdrop scale factor is 1/20, not 1/12

Question 12.
Communicate Mathematical Ideas Explain what each of these algebraic transformations does to a figure.
a. (x, y) → (y, -x)
Type below:
_____________

Answer:
(x, y) → (y, -x)
90º clockwise rotation

Question 12.
b. (x, y) → (-x, -y)
Type below:
_____________

Answer:
(x, y) → (-x, -y)
180º rotation

Question 12.
c. (x, y) → (x, 2y)
Type below:
_____________

Answer:
(x, y) → (x, 2y)
vertically stretches by a factor of 2

Question 12.
d. (x, y) → (\(\frac{2}{3}\)x, y)
Type below:
_____________

Answer:
(x, y) → (\(\frac{2}{3}\)x, y)
horizontally shrinks by a factor of 2/3

Question 12.
e. (x, y) → (0.5x, 1.5y)
Type below:
_____________

Answer:
(x, y) → (0.5x, 1.5y)
horizontally shrinks by a factor of 0.5 and vertically stretches by a factor of 1.5

Question 13.
Communicate Mathematical Ideas Triangle ABC has coordinates A(1, 5), B(-2, 1), and C(-2, 4). Sketch triangle ABC and A′B′C′ for the dilation (x, y) → (-2x, -2y). What is the effect of a negative scale factor?
Type below:
_____________

Answer:
The figure is dilated by a factor of 2, but the orientation of the figure in the coordinate plane is rotated 180°

Guided Practice – Similar Figures – Page No. 330

Question 1.
Apply the indicated sequence of transformations to the square. Apply each transformation to the image of the previous transformation. Label each image with the
letter of the transformation applied.
Go Math Grade 8 Answer Key Chapter 10 Transformations and Similarity Lesson 3: Similar Figures img 10
A: (x, y) → (-x, y)
B: Rotate the square 180° around the origin.
C: (x, y) → (x – 5, y – 6)
D: (x, y) → (\(\frac{1}{2}\)x, \(\frac{1}{2}\)y)
Type below:
_____________

Answer:

Explanation:
A: (x, y) → (-x, y)
coordinates for A
(-7, -8)
(-7, -4)
(-3, -4)
(-3, -8)
B: Rotate the square 180° around the origin.
coordinates for B
(3, 4)
(3, 8)
(7, 8)
(7, 4)
C: (x, y) → (x – 5, y – 6)
coordinates for C
(-2, -2)
(-2, 2)
(2, 2)
(2, -2)
D: (x, y) → (\(\frac{1}{2}\)x, \(\frac{1}{2}\)y)
coordinates for D
(-1, -1)
(-1, 1)
(1, 1)
(1, -1)

Identify a sequence of two transformations that will transform figure A into the given figure.
Go Math Grade 8 Answer Key Chapter 10 Transformations and Similarity Lesson 3: Similar Figures img 11

Question 2.
figure B
Type below:
_____________

Answer:
(x, y) -> (x, -y)
(x, y) -> (x +5, y-6)

Question 3.
figure C
Type below:
_____________

Answer:
(x, y) -> (x, y+6)
rotate 90º counterclockwise

Question 4.
figure D
Type below:
_____________

Answer:
(x, y) -> (1.5x, 1.5y)
(x, y) -> (x+3, y+5)

ESSENTIAL QUESTION CHECK-IN

Question 5.
If two figures are similar but not congruent, what do you know about the sequence of transformations used to create one from the other?
Type below:
_____________

Answer:
At least one transformation must be a dilation with a scale factor other than 1

10.3 Independent Practice – Similar Figures – Page No. 331

Question 6.
A designer creates a drawing of a triangular sign on centimeter grid paper for a new business. The drawing has sides measuring 6 cm, 8 cm, and 10 cm, and angles measuring 37°, 53°, and 90°. To create the actual sign shown, the drawing must be dilated using a scale factor of 40.
Go Math Grade 8 Answer Key Chapter 10 Transformations and Similarity Lesson 3: Similar Figures img 12
a. Find the lengths of the sides of the actual sign.
Type below:
_____________

Answer:
240 cm, 320 cm, and 400 cm

Explanation:
6cm × 40 = 240cm
8cm × 40 = 320cm
10cm × 40 = 400cm
The lengths are 240 cm, 320 cm, and 400 cm

Question 6.
b. Find the angle measures of the actual sign.
Type below:
_____________

Answer:
The angle measures are the same
37º, 53º, and 90º

Question 6.
c. The drawing has the hypotenuse on the bottom. The business owner would like it on the top. Describe two transformations that will do this.
Type below:
_____________

Answer:
Reflect the drawing over the x-axis
Rotate the drawing 180º around the origin.

Question 6.
d. The shorter leg of the drawing is currently on the left. The business owner wants it to remain on the left after the hypotenuse goes to the top. Which transformation in part c will accomplish this?
Type below:
_____________

Answer:
Reflecting over the x-axis

In Exercises 7–10, the transformation of a figure into its image is described. Describe the transformations that will transform the image back into the original figure. Then write them algebraically.

Question 7.
The figure is reflected across the x-axis and dilated by a scale factor of 3.
Type below:
_____________

Answer:
Dilate the image by a scale factor of 1/3 and reflect it back across the x-axis.
(x, y) -> (1/3x, 1/3y)

Question 8.
The figure is dilated by a scale factor of 0.5 and translated 6 units left and 3 units up.
Type below:
_____________

Answer:
Translate the image 3 units down and 6 units right and dilate it by a factor of 2
(x, y) -> (x+6, y-3)
(x, y) -> (2x, 2y)

Question 9.
The figure is dilated by a scale factor of 5 and rotated 90° clockwise.
Type below:
_____________

Answer:
Rotate the image 90 counterclockwise and dilate it by a factor of 1/5.
(x, y) -> (-y, x)
(x, y) -> (1/5x, 1/5y)

Similar Figures – Page No. 332

Question 10.
The figure is reflected across the y-axis and dilated by a scale factor of 4.
Type below:
_____________

Answer:
Dilate the image by a factor of 1/4 and reflect it back across the y-axis.
(x, y) -> (1/4x, 1/4y)
(x, y) -> (-x, y)

FOCUS ON HIGHER ORDER THINKING

Question 11.
Draw Conclusions A figure undergoes a sequence of transformations that include dilations. The figure and its final image are congruent. Explain how this can happen.
Type below:
_____________

Answer:
There must be an even number of dilations and for each dilation applied to the figure, a dilation that has the opposite effect must be applied as well.

Question 12.
Multistep As with geometric figures, graphs can be transformed through translations, reflections, rotations, and dilations. Describe how the graph of y = x shown at the right is changed through each of the following transformations.
Go Math Grade 8 Answer Key Chapter 10 Transformations and Similarity Lesson 3: Similar Figures img 13
a. a dilation by a scale factor of 4
Type below:
_____________

Answer:
original coordinates
(0, -6)
(0, -4)
(4, 0)
(0, 4)
(-4, 4)
(-4, 2)
(-2, 0)
(-4, -2)

Question 12.
b. a translation down 3 units
Type below:
_____________

Answer:
coordinates for A
(0, -4)
(0, -3)
(2, -1)
(0, 1)
(-2, 1)
(-2, 0)
(-1, -1)
(-2, -2)

Question 12.
c. a reflection across the y-axis
Type below:
_____________

Answer:
coordinates for B
(-4, 3)
(-4, 2)
(-2, 0)
(-4, -2)
(-6, -2)
(-6, -1)
(-5, 0)
(-6, 1)

Question 13.
Justify Reasoning The graph of the line y = x is dilated by a scale factor of 3 and then translated up 5 units. Is this the same as translating the graph up 5 units and then dilating by a scale factor of 3? Explain.
Type below:
_____________

Answer:
No; the dilation is not the same reversed

Explanation:
The position of the sketch from 12A will be 1/2 unit above the sketch obtained when the translation occurs first

Ready to Go On? – Model Quiz – Page No. 333

10.1 Properties of Dilations

Determine whether one figure is a dilation of the other. Justify your answer.

Question 1.
Triangle XYZ has angles measuring 54° and 29°. Triangle X′Y′Z′ has angles measuring 29° and 92°.
_____________

Answer:
No; one figure is not a dilation of the other

Explanation:
The triangles have only one pair of congruent angles

Question 2.
Quadrilateral DEFG has sides measuring 16 m, 28 m, 24 m, and 20 m. Quadrilateral D′E′F′G′ has sides measuring 20 m, 35 m, 30 m, and 25 m.
_____________

Answer:
Yes; Quadrilateral D’E’F’G’ is a dilation of quadrilateral DEFG

Explanation:
each side of the second figure is 1.25 times the corresponding side of the original figure.

10.2 Algebraic Representations of Dilations

Dilate each figure with the origin as the center of dilation.

Question 3.
(x, y) → (0.8x, 0.8y)
Go Math Grade 8 Answer Key Chapter 10 Transformations and Similarity Model Quiz img 14
Type below:
_____________

Answer:
Coordinates after dilation
(0, -4)
(4, 0)
(0, 4)
(-4, 0)

Question 4.
(x, y) → (2.5x, 2.5y)
Go Math Grade 8 Answer Key Chapter 10 Transformations and Similarity Model Quiz img 15
Type below:
_____________

Answer:
Coordinates after dilation
(-2.5, 2.5)
(5, 5)
(5, -3)

10.3 Similar Figures

Question 5.
Describe what happens to a figure when the given sequence of transformations is applied to it: (x, y) → (-x, y); (x, y) → (0.5x, 0.5y); (x, y) → (x – 2, y + 2)
Type below:
_____________

Answer:
After the sequencing of transformations, reflection over the y-axis.
dilation with a scale factor of 0.5
translation 2 units left and 2 units up

ESSENTIAL QUESTION

Question 6.
How can you use dilations to solve real-world problems?
Type below:
_____________

Answer:
You can use dilations when drawing or designing

Selected Response – Mixed Review – Page No. 334

Question 1.
A rectangle has vertices (6, 4), (2, 4), (6, –2), and (2, –2). What are the coordinates of the vertices of the image after a dilation with the origin as its center and a scale factor of 1.5?
Options:
a. (9, 6), (3, 6), (9, –3), (3, –3)
b. (3, 2), (1, 2), (3, –1), (1, –1)
c. (12, 8), (4, 8), (12, –4), (4, –4)
d. (15, 10), (5, 10), (15, –5), (5, –5)

Answer:
a. (9, 6), (3, 6), (9, –3), (3, –3)

Explanation:
(9 -3)/(6 -2) = 6/4 = 1.5

Question 2.
Which represents the dilation shown where the black figure is the preimage?
Go Math Grade 8 Answer Key Chapter 10 Transformations and Similarity Mixed Review img 16
Options:
a. (x, y) -> (1.5x, 1.5y)
b. (x, y) -> (2.5x, 2.5y)
c. (x, y) -> (3x, 3y)
d. (x, y) -> (6x, 6y)

Answer:
b. (x, y) -> (2.5x, 2.5y)

Explanation:
5/2 = 2.5
10/4 = 2.5
(x, y) -> (2.5x, 2.5y)

Question 3.
Identify the sequence of transformations that will reflect a figure over the x-axis and then dilate it by a scale factor of 3.
Options:
a. (x, y) -> (-x, y); (x, y) -> (3x, 3y)
b. (x, y) -> (-x, y); (x, y) -> (x, 3y)
c. (x, y) -> (x, -y); (x, y) -> (3x, y)
d. (x, y) -> (x, -y); (x, y) -> (3x, 3y)

Answer:
d. (x, y) -> (x, -y); (x, y) -> (3x, 3y)

Explanation:
Reflection over x-axis (x, y) -> (x, -y)
dilation by scale factor of 3 (x, y) -> (3x, 3y)
(x, y) -> (x, -y); (x, y) -> (3x, 3y)

Question 4.
Solve −a + 7 = 2a − 8.
Options:
a. a = -3
b. a = −\(\frac{1}{3}\)
c. a = 5
d. a = 15

Answer:
c. a = 5

Explanation:
-a + 7 = 2a – 8
2a + a = 8 + 7
3a = 15
a = 15/3
a = 5

Question 5.
Which equation does not represent a line with an x-intercept of 3?
Options:
a. y = −2x + 6
b. y = −\(\frac{1}{3}\)x + 1
c. y = \(\frac{2}{3}\)x − 2
d. y = 3x − 1

Answer:
d. y = 3x − 1

Explanation:
y = -2x + 6
0 = -2x + 6
2x = 6
x = 3
y = -1/3 . x + 1
0 = -1/3 . x + 1
1/3x = 1
x = 3
y = 2/3 . x – 2
0 = 2/3 . x – 2
2/3x = 2
x = 2 . 3/2
x = 3
y = 3x – 1
0 = 3x – 1
3x = 1
x = 1/3

Mini-Task

Question 6.
The square is dilated under the dilation (x, y) → (0.25x, 0.25y).
Go Math Grade 8 Answer Key Chapter 10 Transformations and Similarity Mixed Review img 17
a. Graph the image. What are the coordinates?
Type below:
_____________

Answer:
After dilation:
(-1, 1)
(1, 1)
(1, -1)
(-1, -1)

Question 6.
b. What is the length of a side of the image?
______ units

Answer:
2 units

Explanation:
The length is 2 units

Question 6.
c. What are the perimeter and area of the preimage?
Perimeter = ________ units
Area = ________ square units

Answer:
Perimeter = 32 units
Area = 64 square units

Explanation:
Perimeter = 2l + 2w = 2(8) + 2(8) = 32
area = l.w = 8 .8 = 64

Question 6.
d. What are the perimeter and area of the image?
Perimeter = ________ units
Area = ________ square units

Answer:
Perimeter = 8 units
Area = 4 square units

Explanation:
Perimeter = 2l + 2w = 2(2) + 2(2) = 8
area = l.w = 2 . 2 = 4

Conclusion:

The Go Math Grade 8 Solution Key Chapter 10 Transformations and Similarity help the students to enhance their math skills. All the explanations are given by the best maths experts. So, to learn maths in the best way, you must refer to HMH Go Math Grade 8 Solution Key. Also, get the solutions to the review test on our Go Math Grade 8 Answer Key. Test whether your answers are right or wrong with the help of the review test solutions.

Go Math Grade 6 Answer Key Chapter 3 Understand Positive and Negative Numbers

go-math-grade-6-chapter-3-understand-positive-and-negative-numbers-answer-key

Enhance your knowledge by practicing the problems from Go Math Grade 6 Answer Key Chapter 3 Understand Positive and Negative Numbers. You can get the free pdfs of Go Math Grade 6 Chapter 3 Understand Positive and Negative Numbers Solution Key. We have provided the Go Math Grade 6 Answer Key in pdf format so that you can practice online and offline mode. Take the given resources as references and score well in the exams.

Go Math Grade 6 Chapter 3 Understand Positive and Negative Numbers Answer Key

Improve your math skills with the help of Go Math 6th Standard Answer Key Chapter 3 Understand Positive and Negative Numbers. Unlimited practice with all the maths questions and answers along with the practice questions. It is mandatory to practice with the Grade 6 Chapter 3 Solution key to score maximum marks in the exams. Check out the links given the below sections before you start your preparation.

Lesson 1: Understand Positive and Negative Numbers

Lesson 2: Compare and Order Integers

Lesson 3: Rational Numbers and the Number Line

Lesson 4: Compare and Order Rational Numbers

Mid-Chapter Checkpoint

Lesson 5: Absolute Value

Lesson 6: Compare Absolute Values

Lesson 7: Rational Numbers and the Coordinate Plane

Lesson 8: Ordered Pair Relationships

Lesson 9: Distance on the Coordinate Plane

Lesson 10: Problem Solving • The Coordinate Plane

Chapter 3 Review/Test

Share and Show – Page No. 141

Graph the integer and its opposite on a number line.

Question 1.
−7
Type below:
__________

Answer:
7

Explanation:
The opposite number of -7 is 7

Question 2.
9
Type below:
__________

Answer:
-9

Explanation:
The opposite number of 9 is -9

Name the integer that represents the situation, and tell what 0 represents in that situation.
Go Math Grade 6 Answer Key Chapter 3 Understand Positive and Negative Numbers img 1

Question 3.
Type below:
__________

Answer:
Integer: 24
0 represents: neither gaining nor losing points

Explanation:
Kerri gained 24 pounds during a round of the game show. So, he has a positive integer.

Question 4.
Type below:
__________

Answer:
Integer: -5
0 represents: Ben neither gains nor loses during the summer

Explanation:
Ben lost 5 points during the summer. He has a negative integer.

Question 5.
Type below:
__________

Answer:
Integer: 35
0 represents: No changes in her savings account.

Explanation:
Marcy deposited $35 in her savings account. She has a positive integer.

On Your Own

Write the opposite of the integer.

Question 6.
−98
Type below:
__________

Answer:
98

Explanation:
The integer −98 is on the left side of 0.
So, the opposite of -98 is 98

Question 7.
0
Type below:
__________

Answer:
0

Explanation:
Opposite of 0 is 0

Question 8.
−53
Type below:
__________

Answer:
53

Explanation:
The integer −53 is on the left side of 0.
So, the opposite of -53 is 53

Name the integer that represents the situation, and tell what 0 represents in that situation.
Go Math Grade 6 Answer Key Chapter 3 Understand Positive and Negative Numbers img 2

Question 9.
Type below:
__________

Answer:
Integer: $850
0 represents: Desmond neither gains nor loses at his summer job

Question 10.
Type below:
__________

Answer:
Integer: -300
0 represents: No change from his checking point

Question 11.
Type below:
__________

Answer:
Integer: 2
0 represents: No change of protons than electrons

Write the opposite of the opposite of the integer.

Question 12.
−23
Type below:
__________

Answer:
-23

Explanation:
The opposite integer of the -23 is 23
The opposite integer of the 23 is -23.
So, the opposite of the opposite of the integer -23 is -23.

Question 13.
17
Type below:
__________

Answer:
17

Explanation:
The opposite integer of the 17 is -17
The opposite integer of the -17 is 17.
So, the opposite of the opposite of the integer 17 is 17.

Question 14.
−125
Type below:
__________

Answer:
-125

Explanation:
The opposite integer of the -125 is 125
The opposite integer of the 125 is -125.
So, the opposite of the opposite of the integer -125 is -125.

Question 15.
Suppose you know a certain number’s distance from zero on the number line. Explain how you could find the number’s distance from its opposite.
Type below:
__________

Answer:
The distance between a number’s place on the number line and 0 is called the number’s [absolute value]. To write the absolute value of a number, use short vertical lines (|) on either side of the number. For example, the absolute value of −5 is written |−5|

Problem Solving + Applications – Page No. 142

Wind makes the air temperature seem colder. The chart gives the wind chill temperature (what the temperature seems like) at several air temperatures and wind speeds. Use the chart for 16–18.
Go Math Grade 6 Answer Key Chapter 3 Understand Positive and Negative Numbers img 3

Question 16.
At 6 a.m., the air temperature was 20°F and the wind speed was 55 mi/hr. What was the wind chill temperature at 6 a.m.?
Type below:
__________

Answer:
At 6 a.m., the air temperature was 20°F and the wind speed was 55 mi/hr.
The winds chill temperature at 6 a.m. is -4

Question 17.
At noon, the air temperature was 15°F and the wind speed was 45 mi/hr. At what air temperature and wind speed would the wind chill temperature be the opposite of what it was at noon?
Type below:
__________

Answer:
If the air temperature was 15°F and the wind speed was 45 mi/hr, the wind chill temperature is -9. The opposite number of -9 is 9.
So, at the air temperature was 25°F and the wind speed was 25 mi/hr, the wind chill temperature is the opposite of what it was at noon.

Question 18.
The wind was blowing 35 mi/hr in both Ashton and Fenton. The wind chill temperatures in the two towns were opposites. If the air temperature in Ashton was 25°F, what was the air temperature in Fenton?
Type below:
__________

Answer:
The wind was blowing 35 mi/hr in both Ashton and Fenton.
If the air temperature in Ashton was 25°F, the wind chill temperature is 7.
The wind chill temperatures in the two towns were opposites.
So, the wind chill temperature in Fenton is -7. So, the air temperature in Fenton was 15°F.

Question 19.
Sense or Nonsense? Claudia states that the opposite of any integer is always a different number than the integer. Is Claudia’s statement sense or nonsense? Explain.
Type below:
__________

Answer:
Claudia is correct.
Because the opposite of any integer is always a different number than the integer.
Example: The opposite of 7 is -7.

Question 20.
For numbers 20a−20d, choose Yes or No to indicate whether the situation can be represented by a negative number.
20a. Death Valley is located 282 feet below sea level.
20b. Austin’s golf score was 3 strokes below par.
20c. The average temperature in Santa Monica in August is 75°F.
20d. Janai withdraws $20 from her bank account.
20a. __________
20b. __________
20c. __________
20d. __________

Answer:
20a. Yes
20b. Yes
20c. No
20d. Yes

Understand Positive and Negative Numbers – Page No. 143

Graph the integer and its opposite on a number line.

Question 1.
−6
Type below:
__________

Answer:
6
grade 6 chapter 3 image 1

Explanation:
The opposite number of -6 is 6

Question 2.
3
Type below:
__________

Answer:
-3
grade 6 chapter 3 image 2

Explanation:
The opposite number of -3 is 3

Question 3.
10
Type below:
__________

Answer:
-10
grade 6 chapter 3 image 3

Explanation:
The opposite number of 10 is -10

Question 4.
−8
Type below:
__________

Answer:
8
grade 6 chapter 3 image 4

Explanation:
The opposite number of -8 is 8

Name the integer that represents the situation, and tell what 0 represents in that situation
Go Math Grade 6 Answer Key Chapter 3 Understand Positive and Negative Numbers img 4

Question 5.
Type below:
__________

Answer:
Integer: -60
0 represents: No changes in the account balance

Explanation:

Question 6.
Type below:
__________

Answer:
Integer: 12
0 represents: neither gaining nor losing points

Explanation:

Write the opposite of the opposite of the integer.

Question 7.
−20
Type below:
__________

Answer:
-20

Explanation:
The opposite integer of the -20 is 20
The opposite integer of the 20 is -20.
So, the opposite of the opposite of the integer -20 is -20

Question 8.
4
Type below:
__________

Answer:
4

Explanation:
The opposite integer of the 4 is -4
The opposite integer of the -4 is 4.
So, the opposite of the opposite of the integer -4 is 4.

Question 9.
95
Type below:
__________

Answer:
95

Explanation:
The opposite integer of the 95 is -95
The opposite integer of the -95 is 95.
So, the opposite of the opposite of the integer 95 is 95.

Question 10.
−63
Type below:
__________

Answer:
-63

Explanation:
The opposite integer of the -63 is 63
The opposite integer of the 63 is -63.
So, the opposite of the opposite of the integer -63 is -63.

Problem Solving

Question 11.
Dakshesh won a game by scoring 25 points. Randy scored the opposite number of points as Dakshesh. What is Randy’s score?
Type below:
__________

Answer:
Randy’s score -25.

Explanation:
Dakshesh won a game by scoring 25 points. Randy scored the opposite number of points as Dakshesh.
The opposite number of 25 is -25

Question 12.
When Dakshesh and Randy played the game again, Dakshesh scored the opposite of the opposite of his first score. What is his score?
Type below:
__________

Answer:
25 points

Explanation:
When Dakshesh and Randy played the game again, Dakshesh scored the opposite of the opposite of his first score.
The opposite of the 25 is -25.
The opposite of the -25 is 25.
The opposite of the opposite of his first score is 25

Question 13.
Give three examples of when negative numbers are used in daily life.
Type below:
__________

Answer:
1) negative numbers in weather reports and on food packaging. The temperature -5°C is ‘negative five degrees’ and it means 5 degrees below zero.
2) The floors as you go down in a lift, starting on the third floor you’ll see:
3, 2, 1, 0, -1, -2.
In this example building, -2 is the second floor underground
3) When you spend more money than you have in your bank account it shows up as a negative number.

Lesson Check – Page No. 144

Name the integers that represent each situation.

Question 1.
During their first round of golf, Imani was 7 strokes over par and Peter was 8 strokes below par.
Type below:
__________

Answer:
In the first round of golf, Imani has scored 7 strokes over par. So, it is represented by 7.
Peter was scored 8 strokes below par. So, it is represented by -8.
Therefore, the answer is 7 and -8.

Question 2.
Wyatt earned $15 baby-sitting on Saturday. Wilson spent $12 at the movies.
Type below:
__________

Answer:
He has $3 dollars left because you take 12 from 15 and you get 3
$15 – $12 = $3

Spiral Review

Question 3.
Mr. Nolan’s code for his ATM card is a 4-digit number. The digits of the code are the prime factors of 84 listed from least to greatest. What is the code for Mr. Nolan’s ATM card?
Type below:
__________

Answer:
2237

Explanation:
Mr.Nolan’s code for his ATM card is a 4- digit number.
The digits of the code are the prime factors of 84 listed from least to greatest.
In order to find the code, we have to find the prime factors of 84.
The prime factors of 84 are 2,2,3 and 7.
Therefore, the code=2237

Question 4.
Over a four-year period, a tree grew 2.62 feet. If the tree grows at a constant rate, how many feet did the tree grow each year?
Type below:
__________

Answer:
0.655 feet

Explanation:
Each year the tree grows
( 2.62 ÷ 4 ) feet
= 0.655 feet

Question 5.
Omarion has \(\frac{9}{10}\) of the pages in a book remaining to read for school. He reads \(\frac{2}{3}\) of the remaining pages over the weekend. What fraction of the book does Omarion read over the weekend?
Type below:
__________

Answer:
\(\frac{3}{5}\)

Explanation:
Omarion has 9/10 of pages in a book remaining to read for school and he reads 2/3 of the remaining pages over the weekend.
The fraction of the book trade over the weekend = the fraction of the pages read over the weekend multiplied by the fraction of the book that is remaining to be read.
Therefore, the fraction of the book that Omarion trad over the weekend is 2/3 × 9/10 = 3/5
Thus, the required fraction of the book that Omarion trad over the weekend is 3/5

Question 6.
Marianne has \(\frac{5}{8}\) pound of peas. She cooks \(\frac{2}{3}\) of those peas for 5 people. If each person is served an equal amount, how much peas did each person get?
Type below:
__________

Answer:
\(\frac{1}{12}\) pounds

Explanation:
Marianne has \(\frac{5}{8}\) pound of peas. IShe cooks \(\frac{2}{3}\) of those peas for 5 people.
Marianne cooks 5/8 × 2/3 = 5/12 pounds.
(5/12)/5 = 1/12 pounds

the answer is

Share and Show – Page No. 147

Compare the numbers. Write < or >.

Question 1.
8 _____ 6

Answer:
–8 < 6

Explanation:
-8 is to the left of 6 on the number line.
So, -8 is less than 6.

Question 2.
1 _____ 8

Answer:
1 > –8

Explanation:
1 is to the right of -8 on the number line.
So, 1 is greater than -8.

Question 3.
4 _____ 0

Answer:
-4 < 0

Explanation:
-4 is to the left of 0 on the number line.
So, -4 is less than 0.

Question 4.
3 _____ 7

Answer:
3 > -7

Explanation:
3 is to the right of -7 on the number line.
So, 3 is greater than -7.

Order the numbers from least to greatest.

Question 5.
4, 3, 7
Type below:
__________

Answer:
-7, -3, 4

Explanation:
-7 is to the left of -3 on the number line. -3 is to the left of 4 on the number line.
So, -7 < -3 < 4

Question 6.
0, 1, 3
Type below:
__________

Answer:
-1, 0, 3

Explanation:
-1 is to the left of 0 on the number line. 0 is to the left of 3 on the number line.
So, -1 < 0 < 3

Question 7.
5, 3, 9
Type below:
__________

Answer:
-9, -5, -3

Explanation:
-9 is to the left of -5 on the number line. -5 is to the left of -3 on the number line.
So, -9 < -5 < -3

Order the numbers from greatest to least.

Question 8.
1, 4, 2
Type below:
__________

Answer:
2, -1, -4

Explanation:
2 is to the right of -1 on the number line. -1 is to the right of -4 on the number line.
So, 2 > -1 > -4

Question 9.
5, 0, 10
Type below:
__________

Answer:
10, 5, 0

Explanation:
10 is to the right of 5 on the number line. 5 is to the right of 0 on the number line.
So, 10 > 5 > 0

Question 10.
5, 4, 3
Type below:
__________

Answer:
-3, -4, -5

Explanation:
-3 is to the right of -4 on the number line. -4 is to the right of -5 on the number line.
So, -3 > -4 > -5

On Your Own

Order the numbers from least to greatest.

Question 11.
2, 1, 1
Type below:
__________

Answer:
-1, 1, 2

Explanation:
-1 is to the left of 1 on the number line. 1 is to the left of 2 on the number line.
So, -1 < 1 < 2

Question 12.
6, 12, 30
Type below:
__________

Answer:
-12, -6, 30

Explanation:
-12 is to the left of -6 on the number line. -6 is to the left of 30 on the number line.
So, -12 < -6 < 30

Question 13.
15, 9, 20
Type below:
__________

Answer:
-20, -15, -9

Explanation:
-20 is to the left of -15 on the number line. -15 is to the left of -9 on the number line.
So, -20 < -15 < -9

Order the number from greatest to least.

Question 14.
13, 14, 14
Type below:
__________

Answer:
14, -13, -14

Explanation:
14 is to the right of -13 on the number line. -13 is to the right of -14 on the number line.
So, 14 > -13 > -14

Question 15.
20, 30, 40
Type below:
__________

Answer:
-20, -30, -40

Explanation:
-20 is to the right of -30 on the number line. -30 is to the right of -40 on the number line.
So, -20 > -30 > -40

Question 16.
9, 37, 0
Type below:
__________

Answer:
9, 0, -37

Explanation:
9 is to the right of 0 on the number line. 0 is to the right of -37 on the number line.
So, 9 > 0 > -37

Question 17.
Saturday’s low temperature was −6°F. Sunday’s low temperature was 3°F. Monday’s low temperature was −2°F. Tuesday’s low temperature was 5°F. Which day’s low temperature was closest to 0°F?
Type below:
__________

Answer:
Monday’s temperature was closest to 0°F

Explanation:
Saturday’s low temperature was −6°F. Sunday’s low temperature was 3°F. Monday’s low temperature was −2°F. Tuesday’s low temperature was 5°F.
-2 is closest to 0. So, Monday’s temperature was closest to 0°F.

Question 18.
Use Symbols Write a comparison using < or > to show that South America’s Valdes Peninsula (elevation −131 ft) is lower than Europe’s Caspian Sea (elevation −92 ft).
Type below:
__________

Answer:
South America’s Valdes Peninsula < Europe’s Caspian Sea

Explanation:
South America’s Valdes Peninsula (elevation −131 ft) is lower than Europe’s Caspian Sea (elevation −92 ft).
-131 < -92.
So, South America’s Valdes Peninsula < Europe’s Caspian Sea

Problem Solving + Applications – Page No. 148

What’s the Error?

Question 19.
In the game of golf, the player with the lowest score wins. Raheem, Erin, and Blake played a game of miniature golf. The table shows their scores compared to par.
Go Math Grade 6 Answer Key Chapter 3 Understand Positive and Negative Numbers img 5
At the end of the game, they wanted to know who had won.
Look at how they solved the problem. Find their error.
STEP 1: 0 is greater than both −1 and −5. Since Raheem had the highest score, he did not win.
STEP 2: −1 is less than −5, so Blake’s score was less than Erin’s score. Since Blake had the lowest score, he won the game.
Correct the error by ordering the scores from least to greatest.
So, _____ won. _____ came in second. _____ came in third.
Describe the error that the players made.
Type below:
__________

Answer:
Step 2 is wrong.
In step 2, they mentioned that -1 is less than −5. But -1 is greater than -5.
So, Erin won. Blake came in second. Raheem came in third.

Question 20.
Jasmine recorded the low temperatures for 3 cities.
Go Math Grade 6 Answer Key Chapter 3 Understand Positive and Negative Numbers img 6
Draw a dot on the number line to represent the low temperature of each city. Write the letter of the city above the dot.
Type below:
__________

Answer:
grade 6 chapter 3 image 5

Explanation:
6 > 2 > -4

Compare and Order Integers – Page No. 149

Compare the numbers. Write < or >.

Question 1.
4 ____ 5

Answer:
-4 > -5

Explanation:
-4 is to the right of -5 on the number line.
So, -4 is greater than -5.

Question 2.
0 ____ 1

Answer:
0 > -1

Explanation:
0 is to the right of -1 on the number line.
So, 0 is greater than -1.

Question 3.
4 ____ 6

Answer:
4 > -6

Explanation:
4 is to the right of -6 on the number line.
So, 4 is greater than -6.

Question 4.
9 ____ 8

Answer:
-9 < -8

Explanation:
-9 is to the left of -8 on the number line.
So, -9 is less than -8.

Question 5.
2 ____ 10

Answer:
2 > -10

Explanation:
2 is to the right of -10 on the number line.
So, 2 is greater than -10.

Question 6.
12 ____ 11

Answer:
-12 < -11

Explanation:
-12 is to the left of -11 on the number line.
So, -12 is less than -11.

Question 7.
1 ____ 10

Answer:
1 > -10

Explanation:
1 is to the right of -10 on the number line.
So, 1 is greater than -10.

Order the numbers from least to greatest.

Question 8.
3, 2, 7
Type below:
__________

Answer:
-7, -2, 3

Explanation:
-7 is to the left of -2 on the number line. -2 is to the left of 3 on the number line.
So, -7 < -2 < 3

Question 9.
0, 2, 5
Type below:
__________

Answer:
-5, 0, 2

Explanation:
-5 is to the left of 0 on the number line. 0 is to the left of 2 on the number line.
So, -5 < 0 < 2

Question 10.
9, 12, 10
Type below:
__________

Answer:
-12, -10, -9

Explanation:
-12 is to the left of -10 on the number line. -10 is to the left of -9 on the number line.
So, -12 < -10 < -9

Question 11.
2, 3, 4
Type below:
__________

Answer:
-4, -3, -2

Explanation:
-4 is to the left of -3 on the number line. -3 is to the left of -2 on the number line.
So, -4 < -3 < -2

Question 12.
1, 6, 13
Type below:
__________

Answer:
-13, -6, 1

Explanation:
-13 is to the left of -6 on the number line. -6 is to the left of 1 on the number line.
So, -13 < -6 < 1

Question 13.
5, 7, 0
Type below:
__________

Answer:
0, 5, 7

Explanation:
0 is to the left of 5 on the number line. 5 is to the left of 7 on the number line.
So, 0 < 5 < 7

Question 14.
0, 13, 13
Type below:
__________

Answer:
-13, 0, 13

Explanation:
-13 is to the left of 0 on the number line. 0 is to the left of 13 on the number line.
So, -13 < 0 < 13

Question 15.
11, 7, 5
Type below:
__________

Answer:
-11, -5, 7

Explanation:
-11 is to the left of -5 on the number line. -5 is to the left of 7 on the number line.
So, -11 < -5 < 7

Question 16.
9, 8, 1
Type below:
__________

Answer:
-9, -8, 1

Explanation:
-9 is to the left of -8 on the number line. -8 is to the left of 1 on the number line.
So, -9 < -8 < 1

Problem Solving

Question 17.
Meg and Derek played a game. Meg scored 11 points, and Derek scored 4 points. Write a comparison to show that Meg’s score is less than Derek’s score.
Type below:
__________

Answer:
-11 < 4

Explanation:
Meg and Derek played a game. Meg scored -11 points, and Derek scored 4 points.
-11 < 4

Question 18.
Misha is thinking of a negative integer greater than −4. What number could she be thinking of?
Type below:
__________

Answer:
-3, -2, -1

Explanation:
Misha is thinking of a negative integer greater than −4.
-3, -2, -1

Question 19.
Explain how to use a number line to compare two negative integers. Give an example.
Type below:
__________

Answer:
-> On a number line, numbers always increase (become “more positive”) to the right and decrease (become “more negative”) to the left.
-> Numbers to the right are greater than numbers to the left and numbers to the left are less than numbers to the right.
Example: 2 > -10
2 is to the right of -10 on the number line.
So, 2 is greater than -10.

Lesson Check – Page No. 150

Go Math Grade 6 Answer Key Chapter 3 Understand Positive and Negative Numbers img 7
The chart shows the high temperatures for seven cities on one day in January.

Question 1.
Which city had the lower temperature, Helena or Chicago?
Type below:
__________

Answer:
Helena had a lower temperature

Explanation:
Helena = -1
Chicago = 2
-1 < 2
So, Helena had a lower temperature.

Question 2.
Write the temperatures of the following cities in order from greatest to least: Denver, Helena, Lansing.
Type below:
__________

Answer:
Lansing, Helena, Denver

Explanation:
Denver = -8
Helena = -1
Lansing = 3
3 > -1 > -8
So, Lansing, Helena, Denver is the answer.

Spiral Review

Question 3.
Fiona starts at the beginning of a hiking trail and walks \(\frac{4}{5}\) mile. She counts the mileage markers that are placed every \(\frac{1}{10}\) mile along the trail. How many markers does she count?
______ markers

Answer:
8 markers

Explanation:
Fiona starts at the beginning of a hiking trail and walks \(\frac{4}{5}\) mile. She counts the mileage markers that are placed every \(\frac{1}{10}\) mile along the trail.
Number of markers = (4/5)/(1/10) = 4/5 × 10 = 8

Question 4.
If Amanda hikes at an average speed of 2.72 miles per hour, how long will it take her to hike 6.8 miles?
______ hours

Answer:
2.5 hours

Explanation:
speed times time = distance
distance = 6.8
speed = 2.72
time = s
2.72 times s = 6.8
divide both sides by 2.72
s = 2.5
The answer is 2.5 hours

Question 5.
The area of a rectangle is 5 \(\frac{4}{5}\) square meters. The width of the rectangle is 2 \(\frac{1}{4}\) meter. Which is the best estimate for the length of the rectangle?
______ meters

Answer:
2 \(\frac{26}{45}\) meters

Explanation:
Since the area of a rectangle is, A = l × b
A = 5 \(\frac{4}{5}\) square meters
b = 2 \(\frac{1}{4}\) meter
5 \(\frac{4}{5}\) = l × 2 \(\frac{1}{4}\)
l = \(\frac{29 × 4}{9 × 5}\) = \(\frac{116}{45}\) = 2 \(\frac{26}{45}\)

Question 6.
Lillian bought 2.52 pounds of tomatoes and 1.26 pounds of lettuce to make a salad for 18 people. If each person got the same amount of salad, how much salad did each person get?
______ pounds per person

Answer:
0.21 pounds per person

Explanation:
Lillian bought 2.52 pounds of tomatoes and 1.26 pounds of lettuce to make a salad for 18 people.
2.52 pounds + 1.26 pounds = 3.78 pounds of salad ÷ 18 people = 0.21 pounds of salad per person

Share and Show – Page No. 153

Graph the number on the horizontal number line.

Question 1.
2.25
Type below:
__________

Answer:
grade 6 chapter 3 image 6

Explanation:
-2.25 is in between -2 and -3.
-2.25 is between -2 and -2.5

Question 2.
1 \(\frac{5}{8}\)
Type below:
__________

Answer:
grade 6 chapter 3 image 7

Explanation:
-1 \(\frac{5}{8}\) is in between -1 and -2.
-1 \(\frac{5}{8}\) is closer to -2.

Question 3.
\(\frac{1}{2}\)
Type below:
__________

Answer:
grade 6 chapter 3 image 8

Explanation:
\(\frac{1}{2}\) is in between 0 and 1
\(\frac{1}{2}\) = 0.5

On Your Own

Practice: Copy and Solve Graph the number on a vertical number line.

Question 4.
0.6
Type below:
__________

Answer:
grade 6 chapter 3 image 13

Explanation:
0.6 is in between 0 and 1.
0.6 is closer to 1

Question 5.
1.25
Type below:
__________

Answer:
grade 6 chapter 3 image 10

Explanation:
-1.25 is in between -1 and -2
-1.25 is closer to -1.

Question 6.
1 \(\frac{1}{2}\)
Type below:
__________

Answer:
grade 6 chapter 3 image 11

Explanation:
-1 \(\frac{1}{2}\) is in between -1 and -2
-1 \(\frac{1}{2}\) = -1.5

Question 7.
0.3
Type below:
__________

Answer:
grade 6 chapter 3 image 14

Explanation:
0.3 is in between 0 and 1
0.3 is closer to 0

Question 8.
0.7
Type below:
__________

Answer:
grade 6 chapter 3 image 15

Explanation:
-0.7 is in between 0 and -1
-0.7 is closer to -1

Question 9.
1.4
Type below:
__________

Answer:
grade 6 chapter 3 image 16

Explanation:
1.4 is in between 1 and 2
1.4 is closer to 1

Question 10.
0.5
Type below:
__________

Answer:
grade 6 chapter 3 image 17

Explanation:
−0.5 is in between 0 and -1

Question 11.
− \(\frac{1}{4}\)
Type below:
__________

Answer:
grade 6 chapter 3 image 19

Explanation:
− \(\frac{1}{4}\) is in between 0 and -1
-0.25 is closer to 0

State whether the numbers are on the same or opposite sides of zero.

Question 12.
1.38 and 2.9
Type below:
__________

Answer:
Opposite

Explanation:
-1.38 is a negative number.
2.9 is a positive number.
So, both numbers are on opposite sides of zero.

Question 13.
3 \(\frac{9}{10}\) and 0.99
Type below:
__________

Answer:
Same

Explanation:
−3 \(\frac{9}{10}\) is a negative number.
−0.99 is a negative number.
So, both numbers are on the same sides of zero.

Question 14.
\(\frac{5}{6}\) and 4.713
Type below:
__________

Answer:
Opposite

Explanation:
−4.713 is a negative number.
\(\frac{5}{6}\) is a positive number.
So, both numbers are on opposite sides of zero.

Identify a decimal and a fraction in simplest form for the point.
Go Math Grade 6 Answer Key Chapter 3 Understand Positive and Negative Numbers img 8

Question 15.
Point A
Type below:
__________

Answer:
-1.0

Explanation:
The point A is located at -1.0 = -1

Question 16.
Point B
Type below:
__________

Answer:
0.75 = 3/4

Explanation:
Point B is between 0.5 and 1. It is 0.75

Question 17.
Point C
Type below:
__________

Answer:
-0.25 = 1/4

Explanation:
Point C is in between 0 and -0.5
-0.25

Question 18.
Point D
Type below:
__________

Answer:
-1.25 = 5/4

Explanation:
Point D is in between -1 and -1.5.
-1.25

Question 19.
The roots of 6 corn plants grew to 3.54 feet, 2 \(\frac{4}{5}\) feet, 3.86 feet, 4 \(\frac{1}{8}\) feet, 4.25 feet, and 2 \(\frac{2}{5}\) feet. How many corn plants had roots between 3 and 4 feet deep?
______ plants

Answer:
2 plants

Explanation:
The roots of 6 corn plants grew to −3.54 feet, −2 \(\frac{4}{5}\) feet, −3.86 feet, −4 \(\frac{1}{8}\) feet, −4.25 feet, and −2 \(\frac{2}{5}\) feet.
−3.54 feet, −3.86 feet,
2 corn plants had roots between 3 and 4 feet deep.

Problem Solving + Applications – Page No. 154

A star’s magnitude is a number that measures the star’s brightness. Use the table of star magnitudes for 20–22.
Go Math Grade 6 Answer Key Chapter 3 Understand Positive and Negative Numbers img 9

Question 20.
Between what two integers is the magnitude of Canopus?
Type below:
__________

Answer:
-0.72 is between -0.04 and -1.46

Explanation:
Canopus = -0.72
-0.72 is between -0.04 and -1.46

Question 21.
Model Mathematics
Graph the magnitude of Betelgeuse on the number line.
Type below:
__________

Answer:
grade 6 chapter 3 image 20

Explanation:
Betelgeuse = 0.7

Question 22.
What’s the Error?
Jacob graphed the magnitude of Sirius on the number line. Explain his error. Then graph the magnitude correctly.
Type below:
__________

Answer:
grade 6 chapter 3 image 21

Explanation:
Sirius = -1.46

Question 23.
The flag pole is located at point 0 on a map of Orange Avenue. Other points of interest on Orange Avenue are located on the number line based on their distances, in miles to the right of the flag pole (positive numbers) or to the left of the flag pole (negative numbers). Graph and label each location on the number line.
Go Math Grade 6 Answer Key Chapter 3 Understand Positive and Negative Numbers img 10
Type below:
__________

Answer:
grade 6 chapter 3 image 22

Explanation:
0.4 is the right side of the 0.
1.8 is the right side of the 0.
-1 is the left side of the 0.
-1.3 is the left side of the 0.

Rational Numbers and the Number Line – Page No. 155

Graph the number on the number line.

Question 1.
2 \(\frac{3}{4}\)
Type below:
__________

Answer:
grade 6 chapter 3 Page no. 155 image 1

Explanation:
The number is between the integers -3 and -2.
It is closer to the integer -3.

Question 2.
\(\frac{-1}{4}\)
Type below:
__________

Answer:
grade 6 chapter 3 Page no. 155 image 2

Explanation:
The number is between the integers -0.3 and -0.2.
It is closer to the integer -0.25.

Question 3.
0.5
Type below:
__________

Answer:
grade 6 chapter 3 Page no. 155 image 3JPG

Explanation:
The number is between integers 0 and -1.
It is closer to the integer -0.5.

Question 4.
1.75
Type below:
__________

Answer:
grade 6 chapter 3 Page no. 155 image 4

Explanation:
The number is between integers 1 and 2.
It is closer to the integer 1.75.

Question 5.
1 \(\frac{1}{2}\)
Type below:
__________

Answer:
grade 6 chapter 3 Page no. 155 image 5

Explanation:
The number is between integers 1 and 2.
It is closer to the integer 1.5.

State whether the numbers are on the same or opposite sides of zero.

Question 6.
2.4 and 2.3
Type below:
__________

Answer:
Opposite

Explanation:
-2.4 is a negative number.
2.3 is a positive number.
So, both numbers are on opposite sides of zero.

Question 7.
2 \(\frac{1}{5}\) and 1
Type below:
__________

Answer:
Same

Explanation:
−2 \(\frac{1}{5}\) is a negative number.
-1 is a negative number.
So, both numbers are on the same sides of zero.

Question 8.
0.3 and 0.3
Type below:
__________

Answer:
opposite

Explanation:
-0.3 is a negative number.
0.3 is a positive number.
So, both numbers are on opposite sides of zero.

Question 9.
0.44 and \(\frac{2}{3}\)
Type below:
__________

Answer:
Same

Explanation:
0.44 is a positive number.
\(\frac{2}{3}\) is a positive number.
So, both numbers are on the same sides of zero.

Write the opposite of the number.

Question 10.
5.23
Type below:
__________

Answer:
5.23

Explanation:
The opposite number of -5.23 is 5.23

Question 11.
\(\frac{4}{5}\)
Type below:
__________

Answer:
–\(\frac{4}{5}\)

Explanation:
The opposite number of \(\frac{4}{5}\) is –\(\frac{4}{5}\)

Question 12.
−5
Type below:
__________

Answer:
5

Explanation:
The opposite number of -5 is 5

Question 13.
2 \(\frac{2}{3}\)
Type below:
__________

Answer:
2 \(\frac{2}{3}\)

Explanation:
The opposite number of −2 \(\frac{2}{3}\) is 2 \(\frac{2}{3}\)

Problem Solving

Question 14.
The outdoor temperature yesterday reached a low of −4.5° F. Between what two integers was the temperature?
Type below:
__________

Answer:
An integer is a whole number. -4.5 is not a whole number.
-4.5 is in between -4 and the integer below it is -5.

Question 15.
Jacob needs to graph 6 \(\frac{2}{5}\) on a horizontal number line. Should he graph it to the left or right of 6?
Type below:
__________

Answer:
left

Explanation:
It will on the left because it is negative and on a number line the left side is the least side.

Question 16.
Describe how to plot 3 \(\frac{3}{4}\) on a number line.
Type below:
__________

Answer:
On the number line, negative numbers go to the left. Since -3 3/4 is negative, go 3 spaces to the left.
If there are half marks in between the numbers, plot the point near the half mark. If it’s -3 3/4, count 3 spaces, then go to the half mark of -3 to -4, and plot the 3/4 in between the -3 and -4 half mark. If there is no half mark, place it near the -4 mark.
The red line represents the half mark, the blue line represents where the point would go. Notice how when negative, the numbers go higher as they go left.
grade 6 chapter 3 Page no. 155 image 5

Lesson Check – Page No. 156

Question 1.
What number is the opposite of 0.2?
Type below:
__________

Answer:
-0.2

Explanation:
The opposite of 0.2 is -0.2

Question 2.
Between which two integers would you locate −3.4 on a number line?
Type below:
__________

Answer:
-3.4 is located between -3 and -4

Explanation:
Positive 3.4 lies between 3 and 4 on the number line. It is more than 3 but less than 4. 3.4 is further from 0 than just 3. In the same way and because of the symmetrical arrangement of numbers on the number line, -3.4 lies between -3 and -4.

Spiral Review

Question 3.
Yemi used these pattern blocks to solve a division problem. He found a quotient of 7. Which division problem was he solving?
Go Math Grade 6 Answer Key Chapter 3 Understand Positive and Negative Numbers img 11
Type below:
__________

Answer:
7

Explanation:
3 1/2 ÷ 1/2
First, we transform the mixed number into a fraction, 3 1/2 = 7/2
Then, we divide
7/2 ÷ 1/2 = 7
The quotient of the first division is 7.

Question 4.
Eric had 2 liters of water. He gave 0.42 liter to his friend and then drank 0.32 liter. How much water does he have left?
______ liters

Answer:
1.26 liters

Explanation:
Eric had 2 liters of water. He gave 0.42 liter to his friend and then drank 0.32 liter.
2 – 0.42 – 0.32 = 1.26 L

Question 5.
To pass a math test, students must correctly answer at least 0.6 of the questions. Donald’s score is \(\frac{5}{8}\), Karen’s score is 0.88, Gino’s score is \(\frac{3}{5}\) and Sierra’s score is \(\frac{4}{5}\). How many of the students passed the test?
Type below:
__________

Answer:
4

Explanation:
Donald’s score of 5/8 is equal to 0.625.
Gino scored 3/5 which is 0.6.
Sierra’s score of 4/5 equals 0.8.
Karen’s score is already given, and 0.88 is greater than 0.6.
None of the students obtained lower than 0.6.
If at least does not include scores equal to 0.6, and only scores greater than 0.6, then Gino possibly failed this math test.
If a passing score is equal to or greater than 0.6, then all four students passed the test.

Question 6.
Jonna mixes \(\frac{1}{4}\) gallon of orange juice and \(\frac{1}{2}\) gallon of pineapple juice to make punch. Each serving is \(\frac{1}{16}\) gallon. How many servings can Jonna make?
_____ servings

Answer:
12 servings

Explanation:
The number of a gallon of orange juice is mixed to make punch is given by 1/4
The number of a gallon of pineapple juice is mixed to make punch is given by 1/2
The number of gallon in each serving is given by 1/16
1/4 + 1/2 = 3/4
3/4 ÷ 1/16 = 12
So, there are 12 servings which can be made by Jonna.

Share and Show – Page No. 159

Compare the numbers. Write < or >.

Question 1.
0.3 _____ 0.2

Answer:
-0.3 < 0.2

Explanation:
-0.3 is to the left of 0.2 on the number line.
So, -0.3 is less than 0.2.

Question 2.
\(\frac{1}{3}\) _____ \(\frac{−2}{5}\)

Answer:
\(\frac{1}{3}\) > \(\frac{−2}{5}\)

Explanation:
\(\frac{1}{3}\) is to the right of \(\frac{−2}{5}\) on the number line.
So, \(\frac{1}{3}\) is greater than \(\frac{−2}{5}\).

Question 3.
0.8 _____ 0.5

Answer:
−0.8 < −0.5

Explanation:
-0.8 is to the left of -0.5 on the number line.
So, -0.8 is less than -0.5.

Question 4.
\(\frac{−3}{4}\) _____ −0.7

Answer:
\(\frac{−3}{4}\) < −0.7

Explanation:
\(\frac{−3}{4}\) is to the left of −0.7 on the number line.
So, \(\frac{−3}{4}\) is less than −0.7.

Order the numbers from least to greatest.

Question 5.
3.6, 7.1, 5.9
Type below:
__________

Answer:
-7.1, -5.9, 3.6

Explanation:
-7.1 is to the left of -5.9 on the number line. -5.9 is to the left of 3.6 on the number line.
So, -7.1 < -5.9 < 3.6

Question 6.
\(\frac{-6}{7}, \frac{1}{9}, \frac{-2}{3}\)
Type below:
__________

Answer:
\(\frac{-6}{7}, \frac{-2}{3}, \frac{1}{9}\)

Explanation:
-6/7 = -0.857
1/9 = 0.111
-2/3 = -0.666
-6/7 is to the left of -2/3 on the number line. -2/3 is to the left of 1/9 on the number line.
So, -6/7 < -2/3 < 1/9

Question 7.
5 \(\frac{1}{4}\), 6.5, 5.3
Type below:
__________

Answer:
-6.5, -5.3, −5 \(\frac{1}{4}\)

Explanation:
−5 \(\frac{1}{4}\) = -21/4 = -5.25
-6.5 is to the left of -5.3 on the number line. -5.3 is to the left of -5 \(\frac{1}{4}\) on the number line.
-6.5 < -5.3 < -5.25

On Your Own

Compare the numbers. Write < or >.

Question 8.
\(\frac{−1}{2}\) _____ \(\frac{−3}{7}\)

Answer:
\(\frac{−1}{2}\) < \(\frac{−3}{7}\)

Explanation:
\(\frac{−1}{2}\) = -0.5
\(\frac{−3}{7}\) = -0.428
\(\frac{−1}{2}\) is to the left of \(\frac{−3}{7}\) on the number line.
So, \(\frac{−1}{2}\) is less than \(\frac{−3}{7}\).

Question 9.
23.7 _____ 18.8

Answer:
−23.7 < −18.8

Explanation:
−23.7 is to the left of −18.8 on the number line.
So, −23.7 is less than −18.8.

Question 10.
3 \(\frac{1}{4}\) _____ 4.3

Answer:
−3 \(\frac{1}{4}\) > −4.3

Explanation:
−3 \(\frac{1}{4}\) = -13/4 = -3.25
−3 \(\frac{1}{4}\) is to the right of −4.3 on the number line.
So, −3 \(\frac{1}{4}\) is greater than −4.3.

Order the numbers from greatest to least.

Question 11.
2.4, 1.9, 7.6
Type below:
__________

Answer:
1.9, -2.4, -7.6

Explanation:
1.9 is to the right of -2.4 on the number line. -2.4 is to the right of -7.6 on the number line.
So, 1.9 > -2.4 > -7.6

Question 12.
\(\frac{-2}{5}, \frac{-3}{4}, \frac{-1}{2}\)
Type below:
__________

Answer:
\(\frac{-2}{5}, \frac{-1}{2}, \frac{-3}{4}\)

Explanation:
-2/5 = -0.4; -3/4 = -0.75; -1/2 = -0.5
-2/5 is to the right of -1/2 on the number line. -1/2 is to the right of -3/4 on the number line.
So, -2/5 > -1/2 > -3/4

Question 13.
3, 6 \(\frac{4}{5}\), 3 \(\frac{2}{3}\)
Type below:
__________

Answer:
3, −3 \(\frac{2}{3}\), −6 \(\frac{4}{5}\)

Explanation:
−6 \(\frac{4}{5}\) = -34/5 = -6.8
−3 \(\frac{2}{3}\) = -11/3 = -3.666
3 is to the right of -3 \(\frac{2}{3}\) on the number line. -3 \(\frac{2}{3}\) is to the right of −6 \(\frac{4}{5}\) on the number line.
So, 3 > −3 \(\frac{2}{3}\) > −6 \(\frac{4}{5}\)

Question 14.
Last week, Wednesday’s low temperature was −4.5°F, Thursday’s low temperature was −1.2°F, Friday’s low temperature was −2.7°F, and Saturday’s low temperature was 0.5°F. The average low temperature for the week was −1.5°F. How many of these days had low temperatures less than the average low temperature for the week?
_____ days

Answer:
2 days

Explanation:
Last week, Wednesday’s low temperature was −4.5°F, Thursday’s low temperature was −1.2°F, Friday’s low temperature was −2.7°F, and Saturday’s low temperature was 0.5°F. The average low temperature for the week was −1.5°F.
-4.5 < -1.5; -2.7 < -1.5
2 days had low temperatures less than the average low temperature for the week.

Question 15.
Use Symbols Write a comparison using < or > to show the relationship between an elevation of 12 \(\frac{1}{2}\) ft and an elevation of 16 \(\frac{5}{8}\) ft.
Type below:__________

Answer:
−12 \(\frac{1}{2}\) ft > −16 \(\frac{5}{8}\) ft

Explanation:
−12 \(\frac{1}{2}\) = -25/2 = -12.5
−16 \(\frac{5}{8}\) = -133/8 = -16.625
-12.5 > -16.625

Problem Solving + Applications – Page No. 160

Elevations, in miles, are given for the lowest points below sea level for 4 bodies of water. Use the table for 16–19.
Go Math Grade 6 Answer Key Chapter 3 Understand Positive and Negative Numbers img 12

Question 16.
The lowest point of which has the greater elevation, the Arctic Ocean or Lake Tanganyika?
Type below:
__________

Answer:
Arctic Ocean has the greater elevation

Explanation:
Arctic Ocean = -0.8
Lake Tanganyika = -0.9
-0.8 > -0.9
Arctic Ocean has the greater elevation

Question 17.
Which has a lower elevation, the lowest point of Lake Superior or a point at an elevation of \(\frac{2}{5}\) mi?
Type below:
__________

Answer:
Lake Superior has a lower elevation

Explanation:
Lake Superior = -1/4 = -0.25
\(\frac{2}{5}\) = 0.4
-0.25 < 0.4
Lake Superior has a lower elevation

Question 18.
List the elevations in order from least to greatest.
Type below:
__________

Answer:
-0.9, -0.8, -1/3, -1/4

Explanation:
Article Ocean = -0.8
Lake Superior = -1/4 = -0.25
Lake Tanganyika = -0.9
Red Sea = -1/3 = -0.333
-0.9 < -0.8 < -0.333 < -0.25

Question 19.
A shipwreck is found at an elevation of – 0.75 mile. In which bodies of water could the shipwreck have been found?
Type below:
__________

Answer:
Article Ocean

Explanation:
-0.75 is closer to -0.8
Article Ocean = -0.8

Question 20.
Circle <, >, or =.
20a. \(\frac{−3}{5}\) Ο \(\frac{−4}{5}\)
20b. \(\frac{−2}{5}\) Ο \(\frac{−3}{4}\)
20c. 6.5 Ο 4.2
20d. 2.4 Ο 3.7
\(\frac{−3}{5}\) _____ \(\frac{−4}{5}\)
\(\frac{−2}{5}\) _____ \(\frac{−3}{4}\)
6.5 _____ 4.7
2.4 _____ 3.7

Answer:
\(\frac{−3}{5}\) > \(\frac{−4}{5}\)
\(\frac{−2}{5}\) > \(\frac{−3}{4}\)
−6.5 < −4.7
−2.4 > −3.7

Explanation:
-3/5 = -0.6; -4/5 = -0.8
-0.6 > -0.8
-2/5 = -0.4; -3/4 = -0.75
-0.4 > -0.75
-6.5 < -4.7
-2.4 > -3.7

Compare and Order Rational Numbers – Page No. 161

Compare the numbers. Write < or >.

Question 1.
1\(\frac{1}{2}\) _____ \(\frac{−1}{2}\)

Answer:
−1\(\frac{1}{2}\) < \(\frac{−1}{2}\)

Explanation:
−1\(\frac{1}{2}\) = -3/2 = – 1.5
\(\frac{−1}{2}\) = -0.5
-1.5 < -0.5

Question 2.
0.1 _____ 1.9

Answer:
0.1 > −1.9

Explanation:
0.1 is to the right of -1.9 on the number line.
So, 0.1 is greater than -1.9.

Question 3.
0.4 _____ \(\frac{−1}{2}\)

Answer:
0.4 > \(\frac{−1}{2}\)

Explanation:
0.4 is to the right of \(\frac{−1}{2}\) on the number line.
So, 0.4 is greater than \(\frac{−1}{2}\).

Question 4.
\(\frac{2}{5}\) _____ 0.5

Answer:
\(\frac{2}{5}\) < 0.5

Explanation:
2/5 = 0.4
0.4 < 0.5

Order the numbers from least to greatest.

Question 5.
0.2, 1.7, 1
Type below:
__________

Answer:
-1.7, -1, 0.2

Explanation:
-1.7 is to the left of -1 on the number line. -1 is to the left of 0.2 on the number line.
So, -1.7 < -1 < 0.2

Question 6.
\(2 \frac{3}{4}, \frac{-3}{5}, 1 \frac{3}{4}\)
Type below:
__________

Answer:
\( \frac{-3}{5}, 1\frac{3}{4}, 2 \frac{3}{4}\)

Explanation:
2 3/4 = 11/4 = 2.75
-3/5 = – 0.6
1 3/4 = 7/4 = 1.75
-0.6 < 1.75 < 2.75

Question 7.
0.5, 1 \(\frac{2}{3}\), 2.7
Type below:
__________

Answer:
-2.7, −1 \(\frac{2}{3}\), -0.5

Explanation:
−1 \(\frac{2}{3}\) = -5/3 = -1.666
-2.7 < -1.66, -0.5

Order the numbers from greatest to least.

Question 8.
1, \(\frac{−5}{6}\), 0
Type below:
__________

Answer:
0, \(\frac{−5}{6}\), -1

Explanation:
\(\frac{−5}{6}\) = -0.8333
0 is to the right of \(\frac{−5}{6}\) on the number line. \(\frac{−5}{6}\) is to the right of -1 on the number line.
So, 0 > \(\frac{−5}{6}\) > -1

Question 9.
\(1.82, \frac{-2}{5}, \frac{4}{5}\)
Type below:
__________

Answer:
\(1.82, \frac{4}{5}, \frac{-2}{5}\)

Explanation:
-2/5 = -0.4
4/5 = 0.8
1.82
1.82 > 0.8 > -0.4

Question 10.
2.19, 2.5, 1.1
Type below:
__________

Answer:
1.1, -2.19, -2.5

Explanation:
1.1 is to the right of -2.19 on the number line. -2.19 is to the right of -2.5 on the number line.
So, 1.1 > -2.19 > -2.5

Write a comparison using < or > to show the relationship between the two values.

Question 11.
an elevation of −15 m and an elevation of −20.5 m
Type below:
__________

Answer:
-15m > -20.5m

Explanation:
-15 is to the right of -20.5 on the number line.
-15m > -20.5m

Question 12.
a balance of $78 and a balance of −$42
Type below:
__________

Answer:
$42 < $78

Explanation:
$42 is to the left of $78 on the number line.
So, $42 is less than $78.

Question 13.
a score of −31 points and a score of −30 points
Type below:
__________

Answer:
-31 points < -30 points

Explanation:
-31 is to the left of -30 on the number line.
So, -31 is less than -30.

Problem Solving

Question 14.
The temperature in Cold Town on Monday was 1°C. The temperature in Frosty Town on Monday was −2°C. Which town was colder on Monday?
Type below:
__________

Answer:
Frosty Town

Explanation:
The temperature in Cold Town on Monday was 1°C. The temperature in Frosty Town on Monday was −2°C.
Frosty Town town was colder on Monday.

Question 15.
Stan’s bank account balance is less than −$20.00 but greater than −$21.00. What could Stan’s account balance be?
Type below:
__________

Answer:
From -$20.99 to -$20.01

Explanation:
Stan’s bank account balance is less than −$20.00 but greater than −$21.00. The possible answer is From -$20.99 to -$20.01

Question 16.
Describe two situations in which it would be helpful to compare or order positive and negative rational numbers.
Type below:
__________

Answer:
1) negative numbers in weather reports and on food packaging. The temperature -5°C is ‘negative five degrees’ and it means 5 degrees below zero.
2) When you spend more money than you have in your bank account it shows up as a negative number.

Lesson Check – Page No. 162

Question 1.
The low temperature was —1.8 °C yesterday and −2.1 °C today. Use the symbols < or > to show the relationship between the temperatures.
Type below:
__________

Answer:
The low temperature was —1.8 °C yesterday and −2.1 °C today.
-1.8 > -2.1

Question 2.
The scores at the end of a game are shown. List the scores in order from greatest to least.
Vince: −0.5
Allison: \(\frac{3}{8}\)
Mariah: \(\frac{−7}{20}\)
Type below:
__________

Answer:
\(\frac{3}{8}\), -0.5, \(\frac{−7}{20}\)

Explanation:
\(\frac{3}{8}\) = 0.375
\(\frac{−7}{20}\) = -0.35
-0.5
\(\frac{3}{8}\) > -0.5 > -0.35

Spiral Review

Question 3.
Simone bought 3.42 pounds of green apples and 2.19 pounds of red apples. She used 3 pounds to make a pie. How many pounds of apples are left?
_____ pounds

Answer:
2.61 pounds

Explanation:
She bought 3.42 pounds of green apples, then you can subtract 3 lbs off of that, so she bought .42 lbs of green apples and 2.19 lbs red apples
So now, you just need to add .42 and 2.19
.42 + 2.19 = 2.61, so she has 2.61 lbs of apples left

Question 4.
Kwan bought three rolls of regular wrapping paper with 6.7 square meters of paper each. He also bought a roll of fancy wrapping paper containing 4.18 square meters. How much paper did he have altogether?
_____ square meters

Answer:
24.28 square meters

Explanation:
He bought 3 rolls of regular wrapping paper with 6.7 m². Then the total of this paper is: 3 × 6.7 = 20.1
He also bought a roll of fancy wrapping with 4.18 m². Therefore, to calculate the amount of paper he had together (which you can call ), you must add 20.1 m² and 4.18 m²,
x = 20.1 + 41.8 = 24.28

Question 5.
Eddie needs 223 cups of flour for one batch of pancakes. How much flour does he need for 212 batches?
_____ \(\frac{□}{□}\) cups

Answer:
6\(\frac{4}{6}\) cups

Explanation:
For 1 batch of pancake = 2 2/3 = 8/3 cups
For 2 1/2 = 5/2 pancake = 8/3 × 5/2 = 40/6 cups = 6 4/6 cups

Question 6.
Tommy notices that he reads \(\frac{2}{3}\) page in a minute. At that rate, how long will it take him to read 12 pages?
_____ minutes

Answer:
18 minutes

Explanation:
It will take him 18 minutes.
2/3 of a page in 18 minutes= 12 pages read

Mid-Chapter Checkpoint – Vocabulary – Page No. 163

Choose the best term from the box to complete the sentence.
Go Math Grade 6 Answer Key Chapter 3 Understand Positive and Negative Numbers img 13

Question 1.
Any number that can be written as \(\frac{a}{b}\), where a and b are integers and b≠0 is called a(n) _____.
Type below:
__________

Answer:
rational number

Question 2.
The set of whole numbers and their opposites is the set of _____.
Type below:
__________

Answer:
Integers

Concepts and Skills

Write the opposite of the integer.

Question 3.
72
Type below:
__________

Answer:
72

Explanation:
The integer −72 is on the left side of 0.
So, the opposite of -72 is 72

Question 4.
0
Type below:
__________

Answer:
0

Explanation:

Opposite of 0 is 0

Question 5.
31
Type below:
__________

Answer:
31

Explanation:
The integer −31 is on the left side of 0.
So, the opposite of -31 is 31

Question 6.
27
Type below:
__________

Answer:

Explanation:
The integer 27 is on the right side of 0.
So, the opposite of 27 is -27

Name the integer that represents the situation, and tell what 0 represents in that situation.
Go Math Grade 6 Answer Key Chapter 3 Understand Positive and Negative Numbers img 14

Question 7.
Type below:
__________

Answer:
Integer: 278
0 represents: Neither loses or gains in the video game.

Question 8.
Type below:
__________

Answer:
Integer: -8 degrees
0 represents: No change in the temperature.

Compare the numbers. Write < or >.

Question 9.
3 _____ 4

Answer:
3 > −4

Explanation:
3 is to the right of -4 on the number line.
So, 3 is greater than -4.

Question 10.
6 _____ 5

Answer:
−6 < −5

Explanation:
-6 is to the left of -5 on the number line.
So, -6 is less than -5.

Question 11.
5 _____ 6

Answer:
5 > −6

Explanation:
5 is to the right of -6 on the number line.
So, 5 is greater than -6.

Question 12.
\(\frac{1}{3}\) _____ \(\frac{1}{2}\)

Answer:
\(\frac{1}{3}\) < \(\frac{1}{2}\)

Explanation:
\(\frac{1}{3}\) is to the left of \(\frac{1}{2}\) on the number line.
So, \(\frac{1}{3}\) is less than \(\frac{1}{2}\).

Question 13.
3.1 _____ 4.3

Answer:
−3.1 >−4.3

Explanation:
-3.1 is to the right of -4.3 on the number line.
So, -3.1 is greater than -4.3.

Question 14.
1\(\frac{3}{4}\) _____ 2\(\frac{1}{2}\)

Answer:
1\(\frac{3}{4}\) >−2\(\frac{1}{2}\)

Explanation:
1\(\frac{3}{4}\) is to the right of −2\(\frac{1}{2}\) on the number line.
So, 1\(\frac{3}{4}\) is greater than −2\(\frac{1}{2}\).

Order the numbers.

Question 15.
5, 2, 8
Type below:
__________

Answer:
-8, -2, 5

Explanation:
-8 is to the left of -2 on the number line. -2 is to the left of 5 on the number line.
So, -8 < -2 < 5

Question 16.
0, 3, 1
Type below:
__________

Answer:
-3, 0, 1

Explanation:
-3 is to the left of 0 on the number line. 0 is to the left of 1 on the number line.
So, -3 < 0 < 1

Question 17.
7, 6, 11
Type below:
__________

Answer:
-11, -7, -6

Explanation:
-11 is to the left of -7 on the number line. -7 is to the left of -6 on the number line.
So, -11 < -7 < -6

Question 18.
2.5, 1.7, 4.3
Type below:
__________

Answer:
-4.3, -1.7, 2.5

Explanation:
-4.3 is to the left of -1.7 on the number line. -1.7 is to the left of 2.5 on the number line.
So, -4.3 < -1.7 < 2.5

Question 19.
\(\frac{2}{3} \cdot \frac{1}{4}, \frac{5}{12}\)
Type below:
__________

Answer:
\(\frac{1}{4} \cdot \frac{5}{12}, \frac{2}{3}\)

Explanation:
2/3 = 0.666
1/4 = 0.25
5/12 = 0.4166
1/4 < 5/12 < 2/3

Question 20.
5.2, 3.8, 9.4
Type below:
__________

Answer:
−9.4, −5.2, −3.8

Explanation:
-9.4 is to the left of -5.2 on the number line. -5.2 is to the left of -3.8 on the number line.
So, -9.4 < -5.2 < -3.8

Page No. 164

Question 21.
Judy is scuba diving at −7 meters, Nelda is scuba diving at −9 meters, and Rod is scuba diving at −3 meters. List the divers in order from the deepest diver to the diver who is closest to the surface.
Type below:
__________

Answer:
Judy is scuba diving at −7 meters, Nelda is scuba diving at −9 meters, and Rod is scuba diving at −3 meters.
the higher the value of the negative number, the deepest the diver is.
Nelda (-9)- Judy (-7) -Rod (-3)

Question 22.
A football team gains 8 yards on their first play. They lose 12 yards on the next play. What two integers represent the two plays?
Type below:
__________

Answer:
A football team gains 8 yards on their first play. +8
They lose 12 yards on the next play. -12
The 2 integers are positive 8 and negative 12

Question 23.
The player who scores the closest to 0 points wins the game. The scores of four players are given in the table. Who won the game?
Go Math Grade 6 Answer Key Chapter 3 Understand Positive and Negative Numbers img 15
Type below:
__________

Answer:
Donovan won the game

Explanation:
Donovan because he has-1.5
Myra has -1.93
Amari has -1.66666666
Justine has -1.8
-1.5 is the closest to 0

Question 24.
Which point on the graph represents 3 \(\frac{3}{4}\) ? What number does point C represent?
Go Math Grade 6 Answer Key Chapter 3 Understand Positive and Negative Numbers img 16
Type below:
__________

Answer:
A

Explanation:
−3 \(\frac{3}{4}\) = -15/4 = -3.75
-3.75 is in between -3 and -4.
So, point A is the correct answer

Share and Show – Page No. 167

Find the absolute value.

Question 1.
|2|
Type below:
__________

Answer:
2

Explanation:
The distance from 0 to the point I graphed is 2 units.
|−2| = 2

Question 2.
|6|
Type below:
__________

Answer:
6

Explanation:
The distance from 0 to the point I graphed is 6 units.
|6| = 6

Question 3.
|5|
Type below:
__________

Answer:
5

Explanation:
The distance from 0 to the point I graphed is 6 units.
|-5| = 5

Question 4.
|11|
Type below:
__________

Answer:
11

Explanation:
The distance from 0 to the point I graphed is 6 units.
|-11| = 11

Question 5.
|9|
Type below:
__________

Answer:
9

Explanation:
The distance from 0 to the point I graphed is 6 units.
|9| = 9

Question 6.
|15|
Type below:
__________

Answer:
15

Explanation:
The distance from 0 to the point I graphed is 6 units.
|-15| = 15

On Your Own

Find the absolute value.

Question 7.
|37|
Type below:
__________

Answer:
37

Explanation:
The distance from 0 to the point I graphed is 6 units.
|-37| = 37

Question 8.
|1.8|
Type below:
__________

Answer:
1.8

Explanation:
The distance from 0 to the point I graphed is 6 units.
|1.8| = 1.8

Question 9.
|\(\frac{−2}{3}\)|
Type below:
__________

Answer:
|\(\frac{2}{3}\)|

Explanation:
The distance from 0 to the point I graphed is 6 units.
||\(\frac{−2}{3}\)|| = |\(\frac{2}{3}\)|

Question 10.
|6.39|
Type below:
__________

Answer:
6.39

Explanation:
The distance from 0 to the point I graphed is 6 units.
|-6.39| = 6.39

Question 11.
|5\(\frac{7}{8}\)|
Type below:
__________

Answer:
5\(\frac{7}{8}\)

Explanation:
The distance from 0 to the point I graphed is 5\(\frac{7}{8}\) units.
|−5\(\frac{7}{8}\)| = 5\(\frac{7}{8}\)

Find all numbers with the given absolute value.

Question 12.
13
Type below:
__________

Answer:
13 and -13

Explanation:
13 and -13 are at the same distance from 0.

Question 13.
\(\frac{5}{6}\)
Type below:
__________

Answer:
\(\frac{5}{6}\) and \(\frac{-5}{6}\)

Explanation:
\(\frac{5}{6}\) and \(\frac{-5}{6}\) are at the same distance from 0.

Question 14.
14.03
Type below:
__________

Answer:
14.03 and -14.03

Explanation:
14.03 and -14.03 are at the same distance from 0.

Question 15.
0.59
Type below:
__________

Answer:
0.59 and -0.59

Explanation:
0.59 and -0.59 are at the same distance from 0.

Question 16.
3\(\frac{1}{7}\)
Type below:
__________

Answer:
3\(\frac{1}{7}\) and -3\(\frac{1}{7}\)

Explanation:
3\(\frac{1}{7}\) and -3\(\frac{1}{7}\) are at the same distance from 0.

Use Reasoning Algebra Find the missing number or numbers to make the statement true.

Question 17.
|?| = 10
Type below:
__________

Answer:
10 and -10

Explanation:
|-10| = 10
|10| = 10

Question 18.
|?| = 1.78
Type below:
__________

Answer:
1.78 and -1.78

Explanation:
|-1.78| = 1.78
|1.78| = 1.78

Question 19.
|?| = 0
Type below:
__________

Answer:
0

Explanation:
|0| = 0

Question 20.
|?| = \(\frac{15}{16}\)
Type below:
__________

Answer:
\(\frac{-15}{16}\) and \(\frac{15}{16}\)

Explanation:
|\(\frac{-15}{16}\)| = \(\frac{15}{16}\)
|\(\frac{15}{16}\)| = \(\frac{15}{16}\)

Question 21.
Find all of the integers whose absolute value is less than |–4|.
Type below:
__________

Answer:
3, 2, 1, 0

Explanation:
The absolute value of |–4| = 4.
3, 2, 1, 0 are the integers whose absolute value is less than |–4|.

Unlock The Problem – Page No. 168

Question 22.
The Blue Ridge Trail starts at Park Headquarters in Big Bear Park and goes up the mountain. The Green Creek Trail starts at Park Headquarters and goes down the mountain. The table gives elevations of various points of interest in relation to Park Headquarters. How many points of interest are less than 1 kilometer above or below Park Headquarters?
Go Math Grade 6 Answer Key Chapter 3 Understand Positive and Negative Numbers img 17
a. How can you find how far above or below Park Headquarters a given point of interest is located?
Type below:
__________

Answer:
By knowing the values below 1 km can help you to find how far above or below Park Headquarters a given point of interest is located

Question 22.
b. How can you find the number of points of interest that are less than 1 km above or below Park Headquarters?
Type below:
__________

Answer:
By counting the number of points of interest that are less than 1 km, you can find the number of points of interest that are less than 1 km above or below Park Headquarters.

Question 22.
c. Find how far above or below Park Headquarters each point of interest is located.
Type below:
__________

Answer:
C, D, E, F, G, H

Question 22.
d. How many points of interest are less than 1 kilometer above or below Park Headquarters?
Type below:
__________

Answer:
6

Question 23.
Use Reasoning Name a rational number that can replace ? to make both statements true.
?>3             |?|<|3|
Type below:
__________

Answer:
-2 or -1 >−3          1 or 2 < |−3|

Explanation:
The greatest numbers than -3 are -2 or -1.
|−3| = 3. So, the fewer numbers than 3 are 1, 2

Question 24.
Laila said |4| equals |−4|. Is Laila correct? Use the number line and words to support your answer.
Type below:
__________

Answer:
Laila is correct. The absolute value of |−4| = 4 = |4|

Absolute Value – Page No. 169

Find the absolute value.

Question 1.
|7|
Type below:
__________

Answer:
7

Explanation:

The distance from 0 to the point I graphed is 2 units.
|7| = 7

Question 2.
|8|
Type below:
__________

Answer:
8

Explanation:
The distance from 0 to the point I graphed is 2 units.
|−8| = 8

Question 3.
|16|
Type below:
__________

Answer:
16

Explanation:
The distance from 0 to the point I graphed is 2 units.
|16| = 16

Question 4.
|8.65|
Type below:
__________

Answer:
8.65

Explanation:
The distance from 0 to the point I graphed is 2 units.
|8.65| = 8.65

Question 5.
|4\(\frac{3}{20}\)|
Type below:
__________

Answer:
4\(\frac{3}{20}\)

Explanation:
The distance from 0 to the point I graphed is 2 units.
|4\(\frac{3}{20}\)| = 4\(\frac{3}{20}\)

Question 6.
|5000|
Type below:
__________

Answer:
5000

Explanation:
The distance from 0 to the point I graphed is 2 units.
|−5000| = 5000

Find all numbers with the given absolute value.

Question 7.
12
Type below:
__________

Answer:
12 and -12

Explanation:
12 and -12 are at the same distance from 0.

Question 8.
1.7
Type below:
__________

Answer:
1.7 and -1.7

Explanation:
1.7 and -1.7 are at the same distance from 0.

Question 9.
\(\frac{3}{5}\)
Type below:
__________

Answer:
\(\frac{3}{5}\) and \(\frac{-3}{5}\)

Explanation:
\(\frac{3}{5}\) and \(\frac{-3}{5}\) are at the same distance from 0.

Question 10.
3\(\frac{1}{6}\)
Type below:
__________

Answer:
3\(\frac{1}{6}\) and -3\(\frac{1}{6}\)

Explanation:
3\(\frac{1}{6}\) and -3\(\frac{1}{6}\) are at the same distance from 0.

Question 11.
0
Type below:
__________

Answer:
0

Explanation:
0 is same distance from 0.

Find the number or numbers that make the statement true.

Question 12.
|?| = 17
Type below:
__________

Answer:
17 and -17

Explanation:
|-17| = 17
|17| = 17

Question 13.
|?| = 2.04
Type below:
__________

Answer:
2.04 and -2.04

Explanation:
|-2.04| = 2.04
|2.04| = 2.04

Question 14.
|?| = 1\(\frac{9}{10}\)
Type below:
__________

Answer:
1\(\frac{9}{10}\) and -1\(\frac{9}{10}\)

Explanation:
|-1\(\frac{9}{10}\)| = 1\(\frac{9}{10}\)
|1\(\frac{9}{10}\)| = 1\(\frac{9}{10}\)

Question 15.
|?| = \(\frac{19}{24}\)
Type below:
__________

Answer:
\(\frac{19}{24}\) and \(\frac{-19}{24}\)

Explanation:
|\(\frac{-19}{24}\)| = \(\frac{19}{24}\)
|\(\frac{19}{24}\)| = \(\frac{19}{24}\)

Problem Solving

Question 16.
Which two numbers are 7.5 units away from 0 on a number line?
Type below:
__________

Answer:
7.5 and -7.5 are away from 0 on a number line

Explanation:
|7.5| = 7.5
|-7.5| = 7.5

Question 17.
Emilio is playing a game. He just answered a question incorrectly, so his score will change by −10 points. Find the absolute value of −10.
Type below:
__________

Answer:
10

Explanation:
Emilio is playing a game. He just answered a question incorrectly, so his score will change by −10 points.
|-10| = 10

Question 18.
Write two different real-world examples. One should involve the absolute value of a positive number, and the other should involve the absolute value of a negative number.
Type below:
__________

Answer:
1) If we have a balance of -$35 dollars in an account, we may also choose to represent that as a debt of $35.
2) The temperature of the human body

Lesson Check – Page No. 170

Question 1.
What is the absolute value of \(\frac{8}{9}\)?
Type below:
__________

Answer:
\(\frac{8}{9}\)

Explanation:
|\(\frac{8}{9}\)| = \(\frac{8}{9}\)

Question 2.
What two numbers have an absolute value of 21.63?
Type below:
__________

Answer:
21.63 and -21.63

Explanation:
|-21.63| = 21.63
|21.63| = 21.63

Spiral Review

Question 3.
Rachel earned $89.70 on Tuesday. She spent $55.89 at the grocery store. How much money does she have left?
$ ______

Answer:
$33.81

Explanation:
Rachel earned $89.70 on Tuesday. She spent $55.89 at the grocery store.
89.70 – 55.89 = 33.81
Rachel has $33.81 left

Question 4.
One carton contains \(\frac{17}{20}\) liter of juice. Another carton contains 0.87 liter of juice. Which carton contains the most?
Type below:
__________

Answer:
0.87 is more because 17/20 is 0.85

Explanation:
One carton contains \(\frac{17}{20}\) liter of juice. Another carton contains 0.87 liters of juice.
0.87 is more because 17/20 is 0.85

Question 5.
Maggie jogged \(\frac{7}{8}\) mile on Monday and \(\frac{1}{2}\) of that distance on Tuesday. How far did she jog on Tuesday?
\(\frac{□}{□}\) mile

Answer:
\(\frac{7}{4}\) mile

Explanation:
Maggie jogged \(\frac{7}{8}\) mile on Monday and \(\frac{1}{2}\) of that distance on Tuesday.
\(\frac{7}{8}\) ÷ \(\frac{1}{2}\) = 7/4
7/4 or as a mixed fraction which is 1 3/4 mile

Question 6.
Trygg has \(\frac{3}{4}\) package of marigold seeds. He plants \(\frac{1}{6}\) of those seeds in his garden and divides the rest equally into 10 flowerpots. What fraction of a package of seeds is planted in each flowerpot?
\(\frac{□}{□}\) package

Answer:
\(\frac{1}{16}\) package

Explanation:
He has a 3/4 package and plants 1/6 of the seeds.
3/4 × 1/6 = 1/8
He divides the rest equally into 10 flowerpots.
Subtract 1/8 from 3/4.
The common denominator of 4 and 8 is 8.
Multiply the numerator 3 × 2= 6 with a denominator of 8.
3/4 – 1/8 = 6/8 -1/8 = 5/8
5/8 is left to be divided equally into 10 flowerpots.
5/8 ÷ 10/1
= 5/8 * 1/10
= 5/80
= 1/16

Share and Show – Page No. 173

Question 1.
On Monday, Allie’s bank account balance was – $24. On Tuesday, her account balance was less than it was on Monday. Use absolute value to describe Allie’s balance on Tuesday as a debt.
Type below:
__________

Answer:
On Tuesday, her account balance is less than -$24 means her debt will be bigger than $24 dollars.

Explanation:
On Monday, allies’ bank account balance was -$24.
Balance being negative means he is carrying a debt of $24.
On Tuesday, Allie’s balance account was less than it was on Monday. It means
Her bank account < -$24
So, she must be carrying a debit bigger than $24.
Therefore, on Tuesday, her account balance being less than -$24 means her debt will be greater than $24 dollars.

Question 2.
Matthew scored −36 points in his turn at a video game. In Genevieve’s turn, she scored fewer points than Matthew. Use absolute value to describe Genevieve’s score as a loss.
Type below:
__________

Answer:
Genevieve lost more than 36 points

Explanation:
Matthew scored −36 points in his turn at a video game. In Genevieve’s turn, she scored fewer points than Matthew.
-36 > -40
|-36| < |-40|
36 < 40
Genevieve lost more than 36 points

On Your Own

Question 3.
One of the cats shown in the table is a tabby. The tabby had a decrease in weight of more than 3.3 ounces. Which cat is the tabby?
Go Math Grade 6 Answer Key Chapter 3 Understand Positive and Negative Numbers img 18
Type below:
__________

Answer:
Spot is tabby

Explanation:
|-3.4| = 3.4
So, Spot is tabby

Compare. Write <, >, or =.

Question 4.
−8 _____ |8|

Answer:
−8 < |−8|

Explanation:
|−8| = 8
-8 < 8

Question 5.
13 _____ |13|

Answer:
13 = |−13|

Explanation:
|−13| = 13
13 = 13

Question 6.
|23| _____ |24|

Answer:
|−23| < |−24|

Explanation:
|−23| = 23
|−24| = 24
23 < 24

Question 7.
15 _____ |14|

Answer:
15 > |−14|

Explanation:
|−14| = 14
15 > 14

Question 8.
34 _____ |36|

Answer:
34 < |−36|

Explanation:
|−36| = 36
34 < 36

Question 9.
−5 _____ |6|

Answer:
−5 < |−6|

Explanation:
|−6| = 6
-5 < 6

Question 10.
Write the values in order from least to greatest.
Go Math Grade 6 Answer Key Chapter 3 Understand Positive and Negative Numbers img 19
Type below:
__________

Answer:
1, 2, 3, 6

Explanation:
|-2| = 2
|3| = 3
|-6| = 6
|1| = 1
1 < 2 < 3 < 6

Compare and Contrast – Page No. 174

When you compare and contrast, you look for ways that two or more subjects are alike (compare) and ways they are different (contrast). This helps you to discover information about each subject that you might not have known otherwise. As you read the following passage, think about how the main topics are alike and how they are different.

Trevor mows lawns after school to raise money for a new mountain bike. Last week, it rained every day, and he couldn’t work. While waiting for better weather, he spent some of his savings on lawnmower repairs. As a result, his savings balance changed by −$45. This week, the weather was better, and Trevor returned to work. His savings balance changed by +$45 this week.

Question 11.
The passage has two main parts. Describe them.
Type below:
__________

Answer:
Last week, Trevor couldn’t work, so he spent money to repair the lawnmower!
This week, he goes back to work and earns money again!

Question 12.
Describe the two changes in Trevor’s savings balance
Type below:
__________

Answer:
His savings balance changed by −$45 in one week and his savings balance changed by +$45 in another week.

Question 13.
Reason Quantitatively Compare the two changes in Trevor’s savings balance. How are they alike?
Type below:
__________

Answer:
Each week, Trevor’s balance changed by $45; or his balance is the same distance from 0 each week.

Question 14.
Contrast the two changes in Trevor’s savings balance. How are they different?
Type below:
__________

Answer:
The balances are different because one week the balance had a decrease, while the next week there was an increase in the balance

Compare Absolute Values – Page No. 175

Solve.

Question 1.
Jamie scored −5 points on her turn at a trivia game. In Veronica’s turn, she scored more points than Jamie. Use absolute value to describe Veronica’s score as a loss.
Type below:
__________
Jamie scored −5 points on her turn at a trivia game. In Veronica’s turn, she scored more points than Jamie.

Answer:
In this situation, |-5| represents a loss of 5 points. Veronica lost fewer than 5 points.

Question 2.
The low temperature on Friday was −10°F. The low temperature on Saturday was colder. Use absolute value to describe the temperature on Saturday as a temperature below zero.
Type below:
__________

Answer:
The temperature on Sunday was more than 10 degrees below zero

Explanation:
The low temperature on Friday was −10°F. The low temperature on Saturday was colder. The temperature on Sunday was more than 10 degrees below zero

Question 3.
The table shows changes in the savings accounts of five students. Which student had the greatest increase in money? By how much did the student’s account increase?
Go Math Grade 6 Answer Key Chapter 3 Understand Positive and Negative Numbers img 20
Type below:
__________

Answer:
Carissa; an increase of $15

Compare. Write <, >, or =.

Question 4.
16 _____ |16|

Answer:
−16 < |−16|

Explanation:
|−16| = 16
-16 < 16

Question 5.
20 _____ 20

Answer:
20 = 20

Question 6.
3 _____ |4|

Answer:
3 < |−4|

Explanation:
|−4| = 4
3 < 4

Problem Solving

Question 7.
On Wednesday, Miguel’s bank account balance was −$55. On Thursday, his balance was less than that. Use absolute value to describe Miguel’s balance on Thursday as a debt.
Type below:
__________

Answer:
In this situation, -$55 represents a debt of $55. On Thursday, Miguel had a debt of more than $55.

Explanation:
On Wednesday, Miguel’s bank account balance was −$55. On Thursday, his balance was less than that.
In this situation, -$55 represents a debt of $55. On Thursday, Miguel had a debt of more than $55.

Question 8.
During a game, Naomi lost points. She lost fewer than 3 points. Use an integer to describe her possible score.
Type below:
__________

Answer:
-2, -1

Explanation:
During a game, Naomi lost points. She lost fewer than 3 points.
It may be -2, -1

Question 9.
Give two numbers that fit this description: a number is less than another number but has a greater absolute value. Describe how you determined the numbers.
Type below:
__________

Answer:
Choose a large negative number and a smaller positive number.
Example: Use -14 and 3, -8392 and 274, -1 and 0.5, etc. Even though the negative numbers are technically less, they would have higher absolute values.

Lesson Check – Page No. 176

Question 1.
A temperature of –6° is colder than a temperature of 5°F below zero. Is this statement true or false?
Type below:
__________

Answer:
True

Explanation:
–6° is colder than a temperature of 5°F below zero

Question 2.
Long Beach, California has an elevation of −7 feet. New Orleans, Louisiana is 8 feet below sea level. Which city has a lower elevation?
Type below:
__________

Answer:
New Orleans, Louisiana has a lower elevation

Explanation:
Long Beach, California has an elevation of −7 feet.
New Orleans, Louisiana is 8 feet below sea level. = -8 feet
So, New Orleans, Louisiana has a lower elevation.

Spiral Review

Question 3.
Dawn and Lin took off on skateboards from the same location but traveled in opposite directions. After 20 minutes, Dawn had traveled 6.42 kilometers and Lin had traveled 7.7 kilometers. How far apart were they?
_____ kilometers

Answer:
14.12 kilometers

Explanation:
Distance of Dawn = 6.42 km
Distance from Lin = 7.7 km in the opposite direction.
If they went in opposite directions then they were moving away from each other.
The final distance between the two, d = 6.42 + 7.7 = 14.12 km
After 20 minutes Dawn and Lin were 14.12 km away.

Question 4.
Rico and Josh took off on skateboards going in the same direction. After 20 minutes, Rico had traveled 5.98 kilometers and Josh had gone 8.2 kilometers. How far apart were they?
_____ kilometers

Answer:
2.22 kilometers

Explanation:
Rico and Josh took off on skateboards going in the same direction.
After 20 minutes, Rico had traveled 5.98 kilometers and Josh had gone 8.2 kilometers.
D = 8.2 – 5.98 = 2.22 km
Hence, Rico and Josh were 2.22 km apart from each other.

Question 5.
Etta bought 11.5 yards of fabric selling for $0.90 per yard. What was the total cost?
$ _____

Answer:
$10.35

Explanation:
Multiply 11.5 times 0.90 and get $10.35

Question 6.
Yen calculates the product \(\frac{5}{8} \times \frac{24}{25}\). Before he multiplies, he simplifies all factors. What does the problem look like after he simplifies the factors?
Type below:
__________

Answer:
Yen calculates the product \(\frac{5}{8} \times \frac{24}{25}\).
5/8 = 0.625
24/25 = 0.96
0.625 × 0.96 = 0.6

Share and Show – Page No. 179

Go Math Grade 6 Answer Key Chapter 3 Understand Positive and Negative Numbers img 21

Question 1.
Write the ordered pair for point J.
Type below:
__________

Answer:
(-1.5, 2.5)

Explanation:
To find the x-coordinate, move 1.5 units to the left.
To find the y-coordinate, move 2.5 units up.
Point J is located at (-1.5, 2.5)

Write the ordered pair for the point.

Question 2.
K
Type below:
__________

Answer:
(1, -1.5)

Explanation:
To find the x-coordinate, move 1 unit to the right.
To find the y-coordinate, move 1.5 units down.
Point K is located at (1, -1.5)

Question 3.
L
Type below:
__________

Answer:
(-2, -1.75)

Explanation:
To find the x-coordinate, move 2 units to the left.
To find y-coordinate, move 1.75 units down.
Point L is located at (-2, -1.75)

Question 4.
M
Type below:
__________

Answer:
(1, 0)

Explanation:
To find the x-coordinate, move 1 unit to the right.
To find y-coordinate, move 0 units.
Point M is located at (1, 0)

Graph and label the point on the coordinate plane.

Question 5.
P(-2.5, 2)
Type below:
__________

Answer:
The x-coordinate is negative. Move 2.5 units to the left.
y-coordinate is positive. Move 2 units up

Question 6.
Q(-2, \(\frac{1}{4}\))
Type below:
__________

Answer:
The x-coordinate is negative. Move 2 units to the left.
y-coordinate is positive. Move 0.25 units up

Question 7.
R(0, 1.5)
Type below:
__________

Answer:
The x-coordinate is positive. Move 0 units.
y-coordinate is positive. Move 1.5 units up

Question 8.
S(-1, \(\frac{-1}{2}\))
Type below:
__________

Answer:
The x-coordinate is negative. Move 1 unit to the left.
y-coordinate is negative. Move 0.5 units down

Question 9.
T( 1\(\frac{1}{2}\), -2 )
Type below:
__________

Answer:
The x-coordinate is positive. Move 1.5 units to the right.
y-coordinate is negative. Move 2 units down

Question 10.
U(0.75, 1.25)
Type below:
__________

Answer:
The x-coordinate is positive. Move 0.75 units to the right.
y-coordinate is positive. Move 1.25 units up

Question 11.
V(-0.5, 0)
Type below:
__________

Answer:
The x-coordinate is negative. Move 0.5 units to the left.
y-coordinate is positive. Move 0 units

Question 12.
W(2, 0)
Type below:
__________

Answer:
The x-coordinate is positive. Move 2 units to the right.
y-coordinate is positive. Move 0 units up

Question 13.
X(0, -2)
Type below:
__________

Answer:
The x-coordinate is positive. Move 0 units.
y-coordinate is negative. Move 2 units down

grade 6 chapter 3 Page no. 175 image 1

On Your Own

Write the ordered pair for the point. Give approximate coordinates when necessary.
Go Math Grade 6 Answer Key Chapter 3 Understand Positive and Negative Numbers img 22

Question 14.
A
Type below:
__________

Answer:
(4, 4)

Explanation:
To find the x-coordinate, move 4 units to the right.
To find y-coordinate, move 4 units up.
Point A is located at (4, 4)

Question 15.
B
Type below:
__________

Answer:
(-4, 3)

Explanation:
To find the x-coordinate, move 4 units to the left.
To find y-coordinate, move 3 units up.
Point B is located at (-4, 3)

Question 16.
C
Type below:
__________

Answer:
(-3, 1)

Explanation:
To find the x-coordinate, move 3 units to the left.
To find y-coordinate, move 1 unit up.
Point C is located at (-3, 1)

Question 17.
D
Type below:
__________

Answer:
(-2, -3)

Explanation:
To find the x-coordinate, move 2 units to the left.
To find y-coordinate, move 3 units down.
Point D is located at (-2, -3)

Question 18.
E
Type below:
__________

Answer:
(5, -3)

Explanation:
To find the x-coordinate, move 5 units to the right.
To find y-coordinate, move 3 units down.
Point E is located at (5, -3)

Question 19.
F
Type below:
__________

Answer:
(2.5, 0)

Explanation:
To find the x-coordinate, move 2.5 units to the right.
To find y-coordinate, move 0 units.
Point F is located at (2.5, 0)

Question 20.
G
Type below:
__________

Answer:
(-4, -5)

Explanation:
To find the x-coordinate, move 4 units to the left.
To find y-coordinate, move 5 units down.
Point G is located at (-4, -5)

Question 21.
H
Type below:
__________

Answer:
(0, 3.5)

Explanation:
To find the x-coordinate, move 0 units.
To find y-coordinate, move 3.5 units up.
Point H is located at (0, 3.5)

Question 22.
J
Type below:
__________

Answer:
(0.5, 0.5)

Explanation:
To find the x-coordinate, move 0.5 units to the right.
To find y-coordinate, move 0.5 units up.
Point J is located at (0.5, 0.5)

Graph and label the point on the coordinate plane.

Question 23.
M(-4, 0)
Type below:
__________

Answer:
The x-coordinate is negative. Move 4 units to the left.
y-coordinate is positive. Move 0 units

Question 24.
N(2, 2)
Type below:
__________

Answer:
The x-coordinate is positive. Move 2 units to the right.
y-coordinate is positive. Move 2 units up

Question 25.
P(-3, 3)
Type below:
__________

Answer:
The x-coordinate is positive. Move 3 units to the left.
y-coordinate is positive. Move 3 units up

Question 26.
Q(0, −2\(\frac{1}{2}\))
Type below:
__________

Answer:
The x-coordinate is positive. Move 0 units.
y-coordinate is negative. Move 2.5 units down

Explanation:

Question 27.
R(0.5, 0.5)
Type below:
__________

Answer:
The x-coordinate is positive. Move 0.5 units to the right.
y-coordinate is positive. Move 0.5 units up

Question 28.
S(-5, \(\frac{1}{2}\))
Type below:
__________

Answer:
The x-coordinate is negative. Move 5 units to the left.
y-coordinate is positive. Move 0.5 units up

Question 29.
T(0, 0)
Type below:
__________

Answer:
It is at the origin. T is at the origin

Question 30.
U(3 \(\frac{1}{2}\), 0)
Type below:
__________

Answer:
The x-coordinate is positive. Move 3.5 units to the right.
y-coordinate is positive. Move 0 units

Question 31.
V(-2, -4)
Type below:
__________

Answer:
The x-coordinate is negative. Move 2 units to the left.
y-coordinate is negative. Move 4 units down

grade 6 chapter 3 Page no. 175 image 2

Question 32.
Look for Structure A point lies to the left of the y-axis and below the x-axis. What can you conclude about the coordinates of the point?
Type below:
__________

Answer:
A point lies to the left of the y-axis. So, the x-coordinate is negative.
A point lies below the x-axis. So, y-coordinate is negative.
Both coordinates points are negative

Problem Solving + Applications – Page No. 180

Many of the streets in downtown Philadelphia can be modeled by a coordinate plane, as shown on the map. Each unit on the map represents one block. Use the map for 33 and 34.
Go Math Grade 6 Answer Key Chapter 3 Understand Positive and Negative Numbers img 23

Question 33.
Anita works at the Historical Society. She leaves the building and walks 3 blocks north to a restaurant. What ordered pair represents the restaurant?
Type below:
__________

Answer:
Anita works at the Historical Society. She leaves the building and walks 3 blocks north to a restaurant.
Historical Society = (2, 4)
As she walks 3 blocks north to a restaurant 4-3 = 1
(2, 1) ordered pair represents the restaurant

Question 34.
Pose a Problem Write and solve a new problem that uses a location on the map.
Type below:
__________

Answer:
Anita is at City Hall. She walked 3 blocks to the East and 2 blocks to the North. What ordered pair represents her present location?
She is at the Fabric Workshop & Museum. The ordered pair is (3, 2)

Question 35.
The points A, B, C, and D on a coordinate plane can be connected to form a rectangle. Point A is located at (2, 0), point B is located at (6, 0), and point C is located at (6, –2.5). Write the ordered pair for point D.
Type below:
__________

Answer:
The point D is at (2, -2.5)

Explanation:
grade 6 chapter 3 Page no. 180 image 1
The points A, B, C, and D on a coordinate plane can be connected to form a rectangle. Point A is located at (2, 0), point B is located at (6, 0), and point C is located at (6, –2.5). The point D is at (2, -2.5)

Question 36.
Identify Relationships Explain how you can tell that the line segment connecting two points is vertical without graphing the points.
Type below:
__________

Answer:
The line segment connecting two points is vertical. By calculating the slope, we can say that the line segment connecting two points is vertical without graphing the points.

Question 37.
For numbers 37a–37d, select True or False for each statement.
37a. Point A (2, –1) is to the right of the y-axis and below the x-axis.
37b. Point B (– 5,2) is to the left of the y-axis and below the x-axis.
37c. Point C (3, 2) is to the right of the y-axis and above the x-axis.
37d. Point D (–2, –1) is to the left of the y-axis and below the x-axis.
37a. __________
37b. __________
37c. __________
37d. __________

Answer:
37a. True
37b. False
37c. True
37d. True

Rational Numbers and the Coordinate Plane – Page No. 181

Write the ordered pair for the point. Give approximate coordinates when necessary.
Go Math Grade 6 Answer Key Chapter 3 Understand Positive and Negative Numbers img 24

Question 1.
A
Type below:
__________

Answer:
(1, 0.5)

Explanation:
To find the x-coordinate, move 1 unit to the right.
To find y-coordinate, move 0.5 units up.
Point A is located at (1, 0.5)

Question 2.
B
Type below:
__________

Answer:
(-0.75, -2.5)

Explanation:
To find the x-coordinate, move 0.75 units to the left.
To find y-coordinate, move 2.5 units down.
Point B is located at (-0.75, -2.5)

Question 3.
C
Type below:
__________

Answer:
(2, -1.5)

Explanation:
To find the x-coordinate, move 2 units to the right.
To find y-coordinate, move 1.5 units down.
Point C is located at (2, -1.5)

Question 4.
D
Type below:
__________

Answer:
(-1.5, 0)

Explanation:
To find the x-coordinate, move 1.5 units to the left.
To find y-coordinate, move 0 units.
Point D is located at (-1.5, 0)

Graph and label the point on the coordinate plane.

Question 5.
G(−\(\frac{1}{2}\), 1 \(\frac{1}{2}\))
Type below:
__________

Answer:
The x-coordinate is negative. Move 0.5 units to the left.
y-coordinate is positive. Move 1.5 units up

Explanation:
-1/2 = -0.5
1 1/2 = 3/2 = 1.5

Question 6.
H(0, 2.50)
Type below:
__________

Answer:
The x-coordinate is positive. Move 0 units.
y-coordinate is positive. Move 2.5 units up

Question 7.
J(−1 \(\frac{1}{2}\), \(\frac{1}{2}\))
Type below:
__________

Answer:
The x-coordinate is negative. Move 1.5 units to the left.
y-coordinate is positive. Move 0.5 units up

Explanation:
-1 1/2 = -3/2 = -1.5
1/2 = 0.5

Question 8.
K(1, 2)
Type below:
__________

Answer:
The x-coordinate is positive. Move 1 unit to the right.
y-coordinate is positive. Move 2 units up

Question 9.
L(−1 \(\frac{1}{2}\), −2 \(\frac{1}{2}\))
Type below:
__________

Answer:
The x-coordinate is negative. Move 1.5 units to the left.
y-coordinate is negative. Move 2.5 units down

Explanation:
-1 1/2 = -3/2 = -1.5
-2 1/2 = -5/2 = -2.5

Question 10.
M(1, -0.5)
Type below:
__________

Answer:
The x-coordinate is positive. Move 1 unit to the up.
y-coordinate is negative. Move 0.5 units down

Question 11.
N(\(\frac{1}{4}\), 1 \(\frac{1}{2}\))
Type below:
__________

Answer:
The x-coordinate is positive. Move 0.25 units to the right.
y-coordinate is positive. Move 1.5 units up

Question 12.
P(1.25, 0)
Type below:
__________

Answer:
The x-coordinate is positive. Move 1.25 units to the right.
y-coordinate is positive. Move 0 units

grade 6 chapter 3 Page no. 180 image 2

Problem Solving

Use the map for 13–15.
Go Math Grade 6 Answer Key Chapter 3 Understand Positive and Negative Numbers img 25

Question 13.
What is the ordered pair for the city hall?
Type below:
__________

Answer:
(-1, 0.5)

Explanation:
To find the x-coordinate, move 1 unit to the left.
To find y-coordinate, move 0.5 units up.
City Hall is located at (-1, 0.5)

Question 14.
The post office is located at (\(\frac{−1}{2}\), 2). Graph and label a point on the map to represent the post office.
Type below:
__________

Answer:
grade 6 chapter 3 Page no. 181 image 1

Explanation:
The x-coordinate is negative. Move 0.5 units to the left.
y-coordinate is positive. Move 2 units up

Question 15.
Describe how to graph the ordered pair (−1, 4.5).
Type below:
__________

Answer:
The x-coordinate is negative. Move 1 unit to the left.
y-coordinate is positive. Move 4.5 units up

Lesson Check – Page No. 182

Question 1.
An artist uses a coordinate plane to create a design. As part of the design, the artist wants to graph the point (−6.5, 2). How should the artist graph this point?
Type below:
__________

Answer:
The Artist should go 6.5 units to the left on the x-axis and then 2 units up on the y-axis giving:
graph {((x + 6.5)^2 + (y – 2)^2 – 0.0125) = 0 [-10, 5, -5, 2.5]}

Question 2.
What are the coordinates of the campground?
Go Math Grade 6 Answer Key Chapter 3 Understand Positive and Negative Numbers img 26
Type below:
__________

Answer:
(-1, -1.5)

Explanation:
The x-coordinate is negative. Move 1 unit to the left.
y-coordinate is negative. Move 1.5 units down

Spiral Review

Question 3.
Four students volunteer at the hospital. Casey volunteers 20.7 hours, Danielle 20 \(\frac{3}{4}\) hours, Javier 18 \(\frac{9}{10}\) hours, and Forrest, 20 \(\frac{18}{25}\) hours. Who volunteered the greatest number of hours?
__________

Answer:
Danielle volunteered the greatest number of hours

Explanation:
Danielle volunteered the most. She did 20.75 hours while Forest had 20.72, Casey had 20.7 and Javier had the least at 18.90 hours.

Question 4.
Directions for making a quilt say to cut fifteen squares with sides that are 3.625 inches long. What is the side length written as a fraction?
_____ \(\frac{□}{□}\)

Answer:
3\(\frac{5}{8}\)

Explanation:
3.625 = 3 0.625
3.625 = 3 0.625(1000)/1000
3.625 = 3 625/1000
3.625 = 3 (625/125) / (1000/125)
3.625 = 3 5/8
The decimal length of the side of the squares (3.625 inches long) witten as a fraction is 3 5/8 inches long.

Question 5.
Cam has a piece of plywood that is 6 \(\frac{7}{8}\) feet wide. He is going to cut shelves from the plywood that are each 1 \(\frac{1}{6}\) feet wide. Which is a good estimate for the number of shelves Cam can make?
Type below:
__________

Answer:
5 shelves

Explanation:
6 7/8= 55/8
6 1/6= 7/6
first, find common denominators
55/8 × 3= 165/24
7/6 × 4= 28/24
165/24 divided by 28/24 is the same as 165/24 time 24/28
so that equals 3960/672 = 5.8928
About 5 shelves

Question 6.
Zach has \(\frac{3}{4}\) hour to play video games. It takes him \(\frac{1}{12}\) hour to set up the system. Each round of his favorite game takes \(\frac{1}{6}\) hour. How many rounds can he play?
_____ rounds

Answer:
4 rounds

Explanation:
Zach has 3/4 hour to play video games. it takes him 1/12 hour to set up the system. Each round of his favorite game takes 1/6 hours.
1) Zach has 3/4 hour to play video games
Convert to minutes
1 hour = 60 minutes
3/4 × 60 = 45 minutes
2) It takes him an hour to set up the system
Convert to minutes
1/12 × 60 = 5 minutes
3) 45 minutes – 5 minutes = 40 minutes
4) Each round of his favorite game takes an hour
Convert to minutes
1/6 × 60 = 10 minutes
5) Divide the time available to play video games by the time each round of his favorite game
40/10 = 4 rounds

Share and Show – Page No. 185

Identify the quadrant where the point is located.

Question 1.
(2, -5)
Type below:
__________

Answer:
Quadrant IV

Explanation:
The x-coordinate is positive. So, the point is 2 units to the right of the origin.
Since the point is to the right of the origin, it must be located in either
Quadrant I or Quadrant IV
The y-coordinate is negative, so the point is 5 units down from the origin.
Since the point is down the origin, it must be located in Quadrant IV.
Check by graphing the point (2, -5) on the coordinate plane.
Quadrant IV.

Question 2.
(4, 1)
Type below:
__________

Answer:
Quadrant I

Explanation:
The x-coordinate is positive. So, the point is 4 units to the right of the origin.
Since the point is to the right of the origin, it must be located in either
Quadrant I or Quadrant IV
y-coordinate is positive, so the point is 1 unit up from the origin.
Since the point is up to the origin, it must be located in Quadrant I.
Check by graphing the point (4, 1) on the coordinate plane.
Quadrant I.

Question 3.
(-6, -2)
Type below:
__________

Answer:
Quadrant III

Explanation:
The x-coordinate is negative. So, the point is 6 units to the left of the origin.
Since the point is to the left of the origin, it must be located in either
Quadrant II or Quadrant III
The y-coordinate is negative, so the point is 2 units down from the origin.
Since the point is down the origin, it must be located in Quadrant III.
Quadrant III.

Question 4.
(-7, 3)
Type below:
__________

Answer:
Quadrant II

Explanation:
The x-coordinate is negative. So, the point is 7 units to the left of the origin.
Since the point is to the left of the origin, it must be located in either
Quadrant II or Quadrant III
The y-coordinate is positive, so the point is 3 units up from the origin.
Since the point is up to the origin, it must be located in Quadrant II.
Quadrant II.

Question 5.
(8, 8)
Type below:
__________

Answer:
Quadrant I

Explanation:
The x-coordinate is positive. So, the point is 8 units to the right of the origin.
Since the point is to the right of the origin, it must be located in either
Quadrant I or Quadrant IV
The y-coordinate is positive, so the point is 8 units up from the origin.
Since the point is up to the origin, it must be located in Quadrant I.
Quadrant I.

Question 6.
(1, -1)
Type below:
__________

Answer:
Quadrant IV

Explanation:
The x-coordinate is positive. So, the point is 1 unit to the right of the origin.
Since the point is to the right of the origin, it must be located in either
Quadrant I or Quadrant IV
The y-coordinate is negative, so the point is 1 unit down from the origin.
Since the point is down to the origin, it must be located in Quadrant IV.
Quadrant IV.

The two points are reflections of each other across the x- or y-axis. Identify the axis.

Question 7.
(-1, 3) and (1, 3)
Type below:
__________

Answer:
y-axis

Explanation:
The given x-axis points are -1 and 1
The y-axis points are 3 and 3
The y-axis points are reflections to each other

Question 8.
(4, 4) and (4, -4)
Type below:
__________

Answer:
x-axis

Explanation:
The given x-axis points are 4 and 4
The y-axis points are 4 and -4
The x-axis points are reflections to each other

Question 9.
(2, -9) and (2, 9)
Type below:
__________

Answer:
x-axis

Explanation:
The given x-axis points are 2 and 2
The y-axis points are -9 and 9
The x-axis points are reflections to each other

Question 10.
(8, 1) and (-8, 1)
Type below:
__________

Answer:
y-axis

Explanation:
The given x-axis points are 8 and -8
The y-axis points are 1 and 1
The y-axis points are reflections to each other

On Your Own

Identify the quadrant where the point is located.

Question 11.
(-8, -9)
Type below:
__________

Answer:
Quadrant III

Explanation:
The x-coordinate is negative. So, the point is 8 units to the left of the origin.
Since the point is to the left of the origin, it must be located in either
Quadrant II or Quadrant III
The y-coordinate is negative, so the point is 9 units down from the origin.
Since the point is down to the origin, it must be located in Quadrant III.
Quadrant III.

Question 12.
(12, 1)
Type below:
__________

Answer:
Quadrant I

Explanation:
The x-coordinate is positive. So, the point is 12 units to the right of the origin.
Since the point is to the right of the origin, it must be located in either
Quadrant I or Quadrant IV
The y-coordinate is positive, so the point is 1 unit up from the origin.
Since the point is up to the origin, it must be located in Quadrant I.
Quadrant I.

Question 13.
(-13, 10)
Type below:
__________

Answer:
Quadrant II

Explanation:
The x-coordinate is negative. So, the point is 13 units to the left of the origin. Since the point is to the left of the origin, it must be located in either
Quadrant II or Quadrant III
The y-coordinate is negative, so the point is 10 units up from the origin.
Since the point is up to the origin, it must be located in Quadrant II.
Quadrant II.

Question 14.
(5, -20)
Type below:
__________

Answer:
Quadrant IV

Explanation:
The x-coordinate is positive. So, the point is 5 units to the right of the origin.
Since the point is to the right of the origin, it must be located in either
Quadrant I or Quadrant IV
The y-coordinate is negative, so the point is 20 units down from the origin.
Since the point is down to the origin, it must be located in Quadrant IV.
Quadrant IV.

The two points are reflections of each other across the x- or y-axis. Identify the axis.

Question 15.
(-9, -10) and (-9, 10)
Type below:
__________

Answer:
x-axis

Explanation:
The given x-axis points are -9 and -9
The y-axis points are -10 and 10
The x-axis points are reflections to each other

Question 16.
(21, -31) and (21, 31)
Type below:
__________

Answer:
x-axis

Explanation:
The given x-axis points are 21 and 21
The y-axis points are -31 and 31
The x-axis points are reflections to each other

Question 17.
(15, -20) and (-15, -20)
Type below:
__________

Answer:
y-axis

Explanation:
The given x-axis points are 15 and -15
The y-axis points are -20 and -20
The y-axis points are reflections to each other

Give the reflection of the point across the given axis.

Question 18.
(−7, −7), y-axis
Type below:
__________

Answer:
(7, -7)

Explanation:
The x-axis point is -7.
So, the reflection of point 7

Question 19.
(−15, 18), x-axis
Type below:
__________

Answer:
(-15, -18)

Explanation:
The y-axis point is 18.
So, the reflection of a point -18

Question 20.
(11, 9), x-axis
Type below:
__________

Answer:
(11, -9)

Explanation:
The y-axis point is 9.
So, the reflection of a point -9

Problem Solving + Applications – Page No. 186

Use the map of Gridville for 21–23.

Question 21.
The library’s location has opposite x- and y-coordinates as City Hall. Across which streets could you reflect City Hall’s location to find the library’s location?
Type below:
__________

Answer:
The library’s location has opposite x- and y-coordinates as City Hall.
City Hall = (2, -3)
The opposite x- and y-coordinates of City Hall = (-2, 3)
So, the library’s location is (-2, 3)

Question 22.
Each unit on the map represents 1 mile. Gregory leaves his house at (−5, 4), cycles 4 miles east, 6 miles south, and 1 mile west. In which quadrant of the city is he now?
Type below:
__________

Answer:
Quadrant III

Explanation:
Gregory leaves his house at (−5, 4)
cycles 4 miles east = -5 + 4 = -1; (-1, 4)
6 miles south = (-1, -1)
1 mile west (-2, -1)
So, he is now in Quadrant III

Question 23.
The bus station has the same x-coordinate as City Hall but the opposite y-coordinate. In which quadrant of the city is the bus station located?
Type below:
__________

Answer:
Quadrant I

Explanation:
The bus station has the same x-coordinate as City Hall but the opposite y-coordinate.
City Hall = (2, -3)
The opposite y-coordinate = 3
Bus station located at (2, 3)
Bus station located at Quadrant I

Question 24.
Describe Relationships Describe the relationship between the locations of the points (2, 5) and (2, −5) on the coordinate plane.
Type below:
__________

Answer:
(2, 5) and (2, −5) have the same x-coordinate.
They have the opposite y-coordinate.

Question 25.
Identify the quadrant where each point is located. Write each point in the correct box.
(−1, 3), (4, −2), (−3, −2), (1, −3), (−1, 2), (3, 4)
Type below:
__________

Answer:
(−1, 3) = Quadrant II
(4, −2) = Quadrant IV
(−3, −2) = Quadrant III
(1, −3) = Quadrant IV
(−1, 2) = Quadrant II
(3, 4) = Quadrant I

Problem Solving + Applications – Page No. 187

Identify the quadrant where the point is located.

Question 1.
(10, -2)
Type below:
__________

Answer:
Quadrant IV

Explanation:
The x-coordinate is positive. So, the point is 10 units to the right of the origin.
Since the point is to the right of the origin, it must be located in either
Quadrant I or Quadrant IV
The y-coordinate is negative, so the point is 2 units down from the origin.
Since the point is down the origin, it must be located in Quadrant IV.
Quadrant IV

Question 2.
(-5, -6)
Type below:
__________

Answer:
Quadrant III

Explanation:
The x-coordinate is negative. So, the point is 5 units to the left of the origin.
Since the point is to the left of the origin, it must be located in either
Quadrant II or Quadrant III
The y-coordinate is negative, so the point is 6 units down from the origin.
Since the point is down the origin, it must be located in Quadrant III.
Quadrant III.

Question 3.
(3, 7)
Type below:
__________

Answer:
Quadrant I

Explanation:
The x-coordinate is positive. So, the point is 3 units to the right of the origin.
Since the point is to the right of the origin, it must be located in either
Quadrant I or Quadrant IV
y-coordinate is positive, so the point is 7 units up from the origin.
Since the point is up to the origin, it must be located in Quadrant I.
Quadrant I

The two points are reflections of each other across the x- or y-axis. Identify the axis.

Question 4.
(5, 3) and (−5, 3)
Type below:
__________

Answer:
y-axis

Explanation:
The given x-axis points are 5 and -5
The y-axis points are 3 and 3
The y-axis points are reflections to each other

Question 5.
(−7, 1) and (−7, −1)
Type below:
__________

Answer:
x-axis

Explanation:
The given x-axis points are -7 and -7
The y-axis points are 1 and -1
The x-axis points are reflections to each other

Question 6.
(−2, 4) and (−2, −4)
Type below:
__________

Answer:
x-axis

Explanation:
The given x-axis points are -2 and -2
The y-axis points are 4 and -4
The x-axis points are reflections to each other

Give the reflection of the point across the given axis.

Question 7.
(−6, −10), y-axis
Type below:
__________

Answer:
(6, -10)

Explanation:
The x-axis point is -6.
So, the reflection of the point 6

Question 8.
(−11, 3), x-axis
Type below:
__________

Answer:
(-11, -3)

Explanation:
The y-axis point is -3.
So, the reflection of a point 3

Question 9.
(8, 2), x-axis
Type below:
__________

Answer:
(8, -2)

Explanation:
The y-axis point is 2.
So, the reflection of a point -2

Problem Solving

Question 10 .
A town’s post office is located at the point (7, 5) on a coordinate plane. In which quadrant is the post office located?
Type below:
__________

Answer:
Quadrant I

Explanation:
A town’s post office is located at the point (7, 5) on a coordinate plane.
The x-coordinate is positive. So, the point is 7 units to the right of the origin.
Since the point is to the right of the origin, it must be located in either
Quadrant I or Quadrant IV
The y-coordinate is positive, so the point is 5 units up from the origin.
Since the point is up the origin, it must be located in Quadrant I.
Quadrant I

Question 11.
The grocery store is located at a point on a coordinate plane with the same y-coordinate as the bank but with the opposite x-coordinate. The grocery store and bank are reflections of each other across which axis?
Type below:
__________

Answer:
y-axis

Explanation:
The grocery store is located at a point on a coordinate plane with the same y-coordinate as the bank but with the opposite x-coordinate.
The grocery store and bank are reflections of each other across the y-axis.

Question 12.
Explain to a new student how a reflection across the y-axis changes the coordinates of the original point.
Type below:
__________

Answer:
The coordinate plane with the same y-coordinate remains same but with the opposite x-coordinate.

Lesson Check – Page No. 188

Question 1.
In which quadrant does the point (−4, 15) lie?
Type below:
__________

Answer:
Quadrant II

Explanation:
The x-coordinate is negative. So, the point is 4 units to the left of the origin.
Since the point is to the left of the origin, it must be located in either
Quadrant II or Quadrant III
The y-coordinate is positive, so the point is 15 units up from the origin.
Since the point is up to the origin, it must be located in Quadrant II.
Quadrant II.

Question 2.
What are the coordinates of the point (10, −4) if it is reflected across the y–axis?
Type below:
__________

Answer:
(-10, -4)

Explanation:
coordinates of the point (-10, −4)
If it is reflected across the y–axis, coordinates of the point will be (-10, -4)

Spiral Review

Question 3.
Small juice bottles come in packages of 6. Yogurt treats come in packages of 10. Paula wants to have the exact same number of each item. What is the least number of bottles of juice and individual yogurt treats she will have? How many packages of each will she need?
Type below:
__________

Answer:
5 packages

Explanation:
Number of packages of small juice bottles = 6
Number of packages of yogurt = 10
For this, we will find the L.C.M. of 6 and 10 =30
So, there will be 5 packages of small juice bottles and 3 packages of yogurt.

Question 4.
Alison saves $29.26 each month. How many months will it take her to save enough money to buy a stereo for $339.12?
_____ month

Answer:
11 months

Explanation:
Round 29 and 339 to 30 and 340.
Divide 340 by 30
The answer should be 11.3 repeatings.
11 months

Question 5.
The library is 1.75 miles directly north of the school. The park is 0.6 miles directly south of the school. How far is the library from the park?
Type below:
__________

Answer:
2.35 miles

Explanation:
The library is 1.75 miles directly north of the school. The park is 0.6 miles directly south of the school.
1.75 + 0.6 would be 2.35 miles, the library to the parking.

Question 6.
Tours of the art museum are offered every \(\frac{1}{3}\) hour starting at 10 A.M. The museum closes at 4:00 P.M. How many tours are offered each day?
_____ tours

Answer:
18 tours

Explanation:
Staring time of tours=10 am
Closing time of tours=4 pm
Duration of tours(10 am to 4 pm)=6 hours
Time for each tour= 1/3 hours
Total number of tours offered in a day= 6 × 3 = 18
Therefore, 18 tours offered each day.

Share and Show – Page No. 191

Find the distance between the pair of points.
Go Math Grade 6 Answer Key Chapter 3 Understand Positive and Negative Numbers img 27

Question 1.
(−3, 1) and (2, 1)
_____ units

Answer:
5 units

Explanation:
The points have the same y-coordinate, so they are located on a horizontal line.
Find the distance between the x-coordinates of the point (-3, 1) and the point (2, 1)
|-3| = 3
3 + 2 = 5

Question 2.
(2, 1) and (2, -4)
_____ units

Answer:
5 units

Explanation:
The points have the same x-coordinate, so they are located on a vertical line.
Find the distance between the y-coordinates of the point (2, 1) and the point (2, -4)
|-4| = 4
1 + 4 = 5

Question 3.
(2, -4) and (4, -4)
_____ units

Answer:
2 units

Explanation:
The points have the same y-coordinate, so they are located on a horizontal line.
Find the distance between the x-coordinates of the point (2, -4) and the point (4, -4)
4 – 2 = 2

Question 4.
(-3, 3) and (-3, 1)
_____ units

Answer:
2 units

Explanation:
The points have the same x-coordinate, so they are located on a vertical line.
Find the distance between the y-coordinates of the point (-3, 3) and the point (-3, 1)
3 – 1 = 2

On Your Own

Practice: Copy and Solve Graph the pair of points. Then find the distance between them.

Question 5.
(0, 5) and (0, -5)
_____ units

Answer:
10 units

Explanation:
The points have the same x-coordinate, so they are located on a vertical line.
Find the distance between the y-coordinates of the point (0, 5) and the point (0, -5)
|-5| = 5
5 + 5 = 10

Question 6.
(1, 1) and (1, -3)
_____ units

Answer:
4 units

Explanation:
The points have the same x-coordinate, so they are located on a vertical line.
Find the distance between the y-coordinates of the point (1, 1) and the point (1, -3)
|-3| = 3
1 + 3 = 4

Question 7.
(-2, -5) and (-2, -1)
_____ units

Answer:
4 units

Explanation:
The points have the same x-coordinate, so they are located on a vertical line.
Find the distance between the y-coordinates of the point (-2, -5) and the point (-2, -1)
|-5| = 5
|-1| = 1
5 – 1 = 4

Question 8.
(-7, 3) and (5, 3)
_____ units

Answer:
12 units

Explanation:
The points have the same y-coordinate, so they are located on a horizontal line.
Find the distance between the x-coordinates of the point (-7, 3) and the point (5, 3)
|-7| = 7
7 + 5 = 12

Question 9.
(3, -6) and (3, -10)
_____ units

Answer:
4 units

Explanation:
The points have the same x-coordinate, so they are located on a vertical line.
Find the distance between the y-coordinates of the point (3, -6) and the point (3, -10)
|-6| = 6
|-10| = 10
10 – 6 = 4

Question 10.
(8, 0) and (8, -8)
_____ units

Answer:
8 units

Explanation:
The points have the same x-coordinate, so they are located on a vertical line.
Find the distance between the y-coordinates of the point (8, 0) and the point (8, -8)
|-8| = 8
0 + 8 = 8

Use Reasoning Algebra Write the coordinates of a point that is the given distance from the given point.

Question 11.
4 units from (3, 5)
Type below:
__________

Answer:
1 or 9

Explanation:
4 units from (3, 5)
(3, 9) or (3, 1)

Question 12.
6 units from (2, 1)
Type below:
__________

Answer:
8 or -4

Explanation:
6 units from (2, 1)
(8, 1) or (-4, 1)

Question 13.
7 units from (−4, −1)
Type below:
__________

Answer:
-8 or 6

Explanation:
7 units from (−4, −1)
(-8, -1) or (-8, 6)

Problem Solving + Applications – Page No. 192

An archaeologist is digging at an ancient city. The map shows the locations of several important finds. Each unit represents 1 kilometer. Use the map for 14–18.

Question 14.
How far is it from the stadium to the statue?
Go Math Grade 6 Answer Key Chapter 3 Understand Positive and Negative Numbers img 28
_____ km

Answer:
8 km

Explanation:
Stadium = (4, 5)
statue = (4, -3)
The points have the same x-coordinate, so they are located on a vertical line.
Find the distance between the y-coordinates of the point (4, 5) and the point (4, -3)
|-3| = 3
5 + 3 = 8

Question 15.
The archaeologist drives 3 km south from the palace. How far is he from the market?
_____ km

Answer:
3 km

Explanation:
The palace is at the origin (0, 0)
So, the answer is 3 km

Question 16.
The archaeologist’s campsite is located at (−9, −3). How far is it from the campsite to the market?
_____ km

Answer:
11 km

Explanation:
the campsite is located at (−9, −3)
the market is located at (-2, -3)
The points have the same y-coordinate, so they are located on a horizontal line.
Find the distance between the x-coordinates of the point (−9, −3) and the point (-2, -3)
|-9| = 9
|-2| = 2
9 + 2 = 11
campsite is 11 km far to the market

Question 17.
The archaeologist rode east on a donkey from the Great Gate, at (−11, 4), to the Royal Road. Then he rode south to the palace. How far did the archaeologist ride?
_____ km

Answer:
15 km

Explanation:
The archaeologist rode east on a donkey from the Great Gate, at (−11, 4), to the Royal Road. Then he rode south to the palace.
First, he needs to move |-11| = 11 km
Then, he needs to move 4 km
4 + 11 = 15 km

Question 18.
Generalize Explain how you could find the distance from the palace to any point on the Imperial Highway.
Type below:
__________

Answer:
The distance varies according to the points on the Imperial Highway

Question 19.
Select the pairs of points that have a distance of 10 between them. Mark all that apply.
(3, −6) and (3, 4)
( −3, 8) and (7, 8)
(4, 5) and (6, 5)
(4, 1) and (4, 11)
Type below:
__________

Answer:
(3, −6) and (3, 4)
( −3, 8) and (7, 8)
(4, 5) and (6, 5)

Explanation:
6 + 4 = 10
3 + 7 = 10
4 + 6 = 10

Distance on the Coordinate Plane – Page No. 193

Find the distance between the pair of points.

Question 1.
(1, 4) and (−3, 4)
_____ units

Answer:
4 units

Explanation:
The points have the same y-coordinate, so they are located on a horizontal line.
Find the distance between the x-coordinates of the point (1, 4) and the point (−3, 4)
|-3| = 3
1 + 3 = 4

Question 2.
(7, −2) and (11, −2)
_____ units

Answer:
3 units

Explanation:
The points have the same y-coordinate, so they are located on a horizontal line.
Find the distance between the x-coordinates of the point (7, −2) and the point (11, −2)
11 – 7 = 3

Question 3.
(6, 4) and (6, −8)
_____ units

Answer:
12 units

Explanation:
The points have the same x-coordinate, so they are located on a vertical line.
Find the distance between the y-coordinates of the point (6, 4) and the point (6, −8)
|-8| = 8
4 + 8 = 12

Question 4.
(8, −10) and (5, −10)
_____ units

Answer:
3 units

Explanation:
The points have the same y-coordinate, so they are located on a horizontal line.
Find the distance between the x-coordinates of the point (8, −10) and the point (5, −10)
8 – 5 = 3

Question 5.
(−2, −6) and (−2, 5)
_____ units

Answer:
11 units

Explanation:
The points have the same x-coordinate, so they are located on a vertical line.
Find the distance between the y-coordinates of the point (−2, −6)and the point (−2, 5)
|-6| = 6
6 + 5 = 11

Question 6.
(−5, 2) and (−5, −4)
_____ units

Answer:
6 units

Explanation:
The points have the same x-coordinate, so they are located on a vertical line.
Find the distance between the y-coordinates of the point (−5, 2) and the point (−5, −4)
|-4| = 4
2 + 4 = 6

Write the coordinates of a point that is the given distance from the given point.

Question 7.
5 units from (−1, −2)
Type below:
__________

Answer:
-6 or 4

Explanation:
5 units from (−1, −2)
(-6, -2) or (4, -2)

Question 8.
8 units from (2, 4)
Type below:
__________

Answer:
12 or -4

Explanation:
8 units from (2, 4)
(2, 12) or (2, -4)

Question 9.
3 units from (−7, −5)
Type below:
__________

Answer:
-2 or -8

Explanation:
3 units from (−7, −5)
(-7, -2) or (-7, -8)

Problem Solving

The map shows the locations of several areas in an amusement park. Each unit represents 1 kilometer.
Go Math Grade 6 Answer Key Chapter 3 Understand Positive and Negative Numbers img 29

Question 10.
How far is the Ferris wheel from the rollercoaster?
_____ km

Answer:
4 km

Explanation:
Ferris wheel = (5, 6)
rollercoaster = (5, 2)
6 – 2 = 4

Question 11.
How far is the water slide from the restrooms?
_____ km

Answer:
7 km

Explanation:
water slide = (-3, -4)
restrooms = (4, -4)
3 + 4 = 7

Question 12.
Graph the points (23, 3), (23, 7), and (4, 3) on a coordinate plane. Explain how to find the distance from (23, 3) to (23, 7) and from (23, 3) and (4, 3).
Type below:
__________

Answer:

Explanation:
(23, 3) to (23, 7)
The points have the same x-coordinate, so they are located on a vertical line.
Find the distance between the y-coordinates of the point (23, 3) and the point (23, 7)
7 – 3 = 4
(23, 3) and (4, 3).
The points have the same y-coordinate, so they are located on a horizontal line.
Find the distance between the x-coordinates of the point (23, 3) and the point (4, 3)
4 – 3 = 1

Lesson Check – Page No. 194

Question 1.
What is the distance between (4, −7) and (−5, −7)?
_____ units

Answer:
9 units

Explanation:
(4, −7) and (−5, −7)
The points have the same y-coordinate, so they are located on a horizontal line.
Find the distance between the x-coordinates of the point (4, −7) and the point (−5, −7)
|-5| = 5
5 + 4 = 9

Question 2.
Point A and point B are 5 units apart. The coordinates of point A are (3, −9). The y–coordinate of point B is −9. What is a possible x–coordinate for point B?
Type below:
__________

Answer:
8 or -2

Explanation:
Point A and point B are 5 units apart. The coordinates of point A are (3, −9). The y–coordinate of point B is −9.
The points have the same y-coordinate, so they are located on a horizontal line.
the distance between the x-coordinates = 5
So, 8 or -2

Spiral Review

Question 3.
An apple is cut into 10 pieces. 0.8 of the apple is eaten. Which fraction, in simplest form, represents the amount of apple that is left?
\(\frac{□}{□}\)

Answer:
\(\frac{1}{5}\)

Explanation:
An apple is cut into 10 pieces. 0.8 of the apple is eaten.
10/10 – 8/10 = 2/10 = 1/5 amount of apple left.

Question 4.
A carton contains soup cans weighing a total of 20 pounds. Each can weighs 1 \(\frac{1}{4}\) pounds. How many cans does the carton contain?
_____ cans

Answer:
16 cans

Explanation:
Divide 20 with 5/4 (=1 1/4)
So, 20 × 4/5 which would be 80/5 = 16 cans

Question 5.
List −1, \(\frac{1}{4}\), and −1 \(\frac{2}{3}\) in order from greatest to least.
Type below:
__________

Answer:
\(\frac{1}{4}\), -1, −1 \(\frac{2}{3}\)

Explanation:
\(\frac{1}{4}\) > -1 > −1 \(\frac{2}{3}\)

Question 6.
The point located at (3, −1) is reflected across the y−axis. What are the coordinates of the reflected point?
Type below:
__________

Answer:
(-3, -1)

Explanation:
The point located at (3, −1) is reflected across the y−axis.
(-3, -1)

Share and Show – Page No. 197

Question 1.
Busby County is rectangular. A map of the county on a coordinate plane shows the vertices of the county at (−5, 8), (8, 8), (8, −10), and (−5, −10). Each unit on the map represents 1 mile. What is the county’s perimeter?
_____ miles

Answer:
62 miles

Explanation:
Busby County is rectangular. A map of the county on a coordinate plane shows the vertices of the county at (−5, 8), (8, 8), (8, −10), and (−5, −10).
The distance of (−5, 8) is 8
The distance of (8, 8) is 8
The distance between -5 to 8 is 5 + 8 = 13
The distance of (8, −10) is 10
The distance of (−5, −10) is 10
The distance between -5 to 8 is 5 + 8 = 13
8 + 8 + 13 + 10 + 10 + 13 = 62

Question 2.
What if the vertices of the county were (−5, 8), (8, 8), (8, −6), and (−5, −6)? What would the perimeter of the county be?
_____ miles

Answer:
54 miles

Explanation:
The distance from – 5 to 8 is 5 + 8 = 13
The distance from – 5 to 8 is 5 + 8 = 13
The distance from 8 to -6 is 8 + 6 = 14
The distance from 8 to -6 is 8 + 6 = 14
13 + 14 + 13 + 14 = 54

Question 3.
On a coordinate map of Melville, a restaurant is located at (−9, −5). A laundry business is located 3 units to the left of the restaurant on the map. What are the map coordinates of the laundry business?
Type below:
__________

Answer:
(-12, -5)

Explanation:
On a coordinate map of Melville, a restaurant is located at (−9, −5).
A laundry business is located 3 units to the left of the restaurant on the map.(-12, -5) is the answer

Question 4.
The library is 4 blocks north and 9 blocks east of the school. The museum is 9 blocks east and 11 blocks south of the school. How far is it from the library to the museum?
_____ blocks

Answer:
15 blocks

Explanation:
the library is 4 blocks north = (0, 4)
9 blocks east = (9, 4)
The museum is 9 blocks east = (9, 0)
11 blocks south = (9, -11)
4 + 11 = 15

Problem Solving + Applications – Page No. 198

Question 5.
Make Sense of Problems Diana left her campsite at (2, 6) on a map of Big Trees Park, hiked to Redwood Grove at (−5, 6), and continued on to Bass Lake at (−5, −3). Each unit on the map represents 1 kilometer. How far did Diana hike?
_____ km

Answer:
16 km

Explanation:
Diana left her campsite at (2, 6) on a map of Big Trees Park, hiked to Redwood Grove at (−5, 6), 2 + 5 = 7
and continued on to Bass Lake at (−5, −3), 6 + 3 = 9
7 + 9 = 16 km

Question 6.
Hector left his house at (−6, 13) on a map of Coleville and walked to the zoo at (−6, 2). From there he walked east to his friend’s house. He walked a total distance of 25 blocks. If each unit on the map represents one block, what are the coordinates of Hector’s friend’s house?
Type below:
__________

Answer:
(19,2) should be the answer. He was -6 above the x-axis. Going east for 25 unit means he walked horizontally keeping the y-axis same

Question 7.
In November, the price of a cell phone was double the price in March. In December, the price was $57, which was $29 less than the price in November. What was the price of the cell phone in March?
$ _____

Answer:
$172

Explanation:
In November, the price of a cell phone was double the price in March. In December, the price was $57, which was $29 less than the price in November.
29 + 57 = 86
86 × 2 = $172

Question 8.
A map of the city holding the Olympics is placed on a coordinate plane. Olympic Stadium is located at the origin of the map. Each unit on the map represents 2 miles.
Go Math Grade 6 Answer Key Chapter 3 Understand Positive and Negative Numbers img 30
Graph the locations of four other Olympic buildings.

Max said the distance between the Aquatics Center and the Olympic Village is greater than the distance between the Media Center and the Basketball Arena. Do you agree with Max? Use words and numbers to support your answer
Type below:
__________

Answer:
Max said is correct
Aquatics Center = (8, 4)
Olympic Village = (-8, 4)
The distance = 8 + 8 = 16
Media Center = (4, -5)
Basketball Arena = (-8, -5)
The distance = 4 + 8 = 12

Problem Solving The Coordinate Plane – Page No. 199

Read each problem and solve.

Question 1.
On a coordinate map of Clifton, an electronics store is located at (6, −7). A convenience store is located 7 units north of the electronics store on the map. What are the map coordinates of the convenience store?
Type below:
__________

Answer:
(6, 0)

Explanation:
On a coordinate map of Clifton, an electronics store is located at (6, −7). A convenience store is located 7 units north of the electronics store on the map.
The map coordinates of the convenience store is (6, 0)

Question 2.
Sonya and Lucas walk from the school to the library. They walk 5 blocks south and 4 blocks west to get to the library. If the school is located at a point (9, −1) on a coordinate map, what are the map coordinates of the library?
Type below:
__________

Answer:
(5, -6)

Explanation:
The school is at (9, −1)
5 blocks South mean that you subtract 5 from the y coordinate:
(9, -1-5) = (9, -6)
4 blocks West means that you subtract 4 from the x coordinate:
(9-4, -6) = (5, -6)
The library is at (5, -6)

Question 3.
On a coordinate map, Sherry’s house is at the point (10, −2) and the mall is at point (−4, −2). If each unit on the map represents one block, what is the distance between Sherry’s house and the mall?
_____ blocks

Answer:
14 blocks

Explanation:
(10,-2)
(-4,-2)
x value- 10-(-4)=14
y value- Since both y values are the same, you don’t do anything.

Question 4.
Arthur left his job at (5, 4) on a coordinate map and walked to his house at (5, −6). Each unit on the map represents 1 block. How far did Arthur walk?
_____ blocks

Answer:
10 blocks

Explanation:
He walked 10 blocks. 4 to -6 is 10. 4, 3, 2, 1, 0, -1, -2, -3, -4, -5, -6.

Question 5.
A fire station is located 2 units east and 6 units north of a hospital. If the hospital is located at a point (−2, −3) on a coordinate map, what are the coordinates of the fire station?
Type below:
__________

Answer:
(0, 3)

Explanation:
the hospital is located at a point (−2, −3).
A fire station is located 2 units east and 6 units north of a hospital.
(-2 +2, -3+6) = (0,3)

Question 6.
Xavier’s house is located at the point (4, 6). Michael’s house is 10 blocks west and 2 blocks south of Xavier’s house. What are the coordinates of Michael’s house?
Type below:
__________

Answer:
(-6, 4)

Explanation:
the coordinates are -6,4. (x-10,y-2)

Question 7.
Write a problem that can be solved by drawing a diagram on a coordinate plane.
Type below:
__________

Answer:
On a coordinate map, Sherry’s house is at the point (10, −2) and the mall is at point (−4, −2). If each unit on the map represents one block, what is the distance between Sherry’s house and the mall?

Lesson Check – Page No. 200

Question 1.
The points (−4, −4), (−4, 4), (4, 4), and (4, −4) form a square on a coordinate plane. How long is a side length of the square?
_____ units

Answer:
8 units

Explanation:
-4 + 4 = 8 units
side length of the square is 8 units

Question 2.
On a coordinate map, the museum is located at (−5, 7). A park is located 6 units to the right of the museum on the map. What are the coordinates of the park?
Type below:
__________

Answer:
(1, 7)

Explanation:
On a coordinate map, the museum is located at (−5, 7). A park is located 6 units to the right of the museum on the map.
(1,7)

Spiral Review

Question 3.
On a grid Joe’s house is marked at (−5, −3) and Andy’s house is marked at (1, −3). What is the distance, on the grid, between Joe’s house and Andy’s house?
_____ units

Answer:
6 units

Explanation:
On a grid Joe’s house is marked at (−5, −3) and Andy’s house is marked at (1, −3).
|-5| = 5
5 + 0 = 5
0 + 1 = 1
5 + 1 = 6

Question 4.
In the last two years, Mari grew 2 \(\frac{1}{4}\) inches, Kim grew 2.4 inches, and Kate grew 2 \(\frac{1}{8}\) inches. Write the amounts they grew in order from least to greatest.
Type below:
__________

Answer:
2 \(\frac{1}{8}\), 2 \(\frac{1}{4}\), 2.4

Explanation:
In the last two years, Mari grew 2 \(\frac{1}{4}\) inches, Kim grew 2.4 inches, and Kate grew 2 \(\frac{1}{8}\) inches.
2 \(\frac{1}{4}\) = 9/4 = 2.25
2 \(\frac{1}{8}\) = 17/8 = 2.125
2.125, 2.25, 2.4

Question 5.
A jar of jelly that weighs 4.25 ounces costs $2.89. What is the cost of one ounce of jelly?
$ _____

Answer:
$0.68

Explanation:
A jar of jelly that weighs 4.25 ounces costs $2.89.
$2.89/4.25 = $0.68

Question 6.
Jan began with \(\frac{5}{6}\) pound of modeling clay. She used \(\frac{1}{5}\) of the clay to make decorative magnets. She divided the remaining clay into 8 equal portions. What is the weight of the clay in each portion?
\(\frac{□}{□}\) pounds

Answer:
\(\frac{1}{12}\) pounds

Explanation:
Weight of modelling clay is = 5/6 pounds
Part of clay used to make decorative magnets is = 1/5
Remaining part of clay = 1 – 1/5 = 4/5
So, Remaining part of clay is divided into 8 equal parts so, Weight of each clay is given by 4/5 × 5/6 × 1/8 = 1/12 pounds
So, weight of the clay in each portion is 1/12 pounds

Chapter 3 Review/Test – Page No. 201

Question 1.
For numbers 1a–1d, choose Yes or No to indicate whether the situation can be represented by a negative number.
1a. Sherri lost 100 points answering a question wrong. Yes No
1b. The peak of a mountain is 2,000 feet above sea level. Yes No
1c. Yong paid $25 for a parking ticket. Yes No
1d. A puppy gained 3 pounds. Yes No
1a. __________
1b. __________
1c. __________
1d. __________

Answer:
1a. Yes
1b. No
1c. No
1d. No

Question 2.
The low weekday temperatures for a city are shown.
Go Math Grade 6 Answer Key Chapter 3 Understand Positive and Negative Numbers img 31
Part A
Using the information in the table, order the temperatures from lowest to highest.
Type below:
__________

Answer:
-7, -5, -3, 2, 3

Explanation:
Monday = -5
Tuesday = -3
Wednesday = 2
Thursday = -7
Friday = 3
-7, -5, -3, 2, 3

Question 2.
Part B
Explain how to use a vertical number line to determine the order.
Type below:
__________

Answer:
Place the -3, -5, -7, are below the 0. And place 2 and 3 are above the 0.

Page No. 202

Question 3.
For numbers 3a–3e, choose Yes or No to indicate whether the number is between –1 and –2.
3a. \(\frac{-4}{5}\) Yes No
3b. 1 \(\frac{2}{3}\) Yes No
3c. −1.3 Yes No
3d. −1 \(\frac{1}{4}\) Yes No
3e. −2 \(\frac{1}{10}\) Yes No
3a. __________
3b. __________
3c. __________
3d. __________
3e. __________

Answer:
3a. No
3b. No
3c. Yes
3d. Yes
3e. No

Explanation:
3a. \(\frac{-4}{5}\) = -0.8
3b. 1 \(\frac{2}{3}\) = 1.666
3c. -1.3
3d. −1 \(\frac{1}{4}\) = -1.25
3e. −2 \(\frac{1}{10}\) = -21/10 = -2.1

Question 4.
Compare \(\frac{-1}{5}\) and –0.9. Use numbers and words to explain your answer
Type below:
__________

Answer:
\(\frac{-1}{5}\) = -0.2
-0.9
-0.2 and -0.9 both are negative numbers. They lies between 0 and -1

Question 5.
Jeandre said |3| equals |–3|. Is Jeandre correct? Use a number line and words to support your answer.
Type below:
__________

Answer:
grade 6 chapter 3 Page no. 202 image 1

Explanation:
Yes, he is correct and he is referring to the absolute values of number 3 and -3. And by absolute value, this is the distance of the number from the origin zero (0) which is symbolized by two vertical lines, as |3| or |-3| is equal to 3.
The picture shows a number line where green is the origin zero (0). The purple line is the distance between 0 and 3 which is 3. The pink one is the distance of -3 from 0 which is also 3. Therefore, |3| equals |-3|

Question 6.
Write the values in order from least to greatest.
Go Math Grade 6 Answer Key Chapter 3 Understand Positive and Negative Numbers img 32
Type below:
__________

Answer:
|2| |-4| |8| |-12|

Explanation:
|-4| = 4
|2| = 2
|-12| = 12
|8| = 8
2, 4, 8, 12

Question 7.
For numbers 7a–7d, select True or False for each statement.
7a. The x-coordinate of any point on the y-axis is 0. True False
7b. Point D(–2, 1) is to the left of the y-axis and below the x-axis. True False
7c. The point where the axes intersect is the origin. True False
7d. If both the x- and y- coordinates are positive, the point is to the right of the y-axis and below the x-axis. True False
__________
__________
__________
__________

Answer:
7a. True
7b. False
7c. True
7d. False

Page No. 203

Question 8.
Mia’s house is located at point (3, 4) on a coordinate plane. The location of Keisha’s house is the reflection of the location of Mia’s house across the y-axis. In what quadrant is Keisha’s house in?
Type below:
__________

Answer:
quadrant II

Explanation:
Mia’s house is located at point (3, 4) on a coordinate plane. The location of Keisha’s house is the reflection of the location of Mia’s house across the y-axis.
quadrant II is the answer.

Question 9.
Points A(3, 8) and B(–4, 8) are located on a coordinate plane. Graph the pair of points. Then find the distance between them. Use numbers and words to explain your answer.
Type below:
__________

Answer:

Explanation:
Points A(3, 8) and B(–4, 8) are located on a coordinate plane.
3 + 0 = 3
|-4| = 4
4 + 0 = 4
3 + 4 = 7
7 units

Page No. 204

Question 10.
The map shows the location J of Jose’s house and the location F of the football field. Jose is going to go to Tyrell’s house and then the two of them are going to go to the football field for practice.
Go Math Grade 6 Answer Key Chapter 3 Understand Positive and Negative Numbers img 33
Part A
Tyrell’s house is located at point T, the reflection of point J across the y-axis. What are the coordinates of points T, J, and F?
Type below:
__________

Answer:
coordinates of points T (6, 8)
coordinates of points J (-6, 8), and
coordinates of points F(-5, 6)

Question 10.
Part B
If each unit on the map represents 1 block, what was the distance Tyrell traveled to the football field and what was the distance Jose traveled to the football field? Use numbers and words to explain your answer
Type below:
__________

Answer:
If each unit on the map represents 1 block, the distance Jose traveled to the football field
coordinates of points T (6, 8)
coordinates of points J (-6, 8)
|-6| = 6
6 + 6 = 12 units.
the distance Tyrell traveled to the football field
coordinates of points T (6, 8)
coordinates of points F(-5, 6)
6 + 5 = 11 units

Question 11.
For numbers 11a–11d, choose Yes or No to indicate whether the situation could be represented by the integer +3.
11a. A football team gains 3 yards on a play. Yes No
11b. A golfer’s score is 3 over par. Yes No
11c. A student answers a Yes No 3-point question correctly. Yes No
11d. A cat loses 3 pounds. Yes No
11a. __________
11b. __________
11c. __________
11d. __________

Answer:
11a. Yes
11b. Yes
11c. Yes
11d. No

Page No. 205

Question 12.
Jason used a map to record the elevations of five locations.
Go Math Grade 6 Answer Key Chapter 3 Understand Positive and Negative Numbers img 34
Jason wrote the elevations in order from lowest to highest : -3, 5, 8 -18, -20.
Is Jason correct? Use words and numbers to explain why or why not. If Jason is incorrect, what is the correct order?
Type below:
__________

Answer:
Jason is incorrect.
the elevations in order from lowest to highest: -20, -18, -3, 5, 8

Question 13.
For numbers 13a–13d, select True or False for each statement.
13a. \(\frac{1}{5}\) is between 0 and 1. True False
13b. −2 \(\frac{2}{3}\) is between -1 and -2. True False
13c. −3 \(\frac{5}{8}\) is between -3 and -4. True False
13d. 4 \(\frac{3}{4}\) is between 3 and 4. True False
13a. __________
13b. __________
13c. __________
13d. __________

Answer:
13a. True
13b. False
13c. True
13d. False

Question 14.
Choose <, >, or =.
14a. 0.25 ο \(\frac{3}{4}\)
14b. 2 \(\frac{7}{8}\) ο 2.875
14c. \(\frac{1}{3}\) ο 0.325
14d. \(\frac{-3}{4}\) ο \(\frac{-1}{2}\)
0.25 ____ \(\frac{3}{4}\)
2 \(\frac{7}{8}\) _____ 2.875
\(\frac{1}{3}\) _____ 0.325
\(\frac{-3}{4}\) _____ \(\frac{-1}{2}\)

Answer:
0.25 < \(\frac{3}{4}\)
2 \(\frac{7}{8}\) = 2.875
\(\frac{1}{3}\) > 0.325
\(\frac{-3}{4}\) < \(\frac{-1}{2}\)

Explanation:
\(\frac{3}{4}\) = 0.75
0.25 < \(\frac{3}{4}\)
2 \(\frac{7}{8}\) = 23/8 = 2.875
2 \(\frac{7}{8}\) = 2.875
\(\frac{-3}{4}\) = -0.75
\(\frac{-1}{2}\) = -0.5

Page No. 206

Question 15.
Graph 4 and −4 on the number line.
Go Math Grade 6 Answer Key Chapter 3 Understand Positive and Negative Numbers img 35
Tyler says both 4 and −4 have an absolute value of 4. Is Tyler correct? Use the number line and words to explain why or why not.
Type below:__________

Answer:
Tyler is correct.
|-4| = 4
|4| = 4

Question 16.
Lindsay and Will have online accounts for buying music. Lindsay’s account balance is −$20 and Will’s account balance is −$15. Express each account balance as a debt and explain whose debt is greater.
Type below:
__________

Answer:
Lindsey is 5$ more in dept than Will.
Lindsey= -20$
Will= -15$

Question 17.
Explain how to graph points A(–3, 0), B(0, 0), and C(0, –3) on the coordinate plane. Then, explain how to graph point D, so that ABCD is a square.
Type below:
__________

Answer:
First place the points A(–3, 0), B(0, 0), and C(0, –3) on the coordinate plane.
The length from point A to point B is 3.
A square has equal lengths on each side.
So, to find D, Add 3 units to the left to C or down to A.
D(-3, -3)

Question 18.
Point A(2, –3) is reflected across the x-axis to point B. Point B is reflected across the y-axis to point C. What are the coordinates of point C? Use words and numbers to explain your answer.
Type below:
__________

Answer:
Quadrant III

Explanation:
Point A(2, –3) is reflected across the x-axis to point B. Point B is reflected across the y-axis to point C.
So, Point B is (2,3)
Point C is (-2, 3)
C is in Quadrant III

Conclusion:

We believe that the details provided in the HMH Go Math 6th Grade Answer Key for Chapter 3 Understand Positive and Negative Numbers made you happy. All the explanations are prepared by the math experts as per the latest syllabus. Compare the questions in real-time so that you can understand concepts easily. Stay with us to get the answer keys for all the chapters of grade 6.

Go Math Grade 8 Answer Key Chapter 6 Functions

go-math-grade-8-chapter-6-functions-answer-key

The solutions of Go Math Grade 8 Answer Key Chapter 6 Functions are provided in the pdf format here. HMH Go Math Grade 8 Answer Key Chapter 6 Functions are presented by the professional math experts in an easy manner and with brief explanations. You can find different ways of solving the problems on this page. So, we suggest the students to refer to Go Math Grade 8 Answer Key Chapter 6 Functions now to begin your practice.

Go Math Grade 8 Chapter 6 Functions Answer Key

Test and improve your knowledge by using Go Math Grade 8 Chapter 6 Functions Solution Key. The Go Math Grade 8 Chapter 6 Functions Answer Key consists of the topics like Identifying and representing functions, describing functions, analyzing graphs, etc. Use HMH Go Math Grade 8 Answer Key for the best practice of maths. After completion of your preparation test yourself by solving the problems given in the model quiz.

Lesson 1: Identifying and Representing Functions 

Lesson 2: Describing Functions

Lesson 3: Comparing Functions 

Lesson 4: Analyzing Graphs

Model Quiz 

Mixed Review 

Guided Practice – Identifying and Representing Functions – Page No. 158

Complete each table. In the row with x as the input, write a rule as an algebraic expression for the output. Then complete the last row of the table using the rule.

Question 1.
Go Math Grade 8 Answer Key Chapter 6 Functions Lesson 1: Identifying and Representing Functions img 1
Type below:
_______________

Answer:
Grade 8 Chapter 6 image 1

Explanation:
Unit Cost of ticket = 40/2 = 20
Total cost = 20x where x is the number of tickets.
x = 20x
10 = 20(100) = 200

Question 2.
Go Math Grade 8 Answer Key Chapter 6 Functions Lesson 1: Identifying and Representing Functions img 2
Type below:
_______________

Answer:
Grade 8 Chapter 6 image 2

Explanation:
Number of pages per minute = 1/2 = 0.5
Total cost = 0.5x where x is the number of minutes.
x = 0.5x
30 = 0.5(30) = 15

Question 3.
Go Math Grade 8 Answer Key Chapter 6 Functions Lesson 1: Identifying and Representing Functions img 3
Type below:
_______________

Answer:
Grade 8 Chapter 6 image 3

Explanation:
Units cost of Muffins = 2.25/1 = 2.25
Total cost = 2.25x where x is the number of muffins
x = 2.25x
12 = 2.25(12) = 27

Determine whether each relationship is a function.

Question 4.
Go Math Grade 8 Answer Key Chapter 6 Functions Lesson 1: Identifying and Representing Functions img 4
_______________

Answer:
Function

Explanation:
Each input is assigned to exactly one output.

Question 5.
Go Math Grade 8 Answer Key Chapter 6 Functions Lesson 1: Identifying and Representing Functions img 5
_______________

Answer:
Not a function

Explanation:
The input value is 4 is paired with two outputs 25 and 35

Question 6.
The graph shows the relationship between the weights of 5 packages and the shipping charge for each package. Is the relationship represented by the graph a function? Explain.
Go Math Grade 8 Answer Key Chapter 6 Functions Lesson 1: Identifying and Representing Functions img 6
_______________

Answer:
Function

Explanation:
Each input is assigned to exactly one output.

Essential Question Check-In

Question 7.
What are four different ways of representing functions? How can you tell if a relationship is a function?
Type below:
_______________

Answer:
The function can be represented by an equation, table, graph, and Venn diagram.
If a relationship is a function, each input is paired with exactly one output.

Independent Practice – Identifying and Representing Functions – Page No. 159

Determine whether each relationship represented by the ordered pairs is a function. Explain.

Question 8.
(2, 2), (3, 1), (5, 7), (8, 0), (9, 1)
_______________

Answer:
Function

Explanation:
Each input value is paired with exactly one output value.

Question 9.
(0, 4), (5, 1), (2, 8), (6, 3), (5, 9)
_______________

Answer:
Not a function

Explanation:
The input value is 5 is paired with two outputs 1 and 9

Question 10.
Draw Conclusions
Joaquin receives $0.40 per pound for 1 to 99 pounds of aluminum cans he recycles. He receives $0.50 per pound if he recycles more than 100 pounds. Is the amount of money Joaquin receives a function of the weight of the cans he recycles? Explain your reasoning.
_______________

Answer:
Yes

Explanation:
The amount of money increases with the weight of the cans. No weight will result in the same amount of money earned.

Question 11.
A biologist tracked the growth of a strain of bacteria, as shown in the graph.
Go Math Grade 8 Answer Key Chapter 6 Functions Lesson 1: Identifying and Representing Functions img 7
a. Explain why the relationship represented by the graph is a function.
Type below:
_______________

Answer:
The relationship is a function as each input has been assigned exactly one output. There is only one number of bacteria for each number of hours.

Question 11.
b. What If?
Suppose there was the same number of bacteria for two consecutive hours. Would the graph still represent a function? Explain.
Type below:
_______________

Answer:
Yes. If the number of bacteria for two consecutive hours is the same, one input will still be paired with one output, hence the relationship is still a function.

Question 12.
Multiple Representations
Give an example of a function in everyday life, and represent it as a graph, a table, and a set of ordered pairs. Describe how you know it is a function.
Go Math Grade 8 Answer Key Chapter 6 Functions Lesson 1: Identifying and Representing Functions img 8
Type below:
_______________

Answer:
The cost of a bouquet of flowers and the number of flowers in the bouquet is a function. The unit cost of flowers = $0.85 and x the number of flowers. Hence, C= 0.85x
Grade 8 Chapter 6 image 4
Grade 8 Chapter 6 image 5
(2, 1.7), (4, 3.4), (6, 5.1), (8, 6.8), (10, 8.5)
Each value of the input is paired with exactly one output.

Identifying and Representing Functions – Page No. 160

The graph shows the relationship between the weights of six wedges of cheese and the price of each wedge.
Go Math Grade 8 Answer Key Chapter 6 Functions Lesson 1: Identifying and Representing Functions img 9

Question 13.
Is the relationship represented by the graph a function? Justify your reasoning. Use the words “input” and “output” in your explanation, and connect them to the context represented by the graph.
_______________

Answer:
Yes, the relationship represented by the graph is a function.
Each input (weight) in the graph is paired with exactly one output(price).

Question 14.
Analyze Relationships
Suppose the weights and prices of additional wedges of cheese were plotted on the graph. Might that change your answer to question 13? Explain your reasoning.
Type below:
_______________

Answer:
No. As the weight of the cheese will increase, the cost of wedges of cheese will increase as well. Hence, for each input (weight), there would be exactly one output (price).

H.O.T.

Focus on Higher Order Thinking

Question 15.
Justify Reasoning
A mapping diagram represents a relationship that contains three different input values and four different output values. Is the relationship a function? Explain your reasoning.
_______________

Answer:
No. Since there are three inputs and four outputs, one of the inputs will have more than one output, hence the relationship cannot be a function.

Question 16.
Communicate Mathematical Ideas
An onion farmer is hiring workers to help harvest the onions. He knows that the number of days it will take to harvest the onions is a function of the number of workers he hires. Explain the use of the word “function” in this context.
Type below:
_______________

Answer:
Number of days = f(number of workers)

Explanation:
We know that the more the number of workers will be involved in the harvesting of onion, the lesser days it will take to complete.
Thus the number of workers becomes the independent variable and the number of days becomes the dependent variable.
Here the word function is used to describe that the number of days is dependent on the number of workers.
Number of days = f(number of workers)

Guided Practice – Describing Functions – Page No. 164

Plot the ordered pairs from the table. Then graph the function represented by the ordered pairs and tell whether the function is linear or nonlinear.

Question 1.
y = 5 − 2x
Go Math Grade 8 Answer Key Chapter 6 Functions Lesson 2: Describing Functions img 10
_______________

Answer:
Grade 8 Chapter 6 image 6
Grade 8 Chapter 6 image 7
Graph of a linear function is a straight line
Linear relationship

Explanation:
Given y = 5 – 2x
y = 5 – 2(-1) = 5 + 2 = 7
y = 5 – 2(1) = 5 – 2 = 3
y = 5 – 2(3) = 5 – 6 = -1
y = 5 – 2(5) = 5 – 10 = -5

Question 2.
y = 2 − x2
Go Math Grade 8 Answer Key Chapter 6 Functions Lesson 2: Describing Functions img 11
_______________

Answer:
y = 2 − x2
Grade 8 Chapter 6 image 8
Graph the ordered pairs. Then draw a line through the points to represent the solution.
Grade 8 Chapter 6 image 9
Graph of a linear function is not a straight line
Non-linear relationship

Explanation:
y = 2 − x2
y = 2 – 4 = -2
y = 2 – 1 = 1
y = 2 – 0 = 2
y = 2 – 1 = 1
y = 2 – 4 = -2

Explain whether each equation is a linear equation.

Question 3.
y = x2 – 1
_______________

Answer:
The equation is not in the form of a linear equation, hence is not a linear equation.

Explanation:
Compare the equation with the general linear equation y = mx + b.
The equation is not in the form of a linear equation, hence is not a linear equation.

Question 4.
y = 1 – x
_______________

Answer:
The equation is in the form of a linear equation, hence is a linear equation.

Explanation:
Compare the equation with the general linear equation y = mx + b.
The equation is in the form of a linear equation, hence is a linear equation.

Essential Question Check-In

Question 5.
Explain how you can use a table of values, an equation, and a graph to determine whether a function represents a proportional relationship.
Type below:
_______________

Answer:
From a table, determine the ratio y/x. If it is constant the relationship is proportional.
From a graph, note if the graph passes through the origin. The graph of a proportional relationship must pass through the origin (0, 0).
From an equation, compare with general linear form of equation, y = mx + b. If b = 0, the relationship is proportional.

Independent Practice – Describing Functions – Page No. 165

Question 6.
State whether the relationship between x and y in y = 4x – 5 is proportional or nonproportional. Then graph the function.
Go Math Grade 8 Answer Key Chapter 6 Functions Lesson 2: Describing Functions img 12
_______________

Answer:
Grade 8 Chapter 6 image 10

Explanation:
First, we compare the equation with the general linear equation y = mx + b. y = 4x – 5 is in the form of y = mx + b, with m = 4 and b = -5. Therefore, the equation is a linear equation. Since b is not equal to 0, the relationship is non-proportional.
Then, we choose several values for the input x. We substitute these values of x in the equation to find the output y.
y = 4x – 5
If x = 0; y = 4(0) – 5 = -5; (0, -5)
If x = 2; y = 4(2) – 5 = 3; (2, 3)
If x = 4; y = 4(4) – 5 = 11; (4, 11)
If x = 6; y = 4(6) – 5 = 19; (6, 19)
We graph the ordered pairs and we draw a line through the points to represent the solutions of the function.

Question 7.
The Fortaleza telescope in Brazil is a radio telescope. Its shape can be approximated with the equation y = 0.013x2. Is the relationship between x and y linear? Is it proportional? Explain.
____________
____________

Answer:
Compare the equation with the general linear equation y = mx + b.
The equation is not in the form of a linear equation, hence it is not a linear equation. Since x is squared, it is not proportional.

Question 8.
Kiley spent $20 on rides and snacks at the state fair. If x is the amount she spent on rides, and y is the amount she spent on snacks, the total amount she spent can be represented by the equation x + y = 20. Is the relationship between x and y linear? Is it proportional? Explain.
____________
____________

Answer:
x + y = 20
Rewriting the equation
y = 20 – x
Compare the equation with the general linear equation y = mx + b.
It is linear
Since b is not equal to 0, the relationship is not proportional.

Question 9.
Represent Real-World Problems
The drill team is buying new uniforms. The table shows y, the total cost in dollars, and x, the number of uniforms purchased.
Go Math Grade 8 Answer Key Chapter 6 Functions Lesson 2: Describing Functions img 13
a. Use the data to draw a graph. Is the relationship between x and y linear? Explain.
____________

Answer:
Grade 8 Chapter 6 image 10
The graph of a linear relationship is a straight line.
x and y are linear.

Question 9.
b. Use your graph to predict the cost of purchasing 12 uniforms.
$ ________

Answer:
$720

Explanation:
Grade 8 Chapter 6 image 10
The cost of 12 uniforms is $720

Question 10.
Marta, a whale calf in an aquarium, is fed a special milk formula. Her handler uses a graph to track the number of gallons of formula y the calf drinks in x hours. Is the relationship between x and y linear? Is it proportional? Explain.
Go Math Grade 8 Answer Key Chapter 6 Functions Lesson 2: Describing Functions img 14
____________
____________

Answer:
The relationship is linear
The relationship is proportional

Explanation:
As the data lies on a straight line, the relationship is linear
As the graph passes through the origin, the relationship is proportional

Describing Functions – Page No. 166

Question 11.
Critique Reasoning
A student claims that the equation y = 7 is not a linear equation because it does not have the form y=mx + b. Do you agree or disagree? Why?
Go Math Grade 8 Answer Key Chapter 6 Functions Lesson 2: Describing Functions img 15
____________

Answer:
Disagree; The equation can be written in the form y = mx + b Where m is 0. The graph of the solutions is a horizontal line.

Question 12.
Make a Prediction
Let x represent the number of hours you read a book and y represent the total number of pages you have read. You have already read 70 pages and can read 30 pages per hour. Write an equation relating x hours and y pages you read. Then predict the total number of pages you will have read after another 3 hours.
_______ pages

Answer:
160 pages

Explanation:
Let x represent the number of hours you read a book and y represents the total number of pages you have read. You have already read 70 pages and can read 30 pages per hour.
m = 30; b = 70 pages
y = 30x + 70
x = 3 hrs
y = 30(3) + 70 = 160

H.O.T.

Focus on Higher Order Thinking

Question 13.
Draw Conclusions
Rebecca draws a graph of a real-world relationship that turns out to be a set of unconnected points. Can the relationship be linear? Can it be proportional? Explain your reasoning.
Type below:
______________

Answer:
The relationship is linear if all the points lie on the same line. If the relationship is linear and passes through the origin, it is proportional.

Question 14.
Communicate Mathematical Ideas
Write a real-world problem involving a proportional relationship. Explain how you know the relationship is proportional.
Type below:
______________

Answer:
The amount of money earned at a car wash is a proportional relationship. When there are 0 cars washed, $0 are earned. The amount of money earned increases by the unit cost of a car wash.

Question 15.
Justify Reasoning
Show that the equation y + 3 = 3(2x + 1) is linear and that it represents a proportional relationship between x and y.
Type below:
______________

Answer:
y + 3 = 3(2x + 1)
y +3 = 6x + 3
y = 6x
As b = 0, it is a Proportional Relationship.

Guided Practice – Comparing Functions – Page No. 170

Doctors have two methods of calculating maximum heart rate. With the first method, maximum heart rate, y, in beats per minute is y = 220 − x, where x is the person’s age. Maximum heart rate with the second method is shown in the table.
Go Math Grade 8 Answer Key Chapter 6 Functions Lesson 3: Comparing Functions img 16

Question 1.
Which method gives the greater maximum heart rate for a 70-year-old?
____________ method

Answer:
Second

Explanation:
y = 220 – x
y = 220 – 70 = 150
Find the slope using two points from the grapgh by
Slope m = (y2 -y1)/(x2 – x1) where (x1, y1) = (20, 194) and (x2, y2) = (30, 187)
Slope m = (y2 -y1)/(x2 – x1) = (187 – 194)/(30 – 20) = -7/10 = -0.7
197 = -0.7(20) + b
y-intercept b = 208
Substituting the value of the slope m and y-intercept in the slope-intercept form. y = mx + b where, m = -0.7 and b = 208.
y = -0.7x +208
x = 70yrs
y = -0.7(70) + 208 = 159
150 < 159
Second method gives the greater maximum heart rate for a 70 year ols.

Question 2.
Are heart rate and age proportional or nonproportional for each method?
____________

Answer:
For method 1, the relationship is non-propotional.
For method 2, the relationship is non-propotional.

Explanation:
Compare the equation with the general linear equation y = mx + b.
It is linear
Since b is not equal to 0, the relationship is not proportional.

Aisha runs a tutoring business. With Plan 1, students may choose to pay $15 per hour. With Plan 2, they may follow the plan shown on the graph.
Go Math Grade 8 Answer Key Chapter 6 Functions Lesson 3: Comparing Functions img 17

Question 3.
Describe the plan shown on the graph.
Type below:
______________

Answer:
Choose two points on the graph to find the slope.
Find the slope
m = (y2 -y1)/(x2 – x1)
m = (60 – 40)/(4 – 0) = 20/4 = 5
Read the y-intercept from the graph: b = 40
Use your slope and y-intercept values to write an equation in slope-intercept
form.
y = 5x + 40
Plan 2 has an intial cost of $40 and a rate of $5 per hour.

Question 4.
Sketch a graph showing the $15 per hour option.
Type below:
______________

Answer:
Grade 8 Chapter 6 image 11

Question 5.
What does the intersection of the two graphs mean?
Type below:
______________

Answer:
The intersection of the two graphs represents the number of hours for which both plans will cost the same,

Question 6.
Which plan is cheaper for 10 hours of tutoring?
______________

Answer:
Plan 1
y = 15x
x = 10 hrs
y = 15(10) = $150
Plan 2
y = 5x + 40
y = 5(10) + 40 = $90
$150 > $90
Plane 2 is cheaper

Question 7.
Are cost and time proportional or nonproportional for each plan?
Type below:
______________

Answer:
Comparing with the general linear form of equation y = mx + b. Since b = 0, the relationship is proportional
The cost and time are proportional for Plan 1
Comparing with the general linear form of equation y = mx + b. Since b is not equal to 0, the relationship is proportional
The cost and time are not proportional for Plan 2

Essential Question Check-In

Question 8.
When using tables, graphs, and equations to compare functions, why do you find the equations for tables and graphs?
Type below:
______________

Answer:
The tables and graphs represent a part of the solution of the function. By writing the equation, any value can be a substitute to evaluate the function and compared it with the equations.

Independent Practice – Comparing Functions – Page No. 171

The table and graph show the miles driven and gas used for two scooters.
Go Math Grade 8 Answer Key Chapter 6 Functions Lesson 3: Comparing Functions img 18

Question 9.
Which scooter uses fewer gallons of gas when 1350 miles are driven?
______________

Answer:
Scooter B uses fewer gallons of gas when 1350 miles are driven

Explanation:
The equation for Scooter A Slope m = m = (y2 -y1)/(x2 – x1) where (x1, y1) = (150, 2) and (x2, y2) = (300, 4)
Slope m = (y2 -y1)/(x2 – x1) = (4 – 2)/(300 – 150) = 2/150 = 1/75
2 = 1/75(150) + b
y-intercept b = 0
Substituting the value of the slope m and y-intercept in the slope-intercept form. y = mx + b where, m = 1/75 and b = 0.
y = 1/75x
x = 1350 miles
y = 1/75(1,350)
y = 18gal
The equation for Scooter B Slope m = m = (y2 -y1)/(x2 – x1) where (x1, y1) = (0, 0) and (x2, y2) = (90, 1)
Slope m = (y2 -y1)/(x2 – x1) = (1 – 0)/(90 – 0) = 1/90
2 = 1/90(90) + b
y-intercept b = 0
Substituting the value of the slope m and y-intercept in the slope-intercept form. y = mx + b where, m = 1/90 and b = 0.
y = 1/90x
x = 1350 miles
y = 1/90(1,350)
y = 15gal
Compare the gallons of gas to drive 1,350 miles
18 > 15
Scooter B uses fewer gallons of gas when 1,350 miles are driven.

Question 10.
Are gas used and miles proportional or nonproportional for each scooter?
______________

Answer:
The gas used and miles are proportional to both scooters.

Explanation:
Compare with general linear form of an equation, y = mx + b. If b = 0, the relationship is proportional.
The gas used and miles are proportional to both scooters.

A cell phone company offers two texting plans to its customers. The monthly cost, y dollars, of one plan is y = 0.10x + 5, where x is the number of texts. The cost of the other plan is shown in the table.
Go Math Grade 8 Answer Key Chapter 6 Functions Lesson 3: Comparing Functions img 19

Question 11.
Which plan is cheaper for under 200 texts?
______________

Answer:
Plane 1 is cheaper

Explanation:
Plan 1
y = 0.10x + 5
Subsitute x = 199
y = 0.10(199) + 5 = $24.90
Find the slope using two points from the graph by m = (y2 -y1)/(x2 – x1) where (x1, y1) = (100, 20), (x2, y2) =(200, 25)
Substitute the value of m and (x1, y1) = (100, 20), (x2, y2) =(200, 25)
Slope m = (y2 -y1)/(x2 – x1) = (25 – 20)/(200 – 100) = 5/100 = 0.05
20 = 0.05(100) + b
y-intercept b = 15
Substituting the value of slope (m) and (x, y) in the slope intercept form to find y intercept (b):
y = 0.05x + 15
x = 199
y = 0.05(199) + 15 = $24.95
Compare the cost for two plans for text < 200
$24.90 < $24.95
Plane 1 is cheaper

Question 12.
The graph of the first plan does not pass through the origin. What does this indicate?
Type below:
______________

Answer:
Plan 1
y = 0.10x + 5
The graph that does not pass through the origin indicates that there is a base price of $5 for the plan.

Question 13.
Brianna wants to buy a digital camera for a photography class. One store offers the camera for $50 down and a payment plan of $20 per month. The payment plan for a second store is described by y = 15x + 80, where y is the total cost in dollars and x is the number of months. Which camera is cheaper when the camera is paid off in 12 months? Explain.
______________

Answer:
For first store, the slope interecept form y = mx + b where m = 20 dollars per month and b = 50 dollar.
y = 20x + 50
x = 12 months
y = 20(12) + 50 = $290
Second store
y = 15x + 80
x = 12 months
y = 15(12) + 80 = $260
Compare the cost of camera if it paid off in 12 months $290 > $260
Camera is cheaper at second store

Comparing Functions – Page No. 172

Question 14.
The French club and soccer team are washing cars to earn money. The amount earned, y dollars, for washing x cars is a linear function. Which group makes the most money per car? Explain.
Go Math Grade 8 Answer Key Chapter 6 Functions Lesson 3: Comparing Functions img 20
______________

Answer:

Explanation:
Find the slope using two points from the grapgh by
Slope m = (y2 -y1)/(x2 – x1) where (x1, y1) = (2, 10) and (x2, y2) = (4, 20)
Slope m = (y2 -y1)/(x2 – x1) = (20 – 10)/(4 – 2) = 10/2 = 5
French Club makes $5 per car.
Find the slope using two points from the grapgh by
Slope m = (y2 -y1)/(x2 – x1) where (x1, y1) = (0, 0) and (x2, y2) = (2, 16)
Slope m = (y2 -y1)/(x2 – x1) = (16 – 0)/(2 – 0) = 16/2 = 8
Soccer Club makes $8 per car.
Compare the money earned for washing one car $5 < $8
Soccer club makes the most money per car

H.O.T.

Focus on Higher Order Thinking

Question 15.
Draw Conclusions
Gym A charges $60 a month plus $5 per visit. The monthly cost at Gym B is represented by y = 5x + 40, where x is the number of visits per month. What conclusion can you draw about the monthly costs of the gyms?
__________ is more expensive

Answer:
Gym A is more expensive

Explanation:
Since the rate per visit is the same, the monthly cost of Gyn A is always more than Gym B.

Question 16.
Justify Reasoning
Why will the value of y for the function y = 5x + 1 always be greater than that for the function y = 4x + 2 when x > 1?
Type below:
______________

Answer:
y1 = 5x + 1 and y2 = 4x + 2 Subtracting y2 from y1
y1 – y2 = 5x + 1 – (4x + 2)
y1 – y2 = x -1
For x>= 1 we get x – 1 >= 0
So y1 – y2 >= 0 or y1 >= y2

Question 17.
Analyze Relationships
The equations of two functions are y = −21x + 9 and y = −24x + 8. Which function is changing more quickly? Explain.
______________

Answer:
y = -21x + 9
y = -24x + 8
y = -24x + 8 is changing more quickly as the absolute value of -24 is greater than the absolute value of -21.

Guided Practice – Analyzing Graphs – Page No. 176

In a lab environment, colonies of bacteria follow a predictable pattern of growth. The graph shows this growth over time.
Go Math Grade 8 Answer Key Chapter 6 Functions Lesson 4: Analyzing Graphs img 21

Question 1.
What is happening to the population during Phase 2?
______________

Answer:
For Phase 2, the graph is increasing quickly. This shows a period of rapid growth.

Question 2.
What is happening to the population during Phase 4?
______________

Answer:
In Phase 4, the graph is decreasing, hence the number of bacterias is decreasing.

The graphs give the speeds of three people who are riding snowmobiles. Tell which graph corresponds to each situation.
Go Math Grade 8 Answer Key Chapter 6 Functions Lesson 4: Analyzing Graphs img 22

Question 3.
Chip begins his ride slowly but then stops to talk with some friends. After a few minutes, he continues his ride, gradually increasing his speed.
______________

Answer:
Graph 2

Explanation:
The slope of the graph is increasing, then it becomes constant and starts increasing again.
Graph 2

Question 4.
Linda steadily increases her speed through most of her ride. Then she slows down as she nears some trees.
______________

Answer:
Graph 3

Explanation:
The slope of the graph is increasing and then decreasing.
Graph 3

Question 5.
Paulo stood at the top of a diving board. He walked to the end of the board, and then dove forward into the water. He plunged down below the surface, then swam straight forward while underwater. Finally, he swam forward and upward to the surface of the water. Draw a graph to represent Paulo’s elevation at different distances from the edge of the pool.
Go Math Grade 8 Answer Key Chapter 6 Functions Lesson 4: Analyzing Graphs img 23
Type below:
______________

Answer:
Grade 8 Chapter 6 image 12

Independent Practice – Analyzing Graphs – Page No. 177

Tell which graph corresponds to each situation below.
Go Math Grade 8 Answer Key Chapter 6 Functions Lesson 4: Analyzing Graphs img 24

Question 6.
Arnold started from home and walked to a friend’s house. He stayed with his friend for a while and then walked to another friend’s house farther from home.
______________

Answer:
Graph 3

Explanation:
The graph increases (as Arnold walks from home to friend’s house), then becomes constant (when he stayed with his friend) and then increases again (when he walk to another friend’s house farther away).
Graph 3

Question 7.
Francisco started from home and walked to the store. After shopping, he walked back home.
______________

Answer:
Graph 1

Explanation:
The graph increases (as Francisco walked from home to store), becomes constant (when he shops), and then decreases (as he walked back home)
Graph 1

Question 8.
Celia walks to the library at a steady pace without stopping.
______________

Answer:
Graph 2

Explanation:
The graph increases at a constant rate (as Celia walks to library without any stops)
Graph 2

Regina rented a motor scooter. The graph shows how far away she is from the rental site after each half hour of riding.
Go Math Grade 8 Answer Key Chapter 6 Functions Lesson 4: Analyzing Graphs img 25

Question 9.
Represent Real-World Problems
Use the graph to describe Regina’s trip. You can start the description like this: “Regina left the rental shop and rode for an hour…”
Type below:
______________

Answer:
Regina left the rental shop and rode for an hour. She rested for half an hour and then started back. After half an hour, she changed her mind and rode for another half an hour. She rest for half an hour. Then she started back and ranched the rental site in 2 hours.

Question 10.
Analyze Relationships
Determine during which half hour Regina covered the greatest distance.
Type below:
______________

Answer:
Regina covered the greatest distance between 0.5 to 1hr of the journey. She covered 12 miles.

Analyzing Graphs – Page No. 178

The data in the table shows the speed of a ride at an amusement park at different times one afternoon.
Go Math Grade 8 Answer Key Chapter 6 Functions Lesson 4: Analyzing Graphs img 26

Question 11.
Sketch a graph that shows the speed of the ride over time.
Type below:
______________

Answer:
Grade 8 Chapter 6 image 13

Question 12.
Between which times is the ride’s speed increasing the fastest?
Type below:
______________

Answer:
The speed is increasing the fastest during the 3: 21 and 3: 22

Question 13.
Between which times is the ride’s speed decreasing the fastest?
Type below:
______________

Answer:
The speed is decreasing the fastest during the 3: 23 and 3: 24

H.O.T.

Focus on Higher Order Thinking
Go Math Grade 8 Answer Key Chapter 6 Functions Lesson 4: Analyzing Graphs img 27

Question 14.
Justify Reasoning
What is happening to the fox population before time t? Explain your reasoning.
Go Math Grade 8 Answer Key Chapter 6 Functions Lesson 4: Analyzing Graphs img 28
Type below:
______________

Answer:
The population decreases and then increases before time t

Question 15.
What If?
Suppose at time t, a conservation organization moves a large group of foxes to the island. Sketch a graph to show how this action might affect the population on the island after time t.
Go Math Grade 8 Answer Key Chapter 6 Functions Lesson 4: Analyzing Graphs img 29
Type below:
______________

Answer:
Grade 8 Chapter 6 image 14

Explanation:
Population is decreasing at first, then it is increasing rapidly.

Question 16.
Make a Prediction
At some point after time t, a forest fire destroys part of the woodland area on the island. Describe how your graph from problem 15 might change.
Type below:
______________

Answer:
The population would dramatically decrease if there was a fire due to lack of food supply and good land.

6.1 Identifying and Representing Functions – Model Quiz – Page No. 179

Determine whether each relationship is a function.

Question 1.
Go Math Grade 8 Answer Key Chapter 6 Functions Model Quiz img 30
__________

Answer:
Not a function

Explanation:
A relationship is a function when each input is paired with exactly one output. The input 5 has more than one output.
Not a function

Question 2.
Go Math Grade 8 Answer Key Chapter 6 Functions Model Quiz img 31
__________

Answer:
Function

Explanation:
A relationship is a function when each input is paired with exactly one output.
Each input is paired with only one output.
Function

Question 3.
(2, 5), (7, 2), (−3, 4), (2, 9), (1, 1)
__________

Answer:
Not a function

Explanation:
A relationship is a function when each input is paired with exactly one output. Input 2 has more than one output.
Not a function

6.2 Describing Functions

Determine whether each situation is linear or nonlinear, and proportional or nonproportional.

Question 4.
Joanna is paid $14 per hour.
__________
__________

Answer:
Linear
Proportional

Explanation:
Writing the situation as an equation, where x is the number of hours.
y = 14x
Compare with general linear equation y = mx + b
Linear
Since b = 0, the relationship is proportional.
Proportional

Question 5.
Alberto started out bench pressing 50 pounds. He then added 5 pounds every week.
__________
__________

Answer:
Linear
Non-proportional

Explanation:
Writing the situation as an equation, where x is the number of hours.
y = 5x + 50
Compare with general linear equation y = mx + b
Linear
Since b is not equal to 0, the relationship is non-proportional.
Non-proportional

6.3 Comparing Functions

Question 6.
Which function is changing more quickly? Explain.
Go Math Grade 8 Answer Key Chapter 6 Functions Model Quiz img 32
__________

Answer:
Function 2 is changing more quickly.

Explanation:
Find the rate of change for function 1
Rate of Change = (20 – 0)/(0 – 5) = -4
Find the rate of change for function 1
Rate of Change = (6.5 – 11)/(3 – 2) = -4.5
Althogh -4.5 < -4, the absolute value of -4.5 s greater than -4.
Function 2 is changing more quickly.

6.4 Analyzing Graphs

Question 7.
Describe a graph that shows Sam running at a constant rate.
Type below:
______________

Answer:
The graph would be a straight line

Explanation:
Since Sam is running at a constant rate, distance covered per unit of time remains the same and the relationship is linear and proportional.
The graph would be a straight line

Essential Question

Question 8.
How can you use functions to solve real-world problems?
Type below:
______________

Answer:
If in the equation the power of x is 1 then it is linear otherwise nonlinear.
In a graph, if the points form a line it is linear if they form a curve it is a nonlinear function.

Selected Response – Mixed Review – Page No. 180

Question 1.
Which table shows a proportional function?
Go Math Grade 8 Answer Key Chapter 6 Functions Mixed Review img 33
Options:
a. A
b. B
c. C
d. D

Answer:
c. C

Explanation:
It contains the ordered pair of the origin (0, 0)
Option C represents a proportional relationship.

Question 2.
What is the slope and y-intercept of the function shown in the table?
Go Math Grade 8 Answer Key Chapter 6 Functions Mixed Review img 34
Options:
a. m = -2; b = -4
b. m = -2; b = 4
c. m = 2; b = 4
d. m = 4; b = 2

Answer:
c. m = 2; b = 4

Explanation:
Find the slope using two points from the grapgh by
Slope m = (y2 -y1)/(x2 – x1) where (x1, y1) = (1, 6) and (x2, y2) = (4, 12)
Slope m = (y2 -y1)/(x2 – x1) = (12 – 6)/(4 – 1) = 6/3 = 2
Substituting the value of the slope m and (x, y) to find the slope-intercept form.
12 = 4(2) + b
y-intercept b = 4

Question 3.
The table below shows some input and output values of a function.
Go Math Grade 8 Answer Key Chapter 6 Functions Mixed Review img 35
What is the missing output value?
Options:
a. 20
b. 21
c. 22
d. 23

Answer:
b. 21

Explanation:
Find the rate of change = (17.5 – 14)/(5 – 4) = 3.5
Since the missing output is corresponding to x = 6 and 3.5 to 17.5 (for x = 5)
Output = 17.5 + 3.5 = 21

Question 4.
Tom walked to school at a steady pace, met his sister, and they walked home at a steady pace. Describe this graph.
Options:
a. V-shaped
b. upside down V-shaped
c. Straight line sloping up
d. Straight line sloping down

Answer:
b. upside-down V-shaped

Explanation:
The graph would increase at a constant rate and would decrease at a constant rate.
The graph would be the upside-down V-shaped

Mini-Task

Question 5.
Linear functions can be used to find the price of a building based on its floor area.Below are two of these functions.
y = 40x + 15,000
Go Math Grade 8 Answer Key Chapter 6 Functions Mixed Review img 36
a. Find and compare the slopes.
Type below:
____________

Answer:
Compare the slopes
The slope for the first function is less than the slope of the second function.
y = 40x + 15000
Compare with slope intercept form y = mx + b where m is the slope m = 40
Second function find the slope using given points by Slope m = (y2 -y1)/(x2 – x1) where (x1, y1) = (7, 3) and (x2, y2) = (6, 4)
Slope m = (y2 -y1)/(x2 – x1) = (56000 – 32000)/(700 – 400) = 24000/300 = 80
m = 80

Question 5.
b. Find and compare the y-intercepts.
Type below:
____________

Answer:
y = 40x + 15,000
Compare with slope-intercept form y = mx + b where m is the slope b = 15000
The second function find the slope using given points by Slope m and (x, y) in the slope-intercept form to fins y-intercept b
y = mx + b where (x, y) = (700, 56000) and m = 80
56000 = 80(700) + b
b = 0
Compare y intercepts
The y-intercept of the first function is greater than the y-intercept of the second function

Question 5.
c. Describe each function as proportional or nonproportional.
Type below:
____________

Answer:
Comparable to slope interecept form y = mx + b
First function: y = 40x + 15000
Second function: y = 80x
Since b is not equal to 0
First function is non-proportional
Since b = 0
The second function is proportional.

Conclusion:

we wish the detailed prevailed in HMH Go Math 8th Grade Chapter 6 Functions are helpful for you to score the best in the exams. Download our Go Math Grade 8 Solution Key Chapter 6 Functions PDF for free of cost. Keep in touch with us to get the fastest updates about the Go Math Grade 8 Chapter 6 Functions Answer Key. For any queries, you can post your comments in the below comment section.

Go Math Grade 4 Answer Key Chapter 8 Multiply Fractions by Whole Numbers

go-math-grade-4-chapter-8-multiply-fractions-by-whole-numbers-answer-key

Download Go Math Grade 4 Answer Key Chapter 8 Multiply Fractions by Whole Numbers pdf for free of cost. Check out the 4th Grade HMH Go Math chapter 8 questions during your preparation and know the topics clearly. Solving the Chapter Test, Practice Test covered questions will help students to score more marks in the exams. Also, they can match the answers with the help of the Go Math Grade 4 Answer Key Chapter 8. Grade 4 Go Math Answer Key Chapter 8 Multiply Fractions by Whole Numbers provided Step by Step Solutions helps you to master the concepts and become a pro in the subject.

Go Math Grade 4 Chapter 8 Multiply Fractions by Whole Numbers Answer Key

The topics of Chapter 8 Multiply Fractions by Whole Numbers having different kinds of methods to solve the questions in no time. Concepts and their step-wise answers are provided in our HMH Go Math Grade 4 Ch 8 Answer Key. So, 4th Grade ch 8 will be easy to solve for those students who practice the sums from Go Math Grade 4 Solutions Key Ch 8 Multiply Fractions by Whole Number.

Lesson 1: Multiples of Unit Fractions

Lesson 2: Multiples of Fractions

Mid-Chapter Checkpoint

Lesson 3: Multiply a Fraction by a Whole Number Using Models

Lesson 4: Multiply a Fraction or Mixed Number by a Whole Number

Lesson 5: Problem Solving • Comparison Problems with Fractions

Review/Test

Common Core – New – Page No. 459

Multiples of Unit Fractions

Write the fraction as a product of a whole number and a unit fraction.

Question 1.
Go Math Grade 4 Answer Key Chapter 8 Multiply Fractions by Whole Numbers Common Core - New img 1

Answer:
5 x 1/6

Explanation:
Given that 5/6 or 5 sixth-size parts.
Each sixth-size part of the given fraction can be shown by the unit fraction 1/6.
You can use unit fractions to show 5/6
5/6 = 5 x 1/6.

Question 2.
\(\frac{7}{8}\) =
Type below:
__________

Answer:
7 x 1/8

Explanation:
Given that 7/8 or 7 eighth-size parts.
Each eighth-size part of the given fraction can be shown by the unit fraction 1/8.
You can use unit fractions to show 7/8
7/8 = 7 x 1/8.

Question 3.
\(\frac{5}{3}\) =
Type below:
__________

Answer:
5 x 1/3

Explanation:
Given that 5/3 or 5 third-size parts.
Each third-size part of the given fraction can be shown by the unit fraction 1/3.
You can use unit fractions to show 5/6
5/3 = 5 x 1/3.

Question 4.
\(\frac{9}{10}\) =
Type below:
__________

Answer:
9 x 1/10

Explanation:
Given that 9/10 or 9 tenth-size parts.
Each tenth-size part of the given fraction can be shown by the unit fraction 1/10.
You can use unit fractions to show 9/10
9/10 = 9 x 1/10.

Question 5.
\(\frac{3}{4}\) =
Type below:
__________

Answer:
3 x 1/4

Explanation:
Given that 3/4 or 3 fourth-size parts.
Each fourth-size part of the given fraction can be shown by the unit fraction 1/4.
You can use unit fractions to show 5/6
3/4 = 3 x 1/4.

Question 6.
\(\frac{11}{12}\) =
Type below:
__________

Answer:
11 x 1/12

Explanation:
Given that 11/12 or 11 twelve-size parts.
Each twelve-size part of the given fraction can be shown by the unit fraction 1/12.
You can use unit fractions to show 5/6
11/12 = 11 x 1/12.

Question 7.
\(\frac{4}{6}\) =
Type below:
__________

Answer:
4 x 1/6

Explanation:
Given that 4/6 or 4 sixth-size parts.
Each sixth-size part of the given fraction can be shown by the unit fraction 1/6.
You can use unit fractions to show 4/6
4/6 = 4 x 1/6.

Question 8.
\(\frac{8}{20}\) =
Type below:
__________

Answer:
8 x 1/20

Explanation:
Given that 8/20 or 8 twenty-size parts.
Each twenty-size part of the given fraction can be shown by the unit fraction 1/20.
You can use unit fractions to show 8/20
8/20 = 8 x 1/20.

Question 9.
\(\frac{13}{100}\) =
Type below:
__________

Answer:
13 x 1/100

Explanation:
Given that 13/100 or 13 hundred-size parts.
Each hundred-size part of the given fraction can be shown by the unit fraction 1/100.
You can use unit fractions to show 13/100
13/100 = 13 x 1/100.

List the next four multiples of the unit fraction.

Question 10.
\(\frac{1}{5}\) ,
Type below:
__________

Answer:
2/5, 3/5, 4/5, 5/5

Explanation:
Grade 4 Chapter 8 Multiply Fractions Image 2
2/5, 3/5, 4/5, 5/5

Question 11.
\(\frac{1}{8}\) ,
Type below:
__________

Answer:
2/8, 3/8, 4/8, 5/8

Explanation:
Grade 4 Chapter 8 Multiply Fractions Image 3
2/8, 3/8, 4/8, 5/8

Problem Solving

Question 12.
So far, Monica has read \(\frac{5}{6}\) of a book. She has read the same number of pages each day for 5 days. What fraction of the book does Monica read each day?
\(\frac{□}{□}\) of the book

Answer:
1/6 of the book

Explanation:
Monica has read 5/6 of a book. She has read the same number of pages each day for 5 days.
For 1 day, she read one page. In total, she read 5 pages in 5 days. So, Monica read 1/6 of a book each day.

Question 13.
Nicholas buys \(\frac{3}{8}\) pound of cheese. He puts the same amount of cheese on 3 sandwiches. How much cheese does Nicholas put on each sandwich?
\(\frac{□}{□}\) pound of cheese

Answer:
1/8 pound of cheese

Explanation:
Nicholas buys 3/8 pound of cheese. He bought 3 sandwiches. Then, he applied 3/8 pound of cheese on 3 sandwiches. So, 3 x 1/8 cheese he put on 3 sandwiches. So, for one sandwich he put 1/8 pound of cheese.

Common Core – New – Page No. 460

Lesson Check

Question 1.
Selena walks from home to school each morning and back home each afternoon. Altogether, she walks \(\frac{2}{3}\) mile each day. How far does Selena live from school?
Options:
a. \(\frac{1}{3}\) mile
b. \(\frac{2}{3}\) mile
c. 1 \(\frac{1}{3}\) mile
d. 2 miles

Answer:
a. \(\frac{1}{3}\) mile

Explanation:
Selena walks from home to school each morning and back home each afternoon. Altogether, she walks 2/3 miles each day. The distance between home and school will remain the same. So, 2/3 x 1/2 = 1/3 mile far Selena live from the school.

Question 2.
Will uses \(\frac{3}{4}\) cup of olive oil to make 3 batches of salad dressing. How much oil does Will use for one batch of salad dressing?
Options:
a. \(\frac{1}{4}\) cup
b. \(\frac{1}{3}\) cup
c. 2 \(\frac{1}{3}\) cups
d. 3 cups

Answer:
1/8 pound of cheesa. \(\frac{1}{4}\) cup

Explanation:
Will uses 34 cups of olive oil to make 3 batches of salad dressing. To know the one batch of salad dressing, we need to take one part of salad dressing = 1/3. So, 3/4 x 1/3 = 1/4 cup of olive oil will use for one batch of salad dressing.

Spiral Review

Question 3.
Liza bought \(\frac{5}{8}\) pound of trail mix. She gives \(\frac{2}{8}\) pound of trail mix to Michael. How much trail mix does Liza have left?
Options:
a. \(\frac{1}{8}\) pound
b. \(\frac{2}{8}\) pound
c. \(\frac{3}{8}\) pound
d. \(\frac{4}{8}\) pound

Answer:
c. \(\frac{3}{8}\) pound

Explanation:
Liza bought 58 pound of trail mix. She gives 28 pound of trail mix to Michael.
So, Liza have left 5/8 – 2/8 = 3/8 trail mix.

Question 4.
Leigh has a piece of rope that is 6 \(\frac{2}{3}\) feet long. How do you write 6 \(\frac{2}{3}\) as a fraction greater than 1?
Options:
a. \(\frac{11}{3}\)
b. \(\frac{15}{3}\)
c. \(\frac{20}{3}\)
d. \(\frac{62}{3}\)

Answer:
c. \(\frac{20}{3}\)

Explanation:
Multiply the denominator with the whole number. i.e Multiply 3 with 6 in the given example, 6 (2/3).
3 x 6 =18.
Add 18 + 2 =20.
Keep the Denominator the same i.e. 3.
The obtained fraction is 20/3.

Question 5.
Randy’s house number is a composite number. Which of the following could be Randy’s house number?
Options:
a. 29
b. 39
c. 59
d. 79

Answer:
b. 39

Explanation:
The composite numbers can be defined as the whole numbers that have more than two factors. Whole numbers that are not prime are composite numbers because they are divisible by more than two numbers. 39 is the composite number. 39 is divide by 13 and 3.

Question 6.
Mindy buys 12 cupcakes. Nine of the cupcakes have chocolate frosting and the rest have vanilla frosting. What fraction of the cupcakes have vanilla frosting?
Options:
a. \(\frac{1}{4}\)
b. \(\frac{1}{3}\)
c. \(\frac{2}{3}\)
d. \(\frac{3}{4}\)

Answer:
a. \(\frac{1}{4}\)

Explanation:
Mindy buys 12 cupcakes.
Nine of the cupcakes have chocolate frosting = 9/12.
The rest have vanilla frosting. So, there are 3 cups remained = 3/12 = 1/4.
1/4 cupcakes have vanilla frosting.

Page No. 463

Question 1.
Write three multiples of \(\frac{3}{8}\).
Go Math Grade 4 Answer Key Chapter 8 Multiply Fractions by Whole Numbers img 2
1 × \(\frac{3}{8}\) = \(\frac{■}{8}\)
2 × \(\frac{3}{8}\) = \(\frac{■}{8}\)
3 × \(\frac{3}{8}\) = \(\frac{■}{8}\)
Multiples of \(\frac{3}{8}\) are ____ , ____ , and ____ .
Type below:
__________

Answer:
3/8, 6/8, 9/8, 12/8.

Explanation:
1 x 3/8 = 3/8.
2 x 3/8 = 6/8.
3 x 3/8 = 9/8.
4 x 3/8 = 12/8.
Multiples of 3/8 are 3/8, 6/8, 9/8, 12/8.

List the next four multiples of the fraction.

Question 2.
\(\frac{3}{6}\) ,
Type below:
__________

Answer:
6/6, 9/6, 12/6, 20/6

Explanation:
1 x 3/6 = 3/6.
2 x 3/6 = 6/6.
3 x 3/6 = 9/6.
4 x 3/6 = 12/6.
5 x 4/6 = 20/6.
Next four multiples of 3/6 are 6/6, 9/6, 12/6, 20/6.

Question 3.
\(\frac{2}{10}\) ,
Type below:
__________

Answer:
4/10, 6/10, 8/10, 10/10

Explanation:
1 x 2/10 = 2/10.
2 x 2/10 = 4/10.
3 x 2/10 = 6/10.
4 x 2/10 = 8/10.
5 x 2/10 = 10/10.
The next four multiples of 2/10 are 4/10, 6/10, 8/10, 10/10.

Write the product as the product of a whole number and a unit fraction.

Question 4.
Go Math Grade 4 Answer Key Chapter 8 Multiply Fractions by Whole Numbers img 3
3 × \(\frac{3}{4}\) =
Type below:
__________

Answer:
9/4 = 9 x 1/4

Explanation:
1 group of 3/4 = 3/4
2 groups of 3/4 = 6/4
3 groups of 3/4 = 9/4
3 x 3/4 = 9/4.

Question 5.
Go Math Grade 4 Answer Key Chapter 8 Multiply Fractions by Whole Numbers img 4
2 × \(\frac{4}{6}\) =
Type below:
__________

Answer:
8/6 = 8 x 1/6

Explanation:
1 group of 4/6 = 4/6
2 groups of 4/6 = 8/6
2 x 4/6 = 8/6 = 8 x 1/6.

List the next four multiples of the fraction.

Question 6.
\(\frac{4}{5}\) ,
Type below:
__________

Answer:
8/5, 12/5, 16/5, 20/5

Explanation:
1 x 4/5 = 4/5.
2 x 4/5 = 8/5.
3 x 4/5 = 12/5.
4 x 4/5 = 16/5.
5 x 4/5 = 20/5.
The next four multiples of 4/5 are 8/5, 12/5, 16/5, 20/5.

Question 7.
\(\frac{2}{4}\) ,
Type below:
__________

Answer:
4/4, 6/4, 8/4, 10/4

Explanation:
1 x 2/4 = 2/4.
2 x 2/4 = 4/4.
3 x 2/4 = 6/4.
4 x 2/4 = 8/4.
5 x 2/4 = 10/4.
The next four multiples of 2/4 are 4/4, 6/4, 8/4, 10/4.

Write the product as the product of a whole number and a unit fraction.

Question 8.
Go Math Grade 4 Answer Key Chapter 8 Multiply Fractions by Whole Numbers img 5
4 × \(\frac{2}{8}\) =
Type below:
__________

Answer:
8/8 = 8 x 1/8

Explanation:
1 group of 2/8 = 2/8
2 groups of 2/8 = 4/8
3 groups of 2/8 = 6/8
4 groups of 2/8 = 8/8
4 x 2/8 = 8/8 = 8 x 1/8.

Question 9.
Go Math Grade 4 Answer Key Chapter 8 Multiply Fractions by Whole Numbers img 6
3 × \(\frac{3}{5}\) =
Type below:
__________

Answer:
9/5 = 9 x 1/5

Explanation:
1 group of 3/5 = 3/5
2 groups of 3/5 = 6/5
3 groups of 3/5 = 9/5
3 x 3/5 = 9/5 = 9 x 1/5.

Question 10.
Use Repeated Reasoning Are \(\frac{6}{10}\) and \(\frac{6}{30}\) multiples of \(\frac{3}{10}\)?
Explain.
Type below:
__________

Answer:
3/30

Explanation:
Use Repeated Reasoning Are 6/10 and 6/30 multiples of 3/10 and 3/30.

Question 11.
Which is greater, 4 × \(\frac{2}{7}\) or 3 × \(\frac{3}{7}\)? Explain.
4 × \(\frac{2}{7}\) _____ 3 × \(\frac{3}{7}\)

Answer:
4 × \(\frac{2}{7}\) __<___ 3 × \(\frac{3}{7}\)

Explanation:
8/7 < 9/7
So, 4 x 2/7 < 3 x 3/7

Page No. 464

Question 12.
Josh is watering his plants. He gives each of 2 plants \(\frac{3}{5}\) pint of water. His watering can holds \(\frac{1}{5}\) pint. How many times will he fill his watering can to water both plants?
Go Math Grade 4 Answer Key Chapter 8 Multiply Fractions by Whole Numbers img 7
a. What do you need to find?
Type below:
__________

Answer:
We need to find how many times Josh needs to fill his watering can to water both plants.

Question 12.
b. What information do you need to use?
Type below:
__________

Answer:
Use the Number of plants = 2.
He gives each plant a 3/5 pint of water.
His watering can hold 1/5 pint.

Question 12.
c. How can drawing a model help you solve the problem?
Type below:
__________

Answer:
Go Math Grade 4 Answer Key Chapter 8 Multiply Fractions by Whole Numbers img 6

Question 12.
d. Show the steps you use to solve the problem.
Type below:
__________

Answer:
If Josh gives each plant 3/5 pint, then that’s a total of 6/5 pint.
6/5 = 6 x 1/5.

Question 12.
e. Complete the sentence. Josh will fill his watering can ____ times.
____ times

Answer:
Josh will fill his watering can 6 times.

Question 13.
Alma is making 3 batches of tortillas. She adds \(\frac{3}{4}\) cup of water to each batch. The measuring cup holds \(\frac{1}{4}\) cup. How many times must Alma measure \(\frac{1}{4}\) cup of water to have enough for the tortillas? Shade the model to show your answer.
Go Math Grade 4 Answer Key Chapter 8 Multiply Fractions by Whole Numbers img 8
Alma must measure \(\frac{1}{4}\) cup ______ times.
____ times

Answer:
12 times

Explanation:
Alma is making 3 batches of tortillas. She adds a 3/4 cup of water to each batch. The measuring cup holds 1/4 cup.
Alma must measure 1/4 cup 12 times.

Common Core – New – Page No. 465

Multiples of Fractions

List the next four multiples of the fraction.

Question 1.
\(\frac{3}{5}\) ,
Type below:
__________

Answer:
6/5, 9/5, 12/5, 20/5

Explanation:
1 x 3/5 = 3/5.
2 x 3/5 = 6/5.
3 x 3/5 = 9/5.
4 x 3/5 = 12/5.
5 x 4/5 = 20/5.
The next four multiples of 3/5 are 6/5, 9/5, 12/5, 20/5.

Question 2.
\(\frac{2}{6}\) ,
Type below:
__________

Answer:
4/6, 6/6, 8/6, 10/6

Explanation:
1 x 2/6 = 2/6.
2 x 2/6 = 4/6.
3 x 2/6 = 6/6.
4 x 2/6 = 8/6.
5 x 2/6 = 10/6.
The next four multiples of 2/6 are 4/6, 6/6, 8/6, 10/6.

Question 3.
\(\frac{4}{8}\) ,
Type below:
__________

Answer:
8/8, 12/8, 16/8, 20/8

Explanation:
1 x 4/8 = 4/8.
2 x 4/8 = 8/8.
3 x 4/8 = 12/8.
4 x 4/8 = 16/8.
5 x 4/8 = 20/8.
The next four multiples of 4/8 are 8/8, 12/8, 16/8, 20/8.

Question 4.
\(\frac{5}{10}\) ,
Type below:
__________

Answer:
10/10, 15/10, 20/10, 25/10

Explanation:
1 x 5/10 = 5/10.
2 x 5/10 = 10/10.
3 x 5/10 = 15/10.
4 x 5/10 = 20/10.
5 x 5/10 = 25/10.
The next four multiples of 5/10 are 10/10, 15/10, 20/10, 25/10.

Write the product as the product of a whole number and a unit fraction.

Question 5.
Go Math Grade 4 Answer Key Chapter 8 Multiply Fractions by Whole Numbers Common Core - New img 9
2 × \(\frac{4}{5}\) =
Type below:
__________

Answer:
8/5 = 8 x 1/5

Explanation:
1 group of 4/5 = 4/5
2 groups of 4/5 = 8/5
2 x 4/5 = 8/5 = 8 x 1/5.

Question 6.
Go Math Grade 4 Answer Key Chapter 8 Multiply Fractions by Whole Numbers Common Core - New img 10
5 × \(\frac{2}{3}\) =
Type below:
__________

Answer:
10/3 = 10 x 1/3

Explanation:
1 group of 2/3 = 2/3
2 group of 2/3 = 4/3
3 group of 2/3 = 6/3
4 group of 2/3 = 8/3
5 group of 2/3 = 10/3
5 x 2/3 = 10/3 = 10 x 1/3.

Problem Solving

Question 7.
Jessica is making 2 loaves of banana bread. She needs \(\frac{3}{4}\) cup of sugar for each loaf. Her measuring cup can only hold \(\frac{1}{4}\) cup of sugar. How many times will Jessica need to fill the measuring cup in order to get enough sugar for both loaves of bread?
_____ times

Answer:
6 times

Explanation:
Jessica is making 2 loaves of banana bread. She needs a 3/4 cup of sugar for each loaf.
For 2 loaves, she needs 2 x 3/4 = 6/4 cups of sugar.
Her measuring cup can only hold 1/4 cup of sugar. So, to get the 3/4 cup of sugar, she needs to fill the cup 3 times. 1/4 + 1/4 + 1/4 = 3/4.
So, to fill 2 loaves, she needs to fill cup 3 x 2 = 6 times.

Question 8.
A group of four students is performing an experiment with salt. Each student must add \(\frac{3}{8}\) teaspoon of salt to a solution. The group only has a \(\frac{1}{8}\) teaspoon measuring spoon. How many times will the group need to fill the measuring spoon in order to perform the experiment?
_____ times

Answer:
12 times

Explanation:
A group of four students is performing an experiment with salt. Each student must add a 3/8 teaspoon of salt to a solution. 4 x 3/8 = 12/8 teaspoon of salt required to finish the experiment.
If they have 1/8 teaspoon measuring spoon, 12 x 1/8.
So, the group needs to fill the measuring spoon 12 times in order to perform the experiment.

Common Core – New – Page No. 466

Lesson Check

Question 1.
Eloise made a list of some multiples of \(\frac{5}{8}\). Which of the following lists could be Eloise’s list?
Options:
a. \(\frac{5}{8}, \frac{10}{16}, \frac{15}{24}, \frac{20}{32}, \frac{25}{40}\)
b. \(\frac{5}{8}, \frac{10}{8}, \frac{15}{8}, \frac{20}{8}, \frac{25}{8}\)
c. \(\frac{5}{8}, \frac{6}{8}, \frac{7}{8}, \frac{8}{8}, \frac{9}{8}\)
d. \(\frac{1}{8}, \frac{2}{8}, \frac{3}{8}, \frac{4}{8}, \frac{5}{8}\)

Answer:
b. \(\frac{5}{8}, \frac{10}{8}, \frac{15}{8}, \frac{20}{8}, \frac{25}{8}\)

Explanation:
1 x 5/8 = 5/8.
2 x 5/8 = 10/8.
3 x 5/8 = 15/8.
4 x 5/8 = 20/8.
5 x 5/8 = 25/8.
The next four multiples of 5/8 are 10/8, 15/8, 20/8, 25/8.

Question 2.
David is filling five \(\frac{3}{4}\) quart bottles with a sports drink. His measuring cup only holds \(\frac{1}{4}\) quart. How many times will David need to fill the measuring cup in order to fill the 5 bottles?
Options:
a. 5
b. 10
c. 15
d. 20

Answer:
c. 15

Explanation:
David is filling five 3/4 quart bottles with a sports drink = 5 x 3/4 = 15/4.
His measuring cup only holds 1/4 quart.
So, 15 x 1/4. David needs to fill the measuring cup 15 times in order to fill the 5 bottles.

Spiral Review

Question 3.
Ira has 128 stamps in his stamp album. He has the same number of stamps on each of the 8 pages. How many stamps are on each page?
Options:
a. 12
b. 14
c. 16
d. 18

Answer:
c. 16

Explanation:
Ira has 128 stamps in his stamp album. He has the same number of stamps on each of the 8 pages.
128/8 = 16 stamps on each page.

Question 4.
Ryan is saving up for a bike that costs $198. So far, he has saved $15 per week for the last 12 weeks. How much more money does Ryan need in order to be able to buy the bike?
Options:
a. $ 8
b. $ 18
c. $ 48
d. $ 180

Answer:
b. $ 18

Explanation:
Ryan is saving up for a bike that costs $198.
So far, he has saved $15 per week for the last 12 weeks = $15 x 12 = $180.
$198 – $180 = $18 need in order to buy the bike.

Question 5.
Tina buys 3 \(\frac{7}{8}\) yards of material at the fabric store. She uses it to make a skirt. Afterward, she has 1 \(\frac{3}{8}\) yards of the fabric leftover. How many yards of material did Tina use?
Options:
a. 1 \(\frac{4}{8}\)
b. 2 \(\frac{1}{8}\)
c. 2 \(\frac{4}{8}\)
d. 5 \(\frac{2}{8}\)

Answer:
c. 2 \(\frac{4}{8}\)

Explanation:
Tina buys 3 7/8 yards of material at the fabric store. She uses it to make a skirt. Afterward, she has 1 3/8 yards of the fabric leftover.
3 -1 = 2; 7/8 – 3/8 = 4/8. So, answer is 2 4/8.

Question 6.
Which list shows the fractions in order from least to greatest?
Options:
a. \(\frac{2}{3}, \frac{3}{4}, \frac{7}{12}\)
b. \(\frac{7}{12}, \frac{3}{4}, \frac{2}{3}\)
c. \(\frac{3}{4}, \frac{2}{3}, \frac{7}{12}\)
d. \(\frac{7}{12}, \frac{2}{3}, \frac{3}{4}\)

Answer:
d. \(\frac{7}{12}, \frac{2}{3}, \frac{3}{4}\)

Explanation:
2/3 = 0.666
3/4 = 0.75
7/12 = 0.5833
7/12, 2/3, 3/4

Page No. 467

Choose the best term from the box.
Go Math Grade 4 Answer Key Chapter 8 Multiply Fractions by Whole Numbers img 11

Question 1.
A __________ of a number is the product of the number and a counting number.
__________

Answer:
Multiple

Question 2.
A _________ always has a numerator of 1.
_________

Answer:
Unit Fraction

List the next four multiples of the unit fraction.

Question 3.
\(\frac{1}{2}\) ,
Type below:
_________

Answer:
2/2, 3/2, 4/2, 5/2

Explanation:
1 x 1/2 = 1/2.
2 x 1/2 = 2/2.
3 x 1/2 = 3/2.
4 x 1/2 = 4/2.
5 x 1/2 = 5/2.
The next four multiples of 1/2 are 2/2, 3/2, 4/2, 5/2.

Question 4.
\(\frac{1}{5}\) ,
Type below:
_________

Answer:
2/5, 3/5, 4/5, 5/5

Explanation:
1 x 1/5 = 1/5.
2 x 1/5 = 2/5.
3 x 1/5 = 3/5.
4 x 1/5 = 4/5.
5 x 1/5 = 5/5.
The next four multiples of 1/5 are 2/5, 3/5, 4/5, 5/5.

Write the fraction as a product of a whole number and a unit fraction.

Question 5.
\(\frac{4}{10}\) = _____ × \(\frac{1}{10}\)

Answer:
4

Explanation:
4/10 = 4 x 1/10

Question 6.
\(\frac{8}{12}\) = _____ × \(\frac{1}{12}\)

Answer:
8

Explanation:
8/12 = 8 x 1/12

Question 7.
\(\frac{3}{4}\) = _____ × \(\frac{1}{4}\)

Answer:
3

Explanation:
3/4 = 3 x 1/4

List the next four multiples of the fraction.

Question 8.
\(\frac{2}{5}\) ,
Type below:
_________

Answer:
4/5, 6/5, 8/5, 10/5

Explanation:
1 x 2/5 = 1/5.
2 x 2/5 = 4/5.
3 x 2/5 = 6/5.
4 x 2/5 = 8/5.
5 x 2/5 = 10/5.
The next four multiples of 1/5 are 4/5, 6/5, 8/5, 10/5.

Question 9.
\(\frac{5}{6}\) ,
Type below:
_________

Answer:
10/6, 15/6, 20/6, 25/6

Explanation:
1 x 5/6 = 5/6.
2 x 5/6 = 10/6.
3 x 5/6 = 15/6.
4 x 5/6 = 20/6.
5 x 5/6 = 25/6.
The next four multiples of 5/6 are 10/6, 15/6, 20/6, 25/6.

Write the product as the product of a whole number and a unit fraction.

Question 10.
Go Math Grade 4 Answer Key Chapter 8 Multiply Fractions by Whole Numbers img 12
4 × \(\frac{2}{6}\) =
Type below:
_________

Answer:
8/6 = 8 x 1/6

Explanation:
1 group of 2/6 = 2/6
2 groups of 2/6 = 4/6
3 groups of 2/6 = 6/6
4 groups of 2/6 = 8/6
4 x 2/6 = 8/6 = 8 x 1/6.

Question 11.
Go Math Grade 4 Answer Key Chapter 8 Multiply Fractions by Whole Numbers img 13
3 × \(\frac{3}{8}\) =
Type below:
_________

Answer:
9/8 = 9 x 1/8

Explanation:
1 group of 3/8 = 3/8
2 groups of 3/8 = 6/8
3 groups of 3/8 = 9/8
3 x 3/8 = 9/8 = 9 x 1/8.

Page No. 468

Question 12.
Pedro cut a sheet of poster board into 10 equal parts. His brother used some of the poster board and now \(\frac{8}{10}\) is left. Pedro wants to make a sign from each remaining part of the poster board. How many signs can he make?
______ signs

Answer:
8 signs

Explanation:
Pedro cut a sheet of poster board into 10 equal parts.
His brother uses some of the poster board and now an 8/10 is left.
So, the remaining part of the b\poster board is 8/10 parts.
Pedro can use 8/ 10 parts of the board to make signs.
So, he can make 8 signs.

Question 13.
Ella is making 3 batches of banana milkshakes. She needs \(\frac{3}{4}\) gallon of milk for each batch. Her measuring cup holds \(\frac{1}{4}\) gallon. How many times will she need to fill the measuring cup to make all 3 batches of milkshakes?
______ times

Answer:
9 times

Explanation:
Ella is making 3 batches of banana milkshakes. She needs 3/4 gallon of milk for each batch. So, she needs 3 x 3/4 = 9/4 cups for 3 batches of banana milkshakes. Her measuring cup holds 1/4 gallon.
9/4 = 9 x 1/4.
So, Ella needs to fill the measuring cup 9 times to make all 3 batches of milkshakes.

Question 14.
Darren cut a lemon pie into 8 equal slices. His friends ate some of the pie and now \(\frac{5}{8}\) is left. Darren wants to put each slice of the leftover pie on its own plate. What part of the pie will he put on each plate?
\(\frac{□}{□}\) of the pie on each plate.

Answer:
5/8 of the pie on each plate

Explanation:
Darren cut a lemon pie into 8 equal slices. His friends ate some of the pie and now 5/8 is left. So, 5 pie slices leftover.
Darren can put 5/8 parts of the pie on each plate.

Question 15.
Beth is putting liquid fertilizer on the plants in 4 flowerpots. Her measuring spoon holds \(\frac{1}{8}\) teaspoon. The directions say to put \(\frac{5}{8}\) teaspoon of fertilizer in each pot. How many times will Beth need to fill the measuring spoon to fertilize the plants in the 4 pots?
______ times

Answer:
20 times

Explanation:
Beth is putting liquid fertilizer on the plants in 4 flowerpots. Her measuring spoon holds 1/8 teaspoon.
The directions say to put 5/8 teaspoon of fertilizer in each pot. So, 4 x 5/8 = 20/8.
20/8 = 20 x 1/8. Beth needs to fill the measuring spoon 20 times to fertilize the plants in the 4 pots.

Page No. 471

Question 1.
Find the product of 3 × \(\frac{5}{8}\).
1 group of \(\frac{5}{8}\) = \(\frac{□}{8}\)
2 groups of \(\frac{5}{8}\) = \(\frac{□}{8}\)
3 groups of \(\frac{5}{8}\) = \(\frac{□}{8}\)
3 × \(\frac{5}{8}\) = \(\frac{□}{□}\)

Answer:
15/8

Explanation:
1 group of 5/8 = 2/8
2 groups of 5/8 = 4/8
3 groups of 5/8 = 6/8
3 x 5/8 = 15/8.

Multiply.

Question 2.
2 × \(\frac{4}{5}\) = \(\frac{□}{□}\)

Answer:
8/5

Explanation:
1 group of 4/5 = 4/5
2 groups of 4/5 = 8/5
2 x 4/5 = 8/5.

Question 3.
4 × \(\frac{2}{3}\) = \(\frac{□}{□}\)

Answer:
8/3

Explanation:
1 group of 2/3 = 2/3
2 groups of 2/3 = 4/3
3 groups of 2/3 = 6/3
4 groups of 2/3 = 8/3
4 x 2/3 = 8/3

Question 4.
5 × \(\frac{3}{10}\) = \(\frac{□}{□}\)

Answer:
15/10

Explanation:
1 group of 3/10 = 3/10
2 groups of 3/10 = 6/10
3 groups of 3/10 = 9/10
4 groups of 3/10 = 12/10
5 groups of 3/10 = 15/10
5 x 3/10 = 15/10

Question 5.
4 × \(\frac{5}{6}\) = \(\frac{□}{□}\)

Answer:
20/6

Explanation:
1 group of 5/6 = 5/6
2 groups of 5/6 = 10/6
3 groups of 5/6 = 15/6
4 groups of 5/6 = 20/6
4 x 5/6 = 20/6

Multiply.

Question 6.
2 × \(\frac{7}{12}\) = \(\frac{□}{□}\)

Answer:
7/6

Explanation:
1 group of 7/12 = 7/12
2 groups of 7/12 = 14/12
2 x 7/12 = 14/12 = 7/6

Question 7.
6 × \(\frac{3}{8}\) = \(\frac{□}{□}\)

Answer:
9/4

Explanation:
1 group of 3/8 = 3/8
2 groups of 3/8 = 6/8
3 groups of 3/8 = 9/8
4 groups of 3/8 = 12/8
5 groups of 3/8 = 15/8
6 groups of 3/8 = 18/8
6 x 3/8 = 18/8 = 9/4

Question 8.
5 × \(\frac{2}{4}\) = \(\frac{□}{□}\)

Answer:
5/2

Explanation:
1 group of 2/4 = 2/4
2 groups of 2/4 = 4/4
3 groups of 2/4 = 6/4
4 groups of 2/4 = 8/4
5 groups of 2/4 = 10/4
5 x 2/4 = 10/4 = 5/2

Question 9.
3 × \(\frac{4}{6}\) = \(\frac{□}{□}\)

Answer:
2

Explanation:
1 group of 4/6 = 4/6
2 groups of 4/6 = 8/6
3 groups of 4/6 = 12/6
3 x 4/6 = 12/6 = 2

Question 10.
2 × \(\frac{5}{10}\) = \(\frac{□}{□}\)

Answer:
2

Explanation:
1 group of 5/10 = 5/10
2 groups of 5/10 = 10/10
2 x 10/10 = 2 x 1 = 2

Question 11.
4 × \(\frac{2}{8}\) = \(\frac{□}{□}\)

Answer:
1

Explanation:
1 group of 2/8 = 2/8
2 groups of 2/8 = 4/8
3 groups of 2/8 = 6/8
4 groups of 2/8 = 8/8
4 x 2/8 = 8/8 = 1

Look for a Pattern Algebra Write the unknown number.

Question 12.
□ × \(\frac{2}{3}\) = \(\frac{12}{3}\)
□ = ____

Answer:
6

Explanation:
Let the unknown number is s.
s x 2/3 = 12/3
s = 12/3 x 3/2 = 6.

Question 13.
5 × \(\frac{□}{4}\) = \(\frac{10}{4}\)
□ = ____

Answer:
2

Explanation:
Let the unknown number is s.
5 x s/4 = 10/4
5/4 x s = 10/4
s = 10/4 x 4/5 =2.

Question 14.
2 × \(\frac{7}{□}\) = \(\frac{14}{8}\)
□ = ____

Answer:
8

Explanation:
Let the unknown number is s.
2 x 7/s = 14/8
14/s = 14/8
s x 14/8 = 14
s = 14 x 8/14
s = 8.

Page No. 472

Question 15.
Lisa makes clothes for pets. She needs \(\frac{5}{6}\) yard of fabric to make 1 dog coat. How much fabric does she need to make 3 dog coats?
a. What do you need to find?
Type below:
_________

Answer:
The number of fabric yards required for 3 dog coats

Question 15.
b. What information do you need?
Type below:
_________

Answer:
How much she needs of fabric for 1 dog coat can helps to find for 3 dog coats.

Question 15.
c. Show the steps you use to solve the problem.
Type below:
_________

Answer:
Lisa makes clothes for pets. She needs a 5/6 yard of fabric to make 1 dog coat.
For 3 dogs = 5/6 x 3 =5/2

Question 15.
d. Complete the sentence.
Lisa needs _____ yards of fabric to make 3 dog coats.
\(\frac{□}{□}\)

Answer:
Lisa needs a 5/2 yard of fabric to make 3 dog coats.

Question 16.
Manuel’s small dog eats \(\frac{2}{4}\) bag of dog food in 1 month. His large dog eats \(\frac{3}{4}\) bag of dog food in 1 month. How many bags do both dogs eat in 6 months?
\(\frac{□}{□}\) bags

Answer:
2 bags

Explanation:
Manuel’s small dog eats a 2/4 bag of dog food in 1 month. His large dog eats a 3/4 bag of dog food in 1 month.
In total 2/4 + 3/4 = 5/4 bag of dog food eat in 1 month.
So, for 6 months = 6 x 5/4 = 30/4 = 15/2.
So, 2 bags need for 6 months.

Question 17.
Select the correct product for the equation.
9 × \(\frac{2}{12}\) = □
3 × \(\frac{6}{7}\) = □
6 × \(\frac{4}{7}\) = □
8 × \(\frac{3}{12}\) = □
Type below:
_________

Answer:
8 × \(\frac{3}{12}\) = 2

Explanation:
9 × \(\frac{2}{12}\) = 3/2
3 × \(\frac{6}{7}\) = 18/7
6 × \(\frac{4}{7}\) = 24/7
8 × \(\frac{3}{12}\) = 2

Common Core – New – Page No. 473

Multiply a Fraction by a Whole Number Using Models

Multiply.

Question 1.
Go Math Grade 4 Answer Key Chapter 8 Multiply Fractions by Whole Numbers Common Core - New img 14

Answer:
Go Math Grade 4 Answer Key Chapter 8 Multiply Fractions by Whole Numbers Common Core - New img 14

Question 2.
3 × \(\frac{2}{5}\) = \(\frac{□}{□}\)

Answer:
Grade 4 Chapter 8 Image 1 473
3 x 2/5 = 6/5

Question 3.
7 × \(\frac{3}{10}\) = \(\frac{□}{□}\)

Answer:
Grade 4 Chapter 8 Image 2 473
7 x 3/10 = 21/10

Question 4.
3 × \(\frac{5}{12}\) = \(\frac{□}{□}\)

Answer:
Grade 4 Chapter 8 Image 3 473
3 x 5/12 = 15/12

Question 5.
6 × \(\frac{3}{4}\) = \(\frac{□}{□}\)

Answer:
Grade 4 Chapter 8 Image 4 473
6 x 3/4 = 18/4

Question 6.
4 × \(\frac{2}{8}\) = \(\frac{□}{□}\)

Answer:
Grade 4 Chapter 8 Image 5 473
4 x 2/8 = 8/8

Question 7.
5 × \(\frac{2}{3}\) = \(\frac{□}{□}\)

Answer:
Grade 4 Chapter 8 Image 6 473
5 x 2/3 = 10/3

Question 8.
2 × \(\frac{7}{8}\) = \(\frac{□}{□}\)

Answer:
Grade 4 Chapter 8 Image 7 473
2 x 7/8 = 14/8

Question 9.
6 × \(\frac{4}{5}\) = \(\frac{□}{□}\)

Answer:
Grade 4 Chapter 8 Image 8 473
6 x 4/5 = 28/5

Problem Solving

Question 10.
Matthew walks \(\frac{5}{8}\) mile to the bus stop each morning. How far will he walk in 5 days?
\(\frac{□}{□}\)

Answer:
25/8 miles

Explanation:
Matthew walks 5/8 mile to the bus stop each morning.
In 5 days, 5 x 5/8 = 25/8 miles.

Question 11.
Emily uses \(\frac{2}{3}\) cup of milk to make one batch of muffins. How many cups of milk will Emily use if she makes 3 batches of muffins?
\(\frac{□}{□}\)

Answer:
6/3 cups of milk

Explanation:
Emily uses a 2/3 cup of milk to make one batch of muffins.
Emily use 3 x 2/3 = 6/3 cups of milk to make 3 batches of muffins

Common Core – New – Page No. 474

Lesson Check

Question 1.
Aleta’s puppy gained \(\frac{3}{8}\) pound each week for 4 weeks. Altogether, how much weight did the puppy gain during the 4 weeks?
Options:
a. \(\frac{8}{12}\) pound
b. 1 \(\frac{2}{8}\) pounds
c. \(\frac{12}{8}\) pounds
d. 4 \(\frac{3}{8}\) pounds

Answer:
6/3 cups of milk

Explanation:
Aleta’s puppy gained 3/8 pound each week.
It gained 4 x 3/8 = 12/8 pounds in 4 weeks.

Question 2.
Pedro mixes \(\frac{3}{4}\) teaspoon of plant food into each gallon of water. How many teaspoons of plant food should Pedro mix into 5 gallons of water?
Options:
a. \(\frac{3}{20}\) teaspoon
b. \(\frac{4}{15}\) teaspoon
c. \(\frac{8}{4}\) teaspoons
d. \(\frac{15}{4}\) teaspoons

Answer:
d. \(\frac{15}{4}\) teaspoons

Explanation:
If Pedro mixes 3/4 teaspoon of plant food into each gallon of water, then 5 x 3/4 = 15/4 teaspoon of plant food mix into 5 gallons of water.

Spiral Review

Question 3.
Ivana has \(\frac{3}{4}\) pound of hamburger meat. She makes 3 hamburger patties. Each patty weighs the same amount. How much does each hamburger patty weigh?
Options:
a. \(\frac{1}{4}\) pound
b. \(\frac{1}{3}\) pound
c. 2 \(\frac{1}{4}\) pounds
d. 3 pounds

Answer:
a. \(\frac{1}{4}\) pound

Explanation:
Ivana has 3/4 pound of hamburger meat. She makes 3 hamburger patties.
Each patty weighs the same amount. So, each hamburger patty weight 1/4 pound.

Question 4.
Which of the following expressions is NOT equal to \(\frac{7}{10}\)?
Options:
a. \(\frac{5}{10}+\frac{1}{10}+\frac{1}{10}\)
b. \(\frac{2}{10}+\frac{2}{10}+\frac{3}{10}\)
c. \(\frac{3}{10}+\frac{3}{10}+\frac{2}{10}\)
d. \(\frac{4}{10}+\frac{2}{10}+\frac{1}{10}\)

Answer:
c. \(\frac{3}{10}+\frac{3}{10}+\frac{2}{10}\)

Explanation:
a. \(\frac{5}{10}+\frac{1}{10}+\frac{1}{10}\) = 7/10
b. \(\frac{2}{10}+\frac{2}{10}+\frac{3}{10}\) = 7/10
c. \(\frac{3}{10}+\frac{3}{10}+\frac{2}{10}\) = 8/10
d. \(\frac{4}{10}+\frac{2}{10}+\frac{1}{10}\) = 7/10

Question 5.
Lance wants to find the total length of 3 boards. He uses the expression 3 \(\frac{1}{2}\) + (2 + 4 \(\frac{1}{2}\)). How can Lance rewrite the expression using both the Associative and Commutative Properties of Addition?
Options:
a. 5 + 4 \(\frac{1}{2}\)
b. (3 \(\frac{1}{2}\) + 2) + 4 \(\frac{1}{2}\)
c. 2 + (3 \(\frac{1}{2}\) + 4 \(\frac{1}{2}\))
d. 3 \(\frac{1}{2}\) + (4 \(\frac{1}{2}\) + 2)

Answer:
She can write as (3 \(\frac{1}{2}\) + 2) + 4 \(\frac{1}{2}\)

Question 6.
Which of the following statements is true?
Options:
a. \(\frac{5}{8}>\frac{9}{10}\)
b. \(\frac{5}{12}>\frac{1}{3}\)
c. \(\frac{3}{6}>\frac{4}{5}\)
d. \(\frac{1}{2}>\frac{3}{4}\)

Answer:
6/3 cups of milk

Explanation:
0.625 > 0.9
0.416 > 0.333
0.5 > 0.8
0.5 > 0.75

Page No. 477

Question 1.
2 × 3 \(\frac{2}{3}\) = □
_____ \(\frac{□}{□}\)

Answer:
7\(\frac{1}{3}\)

Explanation:
3 \(\frac{2}{3}\) = 11/3
2 x 11/3 = 22/3
22/3 = 7 and remainder 1. So, 22/3 = 7 (1/3)

Multiply. Write the product as a mixed number.

Question 2.
6 × \(\frac{2}{5}\) = _____ \(\frac{□}{□}\)

Answer:
2\(\frac{2}{5}\)

Explanation:
6 × \(\frac{2}{5}\) = 12/5. 12/5 = 2 and remainder . So, 12/5 = 2 2/5

Question 3.
3 × 2 \(\frac{3}{4}\) = _____ \(\frac{□}{□}\)

Answer:
8\(\frac{1}{4}\)

Explanation:
2 \(\frac{3}{4}\) = 11/4
3 x 11/4 = 33/4. 33/4 = 8 and remainder 1. So, 33/4 = 8 1/4

Question 4.
4 × 1 \(\frac{5}{6}\) = _____ \(\frac{□}{□}\)

Answer:
7 \(\frac{2}{6}\)

Explanation:
1 \(\frac{5}{6}\) = 11/6
4 x 11/6 = 44/6. 44/6 = 7 and remainder 2. So, 44/6 = 7 2/6

Question 5.
4 × \(\frac{5}{8}\) = _____ \(\frac{□}{□}\)

Answer:
2\(\frac{1}{2}\)

Explanation:
4 × \(\frac{5}{8}\) = 5/2. 5/2 = 2 and remainder 1. So, 5/2 = 2 1/2

Question 6.
6 × \(\frac{5}{12}\) = _____ \(\frac{□}{□}\)

Answer:
2\(\frac{1}{2}\)

Explanation:
6 × \(\frac{5}{12}\) = 5/2. 5/2 = 2 and remainder 1. So, 5/2 = 2 1/2

Question 7.
3 × 3 \(\frac{1}{2}\) = _____ \(\frac{□}{□}\)

Answer:
10 \(\frac{1}{2}\)

Explanation:
3 \(\frac{1}{2}\) = 7/2
3 x 7/2 = 21/2. 21/2 = 10 and remainder 1. So, 21/2 = 10 1/2

Question 8.
2 × 2 \(\frac{2}{3}\) = _____ \(\frac{□}{□}\)

Answer:
5\(\frac{1}{3}\)

Explanation:
2 \(\frac{2}{3}\) = 8/3
2 x 8/3 = 16/3. 16/3 = 5 and remainder 1. So, 16/3 = 5 1/3

Question 9.
5 × 1 \(\frac{2}{4}\) = _____ \(\frac{□}{□}\)

Answer:
7 \(\frac{1}{2}\)

Explanation:
1 \(\frac{2}{4}\) = 6/4
5 x 6/4 = 30/4 = 15/2. 15/2 = 7 and remainder 1. So, 15/2 = 7 1/2

Question 10.
4 × 2 \(\frac{2}{5}\) = _____ \(\frac{□}{□}\)

Answer:
9\(\frac{3}{5}\)

Explanation:
2 \(\frac{2}{5}\) = 12/5
4 x 12/5 = 48/5. 48/5 = 9 and remainder 3. So, 48/5 = 9 3/5

Look for a Pattern Algebra Write the unknown number.

Question 11.
□ × 2 \(\frac{1}{3}\) = 9 \(\frac{1}{3}\)
□ = ______

Answer:
4

Explanation:
2 \(\frac{1}{3}\) = 7/3
9 \(\frac{1}{3}\) = 28/3
Let the unknown numer s.
s x 7/3 = 28/3
s = 4

Question 12.
3 × 2 \(\frac{2}{□}\) = 7 \(\frac{2}{4}\)
□ = ______

Answer:
4

Explanation:
7 \(\frac{2}{4}\) = 30/4
Let the unknown number s. If s is 4, 3 × 2 \(\frac{2}{4}\) = 3 x 10/4 = 30/4.
So, the unknown number is 4.

Question 13.
3 × □ \(\frac{3}{8}\) = 4 \(\frac{1}{8}\)
□ = ______

Answer:
1

Explanation:
4 \(\frac{1}{8}\) = 33/8
Let the unknown number is s. If s is 1, 3 × 1 \(\frac{3}{8}\) = 3 x 11/8 = 33/8.

Question 14.
Describe two different ways to write \(\frac{7}{3}\) as a mixed number.
Type below:
_________

Answer:
One is 2\(\frac{1}{3}\)
Another one is 2 + 1/3

Explanation:
7/3 = 2 and the remainder is 1. So, 2 1/3 is one mixed fraction.
Seond method is 3/3 + 3/3 + 1/3 = 2 + 1/3.

Page No. 478

Use the recipe for 15–18.
Go Math Grade 4 Answer Key Chapter 8 Multiply Fractions by Whole Numbers img 15

Question 15.
Otis plans to make 3 batches of sidewalk chalk. How much plaster of Paris does he need?
______ \(\frac{□}{□}\) cups plaster of Paris

Answer:
4\(\frac{1}{2}\) cups plaster of Paris

Explanation:
1\(\frac{1}{2}\) = 3/2 + 3/2 + 3/2 = 9/2
9/2 = 4, the remainder is 1. So, 4 1/2 cups plaster of Paris need for 3 batches of sidewalk chalk.

Question 16.
What’s the Question? The answer is \(\frac{32}{3}\).
Type below:
_________

Answer:
How many tablespoons of powdered paint are needed for 4 batches of chalk?

Question 17.
Patty has 2 cups of warm water. Is that enough water to make 4 batches of sidewalk chalk? Explain how you know without finding the exact product.
______

Answer:
No. 4 x 1/2 = 2 and also 3/4 is greater than 1/2. So, 4 x 3/4 is greater than 2.

Question 18.
Rita makes sidewalk chalk 2 days a week. Each of those days, she spends 1 \(\frac{1}{4}\) hours making the chalk. How much time does Rita spend making sidewalk chalk in 3 weeks?
______ \(\frac{□}{□}\) hours

Answer:
7\(\frac{1}{2}\) hours

Explanation:
Rita makes sidewalk chalk 2 days a week. Each of those days, she spends 1 1/4 hours making the chalk.
1 week = 2 x 5/4 = 10/4 = 5/2.
3 weeks = 3 x 5/2 = 15/2 = 7 1/2.

Question 19.
Oliver has music lessons Monday, Wednesday, and Friday. Each lesson is \(\frac{3}{4}\) of an hour. Oliver says he will have lessons for 3 \(\frac{1}{2}\) hours this week. Without multiplying, explain how you know Oliver is incorrect.
Type below:
__________

Answer:
3/4 is less than 1, and 1 × 3 = 3. So 3/4 × 3 will also be less than 3.
Oliver’s answer, 3 1/2 is greater than 3, so it is incorrect.

Common Core – New – Page No. 479

Multiply a Fraction or Mixed Number by a Whole Number.

Multiply. Write the product as a mixed number.

Question 1.
Go Math Grade 4 Answer Key Chapter 8 Multiply Fractions by Whole Numbers Common Core - New img 16

Answer:
1\(\frac{5}{10}\)

Explanation:
5 x 3/10 = 15/10 = 1 and remainder is 5. So, the mixed fraction is 1 5/10

Question 2.
3 × \(\frac{3}{5}\) =
______ \(\frac{□}{□}\)

Answer:
1\(\frac{4}{5}\)

Explanation:
3 x 3/5 = 9/5 = 1 and remainder is 4. So, the mixed fraction is 1 4/5

Question 3.
5 × \(\frac{3}{4}\) =
______ \(\frac{□}{□}\)

Answer:
3\(\frac{3}{4}\)

Explanation:
15/4 = 3 and remainder is 3. So, the mixed fraction is 3 3/4

Question 4.
4 × 1 \(\frac{1}{5}\) =
______ \(\frac{□}{□}\)

Answer:
4\(\frac{4}{5}\)

Explanation:
1 \(\frac{1}{5}\) = 6/5.
4 x 6/5 = 24/5 = 4 and the remainder is 4. So, the mixed fraction is 4 4/5

Question 5.
2 × 2 \(\frac{1}{3}\) =
______ \(\frac{□}{□}\)

Answer:
4\(\frac{2}{3}\)

Explanation:
2 \(\frac{1}{3}\) = 7/3.
2 x 7/3 = 14/3.
14/3 = 4 and the remainder is 2. So, the mixed fraction is 4 2/3

Question 6.
5 × 1 \(\frac{1}{6}\) =
______ \(\frac{□}{□}\)

Answer:
5\(\frac{5}{6}\)

Explanation:
1 \(\frac{1}{6}\) = 7/6
5 x 7/6 = 35/6.
35/6 = 5 and the remainder is 5. So, the mixed fraction is 5 5/6

Question 7.
2 × 2 \(\frac{7}{8}\) =
______ \(\frac{□}{□}\)

Answer:
6\(\frac{1}{1}\)

Explanation:
2 \(\frac{7}{8}\) = 23/8
2 x 23/8 = 46/8 = 6 1/1

Question 8.
7 × 1 \(\frac{3}{4}\) =
______ \(\frac{□}{□}\)

Answer:
9\(\frac{3}{4}\)

Explanation:
1 \(\frac{3}{4}\) = 7/4
7 x 7/4 = 39/4
39/4 = 9 and the remainder is 3. So, the mixed fraction is 9 3/4

Question 9.
8 × 1 \(\frac{3}{5}\) =
______ \(\frac{□}{□}\)

Answer:
12\(\frac{4}{5}\)

Explanation:
1 \(\frac{3}{5}\) = 8/5
8 x 8/5 = 64/5
64/5 = 12 and the remainder is 4. So, the mixed fraction is 12 4/5

Problem Solving

Question 10.
Brielle exercises for \(\frac{3}{4}\) hour each day for 6 days in a row. Altogether, how many hours does she exercise during the 6 days?
______ \(\frac{□}{□}\)

Answer:
4\(\frac{2}{4}\)

Explanation:
6 x 3/4 = 18/4 = 4 and the remainder is 2. So, the mixed fraction is 4 2/4.

Question 11.
A recipe for quinoa calls for 2 \(\frac{2}{3}\) cups of milk. Conner wants to make 4 batches of quinoa. How much milk does he need?
______ \(\frac{□}{□}\)

Answer:
10\(\frac{2}{3}\)

Explanation:
quinoa calls for 8/3 cups of milk. Conner wants to make 4 batches of quinoa.
So, 4 x 8/3 = 32/3 = 10 and the remainder is 2. So, the mixed fraction is 10 2/3

Common Core – New – Page No. 480

Lesson Check

Question 1.
A mother is 1 \(\frac{3}{4}\) times as tall as her son. Her son is 3 feet tall. How tall is the mother?
Options:
a. 4 \(\frac{3}{4}\) feet
b. 5 \(\frac{1}{4}\) feet
c. 5 \(\frac{1}{2}\) feet
d. 5 \(\frac{3}{4}\) feet

Answer:
b. 5 \(\frac{1}{4}\) feet

Explanation:
A mother is 1 3/4 times as tall as her son. Her son is 3 feet tall.
So, 3 x 7/4 = 21/4 = 5 and the remainder is 1. The mixed fraction is 5 1/4 feet.

Question 2.
The cheerleaders are making a banner that is 8 feet wide. The length of the banner is 1 \(\frac{1}{3}\) times the width of the banner. How long is the banner?
Options:
a. 8 \(\frac{1}{3}\) feet
b. 8 \(\frac{3}{8}\) feet
c. 10 \(\frac{1}{3}\) feet
d. 10 \(\frac{2}{3}\) feet

Answer:
d. 10 \(\frac{2}{3}\) feet

Explanation:
The cheerleaders are making a banner that is 8 feet wide. he length of the banner is 1 1/3 times the width of the banner.
So, 8 x 4/3 = 32/3 =10 and the remainder is 2. The mixed fraction is 10 2/3 feet.

Spiral Review

Question 3.
Karleigh walks \(\frac{5}{8}\) mile to school every day. How far does she walk to school in 5 days?
Options:
a. \(\frac{5}{40}\) mile
b. \(\frac{25}{40}\) mile
c. \(\frac{10}{8}\) miles
d. \(\frac{25}{8}\) miles

Answer:
d. \(\frac{25}{8}\) miles

Explanation:
5 x 5/8 = 25/8.

Question 4.
Which number is a multiple of \(\frac{4}{5}\)?
Options:
a. \(\frac{8}{10}\)
b. \(\frac{12}{15}\)
c. \(\frac{16}{20}\)
d. \(\frac{12}{5}\)

Answer:
d. \(\frac{12}{5}\)

Explanation:
The multiple of \(\frac{4}{5}\) has the denominator 5. So, \(\frac{12}{5}\) is the correct answer.

Question 5.
Jo cut a key lime pie into 8 equal-size slices. The next day, \(\frac{7}{8}\) of the pie is left. Jo puts each slice on its own plate. How many plates does she need?
Options:
a. 5
b. 6
c. 7
d. 8

Answer:
c. 7

Explanation:
Jo cut a key lime pie into 8 equal-size slices. The next day, \(\frac{7}{8}\) of the pie is left. Jo puts each slice on its own plate. She needs 7 plates.

Question 6.
Over the weekend, Ed spent 1 \(\frac{1}{4}\) hours doing his math homework and 1 \(\frac{3}{4}\) hours doing his science project. Altogether, how much time did Ed spend doing homework over the weekend?
Options:
a. 3 hours
b. 2 \(\frac{3}{4}\) hours
c. 2 \(\frac{1}{2}\) hours
d. 2 hours

Answer:
a. 3 hours

Explanation:
5/4 + 7/4 = 12/4 = 3 hours

Page No. 483

Question 1.
Komodo dragons are the heaviest lizards on Earth. A baby Komodo dragon is 1 \(\frac{1}{4}\) feet long when it hatches. Its mother is 6 times as long. How long is the mother?
First, draw a bar model to show the problem.
Type below:
_________

Answer:
Grade 4 Chapter 8 Image 1 483

Question 1.
Then, write the equation you need to solve.
Type below:
_________

Answer:
A baby Komodo dragon is 5/4 feet.
Her mother is 6 x 5/4 = 30/4 feet long.

Question 1.
Finally, find the length of the mother Komodo dragon.
The mother Komodo dragon is _____ feet long.
______ \(\frac{□}{□}\)

Answer:
7\(\frac{2}{4}\)

Explanation:
30/4 = 7 and the remainder is 2. The mixed fraction is 7 2/4 feet.

Question 2.
What if a male Komodo dragon is 7 times as long as the baby Komodo dragon? How long is the male? How much longer is the male than the mother?
______ \(\frac{□}{□}\) feet long
______ \(\frac{□}{□}\) feet longer

Answer:
35/4 feet long
5/4 feet longer

Explanation:
If a male Komodo dragon is 7 times as long as the baby Komodo dragon, then 7 x 5/4 = 35/4.
35/4 – 30/4 = 5/4 feet male Komodo dragon is grater than female Komodo dragon.

Question 3.
The smallest hummingbird is the Bee hummingbird. It has a mass of about 1 \(\frac{1}{2}\) grams. A Rufous hummingbird’s mass is 3 times the mass of the Bee hummingbird. What is the mass of a Rufous hummingbird?
______ \(\frac{□}{□}\) grams

Answer:
9/2 grams

Explanation:

The smallest hummingbird is the Bee hummingbird. It has a mass of about 1 \(\frac{1}{2}\) grams. A Rufous hummingbird’s mass is 3 times the mass of the Bee hummingbird.
3 x 3/2 = 9/2 grams.

Question 4.
Sloane needs \(\frac{3}{4}\) hour to drive to her grandmother’s house. It takes her 5 times as long to drive to her cousin’s house. How long does it take to drive to her cousin’s house?
______ \(\frac{□}{□}\) hours

Answer:
\(\frac{15}{4}\) hours

Explanation:
5 x 3/4 = 15/4
To drive to her cousin’s house, it takes 15/4 hours.

Page No. 484

Use the table for 5 and 6.

Payton has a variety of flowers in her garden. The table shows the average heights of the flowers.
Go Math Grade 4 Answer Key Chapter 8 Multiply Fractions by Whole Numbers img 17

tulip = 5/4 = 1.25
daisy = 5/2 = 2.5
tiger lily = 10/3 = 3.33
sunflower = 31/4 = 7.75

Question 5.
Make Sense of Problems What is the difference between the height of the tallest flower and the height of the shortest flower in Payton’s garden?
______ \(\frac{□}{□}\) feet

Answer:
6\(\frac{2}{4}\) feet

Explanation:
tallest flower = sunflower
shortest flower = tulip
Difference between the tallest flower and shortest flower = 31/4 – 5/4 = 26/4 =6 and the remainder is 2. So, the mixed fraction is 6 2/4 feet.

Question 6.
Payton says her average sunflower is 7 times the height of her average tulip. Do you agree or disagree with her statement? Explain your reasoning.
Type below:
_________

Answer:
I will disagree with her statement. Tulip = 5/4. 7 x 5/4 = 35/4. 31/4 is smaller than 35/4. Som the statement is not correct.

Question 7.
Miguel ran 1 \(\frac{3}{10}\) miles on Monday. On Friday, Miguel ran 3 times as far as he did on Monday. How much farther did Miguel run on Friday than he did on Monday?
______ \(\frac{□}{□}\) miles

Answer:
3\(\frac{9}{10}\) miles

Explanation:
Miguel ran 13/10 miles on Monday.
On Friday, 3 x 13/10 = 39/10 miles = 3 and the remainder is 9. the mixed fraction is 3 9/10 miles

Question 8.
The table shows the lengths of different types of turtles at a zoo.
Go Math Grade 4 Answer Key Chapter 8 Multiply Fractions by Whole Numbers img 18
For numbers 8a–8d, select True or False for each statement.
a. Daisy is 4 times as long as Tuck.
i. True
ii. False

Answer:
ii. False

Explanation:
Tuck = 7/6
Lolly = 35/6
Daisy = 7/2
7/6 x 4 = 28/6.
So, the statement is false.

Question 8.
b. Lolly is 5 times as long as Tuck.
i. True
ii. False

Answer:
i. True

Explanation:
5 x 7/6 = 35/6.
So, the statement is true.

Question 8.
c. Daisy is 3 times as long as Tuck.
i. True
ii. False

Answer:
i. True

Explanation:
3 x 7/6 = 21/6 = 7/2
So, the statement is true.

Question 8.
d. Lolly is 2 times as long as Daisy.
i. True
ii. False

Answer:
ii. False

Explanation:
2 x 7/2 = 7.
So, the statement is false.

Common Core – New – Page No. 485

Problem Solving Comparison

Problems with Fractions

Read each problem and solve.

Question 1.
A shrub is 1 \(\frac{2}{3}\) feet tall. A small tree is 3 times as tall as the shrub. How tall is the tree?
Go Math Grade 4 Answer Key Chapter 8 Multiply Fractions by Whole Numbers Common Core - New img 19

Answer:
5 feet

Explanation:
Go Math Grade 4 Answer Key Chapter 8 Multiply Fractions by Whole Numbers Common Core - New img 19

Question 2.
You run 1 \(\frac{3}{4}\) miles each day. Your friend runs 4 times as far as you do. How far does your friend run each day?
__________ miles

Answer:
7 miles

Explanation:
4 x 7/4 = 7 miles each day

Question 3.
At the grocery store, Ayla buys 1 \(\frac{1}{3}\) pounds of ground turkey. Tasha buys 2 times as much ground turkey as Ayla. How much ground turkey does Tasha buy?
______ \(\frac{□}{□}\) pounds

Answer:
2\(\frac{2}{3}\) pounds

Explanation:
2 x 4/3 = 8/3 = 2 and the remainder is 2. The mixed fraction is 2 2/3 pounds

Question 4.
When Nathan’s mother drives him to school, it takes \(\frac{1}{5}\) hour. When Nathan walks to school, it takes him 4 times as long to get to school. How long does it take Nathan to walk to school?
\(\frac{□}{□}\) hours

Answer:
\(\frac{4}{5}\) hours

Explanation:
4 x 1/5 = 4/5 hour

Common Core – New – Page No. 486

Lesson Check

Question 1.
A Wilson’s Storm Petrel is a small bird with a wingspan of 1 \(\frac{1}{3}\) feet. A California Condor is a larger bird with a wingspan almost 7 times as wide as the wingspan of the petrel. About how wide is the wingspan of the California Condor?
Options:
a. \(\frac{4}{21}\) foot
b. 2 \(\frac{1}{3}\) feet
c. 7 \(\frac{1}{3}\) feet
d. 9 \(\frac{1}{3}\) feet

Answer:
d. 9 \(\frac{1}{3}\) feet

Explanation:
1 1/3 = 4/3.
7 x 4/3 = 28/3 feet = 9 and the remainder is 1. The mixed fraction is 9 1/3

Question 2.
The walking distance from the Empire State Building in New York City to Times Square is about \(\frac{9}{10}\) mile. The walking distance from the Empire State Building to Sue’s hotel is about 8 times as far. About how far is Sue’s hotel from the Empire State Building?
Options:
a. \(\frac{9}{80}\) mile
b. \(\frac{72}{80}\) mile
c. 1 \(\frac{7}{10}\) miles
d. 7 \(\frac{2}{10}\) miles

Answer:
d. 7 \(\frac{2}{10}\) miles

Explanation:
8 x 9/10 mile = 72/10 mile = 7 and the remainder is 2. The mixed fraction is 7 2/10 miles.

Spiral Review

Question 3.
Which of the following expressions is NOT equal to 3 × 2 \(\frac{1}{4}\)?
Options:
a. 3 × \(\frac{9}{4}\)
b. (3 × 2) + (3 × \(\frac{1}{4}\))
c. 6 \(\frac{3}{4}\)
d. 3 × 2 + \(\frac{1}{4}\)

Answer:
d. 3 × 2 + \(\frac{1}{4}\)

Explanation:
3 × 2 \(\frac{1}{4}\) = 3 x 9/4 = 27/4
a. 3 × \(\frac{9}{4}\) = 27/4
b. (3 × 2) + (3 × \(\frac{1}{4}\)) = 6 + 3/4 = 27/4
c. 6 \(\frac{3}{4}\) = 27/4
d. 3 × 2 + \(\frac{1}{4}\) = 6 + 1/4 = 25/4

Question 4.
At a bake sale, Ron sells \(\frac{7}{8}\) of an apple pie and \(\frac{5}{8}\) of a cherry pie. Altogether, how much pie does he sell at the bake sale?
Options:
a. \(\frac{2}{8}\)
b. \(\frac{12}{16}\)
c. \(\frac{12}{8}\)
d. \(\frac{35}{8}\)

Answer:
c. \(\frac{12}{8}\)

Explanation:
7/8 + 5/8 = 12/8
The bake sale 12/8 pie.

Question 5.
On a ruler, which measurement is between \(\frac{3}{16}\) inch and \(\frac{7}{8}\) inch?
Options:
a. \(\frac{1}{16}\) inch
b. \(\frac{1}{8}\) inch
c. \(\frac{11}{16}\) inch
d. \(\frac{15}{16}\) inch

Answer:
c. \(\frac{11}{16}\) inch

Question 6.
Which of the following numbers is composite?
Options:
a. 4
b. 3
c. 2
d. 1

Answer:
a. 4

Explanation:
4 has more than 2 factors.

Page No. 487

Question 1.
What are the next four multiples of \(\frac{1}{8}\)?
Type below:
_________

Answer:
2/8, 3/8, 4/8, 5/8

Explanation:
1 x 1/8 = 1/8.
2 x 1/8 = 2/8.
3 x 1/8 = 3/8.
4 x 1/8 = 4/8.
5 x 1/8 = 5/8.
Next four multiples of 1/8 are 2/8, 3/8, 4/8, 5/8.

Question 2.
Marta is making 3 servings of fruit salad. She adds \(\frac{3}{8}\) cup blueberries for each serving. Her measuring cup holds \(\frac{1}{8}\) cup. How many times must Marta measure \(\frac{1}{8}\) cup of blueberries to have enough for the fruit salad? Shade the models to show your answer.
Go Math Grade 4 Answer Key Chapter 8 Multiply Fractions by Whole Numbers img 20
Marta must measure \(\frac{1}{8}\) _________ cup times.
_________

Answer:
Grade 4 Chapter 8 Image 1 487

Marta must measure \(\frac{1}{8}\) 9 cup times.

Question 3.
Mickey exercises \(\frac{3}{4}\) hour every day. How many hours does he exercise in 8 days?
_____ hours

Answer:
6 hours

Explanation:
8 x 3/4 = 24/4 = 6

Page No. 488

Question 4.
Molly is baking for the Moms and Muffins event at her school. She will bake 4 batches of banana muffins. She needs 1 \(\frac{3}{4}\) cups of bananas for each batch of muffins.
Part A
Molly completed the multiplication below and said she needed 8 cups of bananas for 4 batches of muffins. What is Molly’s error?
\(4 \times 1 \frac{3}{4}=4 \times \frac{8}{4}=\times \frac{32}{4}=8\)
Type below:
_________

Answer:
4 x 1 3/4 = 4 x 8/4 = 8
Molly did not write the mixed number, 1 3/4 as a fraction correctly. 1 3/4 is not equal to 8/4.

Question 4.
Part B
What is the correct number of cups Molly needs for 4 batches of muffins? Explain how you found your answer.
_____ cups

Answer:
7 cups

Explanation:
She will bake 4 batches of banana muffins. She needs 7/4 cups of bananas for each batch of muffins.
So, if she prepares 4 batches of muffins = 4 x 7/4 = 7 cups of banana.

Question 5.
Which fraction is a multiple of \(\frac{1}{9}\)? Mark all that apply.
Options:
a. \(\frac{3}{9}\)
b. \(\frac{9}{12}\)
c. \(\frac{2}{9}\)
d. \(\frac{4}{9}\)
e. \(\frac{9}{10}\)
f. \(\frac{9}{9}\)

Answer:
a. \(\frac{3}{9}\)
c. \(\frac{2}{9}\)
d. \(\frac{4}{9}\)
f. \(\frac{9}{9}\)

Explanation:
The multiples of \(\frac{1}{9}\) have the denominator of 9.

Question 6.
Mimi recorded a soccer game that lasted 1 \(\frac{2}{3}\) hours. She watched it 3 times over the weekend to study the plays. How many hours did Mimi spend watching the soccer game? Show your work.
_____ hours

Answer:
5 hours

Explanation:
3 x 1 2/3 = 3 x 5/3 = 5 hours.

Question 7.
Theo is comparing shark lengths. He learned that a horn shark is 2 \(\frac{3}{4}\) feet long. A blue shark is 4 times as long. Complete the model. Then find the length of a blue shark.
Go Math Grade 4 Answer Key Chapter 8 Multiply Fractions by Whole Numbers img 21
A blue shark is ____ feet long.
_____

Answer:
Grade 4 Chapter 8 Image 2 487
4 x 11/4 = 11.
A blue shark is 11 feet long.

Page No. 489

Question 8.
Joel made a number line showing the multiples of \(\frac{3}{5}\).
Go Math Grade 4 Answer Key Chapter 8 Multiply Fractions by Whole Numbers img 22
The product 2 × \(\frac{3}{5}\) is shown by the fraction _________ on the number line.
\(\frac{□}{□}\)

Answer:
The product 2 × \(\frac{3}{5}\) is shown by the fraction \(\frac{6}{5}\) on the number line.

Question 9.
Bobby has baseball practice Monday, Wednesday, and Friday. Each practice is 2 \(\frac{1}{2}\) hours. Bobby says he will have practice for 4 hours this week.
Part A
Without multiplying, explain how you know Bobby is incorrect.
Type below:
_________

Answer:
Bobby needs to find 3 × 2 1/2. If he estimates 3 × 2 hours, then he finds the practice is at least 6 hours. 6 is greater than 4, so Bobby’s answer is incorrect.

Question 9.
Part B
How long will Bobby have baseball practice this week? Write your answer as a mixed number. Show your work.
______ \(\frac{□}{□}\) hours

Answer:
7\(\frac{1}{2}\) hours

Explanation:
3 x 2 1/2 = 3 x 5/2 = 15/2 = 7 1/2

Question 10.
Look at the number line. Write the missing fractions.
Go Math Grade 4 Answer Key Chapter 8 Multiply Fractions by Whole Numbers img 23
Type below:
_________

Answer:
9/6, 10/6, 11/6, 12/6

Question 11.
Ana’s dachshund weighed 5 \(\frac{5}{8}\) pounds when it was born. By age 4, the dog weighed 6 times as much. Fill each box with a number or symbol from the list to show how to find the weight of Ana’s dog at age 4. Not all numbers and symbols may be used.
Go Math Grade 4 Answer Key Chapter 8 Multiply Fractions by Whole Numbers img 24
Go Math Grade 4 Answer Key Chapter 8 Multiply Fractions by Whole Numbers img 25
Type below:
_________

Answer:
Grade 4 Chapter 8 Image 1 489

Page No. 490

Question 12.
Asta made a fraction number line to help her find 3 × \(\frac{4}{5}\).
Go Math Grade 4 Answer Key Chapter 8 Multiply Fractions by Whole Numbers img 26
Select a way to write 3 × \(\frac{4}{5}\) as the product of a whole number and a unit fraction.
Go Math Grade 4 Answer Key Chapter 8 Multiply Fractions by Whole Numbers img 27
Type below:
_________

Answer:
Grade 4 Chapter 8 Image 1 490
12 × \(\frac{1}{5}\)

Explanation:
3 x 4/5 = 12/5 = 12 x 1/5.

Question 13.
Yusif wanted to give \(\frac{1}{3}\) of his total toy car collection to 2 of his friends. How many of his toy cars will he give away?
\(\frac{□}{□}\)

Answer:
\(\frac{2}{3}\)

Explanation:
Yusif wanted to give \(\frac{1}{3}\) of his total toy car collection to 2 of his friends. He has three toy cars in total. He has given 2 cars out of 3 cars. So, the answer is \(\frac{2}{3}\).

Question 14.
Select the correct product for the equation.
Go Math Grade 4 Answer Key Chapter 8 Multiply Fractions by Whole Numbers img 28
4 × \(\frac{5}{8}\) = □ 4 × \(\frac{4}{8}\) = □
Type below:
_________

Answer:
4 × \(\frac{5}{8}\) = \(\frac{20}{8}\)
4 × \(\frac{4}{8}\) = \(\frac{16}{8}\)

Page No. 491

Question 15.
The lengths of different types of snakes at a zoo are shown in the table.
Go Math Grade 4 Answer Key Chapter 8 Multiply Fractions by Whole Numbers img 29
For numbers 15a–15d, select True or False for the statement.
a. Bobby is 4 times as long as Kenny.
i. True
ii. False

Answer:
ii. False

Explanation:
Kenny = 3/2
Bobby = 9/2
Puck = 15/2
4 x 3/2 =6
So, the statement is false.

Question 15.
b. Bobby is 3 times as long as Kenny.
i. True
ii. False

Answer:
i. True

Explanation:
3 x 3/2 = 9/2
So, the statement is true.

Question 15.
c. Puck is 5 times as long as Kenny.
i. True
ii. False

Answer:
i. True

Explanation:
5 x 3/2 = 15/2
So, the statement is true.

Question 15.
d. Puck is 2 times as long as Bobby.
i. True
ii. False

Answer:
ii. False

Explanation:
2 x 9/2 = 9
So, the statement is false.

Question 16.
Hank used 3 \(\frac{1}{2}\) bags of seed to plant grass in his front yard. He used 3 times as much seed to plant grass in his back yard. How much seed did Hank need for the backyard?
_____ \(\frac{□}{□}\)

Answer:
10\(\frac{1}{2}\)

Explanation:
3 x 7/2 = 21/2 = 10 and the remainder is 1. The answer is 10 1/2.

Question 17.
Jess made a big kettle of rice and beans. He used 1 \(\frac{1}{2}\) cups of beans. He used 4 times as much rice.
Part A
Draw a model to show the problem.
Type below:
_________

Answer:
Grade 4 Chapter 8 Image 1 491

Question 17.
Part B
Use your model to write an equation. Then solve the equation to find the amount of rice Jess needs.
Type below:
_________

Answer:
6 cups

Explanation:
Rice = 4 x 3/2 = 12/2 = 6.
Jess needs 6 cups of rice.

Page No. 492

Question 18.
Mrs. Burnham is making modeling clay for her class. She needs \(\frac{2}{3}\) cup of warm water for each batch.
Part A
Mrs. Burnham has a 1-cup measure that has no other markings. Can she make 6 batches of modeling clay using only the 1-cup measure? Describe two ways you can find the answer.
Type below:
_________

Answer:
Yes. She needs 6 x 2/3 cups of water. 6 x 2/3 = 12/3 = 4 cups.
So, she can use the 1-cup measure 4 times to make 6 batches.

Question 18.
Part B
The modeling clay recipe also calls for \(\frac{1}{2}\) cup of cornstarch. Nikki says Mrs. Burnham will also need 4 cups of cornstarch. Do you agree or disagree? Explain.
Type below:
_________

Answer:
Disagree; 6 x 1/2 = 3 cups of cornstrach.
She doesn’t need 4 cups of cornstarch.

Question 19.
Donna buys some fabric to make place mats. She needs \(\frac{1}{5}\) yard of each type of fabric. She has 9 different types of fabrics to make her design. Use the following equation. Write the number in the box to make the statement true.
\(\frac{9}{5}\) = ______ × \(\frac{1}{5}\)

Answer:
9

Question 20.
Mr. Tuyen uses \(\frac{5}{8}\) of a tank of gas each week to drive to and from his job. How many tanks of gas does Mr. Tuyen use in 5 weeks? Write your answer two different ways.
Mr. Tuyen uses __________ or _________ tanks of gas.
Type below:
_________

Answer:
Mr. Tuyen uses 25/8 or 3\(\frac{1}{8}\) tanks of gas

Explanation:
5 x 5/8 = 25/8 = 3 and the remainder is 1. So, the mixed fraction is 3 1/8.

Question 21.
Rico is making 4 batches of salsa. Each batch needs \(\frac{2}{3}\) cup of corn. He only has a \(\frac{1}{3}\)– cup measure. How many times must Rico measure \(\frac{1}{3}\) cup of corn to have enough for all of the salsa?
______ times

Answer:
8 times

Explanation:
Rico is making 4 batches of salsa. Each batch needs \(\frac{2}{3}\) cup of corn. He only has a \(\frac{1}{3}\)– cup measure.
So, he needs 2x 1/3 cups for one batch. For 4 batches of salsa, 4 x 2 = 8 cups of corn required.

Page No. 497

Question 1.
Write five tenths as a fraction and as a decimal.
Go Math Grade 4 Answer Key Chapter 8 Multiply Fractions by Whole Numbers img 30
Fraction: __________ Decimal: __________
Type below:
_________

Answer:
Grade 4 Chapter 8 Image 1 497
5/10 = 0.5

Write the fraction or mixed number and the decimal shown by the model.

Question 2.
Go Math Grade 4 Answer Key Chapter 8 Multiply Fractions by Whole Numbers img 31
Type below:
_________

Answer:
3\(\frac{2}{10}\)
three and two-tenths

Question 3.
Go Math Grade 4 Answer Key Chapter 8 Multiply Fractions by Whole Numbers img 32
Type below:
_________

Answer:
\(\frac{8}{10}\)
Grade 4 Chapter 8 Image 2 497

Write the fraction or mixed number and the decimal shown by the model.

Question 4.
Go Math Grade 4 Answer Key Chapter 8 Multiply Fractions by Whole Numbers img 33
Type below:
_________

Answer:
4/10 = 0.4

Explanation:
4 boxes are shaded out of 10 boxes. So, the fraction is 4/10.

Question 5.
Go Math Grade 4 Answer Key Chapter 8 Multiply Fractions by Whole Numbers img 34
Type below:
_________

Answer:
1\(\frac{2}{10}\)

Question 6.
Go Math Grade 4 Answer Key Chapter 8 Multiply Fractions by Whole Numbers img 35
Type below:
_________

Answer:
2\(\frac{9}{10}\)

Question 7.
Go Math Grade 4 Answer Key Chapter 8 Multiply Fractions by Whole Numbers img 36
Type below:
_________
Answer:
3\(\frac{4}{10}\)

Practice: Copy and Solve Write the fraction or mixed number as a decimal.

Question 8.
5 \(\frac{9}{10}\) = _____

Answer:
\(\frac{59}{10}\)

Explanation:
Multiply 10 x 5 = 50.
Add 50 + 9 = 59.
The fraction is 59/10

Question 9.
\(\frac{1}{10}\) = _____

Answer:
0.1

Question 10.
\(\frac{7}{10}\) = _____

Answer:
0.7

Question 11.
8 \(\frac{9}{10}\) = _____

Answer:
\(\frac{89}{10}\)

Explanation:
Multiply 10 x 8 = 80.
Add 80 + 9 = 89.
The fraction is 89/10

Question 12.
\(\frac{6}{10}\) = _____

Answer:
0.6

Question 13.
6 \(\frac{3}{10}\) = _____

Answer:
\(\frac{63}{10}\)

Explanation:
Multiply 10 x 6 = 60.
Add 60 + 3 = 63.
The fraction is 63/10

Question 14.
\(\frac{5}{10}\) = _____

Answer:
0.5

Question 15.
9 \(\frac{7}{10}\) = _____

Answer:
\(\frac{97}{10}\)

Explanation:
Multiply 10 x 9 = 90.
Add 90 +7 = 97.
The fraction is 97/10

Page No. 498

Use the table for 16−19.
Go Math Grade 4 Answer Key Chapter 8 Multiply Fractions by Whole Numbers img 37
Go Math Grade 4 Answer Key Chapter 8 Multiply Fractions by Whole Numbers img 38

Question 16.
What part of the rocks listed in the table are igneous? Write your answer as a decimal.
_____

Answer:
0.5

Question 17.
Sedimentary rocks make up what part of Ramon’s collection? Write your answer as a fraction and in word form.
Type below:
_________

Answer:
3/10 and three-tenths

Question 18.
What part of the rocks listed in the table are metamorphic? Write your answer as a fraction and as a decimal.
Type below:
_________

Answer:
2/10 or 0.2

Question 19.
Communicate Niki wrote the following sentence in her report: “Metamorphic rocks make up 2.0 of Ramon’s rock collection.” Describe her error.
Type below:
_________

Answer:
Metamorphic rocks make up 2.0 of Ramon’s rock collection. But from the given table, it is clearly mentioned that the answer is 0.2. So, she made a mistake to make up Ramon’s rock collection.

Question 20.
Josh paid for three books with two $20 bills. He received $1 in change. Each book was the same price. How much did each book cost?
$ _____ each book

Answer:
$19/3 for each book.

Explanation:
Josh paid for three books with two $20 bills. He received $1 in change. So, he paid $19 for three books. As the each book has same price, the answer is $19/3 for each book.

Question 21.
Select a number shown by the model. Mark all that apply.
Go Math Grade 4 Answer Key Chapter 8 Multiply Fractions by Whole Numbers img 39
Type below:
_________

Answer:
1\(\frac{7}{10}\)
1.7

Conclusion:

We understand the knowledge shed regarding Go Math Grade 4 Answer Key Chapter 8 Multiply Fractions by Whole Numbers PDF has benefited you more. If you have any queries you can examine your knowledge using the Grade 4 Chapter 8 Answer Key Homework Practice FL. Visit our site to ask for details about the Go Math Solution Key of different grades.

Go Math Grade 8 Answer Key Chapter 4 Nonproportional Relationships

go-math-grade-8-chapter-4-nonproportional-relationships-answer-key

Are you searching for Go Math 8th Grade Answer Key Chapter 4 Nonproportional Relationships? If yes, then download Go Math Grade 8 Answer Key pdf from here in this article. It helps the students to score good marks in the exams. All the solutions are prepared by the math experts according to the latest edition. Get free access for all topics on Go Math Grade 8 Chapter 4 Nonproportional Relationships from here.

Go Math Grade 8 Answer Key Chapter 4 Nonproportional Relationships

Check the list of the exercises contained in Go Math Grade 8 Answer Key Chapter 4 Nonproportional Relationships. With the help of this Go Math Grade 8 Chapter 4 Nonproportional Relationships Solution Key students can improve their performance in the tests or assignments. Refer to HMH Go Math Grade 8 Chapter 4 Answer Key to complete your homework in time.

Lesson 1: Representing Linear Nonproportional Relationships

Lesson 2: Determining Slope and y-intercept

Lesson 3: Graphing Linear Nonproportional Relationships Using Slope and y-intercept

Lesson 4: Proportional and Nonproportional Situations 

Lesson 5: Representing Linear Nonproportional Relationships – Model Quiz

Mixed Review 

Guided Practice – Representing Linear Nonproportional Relationships – Page No. 98

Make a table of values for each equation.

Question 1.
y = 2x + 5
Go Math Grade 8 Answer Key Chapter 4 Nonproportional Relationships Lesson 1: Representing Linear Nonproportional Relationships img 1
Type below:
____________

Answer:
grade 8 chapter 4 image 13

Explanation:
y = 2x + 5
Choose several values for x and substitute in the equation to find y.
x = 2(-2) + 5 = 1
x = 2(-1) + 5 = 3
x = 2(0) + 5 = 5
x = 2(1) + 5 = 7
x = 2(2) + 5 = 9

Question 2.
y = \(\frac{3}{8}\)x − 5
Go Math Grade 8 Answer Key Chapter 4 Nonproportional Relationships Lesson 1: Representing Linear Nonproportional Relationships img 2
Type below:
____________

Answer:
grade 8 chapter 4 image 14

Explanation:
y = \(\frac{3}{8}\)x − 5
Choose several values for x and substitute in the equation to find y.
x = 3/8(-8) – 5 = -8
x = 3/8(0) – 5 = -5
x = 3/8(8) – 5 = -2
x = 3/8(16) – 5 = 1
x = 3/8(24) – 5 = 4

Explain why each relationship is not proportional.

Question 3.
Go Math Grade 8 Answer Key Chapter 4 Nonproportional Relationships Lesson 1: Representing Linear Nonproportional Relationships img 3
First calculate \(\frac{y}{x}\) for the values in the table.
____________

Answer:
The relationship is not proportional

Explanation:
Find y/x
3/0 = undefined
7/2 = 3.5
11/4 = 2.75
15/6 = 2.5
19/8 = 2.375
The ratio is not constant, hence relationship is not proportional.

Question 4.
Go Math Grade 8 Answer Key Chapter 4 Nonproportional Relationships Lesson 1: Representing Linear Nonproportional Relationships img 4
__________________

Answer:
The graph is a straight line but does not pass through the origin. So, the relationship is not proportional.

Complete the table for the equation. Then use the table to graph the equation.

Question 5.
y = x − 1
Go Math Grade 8 Answer Key Chapter 4 Nonproportional Relationships Lesson 1: Representing Linear Nonproportional Relationships img 5
Go Math Grade 8 Answer Key Chapter 4 Nonproportional Relationships Lesson 1: Representing Linear Nonproportional Relationships img 6
Type below:
____________

Answer:
grade 8 chapter 4 image 15

grade 8 chapter 4 image 16

Explanation:
y = x – 1
Choose several values of x and substitute in the equation to find y to draw a table.
x = -2; y = -2 – 1 = -2
x = -1; y = -1 -1 = -2
x = 0; y = 0 -1 = -1
x = 1; y = 1 – 1 = 0
x = 2; y = 2 -1 = 1
Also, Plot the ordered pairs from the table. Then draw a line connecting the points to represent all the possible solutions

Essential Question Check-In

Question 6.
How can you choose values for x when making a table of values representing a real world situation?
Type below:
____________

Answer:
When choosing values for x in a real-world situation, you choose positive values with an appropriate interval to represent the array of data.

Independent Practice – Representing Linear Nonproportional Relationships – Page No. 99

State whether the graph of each linear relationship is a solid line or a set of unconnected points. Explain your reasoning.

Question 7.
The relationship between the number of $4 lunches you buy with a $100 school lunch card and the money remaining on the card
____________

Answer:
Set of unconnected points.

Explanation:
You cannot buy a fractional part of a lunch.
Set of unconnected points.

Question 8.
The relationship between time and the distance remaining on a 3-mile walk for someone walking at a steady rate of 2 miles per hour.
____________

Answer:
A solid line

Explanation:
The relationship between time and the distance remaining on a 3-mile walk for someone walking at a steady rate of 2 miles per hour. The distance remaining can be a fraction. The time can be in a fraction as well.
A solid line

Question 9.
Analyze Relationships
Simone paid $12 for an initial year’s subscription to a magazine. The renewal rate is $8 per year. This situation can be represented by the equation y = 8x + 12, where x represents the number of years the subscription is renewed and y represents the total cost.
a. Make a table of values for this situation.
Go Math Grade 8 Answer Key Chapter 4 Nonproportional Relationships Lesson 1: Representing Linear Nonproportional Relationships img 7
Type below:
____________

Answer:
grade 8 chapter 4 image 17

Explanation:
y = 8x + 12
Choose several values for x and substitute in the equation to find y.

Question 9.
b. Draw a graph to represent the situation. Include a title and axis labels.
Type below:
____________

Answer:
grade 8 chapter 4 image 18

Explanation:
Plot the ordered pairs from the table. Then draw a line connecting the points to represent all the possible solutions

Question 9.
c. Explain why this relationship is not proportional.
Go Math Grade 8 Answer Key Chapter 4 Nonproportional Relationships Lesson 1: Representing Linear Nonproportional Relationships img 8
Type below:
____________

Answer:
It is not proportional as the graph does not pass through the origin

Explanation:
When a relationship is proportional, the graph of the equation passes through the origin.
It is not proportional as the graph does not pass through the origin

Question 9.
d. Does it make sense to connect the points on the graph with a solid line? Explain.
Type below:
____________

Answer:
No

Explanation:
No; The subscription is rewened for the entire year and cannot be done for a fraction of the year. The number of years must be a whole numb, so the total cost goes up in $8 increments.

Representing Linear Nonproportional Relationships – Page No. 100

Question 10.
Analyze Relationships
A proportional relationship is a linear relationship because the rate of change is constant (and equal to the constant of proportionality). What is required of a proportional relationship that is not required of a general linear relationship?
Type below:
____________

Answer:
The ratio between one quantity to the other quantity should be constant for a proportional linear relationship. The graph should be a straight line that passes through the origin.

Question 11.
Communicate Mathematical Ideas
Explain how you can identify a linear non-proportional relationship from a table, a graph, and an equation.
Type below:
____________

Answer:
In a table, the ratios y/x will not be equal. A graph will not pass through the origin. An equation will be in the form y = mx + b where b is not equal to 0.

Focus on Higher Order Thinking

Question 12.
Critique Reasoning
George observes that for every increase of 1 in the value of x, there is an increase of 60 in the corresponding value of y. He claims that the relationship represented by the table is proportional. Critique George’s reasoning.
Go Math Grade 8 Answer Key Chapter 4 Nonproportional Relationships Lesson 1: Representing Linear Nonproportional Relationships img 9
Type below:
____________

Answer:
The ratio is not constant, hence the relationship cannot be proportional.

Explanation:
Find y/x
90/1 = 90
150/2 = 75
210/3 = 70
270/4 = 67.5
330/5 = 66
The ratio is not constant, hence the relationship cannot be proportional.

Question 13.
Make a Conjecture
Two parallel lines are graphed on a coordinate plane. How many of the lines could represent proportional relationships? Explain.
Type below:
____________

Answer:
Maximum one

Explanation:
When there are two parallel lines, only one can pass through the origin and a line representing a proportional relationship must pass through the origin.
Maximum one

Guided Practice – Determining Slope and y-intercept – Page No. 104

Find the slope and y-intercept of the line in each graph.

Question 1.
Go Math Grade 8 Answer Key Chapter 4 Nonproportional Relationships Lesson 2: Determining Slope and y-intercept img 10
slope m = _____ y-intercept b = _____
m = ____________
b = ____________

Answer:
slope m = -2 y-intercept b = 1
m = -2
b = 1

Explanation:
Find the slope using two points from the grapgh by
Slope m = (y2 -y1)/(x2 – x1) where (x1, y1) = (0, 1) and (x2, y2) = (2, -3)
Slope m = (y2 -y1)/(x2 – x1) = (-3 – 1)/(2 – 0) = -4/2 = -2
From the graph when x = 0
y-intercept (b) = 1

Question 2.
Go Math Grade 8 Answer Key Chapter 4 Nonproportional Relationships Lesson 2: Determining Slope and y-intercept img 11
slope m = _____ y-intercept b = _____
m = ____________
b = ____________

Answer:
slope m = 5 y-intercept b = -15
m = 5
b = -15

Explanation:
Find the slope using two points from the grapgh by
Slope m = (y2 -y1)/(x2 – x1) where (x1, y1) = (3, 0) and (x2, y2) = (0, -15)
Slope m = (y2 -y1)/(x2 – x1) = (-15 – 0)/(0 – 3) = 15/3 = 5
From the graph when x = 0
y-intercept (b) = -15

Question 3.
Go Math Grade 8 Answer Key Chapter 4 Nonproportional Relationships Lesson 2: Determining Slope and y-intercept img 12
slope m = _____ y-intercept b = _____
Type below:
____________

Answer:
slope m = 3/2 y-intercept b = -2
m = 3/2
b = -2

Explanation:
Find the slope using two points from the grapgh by
Slope m = (y2 -y1)/(x2 – x1) where (x1, y1) = (0, -2) and (x2, y2) = (2, 1)
Slope m = (y2 -y1)/(x2 – x1) = (1 – (-2))/(2 – 0) = 3/2
From the graph when x = 0
y-intercept (b) = -2

Question 4.
Go Math Grade 8 Answer Key Chapter 4 Nonproportional Relationships Lesson 2: Determining Slope and y-intercept img 13
slope m = _____ y-intercept b = _____
m = ____________
b = ____________

Answer:
slope m = -3 y-intercept b = 9
m = -3
b = 9

Explanation:
Find the slope using two points from the grapgh by
Slope m = (y2 -y1)/(x2 – x1) where (x1, y1) = (3, 0) and (x2, y2) = (0, 9)
Slope m = (y2 -y1)/(x2 – x1) = (9 – 0))/(0 – 3) = -9/3 = -3
From the graph when x = 0
y-intercept (b) = 9

Find the slope and y-intercept of the line represented by each table.

Question 5.
Go Math Grade 8 Answer Key Chapter 4 Nonproportional Relationships Lesson 2: Determining Slope and y-intercept img 14
slope m = _____ y-intercept b = _____
m = ____________
b = ____________

Answer:
slope m = 3 y-intercept b = 1
m = 3
b = 1

Explanation:
Find the slope using two points from the grapgh by
Slope m = (y2 -y1)/(x2 – x1) where (x1, y1) = (8, 25) and (x2, y2) = (6, 19)
Slope m = (y2 -y1)/(x2 – x1) = (19 – 25)/(6 – 8) = 6/2 = 3
From the graph when x = 0
y-intercept (b) = 1

Question 6.
Go Math Grade 8 Answer Key Chapter 4 Nonproportional Relationships Lesson 2: Determining Slope and y-intercept img 15
slope m = _____ y-intercept b = _____
m = ____________
b = ____________

Answer:
slope m = -4 y-intercept b = 140
m = -4
b = 140

Explanation:
Find the slope using two points from the grapgh by
Slope m = (y2 -y1)/(x2 – x1) where (x1, y1) = (5, 120) and (x2, y2) = (15, 80)
Slope m = (y2 -y1)/(x2 – x1) = (80 – 120)/(15 – 5) = -40/10 = -4
From the graph when x = 0
y-intercept (b) = 140

Essential Question Check-In

Question 7.
How can you determine the slope and the y-intercept of a line from a graph?
Type below:
____________

Answer:
Choose any two points on the line from the graph and use it to find the slope. Determine the point where the line crosses the y-axis to find the y-intercept.

Independent Pratice – Determining Slope and y-intercept – Page No. 105

Question 8.
Some carpet cleaning costs are shown in the table. The relationship is linear. Find and interpret the rate of change and the initial value for this situation.
Go Math Grade 8 Answer Key Chapter 4 Nonproportional Relationships Lesson 2: Determining Slope and y-intercept img 16
Type below:
_____________

Answer:
Find the slope using two points
Slope m = (y2 -y1)/(x2 – x1) where (x1, y1) = (1, 125) and (x2, y2) = (3, 225)
Slope m = (y2 -y1)/(x2 – x1) = (225 – 125)/(3 – 1) = 100/2 = 50
Find the initial value when the value of x is 0
Work backward from x = 1 to x = 0
(175 – 125)/(2 – 1) = 50/1 = 50
Subtract the difference of x and y from the first point.
x = 1 – 1 = 0
y = 125 – 50 = 75
y intercept (b) = 75
The slope/rate of change represents the increase in the cost of cleaning the rooms for a unit increase in the number of rooms. The y-intercept shows the initial cost of carpet cleaning.

Question 9.
Make Predictions
The total cost to pay for parking at a state park for the day and rent a paddleboat are shown.
Go Math Grade 8 Answer Key Chapter 4 Nonproportional Relationships Lesson 2: Determining Slope and y-intercept img 17
a. Find the cost to park for a day and the hourly rate to rent a paddleboat.
Type below:
_____________

Answer:
$5

Explanation:
Find the slope using two points
Slope m = (y2 -y1)/(x2 – x1) where (x1, y1) = (1, 17) and (x2, y2) = (2, 29)
Slope m = (y2 -y1)/(x2 – x1) = (29 – 17)/(2 – 1) = 12/1 = 12
Find the initial value when the value of x is 0
Work backward from x = 1 to x = 0
(29 – 17)/(2 – 1) = 12/1 = 12
Subtract the difference of x and y from the first point.
x = 1 – 1 = 0
y = 17 – 12 = 5
The cost to park for a day is $5.

Question 9.
b. What will Lin pay if she rents a paddleboat for 3.5 hours and splits the total cost with a friend? Explain.
$ _____________

Answer:
$23.5

Explanation:
When Lin paddles for 3.5hr
Total Cost = 3.5(12) + 5 = 47
Lin’s cost = 47/2 = 23.5

Question 10.
Multi-Step
Raymond’s parents will pay for him to take sailboard lessons during the summer. He can take half-hour group lessons or half-hour private lessons. The relationship between cost and number of lessons is linear.
Go Math Grade 8 Answer Key Chapter 4 Nonproportional Relationships Lesson 2: Determining Slope and y-intercept img 18
a. Find the rate of change and the initial value for the group lessons.
Type below:
____________

Answer:
$25

Explanation:
Find the slope using two points
Slope m = (y2 -y1)/(x2 – x1) where (x1, y1) = (1, 55) and (x2, y2) = (2, 85)
Slope m = (y2 -y1)/(x2 – x1) = (85 – 55)/(2 – 1) = 30/1 = 30
Rate of change is $30 for per lesson
Find the initial value when the value of x is 0
Work backward from x = 1 to x = 0
(85 – 55)/(2 – 1) = 30/1 = 30
Subtract the difference of x and y from the first point.
x = 1 – 1 = 0
y = 55 – 30 = 25
The initial value of the group lesson is $25.

Question 10.
b. Find the rate of change and the initial value for the private lessons.
Type below:
_____________

Answer:
$25

Explanation:
Find the slope using two points
Slope m = (y2 -y1)/(x2 – x1) where (x1, y1) = (1, 75) and (x2, y2) = (2, 125)
Slope m = (y2 -y1)/(x2 – x1) = (125 – 75)/(2 – 1) = 50/1 = 50
Rate of change is $50 for per lesson
Find the initial value when the value of x is 0
Work backward from x = 1 to x = 0
(125 – 75)/(2 – 1) = 50/1 = 50
Subtract the difference of x and y from the first point.
x = 1 – 1 = 0
y = 75 – 50 = 25
The initial value of the private lesson is $25.

Question 10.
c. Compare and contrast the rates of change and the initial values.
Type below:
_____________

Answer:
The initial value for both types of lessons is the same. The rate of change is higher for private lessons then group lesson

Explanation:
Compare the results of a and b
The initial value for both types of lessons is the same. The rate of change is higher for private lessons then group lesson

Vocabulary – Determining Slope and y-intercept – Page No. 106

Explain why each relationship is not linear.

Question 11.
Go Math Grade 8 Answer Key Chapter 4 Nonproportional Relationships Lesson 2: Determining Slope and y-intercept img 19
Type below:
_____________

Answer:
The rate of change is not constant, hence the relationship is not linear

Explanation:
Find the rate of change using two points Slope m = (y2 -y1)/(x2 – x1)
(6.5 – 4.5)/(2 – 1) = 2
(8.5 – 6.5)/(3 – 2) = 2
(11.5 – 8.5)/(4 – 3) = 3
The rate of change is not constant, hence the relationship is not linear

Question 12.
Go Math Grade 8 Answer Key Chapter 4 Nonproportional Relationships Lesson 2: Determining Slope and y-intercept img 20
Type below:
_____________

Answer:
The rate of change is not constant, hence the relationship is not linear

Explanation:
Find the rate of change using two points Slope m = (y2 -y1)/(x2 – x1)
(126 – 140)/(5 – 3) = -7
(110 – 126)/(7 – 5) = -8
(92 – 110)/(9 – 7) = -9
The rate of change is not constant, hence the relationship is not linear

Question 13.
Communicate Mathematical Ideas
Describe the procedure you performed to derive the slope-intercept form of a linear equation.
Type below:
_____________

Answer:
Express the slope m between a random point (x, y) on the line and the point (0, b) where the line crosses the y-axis. Then solve the equation for y.

H.O.T.

Focus on Higher Order Thinking

Question 14.
Critique Reasoning
Your teacher asked your class to describe a realworld situation in which a y-intercept is 100 and the slope is 5. Your partner gave the following description: My younger brother originally had 100 small building blocks, but he has lost 5 of them every month since.
a. What mistake did your partner make?
Type below:
_____________

Answer:
If the brother loses 5 blocks every month, the slope would be -5 and not 5.

Explanation:
When the initial value is decreasing, the slope is negative.
If the brother loses 5 blocks every month, the slope would be -5 and not 5.

Question 14.
b. Describe a real-world situation that does match the situation.
Type below:
_____________

Answer:
I bought a 100 card pack and buy 5 additional cards every month.

Explanation:
Real world situation
I bought a 100 card pack and buy 5 additional cards every month.

Question 15.
Justify Reasoning
John has a job parking cars. He earns a fixed weekly salary of $300 plus a fee of $5 for each car he parks. His potential earnings for a week are shown in the graph. At what point does John begin to earn more from fees than his fixed salary? Justify your answer.
Go Math Grade 8 Answer Key Chapter 4 Nonproportional Relationships Lesson 2: Determining Slope and y-intercept img 21
Type below:
_____________

Answer:
After parking 60 cars, John’s earning become $600 double of his initial base salary of $300.
Hence, after parking 61 cars, his earning from the fee becomes more than his fixed salary.

Explanation:
He earns the same in fees as his fixed salary for perking 300/5 = 60
After parking 60 cars, John’s earning become $600 double of his initial base salary of $300.
Hence, after parking 61 cars, his earning from the fee becomes more than his fixed salary.

Guided Practice – Graphing Linear Nonproportional Relationships Using Slope and y-intercept – Page No. 110

Graph each equation using the slope and the y-intercept.

Question 1.
y = \(\frac{1}{2}\)x − 3
slope = _____ y-intercept = _____
Go Math Grade 8 Answer Key Chapter 4 Nonproportional Relationships Lesson 3: Graphing Linear Nonproportional Relationships Using Slope and y-intercept img 22
Type below:
_____________

Answer:
slope = 1/2 y-intercept = -3
Grade 8 Chapter 4 image 1

Explanation:
y = 1/2 x – 3
The y-intercept is b = -3. Plot the point that contains the y-intercept (0, -3)
The slope m = 1/2. Use the slope to find a second point. From (0, -3) count 1 unit up and 2 unit right. The new point is (2, -2)
Draw a line through the points

Question 2.
y = −3x + 2
slope = _____ y-intercept = _____
Go Math Grade 8 Answer Key Chapter 4 Nonproportional Relationships Lesson 3: Graphing Linear Nonproportional Relationships Using Slope and y-intercept img 23
Type below:
_____________

Answer:
slope = -3 y-intercept = 2
Grade 8 Chapter 4 image 2

Explanation:
y = -3x + 2
The y-intercept is b = 2. Plot the point that contains the y-intercept (0, 2)
The slope m = -3/1. Use the slope to find a second point. From (0, 2) count 3 unit down and 1 unit right. The new point is (1, -1)
Draw a line through the points

Question 3.
A friend gives you two baseball cards for your birthday. Afterward, you begin collecting them. You buy the same number of cards once each week. The equation y = 4x + 2 describes the number of cards, y, you have after x weeks.
a. Find and interpret the slope and the y-intercept of the line that represents this situation. Graph y = 4x + 2. Include axis labels.
Go Math Grade 8 Answer Key Chapter 4 Nonproportional Relationships Lesson 3: Graphing Linear Nonproportional Relationships Using Slope and y-intercept img 24
Type below:
_____________

Answer:
Grade 8 Chapter 4 image 3

Explanation:
y = 4x + 2
The y-intercept is b = 2. Plot the point that contains the y-intercept (0, 2)
The slope m = 4. Use the slope to find a second point. From (0, 2) count 4 unit up and 1 unit right. The new point is (1, 6)
Draw a line through the points

Question 3.
b. Discuss which points on the line do not make sense in this situation. Then plot three more points on the line that do make sense.
Type below:
_____________

Answer:
Grade 8 Chapter 4 image 4

Explanation:
The points with a negative value of x or y do not make sense as the number of cards or weeks cannot be negative.

Essential Question Check-In

Question 4.
Why might someone choose to use the y-intercept and the slope to graph a line?
Type below:
_____________

Answer:
When the relationship is given in the form y = mx + b, the y-intercept (b) and the slope (m) are easily accessible and easily calculable. Therefore, it is a good practice to use them to graph the line.

Independent Practice – Graphing Linear Nonproportional Relationships Using Slope and y-intercept – Page No. 111

Question 5.
Science
A spring stretches in relation to the weight hanging from it according to the equation y = 0.75x + 0.25 where x is the weight in pounds and y is the length of the spring in inches.
a. Graph the equation. Include axis labels.
Type below:
_____________

Answer:
grade 8 chapter 4 image 7

Explanation:
y = 0.75x + 0.25
Slope m = 0.75 and y-intercept = 0.25
Plot the point that contains the y-intercept (0, 0.25)
The slope is m = 0.75/1. Use the slope to find a second point. From (0,0.25) count 0.75 unit up and 1 unit right. The new point is (1, 1)

Question 5.
b. Interpret the slope and the y-intercept of the line.
Type below:
_____________

Answer:
The slope represents the increase in the length of spring in inches for each increase of pound of weight. y-intercept represents the unstretched length of the spring When there is no weight attached.

Question 5.
c. How long will the spring be if a 2-pound weight is hung on it? Will the length double if you double the weight? Explain
Type below:
_____________

Answer:
When there is a 2-pound weight hung, the length of the spring would be 1.75 inches. No, When there is a 4-pound weight hung, the length of the spring would be 3.25 inches and not 3.5 inches.

Look for a Pattern

Identify the coordinates of four points on the line with each given slope and y-intercept.

Question 6.
slope = 5, y-intercept = -1
Type below:
_____________

Answer:
(2, 9)
(3, 14)

Explanation:
slope = 5, y-intercept = -1
Plot the point that contains the y-intercept (0, -1)
The slope is m = 5/1. Use the slope to find a second point. From (0, -1) count 5 unit up and 1 unit right. The new point is (1, 4)
Follow the same procedure to find the remaining three points.
(2, 9)
(3, 14)

Question 7.
slope = -1, y-intercept = 8
Type below:
_____________

Answer:
(2, 6)
(3, 5)

Explanation:
slope = -1, y-intercept = 8
Plot the point that contains the y-intercept (0, 8)
The slope is m = -1/1. Use the slope to find a second point. From (0, 8) count 1 unit down and 1 unit right. The new point is (1, 7)
Follow the same procedure to find the remaining three points.
(2, 6)
(3, 5)

Question 8.
slope = 0.2, y-intercept = 0.3
Type below:
_____________

Answer:
(2, 0.7)
(3, 0.9)

Explanation:
slope = 0.2, y-intercept = 0.3
Plot the point that contains the y-intercept (0, 0.3)
The slope is m = 0.2/1. Use the slope to find a second point. From (0, 0.3) count 0.2 unit up and 1 unit right. The new point is (1, 0.5)
Follow the same procedure to find the remaining three points.
(2, 0.7)
(3, 0.9)

Question 9.
slope = 1.5, y-intercept = -3
Type below:
_____________

Answer:
(2, 0)
(3, 1.5)

Explanation:
slope = 1.5, y-intercept = -3
Plot the point that contains the y-intercept (0, -3)
The slope is m = 1.5/1. Use the slope to find a second point. From (0, -3) count 1.5 unit up and 1 unit right. The new point is (1, -1.5)
Follow the same procedure to find the remaining three points.
(2, 0)
(3, 1.5)

Question 10.
slope = −\(\frac{1}{2}\), y-intercept = 4
Type below:
_____________

Answer:
(4, 2)
(6, 1)

Explanation:
slope = −\(\frac{1}{2}\), y-intercept = 4
Plot the point that contains the y-intercept (0, 4)
The slope is m = −\(\frac{1}{2}\)/1. Use the slope to find a second point. From (0, 4) count 1 unit down and 2 unit right. The new point is (2, 3)
Follow the same procedure to find the remaining three points.
(4, 2)
(6, 1)

Question 11.
slope = \(\frac{2}{3}\), y-intercept = -5
Type below:
_____________

Answer:
(6, -1)
(9, 1)

Explanation:
slope = \(\frac{2}{3}\), y-intercept = -5
Plot the point that contains the y-intercept (0, -5)
The slope is m = \(\frac{2}{3}\). Use the slope to find a second point. From (0, -5) count 2 unit up and 3 unit right. The new point is (3, -3)
Follow the same procedure to find the remaining three points.
(6, -1)
(9, 1)

Question 12.
A music school charges a registration fee in addition to a fee per lesson. Music lessons last 0.5 hour. The equation y = 40x + 30 represents the total cost y of x lessons. Find and interpret the slope and y-intercept of the line that represents this situation. Then find four points on the line.
Type below:
_____________

Answer:
y = 40x + 30
Slope = 40
y-intercept = 30
The slope represents the fee of the classes per lesson and y-intercept represents the registration fee.
Plot the point that contains the y-intercept (0, 30)
The slope is m = 40/1. Use the slope to find a second point. From (0, 30) count 40 unit up and 1 unit right. The new point is (1, 70)
Follow the same procedure to find the remaining three points.
(2, 110)
(3, 150)

Graphing Linear Nonproportional Relationships Using Slope and y-intercept – Page No. 112

Question 13.
A public pool charges a membership fee and a fee for each visit. The equation y = 3x + 50 represents the cost y for x visits.
a. After locating the y-intercept on the coordinate plane shown, can you move up three gridlines and right one gridline to find a second point? Explain.
Go Math Grade 8 Answer Key Chapter 4 Nonproportional Relationships Lesson 3: Graphing Linear Nonproportional Relationships Using Slope and y-intercept img 25
Type below:
_____________

Answer:
Yes

Explanation:
Yes; Since the horizontal and vertical gridlines each represents 25 units, hence moving up 3 gridlines and right 1 gridline represent a slope a 75/25 or 3

Question 13.
b. Graph the equation y = 3x + 50. Include axis labels. Then interpret the slope and y-intercept.
Type below:
_____________

Answer:
grade 8 chapter 4 image 8
The slope represents the fee per visit and the y-intercept represents the membership fee.

Explanation:
Slope = 3
y-intercept = 50
The slope represents the fee of the classes per lesson and the y-intercept represents the registration fee.
Plot the point that contains the y-intercept (0, 50)
The slope is m = 3/1. Use the slope to find a second point. From (0, 50) count 3 unit up and 1 unit right. The new point is (1, 53)

Question 13.
c. How many visits to the pool can a member get for $200?
______ visits

Answer:
50 visits

Explanation:
You would get 50 visits for $200
grade 8 chapter 4 image 9

H.O.T.

Focus on Higher Order Thinking

Question 14.
Explain the Error
A student says that the slope of the line for the equation y = 20 − 15x is 20 and the y-intercept is 15. Find and correct the error.
Type below:
_____________

Answer:
The slope is -15 as it represents the change in y per unit change in x. The y-intercept is 20, when x = 0.

Explanation:
y = 20 − 15x
The slope is -15 as it represents the change in y per unit change in x. The y-intercept is 20, when x = 0.

Question 15.
Critical Thinking
Suppose you know the slope of a linear relationship and a point that its graph passes through. Can you graph the line even if the point provided does not represent the y-intercept? Explain.
Type below:
_____________

Answer:
Yes. You can plot the given point and use the slope to find a second point. Connect the points by drawing a line.

Question 16.
Make a Conjecture
Graph the lines y = 3x, y = 3x − 3, and y = 3x + 3. What do you notice about the lines? Make a conjecture based on your observation.
Go Math Grade 8 Answer Key Chapter 4 Nonproportional Relationships Lesson 3: Graphing Linear Nonproportional Relationships Using Slope and y-intercept img 26
Type below:
_____________

Answer:
grade 8 chapter 4 image 10

Explanation:
let’s tale the example
y = 3x
y = 3x – 3
y = 3x + 3
We notice that the lines are parallel to each other: the slopes of the lines are equal but the y-intersection point differs.

Guided Practice – Proportional and Nonproportional Situations – Page No. 117

Determine if each relationship is a proportional or nonproportional situation. Explain your reasoning.

Question 1.
Go Math Grade 8 Answer Key Chapter 4 Nonproportional Relationships Lesson 4: Proportional and Nonproportional Situations img 27
Look at the origin.
_____________

Answer:
Proportional relationship

Explanation:
Proportional relationship
The graph passes through the origin. Graph of a proportional relationship must pass through the origin

Question 2.
Go Math Grade 8 Answer Key Chapter 4 Nonproportional Relationships Lesson 4: Proportional and Nonproportional Situations img 28
_____________

Answer:
Non-proportional relationship

Explanation:
The graph does not pass through the origin. Graph of a proportional relationship must pass through the origin
Non-proportional relationship

Question 3.
q = 2p + \(\frac{1}{2}\)
Compare the equation with y = mx + b.
_____________

Answer:
q = 2p + \(\frac{1}{2}\)
The equation is in the form y = mx + b, with p being used es the variable instead of x and q instead of y. The value of m is 2, and the value b is 1/2. Since b is not 0, the relationship presented through the above equation is non-proportional.

Question 4.
v = \(\frac{1}{10}\)u
_____________

Answer:
Proportional relationship

Explanation:
v = \(\frac{1}{10}\)u
Compare with the form of equation y = mx + b. The equation represent proportional relationship if b = 0
Proportional relationship

Proportional and Nonproportional Situations – Page No. 118

The tables represent linear relationships. Determine if each relationship is a proportional or nonproportional situation.

Question 5.
Go Math Grade 8 Answer Key Chapter 4 Nonproportional Relationships Lesson 4: Proportional and Nonproportional Situations img 29
Find the quotient of y and x.
_____________

Answer:
proportional relationship

Explanation:
Find the ratio y/x
12/3 = 4
36/9 = 4
84/21 = 4
Since the ratio is constant, the relationship is proportional.

Question 6.
Go Math Grade 8 Answer Key Chapter 4 Nonproportional Relationships Lesson 4: Proportional and Nonproportional Situations img 30
_____________

Answer:
non-proportional

Explanation:
Find the ratio y/x
4/22 = 2/11
8/46 = 4/23
10/58 = 5/29
Since the ratio is not constant, the relationship is non-proportional.

Question 7.
The values in the table represent the numbers of households that watched three TV shows and the ratings of the shows. The relationship is linear. Describe the relationship in other ways.
Go Math Grade 8 Answer Key Chapter 4 Nonproportional Relationships Lesson 4: Proportional and Nonproportional Situations img 31
Type below:
_____________

Answer:
proportional relationship

Explanation:
Find the ratio y/x
12/15,000,000 = 0.0000008
16/20,000,000 = 0.0000008
20/25,000,000 = 0.0000008
Since the ratio is constant, the relationship is proportional.

Essential Question Check-In

Question 8.
How are using graphs, equations, and tables similar when distinguishing between proportional and nonproportional linear relationships?
Type below:
_____________

Answer:
The ratio between y to x is constant when the relationship is proportional. Graphs, tables, and equations all can be used to find the ratio. The ratio is not constant when the relationship is non-proportional.

Independent Practice – Proportional and Nonproportional Situations – Page No. 119

Question 9.
The graph shows the weight of a cross-country team’s beverage cooler based on how much sports drink it contains.
Go Math Grade 8 Answer Key Chapter 4 Nonproportional Relationships Lesson 4: Proportional and Nonproportional Situations img 32
a. Is the relationship proportional or nonproportional? Explain.
_____________

Answer:
Non-proportional

Explanation:
The graph does not pass through the origin. Graph of a proportional relationship must pass through the origin
Non-proportional

Question 9.
b. Identify and interpret the slope and the y-intercept.
Type below:
_____________

Answer:
Slope m = (y2 -y1)/(x2 – x1) = (12 – 10)/(4 – 0) = 0.5
y-intercept is the weight of the empty cooler, which is 10 lbs.

Explanation:
Find the slope using two points from the grapgh by
Slope m = (y2 -y1)/(x2 – x1) where (x1, y1) = (0, 10) and (x2, y2) = (4, 12)
Slope m = (y2 -y1)/(x2 – x1) = (12 – 10)/(4 – 0) = 0.5
From the graph when x = 0
y-intercept (b) = 10
y-intercept is the weight of the empty cooler, which is 10 lbs.

In 10–11, tell if the relationship between a rider’s height above the first floor and the time since the rider stepped on the elevator or escalator is proportional or nonproportional. Explain your reasoning.

Question 10.
The elevator paused for 10 seconds after you stepped on before beginning to rise at a constant rate of 8 feet per second.
Go Math Grade 8 Answer Key Chapter 4 Nonproportional Relationships Lesson 4: Proportional and Nonproportional Situations img 33
_____________

Answer:
Non-proportional

Explanation:
As there is a pause of 10 seconds, it would be the y-intercept of the graph (when x = 0)
Non-proportional

Question 11.
Your height, h, in feet above the first floor on the escalator is given by h = 0.75t, where t is the time in seconds.
_____________

Answer:
Proportional

Explanation:
Comparing with y = mx + b, where b = 0
Proportional

Analyze Relationships

Compare and contrast the two graphs.

Question 12.
Graph A       Graph B
y = \(\frac{1}{3}\) x        y = \(\sqrt { x } \)
Go Math Grade 8 Answer Key Chapter 4 Nonproportional Relationships Lesson 4: Proportional and Nonproportional Situations img 34
Go Math Grade 8 Answer Key Chapter 4 Nonproportional Relationships Lesson 4: Proportional and Nonproportional Situations img 35
Type below:
_____________

Answer:
Graph A represents a linear relationship while Graph B represents an exponential relationship. They both pass through the origin and the value of y increases with an increase in x.

Proportional and Nonproportional Situations – Page No. 120

Question 13.
Represent Real-World Problems
Describe a real-world situation where the relationship is linear and nonproportional.
Type below:
_____________

Answer:
The entrance fee to the amusement park is $8 and there is a fee of $2 per ride.

H.O.T.

Focus on Higher Order Thinking

Question 14.
Mathematical Reasoning
Suppose you know the slope of a linear relationship and one of the points that its graph passes through. How can you determine if the relationship is proportional or nonproportional?
Type below:
_____________

Answer:
Use the graph and the given point to determine the second point. Connect the two points by a straight line. If the graph passes through the origin, the relationship is proportional and if the graph does not pass through the origin, the relationship is non-proportional.

Question 15.
Multiple Representations
An entrant at a science fair has included information about temperature conversion in various forms, as shown. The variables F, C, and K represent temperatures in degrees Fahrenheit, degrees Celsius, and kelvin, respectively.
Go Math Grade 8 Answer Key Chapter 4 Nonproportional Relationships Lesson 4: Proportional and Nonproportional Situations img 36
a. Is the relationship between kelvins and degrees Celsius proportional? Justify your answer in two different ways.
_____________

Answer:
No, the relationship is not proportional.

Explanation:
Compare the equation B to the form: y = mx + b. Since b is not equal to 0, the relationship is non-proportional.
Find the ratio between the Kelvin and Degrees Celsius. Since the ration is not constant, the relationship is non-proportional.
281.15/8 = 35.14
288.15/15 = 19.21
309.15/36 = 8.59
No, the relationship is not proportional.

Question 15.
b. Is the relationship between degrees Celsius and degrees Fahrenheit proportional? Why or why not?
_____________

Answer:
No, the relationship is not proportional.

Explanation:
Compare the equation A to the form: y = mx + b. Since b is not equal to 0, the relationship is non-proportional.
No, the relationship is not proportional.

4.1 Representing Linear Nonproportional Relationships – Model Quiz – Page No. 121

Question 1.
Complete the table using the equation y = 3x + 2.
Go Math Grade 8 Answer Key Chapter 4 Nonproportional Relationships Model Quiz img 37
Type below:
_____________

Answer:
grade 8 chapter 4 image 11

Explanation:
Given y = 3x + 2
grade 8 chapter 4 image 11
x = -1; y = 3(-1) + 2 = -3 + 2 = -1
x = 0; y = 3(0) +2 = 2
x = 1; y = 3(1) + 2 = 3 + 2 = 5
x = 2; y = 3(2) + 2 = 6 + 2 = 8
x = 3: y = 3(3) + 2 = 9 + 2 = 11

4.2 Determining Slope and y-intercept

Question 2.
Find the slope and y-intercept of the line in the graph.
Go Math Grade 8 Answer Key Chapter 4 Nonproportional Relationships Model Quiz img 38
Type below:
_____________

Answer:
Slope = 3
y-intercept (b) = 1

Explanation:
Find the slope using two points from the grapgh by
Slope m = (y2 -y1)/(x2 – x1) where (x1, y1) = (0, 1) and (x2, y2) = (1, 4)
Slope m = (y2 -y1)/(x2 – x1) = (4 – 1)/(1 – 0) = 3/1
From the graph when x = 0
y-intercept (b) = 1

4.3 Graphing Linear Nonproportional Relationships

Question 3.
Graph the equation y = 2x − 3 using slope and y-intercept.
Go Math Grade 8 Answer Key Chapter 4 Nonproportional Relationships Model Quiz img 39
Type below:
_____________

Answer:
grade 8 chapter 4 image 12

Explanation:
Slope = 2
y-intercept = -3
Plot the point that contains the y-intercept (0, -3)
The slope is m = 2/1. Use the slope to find a second point. From (0, -3) count 2 unit up and 1 unit right. The new point is (1, -1)
Draw a line through the points

4.4 Proportional and Nonproportional Situations

Question 4.
Does the table represent a proportional or a nonproportional linear relationship?
Go Math Grade 8 Answer Key Chapter 4 Nonproportional Relationships Model Quiz img 40
_____________

Answer:
Since the ratio is constant, the table represents a proportional linear relationship.

Explanation:
Find the ratio y/x
4/1 = 4
8/2 = 4
12/3 = 4
16/4 = 4
20/5 = 4
Since the ratio is constant, the table represents a proportional linear relationship.

Question 5.
Does the graph in Exercise 2 represent a proportional or a nonproportional linear relationship?
_____________

Answer:
It represents a non-proportional linear relationship

Explanation:
The line of the graph does not pass through the origin. The graph of a proportional relationship must pass through the origin.
It represents a non-proportional linear relationship

Question 6.
Does the graph in Exercise 3 represent a proportional or a nonproportional relationship?
_____________

Answer:
It represents a non-proportional linear relationship

Explanation:
The line of the graph does not pass through the origin. The graph of a proportional relationship must pass through the origin
It represents a non-proportional linear relationship

Essential Question

Question 7.
How can you identify a linear nonproportional relationship from a table, a graph, and an equation?
Type below:
_____________

Answer:
In a table, the ratio of y/x is not constant for non-proportional relationship.
In a graph, the line of the graph does not pass through the origin for non-proportional relationship.
In an equation, the b is not equal to for y = mx +b for non-proportional relationship.

Selected Response – Mixed Review – Page No. 122

Question 1.
The table below represents which equation?
Go Math Grade 8 Answer Key Chapter 4 Nonproportional Relationships Mixed Review img 41
Options:
a. y = −x − 10
b. y = −6x
c. y = −4x − 6
d. y = −4x + 2

Answer:
c. y = −4x − 6

Explanation:
From the table, you can see that the y-intercept (when x = 0) is b = -6. Comparable to y = mx + b
The table is represented by Option C y = -4x – 6

Question 2.
The graph of which equation is shown below?
Go Math Grade 8 Answer Key Chapter 4 Nonproportional Relationships Mixed Review img 42
Options:
a. y = −2x + 3
b. y = −2x + 1.5
c. y = 2x + 3
d. y = 2x + 1.5

Answer:
a. y = −2x + 3

Explanation:
From the table, you can see that the y-intercept (when x = 0) is b = 3. Comparable to y = mx + b
The Option B and D are rejected.
Since the graph is slanting downwards, the slope is negative.
Option C is rejected
The graph represents y = -2x + 3

Question 3.
The table below represents a linear relationship.
Go Math Grade 8 Answer Key Chapter 4 Nonproportional Relationships Mixed Review img 43
What is the y-intercept?
Options:
a. -4
b. -2
c. 2
d. 3

Answer:
b. -2

Explanation:
Find the rate of change
(7 – 4)/(3 – 2) = (10 – 7)/(4 – 3) = 3
Find the value of y for x = 0
Works backward from x = 2 to x = 1
x = 2 – 1 = 1
y = 4 – 3 = 1
x = 1 – 1 = 0
y = 1 – 3 = -2
y intercept = -2

Question 4.
Which equation represents a nonproportional relationship?
Options:
a. y = 3x + 0
b. y = −3x
c. y = 3x + 5
d. y = \(\frac{1}{3}\)x

Answer:
c. y = 3x + 5

Explanation:
For a non-proportional relationship, the equation is y = mx + b and b is not equal to 0.
Option C represents a non-proportional relationship y = 3x + 5

Question 5.
The table shows a proportional relationship. What is the missing y-value?
Go Math Grade 8 Answer Key Chapter 4 Nonproportional Relationships Mixed Review img 44
Options:
a. 16
b. 20
c. 18
d. 24

Answer:
c. 18

Explanation:
Find the ratio y/x
6/4 = 3/2
Since the relationship is proportional, the ratio is constant.
Using the ratio to fins missing y
3/2 = y/12
y = 3/2 × 12 = 18

Question 6.
What is 0.00000598 written in scientific notation?
Options:
a. 5.98 × 10-6
b. 5.98 × 10-5
c. 59.8 × 10-6
d. 59.8 × 10-7

Answer:
c. 59.8 × 10-6

Explanation:
0.00000598
Move the decimal 6 points
59.8 × 10-6

Mini-Task

Question 7.
The graph shows a linear relationship.
Go Math Grade 8 Answer Key Chapter 4 Nonproportional Relationships Mixed Review img 45
a. Is the relationship proportional or nonproportional?
____________

Answer:
It represents a non-proportional linear relationship

Explanation:
The line of the graph does not pass through the origin. The graph of a proportional relationship must pass through the origin.
It represents a non-proportional linear relationship

Question 7.
b. What is the slope of the line?
_______

Answer:
Slope m = -2

Explanation:
Find the slope using two points from the grapgh by
Slope m = (y2 -y1)/(x2 – x1) where (x1, y1) = (0, -2) and (x2, y2) = (2, 1)
Slope m = (y2 -y1)/(x2 – x1) = (-3 -1)/(0 + 2) = -4/2 = -2

Question 7.
c. What is the y-intercept of the line?
_______

Answer:
y-intercept (b) = -3

Explanation:
From the graph when x = 0
y-intercept (b) = -3

Question 7.
d. What is the equation of the line?
Type below:
____________

Answer:
y = -2x – 3

Explanation:
Substitute m and b in the form: y = mx + b
y = -2x – 3

Conclusion:

I believe that our Go Math Grade 8 Answer Key Chapter 4 Nonproportional Relationships pdf brought a smile to your face. All the beginners can easily start their practice with our step by step explanations. Bookmark our ccssmathanswers.com to get the latest information about the solutions.

Go Math Grade 3 Answer Key Chapter 4 Multiplication Facts and Strategies

go-math-grade-3-chapter-4-multiplication-facts-and-strategies-answer-key

Go Math Grade 3 Answer Key Chapter 4 Multiplication Facts and Strategies helps both Students and Teachers out there to get all the concepts underlying. HMH Go Math Grade 3 Answer Key will familiarize you with different models of Multiplication Facts and Strategies Questions in Exercises.  Understand the logic behind each of the concepts taking the help of HMH Go Math Solution Key Grade 3. Go Math Answer Key Grade 3 Chapter 4 Multiplication Facts and Strategies will give you problem-solving skills rather than giving you simple answers.

Go Math Grade 3 Chapter 4 Multiplication Facts and Strategies Answer Key

Learning the basics of earlier grade chapters is quite necessary to lay a stronger foundation for higher grades. Practice using the Go Math Grade 3 Answer Key Chapter 4 Multiplication Facts and Strategies as much as possible and clear the assessments or homework with utmost confidence. Prepare which ever concept you wish to learn through the direct links available and make your learning effective.

Lesson 1: Multiply with 2 and 4

Lesson 2: Multiply with 5 and 10

Lesson 3: Multiply with 3 and 6

Lesson 4: Algebra • Distributive Property

Lesson 5: Multiply with 7

Mid-Chapter Checkpoint

Lesson 6: Algebra • Associative Property of Multiplication

Lesson 7: Algebra • Patterns on the Multiplication Table

Lesson 8: Multiply with 8

Lesson 9: Multiply with 9

Lesson 10: Problem Solving • Multiplication

Chapter 4 Review/Test

Multiply with 2 and 4 Page No 195

Write a multiplication sentence for the model.

Question 1.
Go Math Grade 3 Answer Key Chapter 4 Multiplication Facts and Strategies Multiply with 2 and 4 img 1
Think: There are 2 groups of
5 counters.

Answer:
2 × 5 = 10

Explanation:
Draw 2 counters. Place 5 Objectives in 2 counters. Count the total number of objectives and them. Add 5 + 5 to get the answer. Finally, you get 10.

Question 2.
Go Math Grade 3 Answer Key Chapter 4 Multiplication Facts and Strategies Multiply with 2 and 4 img 2
______ × ______ = ______

Find the product.

Answer:
4×3 = 12

Explanation:
Look at the above figure. You can see 4 Counters and 3 objectives on each counter. So, you have to multiply 4×3. When you multiply with 4, you can first multiply with 2 and then double the product. Multiply 3 with 2. You will get 2×3 is 6. Now, double the number 6. The final answer is 12.

Question 3.
2
× 6
——–
________

Answer:
2×6 = 12

Explanation:
When there is a multiplication requires with 2, you can double the given number 6 to get the final answer. The answer is 6+6 = 12.

Question 4.
4
× 8
———
________

Answer:
4×8 = 32

Explanation:
You can double 2×8 to get 4×8. Multiply 8 with 2. You will get 16. Then, double the product to get the answer to 4×8. You will get 16+16 = 32. So, you get the final answer as 32.

Question 5.
2
× 3
——–
________

Answer:
2×3 = 6

Explanation:
Draw two counters. Place three objectives in two counters. Now, add the objectives in two counters. The final answer is 3+3 = 6. So, 2×3=6 is the answer.

Question 6.
4
× 6
———
_________

Answer:
4×6 = 24

Explanation:
You can double 2×6 to get 4×6. Multiply 6 with 2. You will get 12. Then, double the product to get the answer to 4×6. You will get 12+12 = 24. So, you get the final answer as 24.

Question 7.
4
× 4
———
_________

Answer:
4×4 = 16

Explanation:
Multiply 4 with 2. 2×4 = 8. Now, double the number 8. 8+8 = 16. The answer for 4×4 is 16

Question 8.
2
× 7
——–
_________

Answer:
2×7 = 14

Explanation:
The given multiplication is 2×7. The answer to any number multiplies with 2 is double of that number. So, the 2×7 can find by doing 7+7. The answer is 7+7 = 14. So, the final answer is 2×7 = 14.

Question 9.
4
× 5
———
_________

Answer:
4×5 = 20

Explanation:
Firstly, multiply 2×5 to get the answer for 4×5. Then, double the answer of 2×5 to get the final answer. So, do 2×5 = 10. Double the number 10 to get the 4×5 Answer. The double of 10 is 10+10 = 20. The answer to 4×5 is 20.

Question 10.
2
× 4
——–
_________

Answer:
2×4 = 8

Explanation:
The multiplication begins with 2. So, you can double 4 to get the answer of 2×4. The double of 4 is 4+4 = 8. The answer for 2×4 is 8.

Problem Solving

Question 11.
On Monday, Steven read 9 pages of his new book. To finish the first chapter on Tuesday, he needs to read double the number of pages he read on Monday. How many pages does he need to read on Tuesday?
__________ pages

Answer:
18

Explanation:
Steven read 9 pages of his new book on Monday. He wants to finish his first chapter on Tuesday. To finish the chapter, he needs to double the number of pages he read on Monday.

Double of pages read on Monday = 9 x 2 =18.

Steven needs to read 18 pages on Tuesday.

Question 12.
Courtney’s school is having a family game night. Each table has 4 players. There are 7 tables in all. How many players are at the game night?
__________ players

Answer:
28

Explanation:
From the given information Courtney’s school is having a family game night. Each table has 4 players. There are 7 tables in all. So, every table has 4 players. To know the total number of players at the game night, we need to multiply 7 with 4.

Total Number of players at the game night = 7 x 4 = 28.

Multiply with 2 and 4 Lesson Check Page No 196

Lesson Check

Question 1.
Which multiplication sentence matches the model?
Go Math Grade 3 Answer Key Chapter 4 Multiplication Facts and Strategies Multiply with 2 and 4 img 3
Options:
a. 3 × 2 = 6
b. 4 × 2 = 8
c. 4 × 4 = 16
d. 4 × 8 = 32

Answer:
b

Explanation:
The above figure consists of 4 counters. Also, each counter has two objectives in it. So, we need to multiply the number of counters with a number of objectives. The answer is 4×2 = 8.

Question 2.
Find the product.
2
× 8
——-
________
Options:
a. 10
b. 14
c. 16
d. 18

Answer:
c

Explanation:
To solve the 2×8, double the number 8. The answer for 2×8 is 8+8. So, the final answer is 2×8 = 16.

Question 3.
Sean made a picture graph to show his friends’ favorite colors. This is the key for the graph.
Each Ο = 2 friends.
How many friends does Ο Ο Ο Ο stand for?
Options:
a. 4
b. 8
c. 20
d. 40

Answer:
b

Explanation:
Sean made 4 picture graphs to show his friends’ favorite colors. Each Ο represents 2 friends. So, to find the final answer, we need to add all the colors. As there are 4 picture graphs you need to do 2+2+2+2 = 8. There are 8 friends available.

Question 4.
The table shows the lengths of some walking trails.
Go Math Grade 3 Answer Key Chapter 4 Multiplication Facts and Strategies Multiply with 2 and 4 img 4
How many feet longer is Mountain Trail than Harmony Trail?
Options:
a. 216 feet
b. 264 feet
c. 316 feet
d. 528 feet

Answer:
c

Explanation:
In the above table, we can see three different walking trails lengths of three persons. The walking trail length of the Mountain is 844feets. The walking trail length of Lake is 792 feet. Also, the Harmony walking trail length is 528 feet. To get How many feet longer is Mountain Trail than Harmony Trail, we need to subtract Harmony walking trail length from Mountain Trail length. So, now we have to do 844-528 which is equal to 316 feet. So, Mountain Trail walked 316 feet than Harmony trail.

Question 5.
Find the sum.
5 2 7
+ 1 5 4
———-
Options:
a. 373
b. 581
c. 671
d. 681

Answer:
c

Explanation:
Break apart the addends from the given values. Start with the hundreds. Then, add each place value.
Let’s write 527 as 500+20+7 and 154 as 100+50+4.
Sum up the above two values. Then you get 600+70+11. Add the ones and then add the 10’s and 100’s. The final answer is 681.

Question 6.
A bar graph shows that sports books received 9 votes. If the scale is 0 to 20 by twos, where should the bar end for the sports books?
Options:
a. between 8 and 10
b. on 10
c. on 8
d. between 6 and 8

Answer:
c

Explanation:
As per the given data, the sports books received 9 votes. If we take a bar graph with a scale of 0 to 20 by twos, the graph is 0, 2, 4, 6, 8, 10, 12, 14, 16, 18, 20. So, the number 9 will lies between numbers 8 to 10. Therefore, the answer is between 8 and 10.

Multiply with 5 and 10 Page No 201

Find the product.

Question 1.
5 × 7 = 35

Answer:
35

Explanation:
Skip count by 5’s until you say 7 numbers. 5, 10, 15, 20, 25, 30, 35. Now, the count of the number is 7. So, the answer for 5 x 7 is 35.

Question 2.
5 × 1 = _______

Answer:
5

Explanation:
Any number multiplied by 1 is the same number. So, the answer is 5.

Question 3.
2 × 10 = _______

Answer:
20

Explanation:
A multiple of 10 is any product that has 10 as one of its factors. So, the multiplication of any number with 10 is 10’s of that particular number. The answer is 20.

Question 4.
________ = 8 × 5

Answer:
40

Explanation:
From Commutative Law of Multiplication, you can write 8 x 5 = 5 x 8. So, to find the multiplication of 5 x 8, Skip count by 5’s until you say 8 numbers. 5, 10, 15, 20, 25, 30, 35, 40. Now, the count of the number is 8. So, the answer for 5 x 8 is 40. Therefore, the answer for 8 x 5 is 40.

Question 5.
1 × 10 = ________

Answer:
10

Explanation:
Multiplication of any number with 1 is the same number. So, 1 x 10 = 10.

Question 6.
_______ = 4 × 5

Answer:
20

Explanation:
Using the Commutative Law of Multiplication, you can write 4 x 5 = 5 x 4. Now, Skip count by 5’s until you say 4 numbers. 5, 10, 15, 20. Therefore, the answer for 5 x 4 is 20.

Question 7.
5 × 10 = _______

Answer:
50

Explanation:
Skip-count by 5’s 10 times. You can write as 5, 10, 15, 20, 25, 30, 35, 40, 45, 50. The answer for 5 x 10 is 50.

Question 8.
7 × 5 = ________

Answer:
35

Explanation:
Write 7 x 5 as 5 x 7 according to the Commutative Law of Multiplication. Now do multiply for 5 x 7. skip-count by 5’s seven times. Now, you can write as 5, 10, 15, 20, 25, 30, 35. The answer for 5 x 7 is 35. So, the answer for 7 x 5 also 35.

Question 9.
_______ = 5 × 5

Answer:
25

Explanation:
Skip-count by 5’s 5 times. You can get 5, 10, 15, 20, 25. The answer for 5 x 5 is 25.

Question 10.
5 × 8 = _________

Answer:
40

Explanation:
You can Skip-count by 5’s 8 times. Write like 5, 10, 15, 20, 25, 30, 35, 40. So, the answer for 5 x 8 is 40.

Question 11.
______ = 5 × 9

Answer:
45

Explanation:
The multiplication of 5 × 9 is calculated as Skip-count by 5’s 9 times. You can write as 5, 10, 15, 20, 25, 30, 35, 40, 45. The final answer for 5 × 9 is 45.

Question 12.
10 × 0 = _________

Answer:
0

Explanation:
Multiplication of any number with 0 is 0. So, the answer for 10 x 0 is 0.

Question 13.
5
× 6
——–
________

Answer:
30

Explanation:
Skip count by 5’s until you say 6 numbers. 5, 10, 15, 20, 25, 30. So, the answer for 5 x 6 is 35.

Question 14.
10
× 7
———
________

Answer:
70

Explanation:
Add 10’s until you say 7 numbers. Now, you can write as 10 + 10 + 10 + 10 + 10 + 10 + 10. Now, add the 10’s to get the final answer. There are seven 10’s available. So, the answer is 70.

Question 15.
5
× 3
——–
________

Answer:
15

Explanation:
Skip count by 5’s until you say 3 numbers. Write like 5, 10, 15. The answer for 5 x 3 is 15.

Question 16.
10
× 4
——–
_________

Answer:
40

Explanation:
Add 10’s until you say 4 numbers. Then, write like 10 + 10 + 10 + 10 to get the answer. The answer for 10 x 4 is 40.

Question 17.
5
× 0
——-
_________

Answer:
0

Explanation:
Multiplication of any number with 0 is 0. So, the answer for 5 x 0 is 0.

Question 18.
10
× 8
——-
__________

Answer:
80

Explanation:
You have to add 10’s until you say 8 numbers. Write as 10 + 10 + 10 + 10 + 10 + 10 + 10 + 10. Now, add the 10’s to get the final answer. There are eight 10’s available. So, the answer for 10 x 8 is 80.

Question 19.
5
× 2
——-
__________

Answer:
10

Explanation:
Skip count by 5’s until you say 2 numbers. Write like 5, 10. The answer for 5 x 2 is 10.

Question 20.
10
× 6
——–
_________

Answer:
60

Explanation:
Add 10’s until you say 6 numbers. Write as 10 + 10 + 10 + 10 + 10 + 10. Now, add the 10’s to get the final answer. So, the answer for 10 x 6 is 60.

Problem Solving

Question 21.
Ginger takes 10 nickels to buy some pencils at the school store. How many cents does Ginger have to spend?
________ cents

Answer:
50

Explanation:
From the given data, Ginger takes 10 nickels to buy some pencils at the school store. One nickel can be treated as 5 cents. So, 10 nickels value is 10 nickels x 5 cents = 50 cents.

Ginger has to spend 50 cents to buy some pencils at the school store.

Question 22.
The gym at Evergreen School has three basketball courts. There are 5 players on each of the courts. How many players are there in all?
__________ players

Answer:
15

Explanation:
As per the given data, the gym at Evergreen School has three basketball courts. There are 5 players on each of the courts. The number of players in there in all = 3 basketball courts x 5 players = 15 players.

15 players are available in total.

Multiply with 5 and 10 Lesson Check Page No 202

Question 1.
Mrs. Hinely grows roses. There are 6 roses on each of her 10 rose bushes. How many roses in all are on Mrs. Hinely’s rose bushes?
Options:
a. 16
b. 54
c. 60
d. 66

Answer:
60

Explanation:
There are 10 rose bushes available. In each rose bush, Mrs. Hinely growing 6 roses. So, to find the number of roses, you need to multiply 10 x 6. The multiplication of 10 x 6 is 60. The number of roses in all are on Mrs. Hinely’s rose bushes are 60.

Question 2.
Find the product.
5
× 8
——-
Options:
a. 8
b. 16
c. 35
d. 40

Answer:
40

Explanation:
Skip-count by 5’s 8 times. Write like 5, 10, 15, 20, 25, 30, 35, 40. So, the answer for 5 x 8 is 40.

Spiral Review

Question 3.
Mr. Miller’s class voted on where to go for a field trip. Use the picture graph to find which choice had the most votes.
Go Math Grade 3 Answer Key Chapter 4 Multiplication Facts and Strategies Multiply with 5 and 10 img 5
Options:
a. Science Center
b. Aquarium
c. Zoo
d. Museum

Answer:
c. Zoo

Explanation:
From the given figure, Mr. Miller’s class voted for a field trip. From the table, every start represents 2 votes. It is mentioned that for the Science Center, Mr. Miller’s given two starts. As one star represents 2 votes, he gave 4 votes for Science Center. Similarly, Aquarium = 3 and a half stars. The half star represents one vote. So, the number of votes given for the Aquarium is 7. For Zoo, he has given 4 stars. So, every start means 2 votes. The total number of votes is 8. Finally, the number of starts for the Museum is 2. So, the total number of votes is 4.

Mr. Miller’s class is given a high number of votes for Zoo. So, Mr. Miller’s class wishes to go for Zoo.

Question 4.
Zack made this table for his survey.
Go Math Grade 3 Answer Key Chapter 4 Multiplication Facts and Strategies Multiply with 5 and 10 img 6
How many students were surveyed in all?
Options:
a. 38
b. 43
c. 47
d. 49

Answer:
c. 47

Explanation:
Zack prepared a table of flavors and votes. So, every vote represents one person. By adding the total number of votes, we can get the total number of students participated in ina survey.
Add 16 + 10 + 9 + 12 = 47. 47 students were surveyed in all to vote for their favorite juice.

Question 5.
Which of the following numbers is even?
25, 28, 31, 37
Options:
a. 25
b. 28
c. 31
d. 37

Answer:
b. 28

Explanation:
Even numbers are divided by 2. Also, even numbers end with a digit of 0, 2, 4, 6, or 8. So, the given number out of the given numbers is 28. It is ended with 8 and also it can divide by 2.

Question 6.
Estimate the sum.
4 7 9
+ 8 9
———
Options:
a. 568
b. 400
c. 500
d. 600

Answer:
a. 568

Explanation:
Add the ones. Regroup the ones as tens and ones. So, 9 + 9 = 18. Remain the 8 same and add the 1 to the tens. Add the tens. Regroup the tens as hundreds and tens. Now, add 7 + 8 + 1 = 16. place the 6 in the tens place and move the 1 to hundreds. Add the hundreds now. 4 + 1 = 5. So, now place the numbers in order. The final answer is 568.

Note:The option is modified for the above question. There is no correct answer available in the given options. So, option a is changed to the correct answer. The answer is explained with the place value to add two addends method.

Multiply with 3 and 6 Page No 207

Find the product.

Question 1.
6 × 4 = 24
Think: You can use doubles.
3 × 4 = 12
12 + 12 = 24

Answer:
24

Explanation:
By using doubles, we can find a 6 x 4 value. First, multiply the factor with half of 6. So, now we can do 3 x 4 = 12. Now, we can double the value of 3 x 4. That is 12 + 12 = 24. So, the answer for 6 x 4 = 24.

Question 2.
3 × 7 = _______

Answer:
21

Explanation:
Skip count by 3’s until you say 7 numbers. 3, 6, 9, 12, 15, 18, 21. So, the answer for 3 x 7 is 21.

Question 3.
________ = 2 × 6

Answer:
12

Explanation:
With the Commutative Law of Multiplication, you can write 2 x 6 as 6 x 2. Multiply the factor 2 with 5 and then add the factor to it to get the final answer. First, multiply the factor with 5. That is 5 x 2 = 10. Now, add the factor with the answer of 5 x 2. 10 +2 = 12. The answer for 6 x 2 is 12. So, the answer for 2 x 6 is 12.

Question 4.
________ = 3 × 5

Answer:
15

Explanation:
You can skip count by 3’s until you say 5 numbers. 3, 6, 9, 12, 15. So, the final answer for 3 x 5 is 15.

Question 5.
1 × 3 = ________

Answer:
3

Explanation:
Multiplication of any number with 1 is the same number. So, 1 x 3 = 3.

Question 6.
________ = 6 × 8

Answer:
48

Explanation:
Use doubles to find the answer of 6 x 8. Firstly, multiply 3 x 8 = 24. Then, double the value of 3 x 8. 24 + 24 = 48. The answer for 6 x 8 is 48.

Question 7.
3 × 9 = ________

Answer:
27

Explanation:
Skip count by 3’s until you say 9 numbers. Write like 3, 6, 9, 12, 15, 18, 21, 24, 27. The answer for 3 x 9 is 27.

Question 8.
_______ = 6 × 6

Answer:
36

Explanation:
Use doubles to find the answer of 6 x 6. Firstly, multiply 3 x 6 = 18. Then, double the value of 3 x 6. 18 + 18 = 36. The answer for 6 x 6 is 36.

Question 9.
4
× 3
——
________

Answer:
12

Explanation:
Write 4 x 3 as 3 x 4 according to the Commutative Law of Multiplication. Now do multiplication for 3 x 4. skip-count by 3’s four times. Now, you can write as 3, 6, 9, 12. The answer for 3 x 4 is 12. So, the answer for 4 x 3 also 12.

Question 10.
6
× 5
——
_________

Answer:
30

Explanation:
Multiply the given factor 5 with 5. Then, add 5 to the answer to get the answer for 6 x 5. Firstly, multiply 5 x 5 = 25. Add 5 to 25. 5 + 25 = 30. So, the answer for 6 x 5 is 30.

Question 11.
2
× 3
——
_________

Answer:
6

Explanation:
Using the Commutative Law of Multiplication, you can write 2 x 3 = 3 x 2. Now, Skip count by 3’s until you say 2 numbers. 3, 6. Therefore, the answer to 3x 2 is 6. The answer for 2 x 3 is 6.

Question 12.
6
× 3
——–
_________

Answer:
18

Explanation:
Double the value of 3 x 3 to get the answer for 6 x 3. The answer for 3 x 3 = 9. Now, double the value of 9. The answer for 6 x 3 is 9 + 9 = 18.

Question 13.
10
× 6
——-
_________

Answer:
60

Explanation:
Using the Commutative Law of Multiplication, you can write as 10 x 6 = 6 x 10. Now, multiply the 10 with 5. The answer is 5 x 10 = 50. Now, add the 10 to 50 to get the answer. The answer for 6 x 10 is 10 + 50 = 60. So, the answer for 10 x 6 is 60.

Question 14.
3
× 6
——-
__________

Answer:
18

Explanation:
Skip-count by 3’s 6 times. You can write as 3, 6, 9, 12, 15, 18. The answer for 3 x 6 is 18.

Question 15.
7
× 6
——-
__________

Answer:
42

Explanation:
With the help of the Commutative Law of Multiplication, you can change 7 x 6 to 6 x 7. Double the value of 3 x 7 to get 6 x 7. Now, do 3 x 7 = 21. Double the 21. 21 + 21 = 42. The answer for 6 x 7 = 42. So, the answer for 7 x 6 is 42.

Question 16.
3
× 0
——-
_________

Answer:
0

Explanation:
Multiplication of any number with 0 is 0. So, the answer for 3 x 0 is 0.

Question 17.
9
× 6
——-
__________

Answer:
54

Explanation:
Write 9 x 6 as 6 x 9 using the Commutative Law of Multiplication. Multiply the 9 with 5. 5 x 9 = 45. Now add the 45 to the 9. So, the answer for 6 x 9 is 45 + 9 = 54. Therefore, the answer for 9 x 6 is 54.

Question 18.
3
× 3
——-
__________

Answer:
9

Explanation:
Skip count by 3’s until you say 3 numbers. 3, 6, 9. So, the answer for 3 x 3 is 9.

Question 19.
10
× 3
——–
___________

Answer:
30

Explanation:
Use Commutative Law of Multiplication to write 10 x 3 to 3 x 10. Now, skip count by 3’s until you say 10 numbers. 3, 6, 9, 12, 15, 18, 21, 24, 27, 30. So, the answer for 3 x 10 is 30. Therefore, the answer for 10 x 3 is 30.

Question 20.
1
× 6
——–
________

Answer:
6

Explanation:
Multiplication of any number with 1 is the same number. So, 1 x 6 = 6.

Problem Solving

Question 21.
James got 3 hits in each of his baseball games. He has played 4 baseball games. How many hits has he had in all?
_________ hits

Answer:
12 hits

Explanation:
As per the given data, James got 3 hits in each of his baseball games. He has played 4 baseball games. To find the number of hits has he had in all, we need to multiply, 3 hits x 4 baseball games. So, the answer is 3 x 4 = 12 hits.

Question 22.
Mrs. Burns is buying muffins. There are 6 muffins in each box. If she buys 5 boxes, how many muffins will she buy?
__________ muffins

Answer:
30 muffins

Explanation:
From the provided information, Mrs. Burns is buying muffins. She is going to buy 5 boxes each has 6 muffins in it. So, to find the total number of muffins, we need to multiply, number of boxes and number of muffins in it. The total number of muffins = 6 x 5 = 30. She is going to buy 30 muffins.

Multiply with 3 and 6 Lesson Check Page No 208

Question 1.
Paco buys a carton of eggs. The carton has 2 rows of eggs. There are 6 eggs in each row. How many eggs are in the carton?
Options:
a. 8
b. 12
c. 14
d. 24

Answer:
b. 12

Explanation:
From the given data, Paco buys a carton of eggs. The carton has 2 rows in it. Every row has 6 eggs. To find the number of eggs, we need to multiply the number of rows and the number of eggs for each row. Therefore, the number of eggs in the carton are 2 x 6 = 12. 12 eggs are available in the carton.

Question 2.
Find the product.
9
× 3
——
Options:
a. 18
b. 24
c. 27
d. 36

Answer:
c. 27

Explanation:
Use Commutative Law of Multiplication to write 9 x 3 to 3 x 9. Now, skip count by 3’s until you say 9 numbers. 3, 6, 9, 12, 15, 18, 21, 24, 27. So, the answer for 3 x 9 is 27. Therefore, the answer for 9 x 3 is 27.

Spiral Review

Question 3.
Find the difference.
5 6 8
– 2 8 3
———
Options:
a. 285
b. 325
c. 385
d. 851

Answer:
a. 285

Explanation:
Subtract the ones. 8 – 3 = 5. Then, Subtract the tens. 6 < 8. So, regroup 5 hundreds 6 tens equal to 4 hundreds 8 tens. 16 – 8 = 8. Subtract the hundreds and add them to check the answer. 4 – 2 = 2. The final answer is 285.

Question 4.
Dwight made double the number of baskets in the second half of the basketball game than in the first half. He made 5 baskets in the first half. How many baskets did he make in the second half?
Options:
a. 7
b. 9
c. 10
d. 20

Answer:
c. 10

Explanation:
Given that Dwight made double the number of baskets in the second half of the basketball game than in the first half. He made 5 baskets in the first half. So, to find the second half baskets, we need to double the baskets in the first half. Baskets in the second half = 5 x 2 = 10.

Question 5.
In Jane’s picture graph, the ???? smile symbol represents two students. One row in the picture graph has 8 symbols. How many students does that represent?
Options:
a. 40
b. 32
c. 24
d. 16

Answer:
d. 16

Explanation:
From Jane’s picture graph, the ???? smile symbol represents 2 students. The one row in the picture graph has 8 symbols. To find the total number of students, we need to multiply the number of smiles and the number of students for each smile. The number of students for a row is 8 x 2 = 16.

Question 6.
What multiplication sentence does this array show?
Go Math Grade 3 Answer Key Chapter 4 Multiplication Facts and Strategies Multiply with 3 and 6 img 7
Options:
a. 5 × 6 = 30
b. 6 × 6 = 36
c. 5 × 5 = 25
d. 1 × 6 = 6

Answer:
a. 5 × 6 = 30

Explanation:
The given array shows that it has 6 boxes for a row and 5 boxes for a column. So, by multiplying the number of rows x number of columns the correct answer is 6 x 5 = 30. By using the Commutative Law of Multiplication, we can write 6 x 5 = 5 x 6. So, the answer from the given options is 5 × 6 = 30.

Distributive Property Page No 213

Write one way to break apart the array.
Then find the product.

Question 1.
Go Math Grade 3 Answer Key Chapter 4 Multiplication Facts and Strategies Distributive Property img 8
(3 × 7) + (3 × 7)
42

Answer:
42

Explanation:
The given array is 6 x 7. Use the Distributive Property to get the final answer. Break the array rows to make two smaller arrays with the facts 3 and 3. Now, you can write the given array as (3 + 3) x 7. Then, multiply each addend with 7. The equation becomes (3 x 7) +(3 x 7). Now simplify and add them to get the final answer. 21 + 21 = 42.

Question 2.
Go Math Grade 3 Answer Key Chapter 4 Multiplication Facts and Strategies Distributive Property img 9
_________

Answer:
32

Explanation:
The given array has 8 columns and 4 rows. Simplify the answer using Distributive Property. Now, break the array rows to make two smaller arrays with the facts 2 and 2. To get the answer, we have to write 4 x 8. By doing smaller arrays, you can write it as (2 + 2) x 8. Multiply each addend with 8. The equation becomes (2 x 8) +(2 x 8). Now simplify and add them to get the final answer. 16 + 16 = 32.

Question 3.
Go Math Grade 3 Answer Key Chapter 4 Multiplication Facts and Strategies Distributive Property img 10
___________

Answer:
54

Explanation:
The given figure has 9 columns and 6 rows. Use Distributive Property to get the answer of a given array. Let’s break the array rows with two smaller arrays with the facts 3 and 3. Now, write the equation as 6 x 9 = (3 + 3) x 9. Multiply each 3 with 9. 27 + 27. The final answer is 54.

Question 4.
Go Math Grade 3 Answer Key Chapter 4 Multiplication Facts and Strategies Distributive Property img 11
_________

Answer:
56

Explanation:
The given array has 8 columns and 7 rows. With the help of Distributive Property, you can solve the given array. Break the array columns with two smaller arrays with the facts 4 and 4. Write the equation as (7 x 4) + (7 x 4) = 28 + 28 = 56.

Problem Solving

Question 5.
There are 2 rows of 8 chairs set up in the library for a puppet show. How many chairs are there in all? Use the Distributive Property to solve.
____________ chairs

Answer:
16

Explanation:
From the given information, there are 2 rows of 8 chairs set up in the library for a puppet show. So, in the array, there are 2 rows and 8 columns. To find the answer, we have to multiply 2 x 8. Using Distributive Property, we can solve 2 x 8. Sum the number 2 with facts 1 and 1. So, we can write as (1 + 1) x 8. Simplify the answer by multiplying each addend to 8. (1 x 8) + (1 x 8) = 8 + 8 = 16. There are 16 chairs available in 2 rows.

Question 6.
A marching band has 4 rows of trumpeters with 10 trumpeters in each row. How many trumpeters are in the marching band? Use the Distributive Property to solve.
___________ trumpeters

Answer:
40

Explanation:
From the given information, a marching band has 4 rows of trumpeters with 10 trumpeters in each row. To know the number of trumpeters, we have to calculate 4 x 10. With the help of Distributive Property, you can separate the one number with its sums. Now write the 4 with its sums 2 and 2. (2 + 2) x 10. Now, you can multiply each addend with 10. (2 x 10) + (2 x 10) = 20 + 20 = 40. 40 trumpeters are in the marching band.

Distributive Property Lesson Check Page No 214

Question 1.
Which number sentence is an example of the Distributive Property?
Options:
a. 7 × 6 = 6 × 7
b. 7 × (2 × 3) = (7 × 2) × 3
c. 7 × 6 = (7 × 3) + (7 × 3)
d. 7 + 6 = 7 + 3 + 3

Answer:
c. 7 × 6 = (7 × 3) + (7 × 3)

Explanation:
Distributive Property: The Distributive Property states that multiplying a sum by a number is the same as multiplying each addend by the number and then adding the products. So, from the given options, c. 7 × 6 = (7 × 3) + (7 × 3) is the correct answer.

Question 2.
What is one way to break apart the array?
Go Math Grade 3 Answer Key Chapter 4 Multiplication Facts and Strategies Distributive Property img 12
Options:
a. (2 × 6) + (2 × 6)
b. (4 × 2) + (4 × 2)
c. (4 × 4) + (4 × 4)
d. (6 × 3) + (6 × 3)

Answer:
a. (2 × 6) + (2 × 6)

Explanation:
The given figure has 6 columns and 4 rows. By using Distributive Property, you can find the answer easily. Let’s break the array rows with two smaller arrays with the facts 2 and 2. Now, write the equation as 4 x 6 = (2 + 2) x 6. Multiply each 2 with 6. the equation can be written as (2 x 6) + (2 x 6).

Spiral Review

Question 3.
The school auditorium has 448 chairs set out for the third grade performance. What is 448 rounded to the nearest ten?
Options:
a. 500
b. 440
c. 450
d. 400

Answer:
c. 450

Explanation:
Given that the school auditorium has 448 chairs set out for the third-grade performance. The 448 number lies between 440 and 450. 448 is closer to 450 than it is to 440. So, 448 rounds to 450.

Question 4.
Find the difference.
4 0 0
– 2 9 6
———-
Options:
a. 104
b. 114
c. 204
d. 296

Answer:
a. 104

Explanation:
Subtract the ones. 0<6. so regroup 0 tens 0 ones = 1 tens 4 ones. Now, do 10 – 6 = 4. Then, Subtract the tens. 9 = 9. So, so regroup. 4 hundred 9 tens = 3 hundred 4 tens. 9 – 9 = 0. Subtract the hundreds and add them to check the answer. 3 – 2 = 1. The final answer is 104.

Question 5.
There are 622 fruit snacks in one crate and 186 in another crate. How many fruit snacks are there in all?
6 2 2
+ 1 8 6
———-
Options:
a. 436
b. 708
c. 768
d. 808

Answer:
d. 808

Explanation:
Break apart the addends from the given values. Start with the hundreds. Then, add each place value. Let’s write 622 as 600 + 20 + 2 and 186 as 100 + 80 + 6.
Sum up the above two values. Then you get 700 + 100 + 8. Add the ones and then add the 10’s and 100’s. The final answer is 808.

Question 6.
Which sport do 6 students play?
Go Math Grade 3 Answer Key Chapter 4 Multiplication Facts and Strategies Distributive Property img 13
a. Football
b. Baseball
c. Basketball
d. Soccer

Answer:
a. Football

Explanation:
From the given picture, it is mentioned that the sports students play and the number of students for that game. As mentioned, Football needs 6 students. Baseball needs 12 students. Basketball needs 10 students. Also, Soccer needs 16 students. So, the answer is Football sport required 6 students to play.

Multiply with 7 Page No 219

Find the product.

Question 1.
6 × 7 = 42

Answer:
42

Explanation:
By using doubles, we can find a 6 x 7 value. First, multiply the factor with half of 6. So, now we can do 3 x 7 = 21. Now, we can double the value of 3 x 7. That is 21 + 21 = 42. So, the answer for 6 x 7 = 42.

Question 2.
________ = 7 × 9

Answer:
63

Explanation:
Use the Distributive Property to find the value of 7 x 9. If we take 7 rows and 9 columns of an array, then we can break the array into two smaller arrays with the facts of 9. Now, write the 7 x 9 as 7 x (6 + 3). Multiply the facts with the 7. Then, you get (7 x 6) + (7 x 3). Therefore, you get the answer as 42 + 21 = 63.

Question 3.
_______ = 1 × 7

Answer:
7

Explanation:
Multiplication of any number with 1 is the same number. So, 1 x 7 = 7.

Question 4.
3 × 7 = _______

Answer:
21

Explanation:
Skip count by 3’s until you say 7 numbers. 3, 6, 9, 12, 15, 18, 21. So, the answer for 3 x 7 is 21.

Question 5.
7 × 7 = ________

Answer:
49

Explanation:
Let us take the array has 7 columns and 7 rows. Simplify the answer using Distributive Property. Now, break the array rows to make two smaller arrays with the facts 4 and 3. To get the answer, we have to write 7 x 7. By doing smaller arrays, you can write it as (4 + 3) x 7. Multiply each addend with 7. The equation becomes (4 x 7) +(3 x 7). Now simplify and add them to get the final answer. 28 + 21 = 49.

Question 6.
_______ = 2 × 7

Answer:
14

Explanation:
Draw two counters. Place seven objectives in two counters. Now, add the objectives in two counters. The final answer is 7 + 7 = 14. So, 2×7 = 14 is the answer.

Question 7.
7 × 8 = _________

Answer:
56

Explanation:
Distributive Property: The Distributive Property states that multiplying a sum by a number is the same as multiplying each addend by the number and then adding the products. So, from the given options, c. 7 × 8 = (7 × 4) + (7 × 4) = 28 + 28 = 56.

Question 8.
______ = 4 × 7

Answer:
28

Explanation:
Firstly, multiply 2×7 to get the answer for 4×7. Then, double the answer of 2×7 to get the final answer. So, do 2×7 = 14. Double the number 14 to get the 4×7 Answer. The double of 14 is 14 +14 = 28. The answer to 4×7 is 28.

Question 9.
7
× 5
——-
__________

Answer:
35

Explanation:
From Commutative Law of Multiplication, you can write 7 x 5 = 5 x 7. So, to find the multiplication of 5 x 7, skip count by 5’s until you say 7 numbers. 5, 10, 15, 20, 25, 30, 35. Now, the count of the number is 7. So, the answer for 5 x 7 is 35. Therefore, the answer for 7 x 5 is 35.

Question 10.
7
× 1
——-
_________

Answer:
7

Explanation:
Using the Commutative Law of Multiplication, you can write 7 x 1 = 1 x 7. Multiplication of any number with 1 is the same number. So, 1 x 7 = 7.

Question 11.
6
× 7
——-
__________

Answer:
42

Explanation:
Use doubles to find the answer of 6 x 7. Firstly, multiply 3 x 7 = 21. Then, double the value of 3 x 7. 21 + 21 = 42. The answer for 6 x 7 is 42.

Question 12.
7
× 4
——-
__________

Answer:
28

Explanation:
Write 7 x 4 as 4 x 7 according to the Commutative Law of Multiplication. Now, do multiplication for 4 x 7. You can double 2×7 to get 4×7. Multiply 7 with 2. You will get 14. Then, double the product to get the answer to 4×7. You will get 14 +14 = 28. So, you get the final answer as 28.

Question 13.
2
× 7
——-
___________

Answer:
14

Explanation:
The given multiplication is 2×7. The answer to any number multiplies with 2 is double of that number. So, the 2×7 can find by doing 7+7. The answer is 7+7 = 14. So, the final answer is 2×7 = 14.

Question 14.
10
× 7
——
____________

Answer:
70

Explanation:
A multiple of 10 is any product that has 10 as one of its factors. So, the multiplication of any number with 10 is 10’s of that particular number. The answer is 70.

Question 15.
3
× 7
——-
____________

Answer:
21

Explanation:
Skip count by 3’s until you say 7 numbers. Write like 3, 6, 9, 12, 15, 18, 21. The answer for 3 x 7 is 21.

Question 16.
7
× 9
——
___________

Answer:
63

Explanation:
Use the Distributive Property to find the value of 7 x 9. If we take 7 rows and 9 columns of an array, then we can break the array into two smaller arrays with the facts of 9. Now, write the 7 x 9 as 7 x (6 + 3). Multiply the facts with the 7. Then, you get (7 x 6) + (7 x 3). Therefore, you get the answer as 42 + 21 = 63.

Question 17.
8
× 7
——-
__________

Answer:
56

Explanation:
Distributive Property: The Distributive Property states that multiplying a sum by a number is the same as multiplying each addend by the number and then adding the products. So, from the given options, c. 8 x 7 = (4 + 4) x 7 = (4 x 7) + (4 x 7) = 28 + 28 = 56 is the correct answer.

Question 18.
7
× 0
——
__________

Answer:
0

Explanation:
Multiplication of any number with 0 is 0. So, the answer for 7 x 0 is 0.

Problem Solving

Question 19.
Julie buys a pair of earrings for $7. Now she would like to buy the same earrings for 2 of her friends. How much will she spend for all 3 pairs of earrings?
$ __________

Answer:
$21

Explanation:
From the given information, Julie buys a pair of earrings for $7. She also wants to buy 2 more pairs of earrings for her 2 friends. So, a total she needs to but 3 pairs of earrings. To know the total amount she is going to spend on earrings, we need to multiply the total number of earrings pair with a cost for each earring. So, we can do 3 x $7 = $21. Julie needs to spend $21 to buy 3 pairs of earrings.

Question 20.
Owen and his family will go camping in 8 weeks. There are 7 days in 1 week. How many days are in 8 weeks?
_________ days

Answer:
56 days

Explanation:
Given that Owen and his family will go camping in 8 weeks. There are 7 days in 1 week. So, to calculate the total number of days in 8 weeks, multiply 8 x 7 = 56. There are 56 days in 8 weeks.

Multiply with 7 Lesson Check Page No 220

Question 1.
Find the product.
7
× 8
——
Options:
a. 54
b. 56
c. 64
d. 66

Answer:
b. 56

Explanation:
Distributive Property: The Distributive Property states that multiplying a sum by a number is the same as multiplying each addend by the number and then adding the products. So, from the given options, c. 7 × 8 = (7 × 4) + (7 × 4) = 28 + 28 = 56.

Question 2.
What product does the array show?
Go Math Grade 3 Answer Key Chapter 4 Multiplication Facts and Strategies Multiply with 7 img 14
Options:
a. 14
b. 17
c. 21
d. 24

Answer:
c. 21

Explanation:
The given array has 7 columns and 3 rows. Simplify the answer using Distributive Property. Now, break the array rows to make two smaller arrays with the facts 1 and 2. To get the answer, we have to write 3 x 7. By doing smaller arrays, you can write it as (1 + 2) x 7. Multiply each addend with 8. The equation becomes (1 x 7) +(2 x 7). Now simplify and add them to get the final answer. 7 + 14 = 21.

Spiral Review

Question 3.
Which statement is true about the numbers below?
6, 12, 18, 24, 30
Options:
a. All of the numbers are odd.
b. Some of the numbers are odd.
c. All of the numbers are even.
d. Some of the numbers are even.

Answer:
c. 21

Explanation:
All the given numbers are divided by 2. So, the answer is All of the numbers are even

Question 4.
How many more people chose retriever than poodle?
Go Math Grade 3 Answer Key Chapter 4 Multiplication Facts and Strategies Multiply with 7 img 15
Options:
a. 31
b. 39
c. 41
d. 49

Answer:
b. 39

Explanation:
To know more people chose retriever than poodle, we have to do subtraction from retriever to poodle. So, 65 – 26 = 39. 39 more people chose retriever than poodle.

Question 5.
What is 94 rounded to the nearest ten?
Options:
a. 90
b. 94
c. 95
d. 100

Answer:
a. 90

Explanation:

94 is between 90 and 100. 94 is closer to 90 than is to 100. 94 rounded to the nearest ten is 90.

Question 6.
Jack has 5 craft sticks. He needs 4 times that number for a project. How many craft sticks does Jack need altogether?
Options:
a. 9
b. 16
c. 20
d. 24

Answer:
c. 20

Explanation:
Jack has 5 craft sticks. He needs 4 times that number for a project. To find the total number of craft sticks does Jack needs altogether are 5 x 4 = 20. 20 craft sticks need to Jack.

Mid-Chapter Checkpoint Page No 221

Vocabulary

Choose the best term from the box to complete the sentence.
Go Math Grade 3 Answer Key Chapter 4 Multiplication Facts and Strategies Mid -Chapter Checkpoint img 16

Question 1.
A __________ of 4 is any product that has 4 as one of its factors.
_________

Answer:
Commutative Property of Multiplication

Question 2.
This is an example of the ______________ .
3 × 8 = (3 × 6) + (3 × 2)
This property states that multiplying a sum by a number is the same as multiplying each addend by the number and then adding the products.
__________

Answer:
Distributive Property of Multiplication

Concepts and Skills

Write one way to break apart the array.
Then find the product.

Question 3.
Go Math Grade 3 Answer Key Chapter 4 Multiplication Facts and Strategies Mid -Chapter Checkpoint img 17
__________

Answer:
7 x 5 = 35

Explanation:

The provided array has 7 columns and 5 rows. Let’s simplify the answer by using Distributive Property. Now, break the array rows to make two smaller arrays with the facts 2 and 3. Now, do 7 x 5. By doing smaller arrays, you can write it as 7 x (2 + 3). Multiply 7 with 2 and 7 with 3. Write (7 x 2) + (7 x 3). Now add them to get the final answer. 14 + 21 = 35.

Question 4.
Go Math Grade 3 Answer Key Chapter 4 Multiplication Facts and Strategies Mid -Chapter Checkpoint img 18
__________

Answer:
9 x 4 = 36

Explanation:
The given array has 9 columns and 4 rows. Now, do 9 x 4. Using Distributive Property break the array rows to make two smaller arrays with the 4 facts 2 and 2. Now, add 9 x (2 + 2). Multiply 9 with 2 and 9 with 2. Write (9 x 2) + (9 x 2). Now add them to get the final answer. 18 + 18 = 36.

Find the product.

Question 5.
3 × 1 = __________

Answer:
3

Explanation:
Using Commutative Law of Multiplication, Write 3 x 1 = 1 x 3. Multiplication of any number with 1 is the same number. So, 1 x 3 = 3.

Question 6.
5 × 6 = _________

Answer:
30

Explanation:
Skip-count by 5’s 6 times. You can write as 5, 10, 15, 20, 25, 30. The answer for 5 x 6 is 30.

Question 7.
________ = 7 × 7

Answer:
49

Explanation:
Use Distributive Property, to get the answer. Given 7 x 7. Write the facts for 7. 7 = 4 + 3. Now, write 7 x 7 = 7 x (4 + 3). Multiply 7 with 4 and 7 with 3. (7 x 4) + (7 x 3) = 28 + 21 = 49.

Question 8.
2 × 10 = _________

Answer:
20

Explanation:
Double the 10 to get the answer for 2 x 10. So, now write 10 + 10 = 20. The answer for 2 x 10 = 20.

Question 9.
2
× 1
——–
_________

Answer:
2

Explanation:
Add 1 + 1 to get 2 x 1. 1 + 1 = 2. The answer for 2 x 1 = 2.

Question 10.
6
× 6
——–
_________

Answer:
36

Explanation:
Use doubles to get of 6 x 6. First, multiply 3 x 6 = 18. Then, double the value of 3 x 6. 18 + 18 = 36. The answer for 6 x 6 is 36.

Question 11.
8
× 7
——-
__________

Answer:
56

Explanation:
Use Distributive Property to find 8 x 7 = (4 + 4) x 7 = (4 x 7) + (4 x 7) = 28 + 28 = 56 is the correct answer.

Question 12.
6
× 0
——
___________

Answer:
0

Explanation:

Multiplication of any number with 0 is 0. So, the answer for 10 x 0 is 0.

Question 13.
3
× 8
——-
__________

Answer:
24

Explanation:
Skip-count by 3’s 8 times. You can write as 3, 6, 9, 12, 15, 18, 21, 24. The answer for 3 x 8 is 24.

Mid-Chapter Checkpoint Page No 222

Question 14.
Lori saw 6 lightning bugs. They each had 6 legs. How many legs did the lightning bugs have in all?
__________ legs

Answer:
36

Explanation:
Given that Lori saw 6 lightning bugs. Each lightning bugs had 6 legs. To find the total number of legs, do 6 x 6 = 36. The lightning bugs have 36 in all.

Question 15.
Zach walked his dog twice a day, for 7 days. Moira walked her dog three times a day for 5 days. Whose dog was walked more times? How many more?
Type below:
_________

Answer:
Moira’s dog walked more than Zach’s dog. It walked one time more than Zach’s dog.

Explanation:
Zach walked his dog twice a day, for 7 days. So, Zach dog walked 7 x 2 = 14 times in total. Moira walked her dog three times a day for 5 days. Moira dog walked 3 x 5 = 15 days. So, Moira’s dog walked more times than Zach dog. It walked one time more than Zach’s dog.

Question 16.
Annette buys 4 boxes of pencils. There are 8 pencils in each box. Jordan buys 3 boxes of pencils with 10 pencils in each box. Who buys more pencils? How many more?
Type below:
__________

Answer:
Annette has more pencils than Jordan. He has 2 pencils more than Jordan.

Explanation:
Annette buys 4 boxes of pencils. Each box has 8 pencils in it. So, Annette has 4 x 8 = 32 pencils. Jordan buys 3 boxes of pencils with 10 pencils in each box. So, he has 3 x 10 = 30 pencils. Annette has 2 pencils more than Jordan.

Question 17.
Shelly can paint 4 pictures in a day. How many pictures can she paint in 7 days?
_________ pictures

Answer:
28

Explanation:
Shelly can paint 4 pictures in a day. In 7 days, she can paint 7 x 4 = 28 pictures.

Associative Property of Multiplication Page No 227

Write another way to group the factors.
Then find the product.

Question 1.
(3 × 2) × 5
3 × (2 × 5)
30

Answer:
30

Explanation:
Using Associative Property of Multiplication, we can write (3 × 2) × 5 = 3 × (2 × 5).
Find (3 × 2) × 5. Multiply 3 x 2 = 6. Then, multiply 6 x 5 = 30.
Find 3 x (2 x 5). Multiply 2 x 5 = 10. Then, multiply 3 x 10 = 30.
So, (3 × 2) × 5 = 3 × (2 × 5). The product value is 30.

Question 2.
(4 × 3) × 2 =
________

Answer:
(4 × 3) × 2 = 4 x (3 x 2)
24

Explanation:
Use Associative Property of Multiplication to write other group the factors. (4 × 3) × 2 = 4 x (3 x 2). Now, multiply 4 x 3 = 12. Then, multiply 12 x 2 = 24. So, (4 × 3) × 2 = 24.

Question 3.
2 × (2 × 8) =
________

Answer:
2 × (2 × 8) = (2 x 2) x 8
32

Explanation:
With Associative Property of Multiplication, 2 × (2 × 8) = (2 x 2) x 8. Now, multiply 2 x 8 = 16. Then, multiply 16 x 2 = 32. So, 2 × (2 × 8) = 32.

Question 4.
9 × (2 × 1) =
________

Answer:
9 × (2 × 1) = (9 × 2) × 1
18

Explanation:
Using Associative Property of Multiplication, we can write 9 × (2 × 1) = (9 × 2) × 1. Now, multiply 2 x 1 = 2. Then, multiply 2 x 9 = 18. So, 9 × (2 × 1) = 18.

Question 5.
2 × (3 × 6) =
________

Answer:
2 × (3 × 6) = (2 x 3) x 6
36

Explanation:
With the help of Associative Property of Multiplication, we can write 2 × (3 × 6) = (2 x 3) x 6. Now, multiply 2 x 3 = 6. Then, multiply 6 x 6 = 36. So, 2 × (3 × 6) = 36.

Question 6.
(4 × 2) × 5 =
________

Answer:
(4 × 2) × 5 = 4 x (2 x 5)
40

Explanation:
Use Associative Property of Multiplication, to write (4 × 2) × 5 = 4 x (2 x 5). Now, multiply 2 x 5 = 10. Then, multiply 4 x 10 = 40. So, (4 × 2) × 5 = 40.

Use parentheses and multiplication properties.
Then, find the product.

Question 7.
9 × 1 × 5
( 9 × 1 ) × 5 = ________

Answer:
( 9 × 1 ) × 5 = 9 x (1 x 5)
45

Explanation:
Use Associative Property of Multiplication. ( 9 × 1 ) × 5 = 9 x (1 x 5). Now, multiply 9 x 1 = 9. Then, multiply 9 x 5 = 45. So, ( 9 × 1 ) × 5 = 45.

Question 8.
3 × 3 × 2
( 3 × 3 ) × 2 = _________

Answer:
( 3 × 3 ) × 2 =
18

Explanation:
Use Associative Property of Multiplication. ( 3 × 3 ) × 2 = 3 x (3 x 2). Now, multiply 3 x 3 = 9. Then, multiply 9 x 2 = 18. So, ( 3 × 3 ) × 2 = 18.

Question 9.
2 × 4 × 3
( 2 × 4 ) × 3 = _________

Answer:
( 2 × 4 ) × 3 = 2 x (4 x 3)
24

Explanation:
Now use Associative Property of Multiplication. ( 2 × 4 ) × 3 = 2 x (4 x 3). Now, multiply 2 x 4 = 8. Then, multiply 8 x 3 = 24. So, ( 2 × 4 ) × 3 = 24.

Question 10.
5 × 2 × 3
( 5 × 2 ) × 3 = _________

Answer:
( 5 × 2 ) × 3 = 5 x (2 x 3)
30

Explanation:
Use Associative Property of Multiplication. ( 5 × 2 ) × 3 = 5 x (2 x 3). Now, multiply 5 x 2 = 10. Then, multiply 10 x 3 = 30. So, ( 5 × 2 ) × 3 = 30.

Question 11.
7 × 1 × 5
( 7 × 1 ) × 5 = ________

Answer:
( 7 × 1 ) × 5 = 7 x (1 x 5)
35

Explanation:
Use Associative Property of Multiplication. ( 7 × 1 ) × 5 = 7 x (1 x 5). Now, multiply 7 x 1 = 7. Then, multiply 7 x 5 = 35. So, ( 7 × 1 ) × 5 = 35.

Question 12.
8 × 2 × 3
( 8 × 2 ) × 3 = _________

Answer:
( 8 × 2 ) × 3 = 8 x (2 x 3)
48

Explanation:
Use Associative Property of Multiplication. ( 8 × 2 ) × 3 = 8 x (2 x 3). Now, multiply 8 x 2 = 16. Then, multiply 16 x 3 = 48. So, ( 8 × 2 ) × 3 = 48.

Question 13.
7 × 2 × 3
( 7 × 2 ) × 3 = ________

Answer:
( 7 × 2 ) × 3 = 7 x (2 x 3)
42

Explanation:
Use Associative Property of Multiplication. ( 7 × 2 ) × 3 = 7 x (2 x 3). Now, multiply 2 x 3 = 6. Then, multiply 7 x 6 = 42. So, ( 7 × 2 ) × 3 = 42.

Question 14.
4 × 1 × 3
( 4 × 1 ) × 3 = ________

Answer:
12

Explanation:
Use Associative Property of Multiplication. ( 4 × 1 ) × 3 = 4 x (1 x 3). Now, multiply 4 x 1 = 4. Then, multiply 4 x 3 = 12. So, ( 4 × 1 ) × 3 = 12.

Question 15.
10 × 2 × 4
( 10 × 2 ) × 4 = ________

Answer:
80

Explanation:
Use Associative Property of Multiplication. ( 10 × 2 ) × 4 = 10 x (2 x 4). Now, multiply 2 x 4 = 8. Then, multiply 10 x 8 = 80. So, ( 10 × 2 ) × 4 = 80.

Problem Solving

Question 16.
Beth and Maria are going to the county fair. Admission costs $4 per person for each day. They plan to go for 3 days. How much will the girls pay in all?
$ _________

Answer:
$24

Explanation:
From the given information, County fair admission costs $4 per person for each day. To go for 3 days, it costs, 3 x $4 = $12. Beth and Maria are going to the county fair. So, 2 members are going to the county fair. The total amount is 2 X $12 = $24.

Question 17.
Randy’s garden has 3 rows of carrots with 3 plants in each row. Next year he plans to plant 4 times the number of rows of 3 plants. How many plants will he have next year?
_________ plants

Answer:
36

Explanation:
From the given information, Randy’s garden has 3 rows of carrots with 3 plants in each row. So, 3 x 3 = 9. To plant 4 times the number of rows of 3 plants, 9 x 4 = 36. 36 plants will have to plant by Randy’s garden next year.

Associative Property of Multiplication Lesson Check Page No 228

Question 1.
There are 2 benches in each car of a train ride. Two people ride on each bench. If a train has 5 cars, how many people in all can be on a train?
Options:
a. 4
b. 9
c. 10
d. 20

Answer:
d. 20

Explanation:
Given that there are 2 benches in each car of a train ride. If a train has 5 cars in total, the number of benches = 5 x 2 = 10. From the given information, two people ride on each bench. So, 10 x 2 = 20 people will travel on a train if the train has 5 cars.

Question 2.
Crystal has 2 CDs in each box. She has 3 boxes on each of her 6 shelves. How many CDs does Crystal have in all?
Options:
a. 6
b. 12
c. 18
d. 36

Answer:
d. 36

Explanation:
The crystal has 3 boxes on each of her 6 shelves. So, she has 3 x 6 = 18 boxes with her. Each box has 2 CDs in it. So, 2 x 18 = 36 CDs available at crystal.

Spiral Review

Question 3.
Find the sum.
4 7 2
+ 1 8 6
——–
Options:
a. 658
b. 648
c. 558
d. 286

Answer:
30

Explanation:

Add the ones. Add 2 + 6 =8. Then, Add the tens. Regroup hundreds and tens. Add 7 + 8 = 15. Remember to carry over. Add the hundreds. Add 4 + 1 + 1 = 6. Place the one’s, ten’s and hundreds to get the sum. The sum is 658.

Question 4.
Trevor made a picture graph to show how many minutes each student biked last week. This is his key.
Each Go Math Grade 3 Answer Key Chapter 4 Multiplication Facts and Strategies Associative Property of Multiplication img 19 = 10 minutes.
What Go Math Grade 3 Answer Key Chapter 4 Multiplication Facts and Strategies Associative Property of Multiplication img 20 does stand for?
Options:
a. 2 minutes
b. 10 minutes
c. 20 minutes
d. 25 minutes

Answer:
d. 25 minutes

Explanation:
Given Go Math Grade 3 Answer Key Chapter 4 Multiplication Facts and Strategies Associative Property of Multiplication img 19 = 10 minutes. So, half time symbol = 5 minutes.  Therefore, Go Math Grade 3 Answer Key Chapter 4 Multiplication Facts and Strategies Associative Property of Multiplication img 20 = 10 + 10 + 5 = 25 minutes.

Question 5.
Madison has 142 stickers in her collection. What is 142 rounded to the nearest ten?
Options:
a. 40
b. 140
c. 150
d. 200

Answer:
b. 140

Explanation:
The 142 lies between 140 and 150. But the 142 is closer to 140 than it is to 150. So, 142 is rounded to the 140.

Question 6.
There are 5 pages of photos. Each page has 6 photos. How many photos are there in all?
Options:
a. 12
b. 20
c. 24
d. 30

Answer:
d. 30

Explanation:
Given that there are 5 pages of photos. And, each page has 6 photos. Total photos = 5 x 6 = 30.

Patterns on the Multiplication Table Page No 233

Is the product even or odd? Write even or odd.

Question 1.
2 × 7 = even
Think: Products with 2 as a factor are even.

Answer:
even

Explanation:
Products with 2 as a factor are even.

Question 2.
4 × 6 = ________

Answer:
even

Explanation:
The numbers end with 0, 2, 4, 6, 8 are even numbers. So, 24 is even number. The 4 × 6 an even number.

Question 3.
8 × 3 = ________

Answer:
even

Explanation:
8 is an even number. 3 is an odd number. The product of an odd number and an even number is even. The answer is even.

Question 4.
2 × 3 = _________

Answer:
even

Explanation:
Products with 2 as a factor are even.

Question 5.
9 × 9 = ________

Answer:
odd

Explanation:
9 is an odd number. The product of two odd numbers is an odd number. The answer is odd.

Question 6.
5 × 7 = _________

Answer:
odd

Explanation:
The numbers end with 1,3, 5, 7, 9 are odd numbers. So, 35 is an odd number. The 5 × 7 an odd number.

Question 7.
6 × 3 = ________

Answer:
even

Explanation:
6 is an even number. 3 is an odd number. The product of an odd number and an even number is even. The answer is even.

Use the multiplication table. Describe a pattern you see.

Go Math Grade 3 Answer Key Chapter 4 Multiplication Facts and Strategies Patterns on the Multiplication Table img 21

Question 8.
in the column for 5
_________

Answer:
The one’s digits repeat 0 and 5. Each number is 5 more than the number above it.

Explanation:
Each number is added with 5. One’s digit is repeated with 0 and 5.

Question 9.
in the row for 10
________

Answer:
Add 10. All the products are even. The one’s digit is always 0.

Explanation:
All the products are even. Also, the one’s digit is always 0. Add 10 for every product to get the next number to it.

Question 10.
in the rows for 3 and 6
Type below:
_________

Answer:
The products of 6 are the products of 3 doubled.

Explanation:
Add 3 for row 3. The products of 6 are the products of 3 doubled.

Question 11.
Carl shades a row in the multiplication table. The products in the row are all even. The ones digits in the products repeat 0, 4, 8, 2, 6. What row does Carl shade?
Carl shaded row for _________

Answer:
The row for 4.

Explanation:
From the given data, the answer is row 4.

Question 12.
Jenna says that no row or column contains products with only odd numbers. Do you agree? Explain.
_________

Answer:
Yes, Either the products are all even, or there is an even and odd number pattern.

Explanation:
Jenna said correctly. Either the products are all even, or there is an even and odd number pattern.

Patterns on the Multiplication Table Lesson Check Page No 234

Question 1.
Which has an even product?
Options:
a. 1 × 9
b. 3 × 3
c. 5 × 7
d. 4 × 9

Answer:
d. 4 × 9

Explanation:
The product of an odd number and an even number is even. The answer is 4 × 9.

Question 2.
Which describes this pattern?
10, 15, 20, 25, 30
Options:
a. Even and then odd
b. Add 10.
c. Subtract 5.
d. Multiply by 5.

Answer:
a. Even and then odd

Explanation:
The given pattern is the combination of even and add.

Spiral Review

Question 3.
Lexi has 2 cans of tennis balls. There are 3 tennis balls in each can. She buys 2 more cans. How many tennis balls does she now have in all?
Options:
a. 12
b. 9
c. 7
d. 6

Answer:
a. 12

Explanation:
Lexi has 2 cans of tennis balls. There are 3 tennis balls in each can. So, she has 2 x 3 = 6 balls. She buys 2 more cans. So, again 2 cans with 3 tennis balls = 2 x 3 =6. Total = 6 + 6 = 12. 12 tennis balls she is going to have with her.

Question 4.
Use the picture graph.
Go Math Grade 3 Answer Key Chapter 4 Multiplication Facts and Strategies Patterns on the Multiplication Table img 22
How many students have green eyes?
Options:
a. 4
b. 8
c. 12
d. 16

Answer:
d. 16

Explanation:
From the given picture, Green eyes have 4 circles. Each circle =4. 4 x 4 = 16. 16 students have green eyes.

Question 5.
Sasha bought 3 boxes of pencils. If each box has 6 pencils, how many pencils did Sasha buy in all?
Options:
a. 9
b. 12
c. 18
d. 24

Answer:
b. 12

Explanation:
Sasha bought 3 boxes of pencils. If each box has 6 pencils, 3 x 6 = 12 pencils Sasha can buy in all.

Question 6.
Find the sum.
2 1 9
+ 7 6 3
———–
Options:
a. 992
b. 982
c. 976
d. 972

Answer:
b. 982

Explanation:
Break apart the addends from the given values. Start with the hundreds. Then, add each place value.
Let’s write 219 as 200+10+9 and 763 as 700+60+3.
Sum up the above two values. Then you get 900+70+12. Add the ones and then add the 10’s and 100’s. The final answer is 982.

Multiply with 8 Page No 239

Find the product.

Question 1.
8 × 10 = 80

Answer:
80

Explanation:
8 × 10 = (2 x 4) x 10
Use the Associative Property.
8 × 10 = 2 x (4 x 10)
Multiply. 4 × 10
8 × 10 = 2 x 40
Double the product.
8 × 10 = 40 + 40
8 × 10 = 80

Question 2.
8 × 8 = ________

Answer:
64

Explanation:
Factor 8 is an even number. 4+ 4
8 x 4 = 32.
32 doubled is 64.
8 x 8 = 64.

Question 3.
8 × 5 = ________

Answer:
40

Explanation:
8 × 5 = (2 x 4) x 5
Use the Associative Property.
8 × 5 = 2 x (4 x 5)
Multiply. 4 × 5
8 × 5 = 2 x 20
Double the product.
8 × 5 = 20 + 20
8 × 5 = 40

Question 4.
3 × 8 = ________

Answer:
24

Explanation:
Factor 8 is an even number. 4+ 4
3 x 4 = 12.
12 doubled is 24.
3 x 8 = 24.

Question 5.
_______ = 4 × 8

Answer:
32

Explanation:
Factor 8 is an even number. 4+ 4
4 x 4 = 16.
16 doubled is 32.
4 x 8 = 32.

Question 6.
8 × 7 = ________

Answer:
56

Explanation:
8 × 7 = (2 x 4) x 7
Use the Associative Property.
8 × 7 = 2 x (4 x 7)
Multiply. 4 × 7
8 × 7 = 2 x 28
Double the product.
8 × 7 = 28 + 28
8 × 7 = 56.

Question 7.
6 × 8 = ________

Answer:
48

Explanation:
Factor 8 is an even number. 4+ 4
6 x 4 = 24.
24 doubled is 48.
6 x 8 = 48.

Question 8.
_______ = 9 × 8

Answer:
72

Explanation:
Factor 8 is an even number. 4+ 4
9 x 4 = 36.
36 doubled is 72.
9 x 8 = 72.

Question 9.
8
× 2
——
________

Answer:
16

Explanation:
Using the Commutative Law of Multiplication, 2 x 8 = 8 x 2.
Factor 8 is an even number. 4+ 4
2 x 4 = 8.
8 doubled is 16.
2 x 8 = 16.
So, 8 x 2 =16.

Question 10.
6
× 8
——-
________

Answer:
48

Explanation:
Factor 8 is an even number. 4+ 4
6 x 4 = 24.
24 doubled is 48.
6 x 8 = 48.

Question 11.
8
× 7
——
_________

Answer:
56

Explanation:
Using the Commutative Law of Multiplication, 8 x 7 = 7 x 8.
Factor 8 is an even number. 4+ 4
7 x 4 = 28.
28 doubled is 56.
7 x 8 = 56.

Question 12.
0
× 8
——-
____________

Answer:
0

Explanation:

Multiplication of any number with 0 is 0. So, the answer for 0 x 8 is 0.

Question 13.
8
× 5
——
___________

Answer:
40

Explanation:
8 × 5 = (2 x 4) x 5
Use the Associative Property.
8 × 5 = 2 x (4 x 5)
Multiply. 4 × 5
8 × 5 = 2 x 20
Double the product.
8 × 5 = 20 + 20
8 × 5 = 40.

Question 14.
8
× 8
——–
___________

Answer:
64

Explanation:
Factor 8 is an even number. 4+ 4
8 x 4 = 32.
32 doubled is 64.
8 x 8 = 64.

Question 15.
9
× 8
——-
___________

Answer:
72

Explanation:
Factor 8 is an even number. 4+ 4
9 x 4 = 36.
36 doubled is 72.
9 x 8 = 72.

Question 16.
8
× 3
——
___________

Answer:
24

Explanation:
8 × 3 = (2 x 4) x 3
Use the Associative Property.
8 × 3 = 2 x (4 x 3)
Multiply. 4 × 3
8 × 3 = 2 x 12
Double the product.
8 × 3 = 12 + 12
8 × 3 = 24.

Question 17.
8
× 1
——
___________

Answer:
8

Explanation:

Multiplication of any number with 1 is the same number. So, 1 x 8 = 8.

Question 18.
4
× 8
——
___________

Answer:
32

Explanation:
Factor 8 is an even number. 4+ 4
4 x 4 = 16.
16 doubled is 32.
4 x 8 = 32.

Problem Solving

Question 19.
There are 6 teams in the basketball league. Each team has 8 players. How many players are there in all?
____________ players

Answer:
48

Explanation:
Multiply 6 x 8 to get the total number of players. 6 x 8 = 48 players are in the basketball league.

Question 20.
Lynn has 4 stacks of quarters. There are 8 quarters in each stack. How many quarters does Lynn have in all?
___________ quarters

Answer:
32

Explanation:
Multiply 4 x 8 to find quarters in stacks. Lynn has 4 x 8 = 32 quarters in total.

Question 21.
Tomas is packing 7 baskets for a fair. He is placing 8 apples in each basket. How many apples are there in all?
___________ apples

Answer:
56

Explanation:
Multiply 7 x 8 to get the total number of apples. Tomas have 7 x 8 = 56 apples.

Question 22.
There are 10 pencils in each box. If Jenna buys 8 boxes, how many pencils will she buy?
_____________ pencils

Answer:
80

Explanation:
To get total pencils, if Jenna buys 8 boxes, multiply 10 x 8. So, Jenna buys 10 x 8 = 80 pencils.

Multiply with 8 Lesson Check Page No 240

Question 1.
Find the product.
5 × 8 = ■
Options:
a. 30
b. 32
c. 42
d. 40

Answer:
d. 40

Explanation:
Using Commutative Property of Multiplication, write 5 x 8 = 8 x 5
8 × 5 = (2 x 4) x 5
Use the Associative Property.
8 × 5 = 2 x (4 x 5)
Multiply. 4 × 5
8 × 5 = 2 x 20
Double the product.
8 × 5 = 20 + 20
8 × 5 = 40

Question 2.
There are 7 tarantulas in the spider exhibit at the zoo. Each tarantula has 8 legs. How many legs do the 7 tarantulas have in all?
Options:
a. 15
b. 49
c. 56
d. 63

Answer:
c. 56

Explanation:
Multiply 7 x 8 to find the 7 tarantulas legs. 7 tarantulas have 7 x 8 = 56 legs.

Spiral Review

Question 3.
Find the difference.
6 5 2
– 9 9
———
Options:
a. 99
b. 552
c. 553
d. 653

Answer:
c. 553

Explanation:
Subtract the ones. 2<9. so regroup 5 tens 2 ones = 4 tens 3 ones. Now, do 12 – 9 = 3. Then, Subtract the tens. 4 < 9. So, regroup 6 hundred 5 tens = 5 hundred 5 tens. 14 – 9 = 5. Subtract the hundreds and add them to check the answer. 5 – 0 = 5. The final answer is 553.

Question 4.
The school library received an order of 232 new books. What is 232 rounded to the nearest ten?
Options:
a. 200
b. 230
c. 240
d. 300

Answer:
b. 230

Explanation:
The 232 lies between 230 and 240. But the 232 is closer to 230 than it is to 240. So, 232 is rounded to the 230.

Question 5.
Sam’s picture graph shows that 8 students chose pizza as their favorite lunch. This is the key for the graph.
Each ☺ smile = 2 students.
How many ☺ smile should be next to pizza on Sam’s graph?
Options:
a. 2
b. 4
c. 6
d. 8

Answer:
b. 4

Explanation:
Given 1 smile = 2 students.
2 smiles = 4 students
3 smiles = 6 students
4 smiles = 8 students
4 smiles required for Sam for her next pizza on the graph.

Question 6.
Tashia buys 5 packages of oranges. Each package has 4 oranges. How many oranges in all does Tashia buy?
Options:
a. 1
b. 9
c. 20
d. 25

Answer:
d. 25

Explanation:
To know the total number of oranges, multiply 5 x 5. Tashia buys 5 x 5 = 25 oranges.

Multiply with 9 Page No 245

Find the product.

Question 1.
10 × 9 = 90

Answer:
90

Explanation:
The tens digit is 1 less than the factor that is multiplied by 9. So, ten’s digit number is 10 – 1 = 9. The sum of the digits in the product is always 9. So, to multiply 10 × 9, think the tens digit is 9 and the one’s digit is 0. The product is 90.

Question 2.
2 × 9 = ________

Answer:
18

Explanation:
9 = 3 + 6
2 × 9 = 2 x (3 + 6)
Multiply each addend by 2.
2 × 9 = (2 × 3) + (2 × 6)
Add the products.
2 × 9 = 6 + 12
2 × 9 = 18.

Question 3.
9 × 4 = ________

Answer:
36

Explanation:
9 = 3 + 6
9 × 4 = (3 + 6) x 4
Multiply each addend by 4.
9 × 4 = (3 x 4) + (6 x 4)
Add the products.
9 × 4 = 12 + 24
9 × 4 = 36.

Question 4.
0 × 9 = ________

Answer:
0

Explanation:

Multiplication of any number with 0 is 0. So, the answer for 0 x 9 is 0.

Question 5.
1 × 9 = ________

Answer:
9

Explanation:

Multiplication of any number with 1 is the same number. So, 1 x 9 = 9.

Question 6.
8 × 9 = ________

Answer:
72

Explanation:
9 = 3 + 6
8 × 9 = 8 x (3 + 6)
Multiply each addend by 8.
8 × 9 = (8 × 3) + (8 × 6)
Add the products.
8 × 9 = 24 + 48
8 × 9 = 72.

Question 7.
9 × 5 = ________

Answer:
45

Explanation:
9 = 3 + 6
9 × 5 = (3 + 6) x 5
Multiply each addend by 5.
9 × 5 = (3 x 5) + (6 x 5)
Add the products.
9 × 5 = 15 + 30
9 × 5 = 45.

Question 8.
6 × 9 = ________

Answer:
54

Explanation:
9 = 3 + 6
6 × 9 = 6 x (3 + 6)
Multiply each addend by 6.
6 × 9 = (6 × 3) + (6 × 6)
Add the products.
6 × 9 = 18 + 36
6 × 9 = 54.

Question 9.
9
× 4
——–
________

Answer:
36

Explanation:
9 = 3 + 6
9 × 4 = (3 + 6) x 4
Multiply each addend by 4.
9 × 4 = (3 x 4) + (6 x 4)
Add the products.
9 × 4 = 12 + 24
9 × 4 = 36.

Question 10.
5
× 9
——–
________

Answer:
45

Explanation:
9 = 3 + 6
5 × 9 = 5 x (3 + 6)
Multiply each addend by 5.
5 × 9 = (5 × 3) + (5 × 6)
Add the products.
5 × 9 = 15 + 30
5 × 9 = 45.

Question 11.
9
× 7
——–
________

Answer:
63

Explanation:
9 = 3 + 6
9 × 7 = (3 + 6) x 7
Multiply each addend by 7.
9 × 7 = (3 x 7) + (6 x 7)
Add the products.
9 × 7 = 21 + 42
9 × 7 = 63.

Question 12.
2
× 9
——–
________

Answer:
18

Explanation:
9 = 3 + 6
2 × 9 = 2 x (3 + 6)
Multiply each addend by 2.
2 × 9 = (2 × 3) + (2 × 6)
Add the products.
2 × 9 = 6 + 12
2 × 9 = 18.

Question 13.
9
× 9
——–
________

Answer:
81

Explanation:
The tens digit is 1 less than the factor that is multiplied by 9. So, ten’s digit number is 9 – 1 = 8. The sum of the digits in the product is always 9. So, add 1 to 8 to get one’s digit 1 + 8 = 9. The product is 81.

Question 14.
10
× 9
——–
________

Answer:
90

Explanation:
The tens digit is 1 less than the factor that is multiplied by 9. So, ten’s digit number is 10 – 1 = 9. The sum of the digits in the product is always 9. So, to multiply 10 × 9, think the tens digit is 9 and the one’s digit is 0. The product is 90.

Question 15.
3
× 9
——–
________

Answer:
27

Explanation:
9 = 3 + 6
3 × 9 = 3 x (3 + 6)
Multiply each addend by 3.
3 × 9 = (3 × 3) + (3 × 6)
Add the products.
3 × 9 = 9 + 18
3 × 9 = 27.

Question 16.
9
× 8
——–
________

Answer:
72

Explanation:
9 = 3 + 6
8 × 9 = 8 x (3 + 6)
Multiply each addend by 8.
8 × 9 = (8 × 3) + (8 × 6)
Add the products.
8 × 9 = 24 + 48
8 × 9 = 72.

Question 17.
6
× 9
——–
________

Answer:
54

Explanation:
9 = 3 + 6
6 × 9 = 6 x (3 + 6)
Multiply each addend by 6.
6 × 9 = (6 × 3) + (6 × 6)
Add the products.
6 × 9 = 18 + 36
6 × 9 = 54.

Question 18.
9
× 1
——–
________

Answer:
9

Explanation:
Multiplication of any number with 1 is the same number. So, 1 x 9 = 9.

Problem Solving

Question 19.
There are 9 positions on the softball team. Three people are trying out for each position. How many people in all are trying out?
___________ people

Answer:
27

Explanation:
To find the total number of people trying for a position on the softball team, multiply 9 x 3. 9 x 3 = 27 people are trying for a softball team position.

Question 20.
Carlos bought a book for $9. Now he would like to buy 4 other books for the same price. How much will he have to pay in all for the other 4 books?
$ _____________

Answer:
$36

Explanation:
Carlos bought a book for $9. Now he would like to buy 4 other books for the same price. So, to get them all 4 books, he needs to pay 4 x $9 = $36.

Multiply with 9 Lesson Check Page No 246

Question 1.
Find the product.
7 × 9 = ■
Options:
a. 63
b. 56
c. 45
d. 36

Answer:
a. 63

Explanation:
9 = 3 + 6
9 × 7 = (3 + 6) x 7
Multiply each addend by 7.
9 × 7 = (3 x 7) + (6 x 7)
Add the products.
9 × 7 = 21 + 42
9 × 7 = 63.

Question 2.
Clare buys 5 tickets for the high school musical. Each ticket costs $9. How much do the tickets cost in all?
Options:
a. $36
b. $40
c. $45
d. $52

Answer:
c. $45

Explanation:
Multiply 5 with $9 to get the total number of tickets. Clare buys 5 tickets for 5 x $9 = $45.

Spiral Review

Question 3.
The table shows the hair color of girls in Kim’s class. How many girls have brown hair?
Go Math Grade 3 Answer Key Chapter 4 Multiplication Facts and Strategies Multiply with 9 img 23
Options:
a. 1
b. 3
c. 4
d. 6

Answer:
d. 6

Explanation:
From the given figure, there are 6 bars available in the Number of Girls for Brown. So, the answer is 6.

Question 4.
Miles picked up 9 shirts from the dry cleaners. It costs $4 to clean each shirt. How much did Miles spend to have all the shirts cleaned?
Options:
a. $13
b. $22
c. $36
d. $45

Answer:
c. $36

Explanation:
Miles spend 9 x $4 = $36 to have all the shirts cleaned.

Question 5.
In a picture graph, each picture of a baseball is equal to 5 games won by a team. The row for the Falcons has 7 baseballs. How many games have the Falcons won?
Options:
a. 40
b. 35
c. 12
d. 7

Answer:
a. 63

Explanation:
From given data, 1 baseball = 5 games. The row for the Falcons has 7 baseballs. So, Falcons won 7 x 5 = 35 games.

Question 6.
An array has 8 rows with 4 circles in each row. How many circles are in the array?
Options:
a. 12
b. 24
c. 32
d. 36

Answer:
a. 63

Explanation:
8 x 4 = 32 circles are in the array.

Multiplication Page No 251

Solve.

Question 1.
Henry has a new album for his baseball cards. He uses pages that hold 6 cards and pages that hold 3 cards. If Henry has 36 cards, how many different ways can he put them in his album?
Henry can put the cards in his album 5 ways.
Go Math Grade 3 Answer Key Chapter 4 Multiplication Facts and Strategies Multiplication img 24

Answer:
5 ways

Explanation:
Henry can put the cards in 5 ways. They are
1. (1 x 6 cards pages) x (10 x 3 cards pages) = 6 cards + 30 cards = 36 cards.
2. (2 x 6 cards pages) x (8 x 3 cards pages) = 12 cards + 24 cards = 36 cards.
3. (3 x 6 cards pages) x (6 x 3 cards pages) = 18 cards + 18 cards = 36 cards.
4. (4 x 6 cards pages) x (4 x 3 cards pages) = 24 cards + 12 cards = 36 cards.
5. (5 x 6 cards pages) x (2 x 3 cards pages) = 30 cards + 6 cards = 36 cards.

Question 2.
Ms. Hernandez has 17 tomato plants that she wants to plant in rows. She will put 2 plants in some rows and 1 plant in the others. How many different ways can she plant the tomato plants? Make a table to solve.

Rows With 2 Plants ___8__ ___7___ ____6__ ___5___ __4____ ____3__ _____2_ _____1_
Rows With 1 Plants ____1__ ___3___ ___5___ ___7___ ___9___ ____11__ ___13___ ____15__
Total Plants ____17__ ___17___ __17____ __17____ ___17___ ___17___ ____17__ ____17__
Ms. Hernandez can plant the tomato plants ___8___ Ways.

Answer:
8 ways

Explanation:
Ms. Hernandez wants to grow 2 tomoto plants in one row and 1 in other row. The total number of plants should be 17. So,
1. (8 rows x 2 plants) x (1 row x 1plant) = 16 plants + 1 plant = 17 plants.
2. (7 rows x 2 plants) x (3 rows x 1plant) = 14 plants + 3 plant = 17 plantst.
3. (6 rows x 2 plants) x (5 rows x 1plant) = 12 plants + 5 plant = 17 plants.
4. (5 rows x 2 plants) x (7 rows x 1plant) = 10 plants + 7 plant = 17 plants.
5. (4 rows x 2 plants) x (9 rows x 1plant) = 8 plants + 9 plant = 17 plants.
6. (3 rows x 2 plants) x (11 rows x 1plant) = 6 plants + 11 plant = 17 plants.
7. (2 rows x 2 plants) x (13 rows x 1plant) = 4 plants + 13 plant = 17 plants.
8. (1 rows x 2 plants) x (15 rows x 1plant) = 2 plants + 15 plant = 17 plants.

Question 3.
Bianca has a total of 25¢. She has some nickels and pennies. How many different combinations of nickels and pennies could Bianca have? Make a table to solve.

Number of Nickels ___1___ ___2___ ____3__ ____4__
Number of Pennies ___20___ ___15___ ___10___ ____5__
Total Value ___25¢___ ___25¢___ ___25¢___ ___25¢___
Bianca could have ___4___ Combination of 25 ¢.

Answer:
4 ways

Explanation:
1 Nickel = 5 pennies.
Bianca can have combinations of nickels and pennies to get 25¢ are
1. 1 Nickel + 20 Pennies = 5 Pennies + 20 Pennies = 25 Pennies = 25¢
2. 2 Nickels + 15 Pennies = 10 Pennies + 15 Pennies = 25 Pennies = 25¢
3. 3 Nickels + 10 Pennies = 15 Pennies + 10 Pennies = 25 Pennies = 25¢
4. 4 Nickels + 5 Pennies = 20 Pennies + 5 Pennies = 25 Pennies = 25¢

Multiplication Lesson Check Page No 252

Question 1.
The table shows different ways that Cameron can display his 12 model cars on shelves. How many shelves will display 2 cars if 8 of the shelves each display 1 car?
Options:
a. 1
b. 2
c. 3
d. 4

Question 1.
Go Math Grade 3 Answer Key Chapter 4 Multiplication Facts and Strategies Multiplication img 25

Answer:
b. 2

Explanation:
1. (Shelves with 1 car x 2) + (Shelves with 2 cars x 5) = 2 Cars + 10 Cars = 12 Cars
2. (Shelves with 1 car x 4) + (Shelves with 2 cars x 4) = 4 Cars + 8 Cars = 12 Cars
3. (Shelves with 1 car x 6) + (Shelves with 2 cars x 3) = 6 Cars + 6 Cars = 12 Cars
4. (Shelves with 1 car x 8) + (Shelves with 2 cars x 2) = 8 Cars + 4 Cars = 12 Cars
5. (Shelves with 1 car x 10) + (Shelves with 2 cars x 1) = 10 Cars + 2 Cars = 12 Cars
Therefore, 2 shelves will display 2 cars if 8 of the shelves each display 1 car.

Spiral Review

Question 2.
Find the sum.
3 1 7
+ 1 5 1
——–
Options:
a. 166
b. 268
c. 468
d. 568

Answer:
c. 468

Explanation:
Add the ones. Add 7 + 1 =8. Then, Add the tens. Add 1 + 5 = 6. Add the hundreds. Add 3 + 1 = 4. Place the one’s, ten’s and hundreds to get the sum. The sum is 468.

Question 3.
The school cafeteria has an order for 238 hot lunches. What is 238 rounded to the nearest ten?
Options:
a. 300
b. 240
c. 230
d. 200

Answer:
b. 240

Explanation:
The 238 lies between 230 and 240. But the 238 is closer to 240 than it is to 230. So, 238 is rounded to the 240.

Question 4.
Tyler made a picture graph to show students’ favorite colors. This is the key for his graph.
Each Go Math Grade 3 Answer Key Chapter 4 Multiplication Facts and Strategies Multiplication img 26 = 3 votes.
If 12 students voted for green, how many Go Math Grade 3 Answer Key Chapter 4 Multiplication Facts and Strategies Multiplication img 27 should there be in the green row of the graph?
Options:
a. 3
b. 4
c. 9
d. 12

Answer:
b. 4

Explanation:
Each circle = 3 votes. If 12 students voted for green, total votes = 12. Write 12 = 3 + 3 + 3 +3. So, 4 circles represent 12 votes.

Question 5.
There are 5 bikes in each bike rack at the school. There are 6 bike racks. How many bikes in all are in the bike racks?
Options:
a. 11
b. 24
c. 25
d. 30

Answer:
d. 30

Explanation:
To find the total bikes in the bike rack at the school, multiply 5 bikes x 6 bike racks. 5 x 6 = 30 bikes are available in bike racks.

Review/Test Page No 253

Question 1.
Mrs. Ruiz sorted spools of thread into 4 boxes. Each box holds 5 spools. How many spools of thread does Mrs. Ruiz have? Draw circles to model the problem. Then solve.
________ spools

Answer:
20 spools

Explanation:
4 boxes of spools x 5 spools = 20 spools. Mrs. Ruiz has 20 spools of thread.

Question 2.
For numbers 2a–2d, select True or False for each multiplication sentence.
a. 2 × 8 = 16
i. True
ii. False

Answer:
i. True

Explanation:
Double 8.
8 + 8 = 16.
So, 2 x 8 = 16.

Question 2.
b. 5 × 8 = 40
i. True
ii. False

Answer:
i. True

Explanation:
Skip-count by 5’s 8 times. Write like 5, 10, 15, 20, 25, 30, 35, 40. So, the answer for 5 x 8 is 40.

Question 2.
c. 6 × 8 = 56
i. True
ii. False

Answer:
ii. False

Explanation:

Use doubles to find the answer of 6 x 8. Firstly, multiply 3 x 8 = 24. Then, double the value of 3 x 8. 24 + 24 = 48. The answer for 6 x 8 is 48. So, 6 × 8 = 56 is false.

Question 2.
d. 8 × 8 = 64
i. True
ii. False

Answer:
i. True

Explanation:
Factor 8 is an even number. 4+ 4
8 x 4 = 32.
32 doubled is 64.
8 x 8 = 64.

Question 3.
Bella is planning to write in a journal. Some pages will have one journal entry on them, and other pages will have two journal entries on them. If Bella wants to make 10 entries, how many different ways can she write them in her journal?
________ different ways

Answer:
4 ways

Explanation:
1. (1 journal entry x 2) + (2 journal entries x 4) = 2 journals + 8 journals = 10 journals.
2. (1 journal entry x 4) + (2 journal entries x 3) = 4 journals + 6 journals = 10 journals.
3. (1 journal entry x 6) + (2 journal entries x 2) = 6 journals + 4 journals = 10 journals.
4. (1 journal entry x 8) + (2 journal entries x 1) = 8 journals + 2 journals = 10 journals.

Bella can use 4 ways to write journals.

Question 4.
There are 7 days in 1 week. How many days are there in 4 weeks?
_______ days

Answer:
28 days

Explanation:
1 week = 1 x 7 = 7 days.
4 weeks = 4 x 7 = 28 days.

Review/Test Page No 254

Question 5.
Circle groups to show 3 × (2 × 3).
Go Math Grade 3 Answer Key Chapter 4 Multiplication Facts and Strategies Review/Test img 28
Type below:
__________

Answer:
18

Explanation:

3 × (2 × 3) = 3 x 6 = 18.

Question 6.
Dale keeps all of his pairs of shoes in his closet. Select the number of shoes that Dale could have in his closet. Mark all that apply.
Options:
a. 3
b. 4
c. 6
d. 7
e. 8

Answer:
b. 4
c. 6
e. 8

Explanation:
Pair of shoes have 2 in number. So, the possible ways are always even. Dale could have 4, 6, 8 in his closet.

Question 7.
Lisa completed the table to describe the product of a mystery one-digit factor and each number.
Go Math Grade 3 Answer Key Chapter 4 Multiplication Facts and Strategies Review/Test img 29
Part A
Give all of the possible numbers that could be Lisa’s mystery one-digit factor.
Type below:
__________

Answer:
0, 2, 4, 6, 8

Question 7.
Part B
Explain how you know that you have selected all of the correct possibilities.
Type below:
__________

Answer:
Given that the product numbers are even. The product of an even number with another number is even. So, we should take the mystery factor as an even number.

Review/Test Page No 255

Question 8.
Kate drew 7 octagons. An octagon has 8 sides. How many sides did Kate draw?
Go Math Grade 3 Answer Key Chapter 4 Multiplication Facts and Strategies Review/Test img 30
_________ sides

Answer:
56

Explanation:
Kate drew 7 octagons. An octagon has 8 sides. Kate draw 8 x 7 = 56 sides.

Question 9.
José buys 6 bags of flour. Each bag weighs 5 pounds. How many pounds of flour did José buy?
_________ pounds

Answer:
30 pounds

Explanation:
6 x 5 = 30. José spends 30 pounds to buy flours.

Question 10.
Break apart the array to show 8 × 6 = (4 × 6) + (4 × 6).
Go Math Grade 3 Answer Key Chapter 4 Multiplication Facts and Strategies Review/Test img 31

Answer:

 

Explanation:
Break the array of 4 columns and 6 rows to get the answer.

Question 11.
Circle the symbol that makes the multiplication sentence true.
9 × 6Go Math Grade 3 Answer Key Chapter 4 Multiplication Facts and Strategies Review/Test img 32 3 × (3 × 9)
_________

Answer:
<

Explanation:
9 x 6 = 54
3 x 27 = 81
54 < 81.
So, 9 x 6 < 3 x (3 x 9)

Question 12.
Roberto wants to make $2.00 using dollars, half dollar, and quarters. How many different ways can he make $2.00?
__________ different ways

Answer:
18

Explanation:
0.25  – 8 |6 |4 |4 |2 |2 |0 |0 |0
0.50 – 0 |1 |2 |0 |3 |1 |2 |4 |0
1        -0 |0 |0 |1 |0 |1 |1 |0 |2

Review/Test Page No 256

Question 13.
A carpenter builds stools that have 3 legs each. How many legs does the carpenter use to build 5 stools? Use the array to explain how you know your answer is correct.
_________ legs

Answer:
15 legs

Explanation:
Each stool has 3 legs. To build 5 stools, 5 x 3 = 15 legs. 15 legs need to build 5 stools.

Question 14.
Etta buys some ribbon and cuts it into 7 pieces that are the same length. Each piece is 9 inches long. How long was the ribbon that Etta bought?
_________ inches

Answer:
63 inches

Explanation:
The length of the ribbon is 7 x 9 inches = 63 inches.

Question 15.
Antoine and 3 friends divide some pennies evenly among themselves. Each friend separates his pennies into 3 equal stacks with 5 pennies in each stack.
Write a multiplication sentence that shows the total number of pennies.
Type below:
_________

Answer:
4 x (5 x 3)

Explanation:
5 pennies into 3 equal stacks = 5 x 3 = 15.
15 stacks shared between 4 friends = 4 x 15 = 4 x (5 x 3).

Question 16.
Luke is making 4 first-aid kits. He wants to put 3 large and 4 small bandages in each kit. How many bandages does he need for the kits? Show your work.
_________ bandages

Answer:
28 bandages

Explanation:
Luke wants to put 3 large and 4 small bandages in each kit. So, total = 3 + 4 = 7 bandages.
He is making 4 first-aid kits. So, 4 x 7 bandages = 28 bandages.

Review/Test Page No 257

Question 17.
For numbers 17a–17d, select True or False for each equation.
a. 3 × 7 = 21
i. True
ii. False

Answer:
i. True

Explanation:

Skip count by 3’s until you say 7 numbers. 3, 6, 9, 12, 15, 18, 21. So, the answer for 3 x 7 is 21. The answer is true.

Question 17.
b. 5 × 7 = 28
i. True
ii. False

Answer:
ii. False

Explanation:

Skip count by 5’s until you say 7 numbers. 5, 10, 15, 20, 25, 30, 35. Now, the count of the number is 7. So, the answer for 5 x 7 is 35. The answer is False.

Question 17.
c. 8 × 7 = 49
i. True
ii. False

Answer:
ii. False

Explanation:

Use Distributive Property to find 8 x 7 = (4 + 4) x 7 = (4 x 7) + (4 x 7) = 28 + 28 = 56 is the correct answer. The answer is False.

Question 17.
d. 9 × 7 = 63
i. True
ii. False

Answer:
i. True

Explanation:
9 = 3 + 6
9 × 7 = (3 + 6) x 7
Multiply each addend by 7.
9 × 7 = (3 x 7) + (6 x 7)
Add the products.
9 × 7 = 21 + 42
9 × 7 = 63.
The answer is true.

Question 18.
Circle the number that makes the multiplication sentence true.
10 × Go Math Grade 3 Answer Key Chapter 4 Multiplication Facts and Strategies Review/Test img 33 = 40

Answer:
4

Explanation:
The value of 10 x4 = 40. So, the answer is 4.

Question 19.
For numbers 19a–19d, select Yes or No to indicate whether the number sentence has the same value as 8 × 6.
a. 8 + (4 × 2) = ■
i. yes
ii. no

Answer:
ii. no

Explanation:
8 x 6 = 48.
8 + (4 x 2) = 8 + 8 =16. The answer is no.

Question 19.
b. (8 × 4) + (8 × 2) = ■
i. yes
ii. no

Answer:
i. yes

Explanation:
8 x 6 = 48.
(8 x 4) + (8 x 2) = 32 + 16 = 48. The answer is yes.

Question 19.
c. (6 × 4) + (6 × 2) = ■
i. yes
ii. no

Answer:
ii. no

Explanation:
8 x 6 = 48.
(6 × 4) + (6 × 2) = 24 + 12 = 36. The answer is no.

Question 19.
d. 6 × (4 + 4) = ■
i. yes
ii. no

Answer:
i. yes

Explanation:
8 x 6 = 48.
6 × (4 + 4) = 6 x 8 = 48. The answer is yes.

Question 20.
Chloe bought 4 movie tickets. Each ticket cost $6. What was the total cost of the movie tickets?
$ _________

Answer:
$24

Explanation:
The total cost of the movie tickets = 4 x $6 = $24.

Question 21.
Write a multiplication sentence using the following numbers and symbols.
Go Math Grade 3 Answer Key Chapter 4 Multiplication Facts and Strategies Review/Test img 34
Type below:
__________

Answer:
6 x (5 x 2) = 60

Explanation:
We can write as 6 x (5 x 2) = 60.

Review/Test Page No 258

Question 22.
Louis started a table showing a multiplication pattern.
Part A
Complete the table. Describe a pattern you see in the products.
Go Math Grade 3 Answer Key Chapter 4 Multiplication Facts and Strategies Review/Test img 35
Type below:
__________

Answer:
12, 15, 18, 21, 24, 27, 30. Add 3 to the product to get the next product.

Explanation:
Add 3 to the product to get the next product.
9+3 = 12.
12 + 3 = 15.
15 + 3 = 18.
18 + 3 = 21.
21 + 3 = 24.
24 + 3 = 27.
27 + 3 = 30.

Question 22.
Part B
If you multiplied 3 × 37, would the product be an even number or an odd number? Use the table to explain your reasoning.
__________

Answer:
When an even number is multiplied by 3 (an odd number), the product is even. When an odd number is multiplied by 3, the product is odd, so the product of 3 × 37 would be odd.

Question 23.
Use the number line to show the product of 4 × 8.
Go Math Grade 3 Answer Key Chapter 4 Multiplication Facts and Strategies Review/Test img 36
4 × 8 = _______

Answer:
32

Explanation:
Skip 2 numbers to get the product of 4 multiples. If you skip 2 numbers until you count 8, you can get 32. The answer is 32.

Conclusion

Go Math Grade 3 Answer Key Chapter 4 Multiplication Facts and Strategies include a clear-cut explanation for all the questions in it. We tried our best to help you understand the concept better by taking enough images, graphs. Firstly solve the problems on your own and tally your answers with the Go Math 3rd Grade Answer Key Ch 4 Multiplication Facts and Strategies.

In order to provide you a smooth learning experience, we have compiled all the Questions in Grade 3 Chapter 4 Multiplication Facts and Strategies in a comprehensive manner. Grab the opportunity and learn from Go Math Grade 3 Answer Key Chapter 4 Multiplication Facts and Strategies Extra Practice PDF too to make your learning fun.

Go Math Grade 8 Answer Key Chapter 15 Two-Way Tables

go-math-grade-8-chapter-15-two-way-tables-answer-key

Hey guys! Are you searching for the Go Math Grade 8 Answer Key Chapter 15 Two-Way Tables on various sites? If your answer is yes, then don’t worry you are on the right page. Here you can get the best answers for Go Math Grade 8 Chapter 15 Two-Way Tables. Students who are lagging in two-way tables can get them in this article. Make use of Go Math Grade 8 Answer Key Chapter 15 Two-way Tables and Download them for free.

Go Math Grade 8 Chapter 15 Two-Way Tables Answer Key

You can learn the concepts of frequency tables in Go Math Grade 8 Answer Key Chapter 15 Two-Way Tables. It is the second essential teaching practice for the students of 8th standard. So, every student who love to learn math must go through the HMH Go Math Grade 8 Chapter 15 in an easy way. You can download and practice Go Math Grade 8 Solution Key Chapter 15 Two-Way Tables for free of cost. Click on the HMH Go Math Grade 8 Answer Key links and kickstart your preparation.

Lesson 1: Two-Way Frequency Tables

 Lesson 2: Two-Way Relative Frequency Tables

Model Quiz

Mixed Review

Guided Practice – Two-Way Frequency Tables – Page No. 454

Question 1.
In a survey of 50 students, 60% said that they have a cat. Of the students who have a cat, 70% also have a dog. Of the students who do not have a cat, 75% have a dog. Complete the two-way table.
Go Math Grade 8 Answer Key Chapter 15 Two-Way Tables Lesson 1: Two-Way Frequency Tables img 1
a. Enter the total number of students surveyed in the bottom right cell of the table.
Type below:
_______________

Answer:
grade 8 chapter 15 image 1

Explanation:
In a survey of 50 students, 60% said that they have a cat.
In mathematical terms:
Cat = 0.6×50 = 30
If 60% have a cat, then 40% don’t have a cat
No Cat = (1 – 0.6) × 50 = 20
Because there are 2 options, Adding them will give the total amount of students
Total = Cat + No Cat = 50
Of the students who have a cat, 70% also have a dog. Of the students who do not have a cat, 75% have a dog. So, in mathematical terms:
Dog = Cat × 0.7 + No Cat ×0.75 = 30 × 0.7 + 20 × 0.75 = 36
Following the same logic as before, if 70% of students who have a cat also have a dog, then 30% of them don’t have a dog. The same analysis for the students who do not have a cat.
No Dog = Cat × (1-0.7) + No Cat × (1-0.75) = 30 × (1-0.7) + 20 × (1-0.75) = 14
Again, the addition of the 2 options has to give the total amount of students
Total = 50

Question 1.
b. Fill in right column.
Type below:
_______________

Answer:
Of the students who have a cat, 70% also have a dog. Of the students who do not have a cat, 75% have a dog. In mathematical terms:
Dog = Cat × 0.7 + No Cat ×0.75 = 30 × 0.7 + 20 × 0.75 = 36

Question 1.
c. Fill in top row.
Type below:
_______________

Answer:
In a survey of 50 students, 60% said that they have a cat. In mathematical terms:
Cat = 0.6×50 = 30

Question 1.
d. Fill in second row.
Type below:
_______________

Answer:
If 60% have a cat, then 40% don’t have a cat
No Cat = (1 – 0.6) × 50 = 20

Question 1.
e. Fill in last row.
Type below:
_______________

Answer:
Because there are 2 options, the addition of them has to give the total amount of students
Total = Cat + No Cat = 50

Question 2.
The results of a survey at a school are shown. Is there an association between being a boy and being left-handed? Explain.
Go Math Grade 8 Answer Key Chapter 15 Two-Way Tables Lesson 1: Two-Way Frequency Tables img 2
_______________

Answer:
No, there isn’t any association between being a boy and being left-handed.
Boys are no more likely to be left-handed than right-handed.

ESSENTIAL QUESTION CHECK-IN

Question 3.
Voters were polled to see whether they supported Smith or Jones. Can you construct a two-way table of the results? Why or why not?
_______________

Answer:
You cannot construct a two-way table of the results. Because there is only one variable; voters. If there were two variables, such as men and women, a two-way table could be constructed.

15.1 Independent Practice – Two-Way Frequency Tables – Page No. 455

Question 4.
Represent Real-World Problems One hundred forty students were asked about their language classes. Out of 111 who take French, only 31 do not take Spanish. Twelve take neither French nor Spanish. Use this information to make a two-way table.
Go Math Grade 8 Answer Key Chapter 15 Two-Way Tables Lesson 1: Two-Way Frequency Tables img 3
Type below:
_______________

Answer:
grade 8 chapter 15 image 2

Question 5.
Represent Real-World Problems Seventh- and eighth-grade students were asked whether they preferred science or math.
a. Complete the two-way table.
Go Math Grade 8 Answer Key Chapter 15 Two-Way Tables Lesson 1: Two-Way Frequency Tables img 4
Type below:
_______________

Answer:
grade 8 chapter 15 image 3

Question 5.
b. Is there an association between being in eighth grade and preferring math? Explain.
_______________

Answer:
There is no association as such between being in eighth grade and preferring maths. But due to the total no. of eighth-grade students choosing maths is greater than the total no. of students in seventh grade preferring science. So, the eighth-grade students preferred.

Question 6.
Persevere in Problem Solving The table gives partial information on the number of men and women who play in the four sections of the Metro Orchestra.
a. Complete the table.
Go Math Grade 8 Answer Key Chapter 15 Two-Way Tables Lesson 1: Two-Way Frequency Tables img 5
Type below:
_______________

Answer:
grade 8 chapter 15 image 4

Question 6.
b. Is there an association between being a woman and playing strings? Explain.
_______________

Answer:
There is no association between being a woman and playing strings since the number of men playing string is less than women.

Two-Way Frequency Tables – Page No. 456

FOCUS ON HIGHER ORDER THINKING

Question 7.
Multi-Step The two-way table below shows the results of a survey of Florida teenagers who were asked whether they preferred surfing or snorkeling.
a. To the right of the number in each cell, write the relative frequency of the number compared to the total for the row the number is in. Round to the nearest percent.
Go Math Grade 8 Answer Key Chapter 15 Two-Way Tables Lesson 1: Two-Way Frequency Tables img 6
Type below:
_______________

Answer:
grade 8 chapter 15 image 9

Question 7.
b. Explain the meaning of the relative frequency you wrote beside 28.
Type below:
_______________

Answer:
The relative frequency shows the percentage of people aged 16-18 that prefer snorkeling.

Question 7.
c. To the right of each number you wrote in part a, write the relative frequency of each number compared to the total for the column the number is in. Are the relative frequencies the same? Why or why not?
Type below:
_______________

Answer:
grade 8 chapter 15 image 10

Question 7.
d. Explain the meaning of the relative frequency you wrote beside 28.
Type below:
_______________

Answer:
The relative frequency represents the percentage of people that prefer snorkeling that is aged 16-18.

Guided Practice – Two-Way Relative Frequency Tables – Page No. 462

Question 1.
In a class survey, students were asked to choose their favorite vacation destination. The results are displayed by gender in the two-way frequency table.
Go Math Grade 8 Answer Key Chapter 15 Two-Way Tables Lesson 2: Two-Way Relative Frequency Tables img 7
a. Find the total for each gender by adding the frequencies in each row. Write the row totals in the Total column.
Type below:
_______________

Answer:
grade 8 chapter 15 image 5
Girl = 7 + 3 + 2 = 12
Boy = 5 + 2 + 6 = 13

Question 1.
b. Find the total for each preferred vacation spot by adding the frequencies in each column. Write the column totals in the Total row.
Type below:
_______________

Answer:
Seashore = 7 + 5 = 12
Mountains = 3 + 2 = 5
Other = 2 + 6 = 8

Question 1.
c. Write the grand total (the sum of the row totals and the column totals) in the lower-right corner of the table.
Type below:
_______________

Answer:
grand total = 25

Question 1.
d. Create a two-way relative frequency table by dividing each number in the above table by the grand total. Write the quotients as decimals.
Go Math Grade 8 Answer Key Chapter 15 Two-Way Tables Lesson 2: Two-Way Relative Frequency Tables img 8
Type below:
_______________

Answer:
grade 8 chapter 15 image 6

Explanation:
7/25 = 0.28, 3/25 = 0.12; 2/25 = 0.08; 12/25 = 0.48
5/25 = 0.2; 2/25 = 0.08; 6/25 = 0.24; 13/25 = 0.52
12/25 = 0.48; 5/25 = 0.2; 8/25= 0.32; 25/25 = 1

Question 1.
e. Use the table to find the joint relative frequency of students surveyed who are boys and who prefer vacationing in the mountains.
_________

Answer:
Joint relative frequency of boys = 2/25 = 0.08
These boys who prefer vacationing in the mountains.

Question 1.
f. Use the table to find the marginal relative frequency of students surveyed who prefer vacationing at the seashore.
_________

Answer:
The marginal relative frequency of studnets = 12/25 = 0.48
These are the number of students who prefer vacationing in the seashore.

Question 1.
g. Find the conditional relative frequency that a student surveyed prefers vacationing in the mountains, given that the student is a girl. Interpret this result.
_________

Answer:
The condition relative frequency of girls of row = 3/12 = 0.25
And that of the column is 3/5 = 0.6
These are the number of girls who preferred vacationing in the mountains.

ESSENTIAL QUESTION CHECK-IN

Question 2.
How can you use a two-way frequency table to learn more about its data?
Type below:
_______________

Answer:
The two-way frequency table gives perfection and accuracy in calculating the data. It helps to calculate the total value two times while calculating the data of the row and to calculate the data of the column.

15.2 Independent Practice – Two-Way Relative Frequency Tables – Page No. 463

Stefan surveyed 75 of his classmates about their participation in school activities as well as whether they have a part-time job. The results are shown in the two-way frequency table. Use the table for Exercises 3–6.
Go Math Grade 8 Answer Key Chapter 15 Two-Way Tables Lesson 2: Two-Way Relative Frequency Tables img 9

Question 3.
a. Complete the table.
Type below:
_______________

Answer:
grade 8 chapter 15 image 7

Question 3.
b. Explain how you found the correct data to enter in the table.
Type below:
_______________

Answer:
1) In the first row of yes the values of sports only, Neither and total were provide. Also, in the 1st column of cubes only the values of No and total were providers. So, these values were subtracted and the value of yes was known.
2) The values in the 1st row of yes were added and subtracted from the total column. Hence the value in both columns was known. So, similarly, by adding and subtracting the values in the rows and columns the vacant values were known.

Question 4.
Create a two-way relative frequency table using decimals. Round to the nearest hundredth.
Go Math Grade 8 Answer Key Chapter 15 Two-Way Tables Lesson 2: Two-Way Relative Frequency Tables img 10
Type below:
_______________

Answer:
grade 8 chapter 15 image 8

Explanation:
Using the frequency table in problem 3, divide each number in each cell by 75, the grand total, and round to the nearest hundredth.
Job and clubs only: 10/75 = 0.13
Job and sports only: 12/75 = 0.16
Job and both clubs not sports: 20/75 = 0.27
Job and neither clubs nor sports: 9/75 = 0.12
Job total: 51/75 = 0.68
No jobs and clubs only: 5/75 = 0.07
No Job and sports only: 6/75 = 0.08
No job and both clubs and sports: 10/75 = 0.13
No Job and neither clubs nor sports: 3/75 = 0.04
No Job total: 24/75 = 0.32
Clubs only total: 15/75 = 0.2
Sports only total: 18/75 = 0.24
Both clubs and sports total: 30/75 = 0.4
Neither clubs nor sports total: 12/75 = 0.16
Total: 75/75 = 1.00
Use the totals above to create a two-way frequency table.

Question 5.
Give each relative frequency as a percent.
a. the joint relative frequency of students surveyed who participate in school clubs only and have part-time jobs
_________ %

Answer:
13%

Explanation:
The joint relative frequency of students surveyed who participate in school clubs only and have part-time jobs 0.13 or 13%
(Job and clubs only: 10/75 = 0.13)

Question 5.
b. the marginal frequency of students surveyed who do not have a part-time job
_________ %

Answer:
32%

Explanation:
The marginal frequency of students surveyed who do not have a part time job is 0.32 or 32%
(No job total: 24/75 = 0.32)

Question 5.
c. the conditional relative frequency that a student surveyed participates in both school clubs and sports, given that the student has a part-time job
_________ %

Answer:
39%

Explanation:
The conditional relative frequency that a student surveyed participates in both school clubs and sports, given that the student has a part-time job is 0.39 or 39%
(20/51 = 0.39)

Two-Way Relative Frequency Tables – Page No. 464

Question 6.
Discuss possible influences of having a part-time job on participation in school activities. Support your response with an analysis of the data.
Type below:
_______________

Answer:
The joint relative frequency of students surveyed who participate in school activities and have part-time jobs is 0.27 or 27%.
The joint relative frequency of students surveyed who participate in school activities and do not have part-time jobs is 0.13 or 13%.
This means that the students who have jobs are more likely to participate in school activities than the students who do not have jobs.

FOCUS ON HIGHER ORDER THINKING

Question 7.
The head of quality control for a chair manufacturer collected data on the quality of two types of wood that the company grows on its tree farm. The table shows the acceptance and rejection data.
Go Math Grade 8 Answer Key Chapter 15 Two-Way Tables Lesson 2: Two-Way Relative Frequency Tables img 11
a. Critique Reasoning To create a two-way relative frequency table for this data, the head of quality control divided each number in each row by the row total. Is this correct? Explain.
_______________

Answer:
No, it is not correct for the head of quality control to divide each number in each row by the row total to create a two-way relative frequency table. Each data value should have been divided by 600, the grand total, not by the row total.

Question 7.
b. Draw Conclusions Are any of the data the head of quality control entered into the two-way relative frequency table correctly? If so, which is and which isn’t? Explain.
Type below:
_______________

Answer:
Since the head of quality control divided incorrectly, the top two rows are incorrect. However, the bottom row has correct data because each number in the bottom row was divide by the grand total.

Question 8.
Analyze Relationships What is the difference between relative frequency and conditional relative frequency?
Type below:
_______________

Answer:
Relative frequency is found by dividing a frequency by the grand total while conditional relative frequency is found by dividing a frequency that is not in the Total row or the Total column by the frequency’s row total or column total.

Ready to Go On? – Model Quiz – Page No. 465

15.1 Two-Way Frequency Tables

Martin collected data from students about whether they played a musical instrument. The table shows his results. Use the table for Exercises 1–4.
Go Math Grade 8 Answer Key Chapter 15 Two-Way Tables Model Quiz img 12

Question 1.
Of the students surveyed, how many played an instrument?
__________ students

Answer:
90 students

Explanation:
Of the students surveyed, 90 students played an instrument

Question 2.
How many girls surveyed did NOT play an instrument?
__________ girls

Answer:
40 girls

Explanation:
(No. of boys did not play a musical instrument) + (No.of girls did not play a musical instrument) = (Total no. of students did not play an instrument)
70 + x = 110
x = 40
So, the no. of girls did not play a musical instrument = 40.

Question 3.
What is the relative frequency of a student playing an instrument? Write the answer as a percent.
________ %

Answer:
45%

Explanation:
The relative frequency of a student playing an instrument in this case, 90 out of 200 students play a musical instrument
(90/200) . 100 = 45%

Question 4.
What is the relative frequency of playing an instrument among boys? Write the answer as a decimal.
________ %

Answer:
38%

Explanation:
the relative frequency of playing an instrument among boys
(42/112) . 100 = 37.5% or 38%

15.2 Two-Way Relative Frequency Tables

Students were asked how they traveled to school. The two-way relative frequency table shows the results. Use the table for Exercises 5–7. Write answers as decimals rounded to the nearest hundredth.
Go Math Grade 8 Answer Key Chapter 15 Two-Way Tables Model Quiz img 13

Question 5.
What is the joint relative frequency of high school students who ride the bus?
________

Answer:
The joint relative frequency of high school students who ride the bus is 0.12

Question 6.
What is the marginal relative frequency of students surveyed who are in middle school?
________

Answer:
The marginal frequency of students surveyed in middle school is 0.42

Question 7.
What is the conditional relative frequency that a student rides the bus, given that the student is in middle school?
________

Answer:
0.62

Explanation:
The conditional relative frequency that a student rides the bus given that the student is in middle school.
Total no. of students going to bus = 0.26
Students who ride the bus (who are a middle school) = 0.42
Conditional relative frequency = 0.26/0.42 = 0.62

ESSENTIAL QUESTION

Question 8.
How can you use two-way tables to solve real-world problems?
Type below:
_______________

Answer:
Accuracy in the calculation of the data as it maintains decently in a suitable format. It helps to measure each and every frequency easily as the values are placed individually. Also, helps to measure the total of each row and column separately. So, since the data represented is suitable it makes the person understand and solve the problem.

Selected Response – Mixed Review – Page No. 466

The table gives data on the length of time that teachers at Tenth Avenue School have taught. Use the table for Exercises 1–5.
Go Math Grade 8 Answer Key Chapter 15 Two-Way Tables Mixed Review img 14

Question 1.
How many female teachers have taught for fewer than 10 years?
Options:
a. 4
b. 9
c. 21
d. 30

Answer:
c. 21

Explanation:
(No. of male teachers who have taught fewer than 10 years) + (No. of female teachers who have taught fewer than 10 years) = 30
9 + x = 30
x = 21
The number of female teachers who have taught fewer than 10 years is 21.

Question 2.
What is the relative frequency of teachers who have taught for 10 or more years?
Options:
a. 10%
b. 25%
c. 30%
d. 60%

Answer:
b. 25%

Explanation:
The relative frequency of teachers who have taught more than 10 or more years.
Total no. of teachers = 40
No. of teachers who taught more than 10 years = 10
Relative frequency = (10/40) . 100 = 25%

Question 3.
What is the relative frequency of having taught for fewer than 10 years among male teachers?
Options:
a. 0.09
b. 0.225
c. 0.6
d. 1.50

Answer:
c. 0.6

Explanation:
The relative frequency of male teachers who have taught fewer than 10 or more years.
Total no. of teachers = 15
No. of male teachers who taught for fewer than 10 years = 9
Relative frequency = (9/15) = 0.6

Question 4.
What is the joint relative frequency of female teachers who have taught for more than 10 years?
Options:
a. 4%
b. 10%
c. 16%
d. 25%

Answer:
b. 10%

Explanation:
The relative frequency of female teachers who taught for more than 10 years is 4/40 = 1/10 = 0.1 × 100 to calculate the data in percentage
10%

Question 5.
What is the marginal relative frequency of teachers who are female?
Options:
a. 0.16
b. 0.25
c. 0.4
d. 0.625

Answer:
d. 0.625

Explanation:
The total no. of teachers who are female = 25
Total no. of teachers = 40
Marginal frequency = 25/40 = 0.625

Question 6.
A triangle has an exterior angle of x°. Which of the following represents the measure of the interior angle next to it?
Options:
a. (180 − x)°
b. (x − 180)°
c. (90 − x)°
d. (x − 90)°

Answer:
a. (180 − x)°

Explanation:
The triangle has an exterior angle of x°. Let that angle be Angle ACD. So, the angle next to it is
Angle ACD + Angle ACB = 180º
Angle ACB = (180 − x)°

Question 7.
What is the volume of a cone that has a diameter of 12 cm and a height of 4 cm? Use 3.14 for π and round to the nearest tenth.
Options:
a. 25.12 cm3
b. 602.88 cm3
c. 150.72 cm3
d. 1,808.64 cm3

Answer:
c. 150.72 cm3

Explanation:
Diameter = 12cm
Radius r = 6cm
height h = 4cm
So, the volume of the cone = 1/3 . π . r². h
= 1/3 . 6 . 6 . 4 . 3.14 = 150.72 cm³

Mini-Task

Question 8.
The table gives data on books read by members of the Summer Reading Club.
Go Math Grade 8 Answer Key Chapter 15 Two-Way Tables Mixed Review img 15
a. Find the relative frequency of a club member reading fewer than 25 books.
________ %

Answer:
25%

Explanation:
The relative frequency of a club member reading fewer than 25 books is
Total of 16 members read fewer than 25 books
16/64 = 0.25 or 25%

Question 8.
b. Find the relative frequency of reading fewer than 25 books among girl club members.
________ %

Answer:
14%

Explanation:
The relative frequency of a girl club member reading fewer than 25 books is
9/64 = 0.14 or 14%

Question 8.
c. Is there an association between being a girl and reading fewer than 25 books? Explain.
____________

Answer:
No, there isn’t any association between being a girl and reading fewer than 25 books. Because it is a choice depending on an individual to read as many books as he/she can and comparing with the boys reading fewer than 25 books because the number of girls reading these books is comparatively greater.

Conclusion:

Keep in touch with us to get the step by step explanation for Go Math Grade 8 Answer Key Chapter 15 Two-Way Tables. Hope the information given by our team is beneficial for all the students of 8th standard. Build your self-confidence and prove yourself with the help of our Go Math Grade 8 Answer Key Chapter 15 Two-Way Tables. After your preparation, we suggest you to solve the problems given in the review test. All the Best!!!

Go Math Grade 4 Answer Key Chapter 6 Fraction Equivalence and Comparison

go-math-grade-4-chapter-6-fraction-equivalence-and-comparison-answer-key

Gain some basic knowledge about the Fraction Equivalence and Comparison topics by accessing the free Go Math Grade 4 Answer Key Chapter 6 Fraction Equivalence and Comparison. This resource of Go Math Grade 4 Answer Key aid your preparation for the standard tests. All the lessons covered in chapter 6 Fraction Equivalence and Comparison HMH Go Math Grade 4 Solution Key can be more efficient while your practice sessions. So, get the  Homework Help needed by referring to the Go Math Grade 4 Answer Key Chapter 6 Fraction Equivalence and Comparison.

Go Math Grade 4 Chapter 6 Fraction Equivalence and Comparison Answer Key

Download Go Math Grade 4 Solution Key Chapter 6 Fraction Equivalence and Comparison and prepare the concepts whenever you wish. Take the help from the given resource and solve the Grade 4 chapter 6 Fraction Equivalence and Comparison regularly to score high. Refer to the detailed Solutions presented here in Go Math Grade 4 Chapter 6 Fraction Equivalence and Comparison Answer Key and review your answers.

Lesson 1: Investigate • Equivalent Fractions

Lesson 2: Generate Equivalent Fractions

Lesson 3: Simplest Form

Lesson 4: Common Denominators

Lesson 5: Problem Solving • Find Equivalent Fractions

Mid-Chapter Checkpoint

Lesson 6: Compare Fractions Using Benchmarks

Lesson 7: Compare Fractions

Lesson 8: Compare and Order Fractions

Review/Test

Common Core – Equivalent Fractions – Page No. 331

Equivalent Fractions
Use the model to write an equivalent fraction.

Question 1.
Go Math Grade 4 Answer Key Chapter 6 Fraction Equivalence and Comparison Common Core Equivalent Fractions img 1
\(\frac{4}{6}=\frac{2}{3}\)

Answer:
\(\frac{4}{6}=\frac{2}{3}\)

Explanation:
The first image has 4 parts shaded our of 6 parts. Divide \(\frac{8}{10}\) with 2. You will get \(\frac{2}{3}\). That means 2 parts are shaded out of 3 parts.

Question 2.
Go Math Grade 4 Answer Key Chapter 6 Fraction Equivalence and Comparison Common Core Equivalent Fractions img 2
\(\frac{3}{4}\) = \(\frac{□}{□}\)

Answer:
\(\frac{3}{4}\) = \(\frac{6}{8}\)

Explanation:
The first image has 3 parts shaded our of 4 parts. Multiply \(\frac{8}{10}\) with 2. You will get \(\frac{6}{8}\). That means 6 parts are shaded out of 8 parts.

Tell whether the fractions are equivalent. Write = or ≠.

Question 3.
\(\frac{8}{10}\) _______ \(\frac{4}{5}\)

Answer:
\(\frac{8}{10}\) = \(\frac{4}{5}\)

Explanation:
Multiply the numerator and denominator of 4 / 5 with 2
8 / 10 = (2 / 2 ) x (4 / 5 )
= 8 / 10
So, 8 / 10 = 4 / 5.

Question 4.
\(\frac{1}{2}\) _______ \(\frac{7}{12}\)

Answer:
\(\frac{1}{2}\) ≠ \(\frac{7}{12}\)

Explanation:
Multiply the numerator and denominator of 1 / 2 with 6
1 / 2 = (6 / 6) x (1 / 2)
= (6 / 12)
So, 1/2 ≠ 7 / 12

Question 5.
\(\frac{3}{4}\) _______ \(\frac{8}{12}\)

Answer:
\(\frac{3}{4}\) ≠ \(\frac{8}{12}\)

Explanation:
Multiply the numerator and denominator of 3 / 4 with 3
3 / 4 = (3 / 3) x (3 / 4)
= (9 / 12)
So, 3 / 4 ≠ 8 / 12

Question 6.
\(\frac{2}{3}\) _______ \(\frac{4}{6}\)

Answer:
\(\frac{2}{3}\) = \(\frac{4}{6}\)

Explanation:
Multiply the numerator and denominator of 2 / 3 with 2
2 / 3 = (2 / 2) x ( 2 / 3 )
= 4 / 6
So, 2 / 3 = 4 / 6.

Question 7.
\(\frac{5}{8}\) _______ \(\frac{4}{10}\)

Answer:
\(\frac{5}{8}\) ≠ \(\frac{4}{10}\)

Explanation:
Multiply the numerator and denominator of 5 / 8 with 2
5 / 8 =(2 / 2) x (5 / 8)
= (10 / 16)
So, 5 / 8 ≠ 4 / 10

Question 8.
\(\frac{2}{6}\) _______ \(\frac{4}{12}\)

Answer:
\(\frac{2}{6}\) = \(\frac{4}{12}\)

Explanation:
Multiply the numerator and denominator of 2 / 6 with 2
2 / 6 = (2 / 2) x (2 / 6)
= (4 / 12)
So, 2 / 6 = 4 / 12.

Question 9.
\(\frac{20}{100}\) _______ \(\frac{1}{5}\)

Answer:
\(\frac{20}{100}\) = \(\frac{1}{5}\)

Explanation:
Cross Multiply the 20 / 100 with 20 / 20
20 / 100 = (20 / 20) x (20 / 100)
= (1 / 5)
So, 20 / 100 = 1 / 5.

Question 10.
\(\frac{5}{8}\) _______ \(\frac{9}{10}\)

Answer:
\(\frac{5}{8}\) ≠ \(\frac{9}{10}\)

Explanation:
Multiply the numerator and denominator of 5 / 8 with 2
5 / 8 = (2 / 2) x (5 / 8)
= 10 / 16
So, 5 / 8 ≠ 9 / 10

Question 11.
Jamal finished \(\frac{5}{6}\) of his homework. Margaret finished \(\frac{3}{4}\) of her homework, and Steve finished \(\frac{10}{12}\) of his homework. Which two students finished the same amount of homework?
_______

Answer:
Jamal and Steve

Explanation:
As per the given data,
Jamal finished work = 5 /6 of his homework
Margaret finished work = 3 / 4th of her homework
Steve finished work = 10 / 12 of his homework
Multiply the numerator and denominator of 5/ 6 with 2
Then, (2 / 2) x (5 / 6) = 10 / 12
Then, Jamal and Steve finished the same amount of homework.

Question 12.
Sophia’s vegetable garden is divided into 12 equal sections. She plants carrots in 8 of the sections. Write two fractions that are equivalent to the part of Sophia’s garden that is planted with carrots.
Type below:
___________

Answer:
\(\frac{2}{3}\) and \(\frac{4}{6}\)

Explanation:
As per the given data,
Sophia’s vegetable garden is divided into 12 equal sections
She plants carrots in 8 of the sections out of 12 sections = 8 / 12
By simplifying the 8 / 12, we will get 4 / 6
Again simplify the 4 /6 by dividing method, you will get 2 /3
2 / 3 = (2 / 2) x (2 / 3)
= 4 / 6
Then, the equivalent fractions are 2 / 3, 4 /6

Common Core – Equivalent Fractions – Page No. 332

Question 1.
A rectangle is divided into 8 equal parts. Two parts are shaded. Which fraction is equivalent to the shaded area of the rectangle?
Options:
a. \(\frac{1}{4}\)
b. \(\frac{1}{3}\)
c. \(\frac{2}{6}\)
d. \(\frac{3}{4}\)

Answer:
a. \(\frac{1}{4}\)

Explanation:
As per the given data,
A rectangle is divided into 8 equal parts
Two parts are shaded
Then, the shaded area of rectangle = 2 / 8
By simplifying the 2/ 8, you will get 1/ 4
So, the shaded area of rectangle = 1 / 4

Question 2.
Jeff uses 3 fifth-size strips to model \(\frac{3}{5}\). He wants to use tenth-size strips to model an equivalent fraction. How many tenth-size strips will he need?
Options:
a. 10
b. 6
c. 5
d. 3

Answer:
b. 6

Explanation:
From the given data,
Jeff uses 3 fifth –size strips to model = 3 / 5 size strips
If he want to use tenth – size strips to an equivalent fraction = 1 / 10 size strips
The number of strips = x
(1 / 10) x = 3 / 5
x = 30/5
then, required number of tenth size trips = 6

Question 3.
Cassidy places 40 stamps on each of 8 album pages. How many stamps does she place in all?
Options:
a. 300
b. 320
c. 360
d. 380

Answer:
b. 320

Explanation:
As per the given data,
Cassidy places 40 stamps on each of 8 album pages = 8 x 40
= 320
So, total placed stamps on album pages by Cassidy = 320 stamps

Question 4.
Maria and 3 friends have 1,200 soccer cards. If they share the soccer cards equally, how many will each person receive?
Options:
a. 30
b. 40
c. 300
d. 400

Answer:
c. 300

Explanation:
As per the given data,
Maria and 3 friends have 1200 soccer cards
If soccer cards shared equally by four members = 1200/4
= 300
Then, each person received soccer cards = 300

Question 5.
Six groups of students sell 162 balloons at the school carnival. There are 3 students in each group. If each student sells the same number of balloons, how many balloons does each student sell?
Options:
a. 9
b. 18
c. 27
d. 54

Answer:
a. 9

Explanation:
As per the given, data,
Six groups of students sell 162 balloons at the school carnival
There are 3 students in each group
Then, total number of students in 6 groups = 6 x 3 = 18
If each student sells the same number of balloons = 162 / 18
= 9
Number of balloons sells by each student = 9

Question 6.
Four students each made a list of prime numbers.
Eric: 5, 7, 17, 23
Maya: 3, 5, 13, 17
Bella: 2, 3, 17, 19
Jordan: 7, 11, 13, 21
Who made an error and included a composite number?
Options:
a. Eric
b. Maya
c. Bella
d. Jordan

Answer:
d. Jordan

Explanation:
As per the given data,
Four students each made a list of prime numbers.
Eric: 5, 7, 17, 23
Maya: 3, 5, 13, 17
Bella: 2, 3, 17, 19
Jordan: 7, 11, 13, 21
21 is not a prime number
So, An error made by Jordan

Page No. 335

Question 1.
Complete the table below.
Go Math Grade 4 Answer Key Chapter 6 Fraction Equivalence and Comparison img 3
Type below:
___________

Answer:
chapter 6 - Common Core - Image 3. jpg

Write two equivalent fractions.

Question 2.
\(\frac{4}{5}\)
\(\frac{4}{5}\) = \(\frac { 4×□ }{ 5×□ } \) = \(\frac{□}{□}\)
\(\frac{4}{5}\) = \(\frac { 4×□ }{ 5×□ } \) = \(\frac{□}{□}\)
\(\frac{4}{5}\) = \(\frac{□}{□}\) = \(\frac{□}{□}\)
Type below:
___________

Answer:
\(\frac{4}{5}\) = \(\frac{8}{10}\) = \(\frac{80}{100}\)

Explanation:
Two equivalent fractions of 4/5,
(4/5) x (2/2) = 8/10
And
(4/5) x (20/20) = 80/100
8/10 = (8/10) (10/10)
= (80/100)
So, the equivalent fractions of 4/5 = 8/10, 80/100

Question 3.
\(\frac{2}{4}\)
\(\frac{2}{4}\) = \(\frac { 2×□ }{ 4×□ } \) = \(\frac{□}{□}\)
\(\frac{2}{4}\) = \(\frac { 2×□ }{ 4×□ } \) = \(\frac{□}{□}\)
\(\frac{2}{4}\) = \(\frac{□}{□}\) = \(\frac{□}{□}\)
Type below:
___________

Answer:
\(\frac{2}{4}\) = \(\frac{4}{8}\) = \(\frac{8}{16}\)

Explanation:
Two equivalent fractions of 2/4,
(2/4) x (2/2) = 4/8
And
(2/4) x (4/4) = 8/16
4/8 = (4/8) (2/2)
= (8/16)
So, the equivalent fractions of 2/4 = 4/8, 8/16

Write two equivalent fractions.

Question 4.
\(\frac{3}{6}\)
\(\frac{3}{6}\) = \(\frac{□}{□}\) = \(\frac{□}{□}\)
Type below:
___________

Answer:
\(\frac{3}{6}\) = \(\frac{6}{12}\) = \(\frac{12}{24}\)

Explanation:
Two equivalent fractions of 3/6,
(3/ 6) x (2/2) = 6/12
And
(3/6) x (4/ 4) = 12/24
6/12 = (6/12) (2/2)
= (12/24)
So, the equivalent fractions of 3/6 = 6/12, 12/24

Question 5.
\(\frac{3}{10}\)
\(\frac{3}{10}\) = \(\frac{□}{□}\) = \(\frac{□}{□}\)
Type below:
___________

Answer:
\(\frac{3}{10}\) = \(\frac{6}{20}\) = \(\frac{12}{40}\)

Explanation:
Two equivalent fractions of 3/10,
(3/ 10) x (2/2) = 6/20
And
(3/10) x (4/ 4) = 12/40
6/20 = (6/20) (2/2)
= (12/40)
So, the equivalent fractions of 3/10 = 6/20, 12/40

Question 6.
\(\frac{2}{5}\)
\(\frac{2}{5}\) = \(\frac{□}{□}\) = \(\frac{□}{□}\)
Type below:
___________

Answer:
\(\frac{2}{5}\) = \(\frac{4}{10}\) = \(\frac{8}{20}\)

Explanation:
Two equivalent fractions of 2/5,
(2/ 5) x (2/2) = 4/10
And
(2/5) x (4/ 4) = 8/20
4/10 = (4/10) (2/2)
= (8/20)
So, the equivalent fractions of 2/5 = 4/10, 8/20

Tell whether the fractions are equivalent. Write = or ≠.

Question 7.
\(\frac{5}{6}\) ______ \(\frac{10}{18}\)

Answer:
\(\frac{5}{6}\) ≠ \(\frac{10}{18}\)

Explanation:
Multiply the numerator and denominator of 5/6 with 2
5/6 =(2/2) x (5/6)
= (10/12)
So, 5/6 ≠ 10/ 18

Question 8.
\(\frac{4}{5}\) ______ \(\frac{8}{10}\)

Answer:
\(\frac{4}{5}\) = \(\frac{8}{10}\)

Explanation:
Multiply the numerator and denominator of 4/5 with 2
4/5 =(2/2) x (4/5)
= (8/10)
So, 4/5 = 8/10

Question 9.
\(\frac{1}{5}\) ______ \(\frac{4}{10}\)

Answer:
\(\frac{1}{5}\) ≠ \(\frac{4}{10}\)

Explanation:
Multiply the numerator and denominator of 1/5 with 4
1/5 =(4/4) x (1/5)
= (4/20)
So, 1/5 ≠ 4/10

Question 10.
\(\frac{1}{4}\) ______ \(\frac{2}{8}\)

Answer:
\(\frac{1}{4}\) = \(\frac{2}{8}\)

Explanation:
Multiply the numerator and denominator of 1/4 with 2
1/4 =(2/2) x (1/4)
= (2/8)
So, 1/4 = 2/8

Page No. 336

Use the recipe for 11–12.
Go Math Grade 4 Answer Key Chapter 6 Fraction Equivalence and Comparison img 4

Question 11.
Kim says the amount of flour in the recipe can be expressed as a fraction. Is she correct? Explain.
______

Answer:
As per the given data, Kim says the amount of flour in the recipe can be expressed as a fraction. But in the recipe, 1 tablespoon flour is added. So, Kim says wrong.

Question 12.
How could you use a \(\frac{1}{8}\) – cup measuring cup to measure the light corn syrup?
Type below:
_________

Answer:
As per the given data,
By using the 1/8 cup measure the 9/12 cup light corn syrup
(9/12)/(1/8) = (9 x 8)/12
= (3 x 8)/4
= (3 x 2)
= 6
So, required 6 cups of 1/8 to measure the light corn syrup of 9/12.

Question 13.
Communicate Explain using words how you know a fraction is equivalent to another fraction.
Type below:
_________

Answer:
If you multiply the numerator and denominator of the first fraction by the same number and the products are the numerator and denominator of the second fraction, then the fractions are equivalent

Question 14.
Kyle drank \(\frac{2}{3}\) cup of apple juice. Fill in each box with a number from the list to generate equivalent fractions for \(\frac{2}{3}\). Not all numbers will be used.
Type below:
_________

Answer:
\(\frac{4}{6}\) and \(\frac{12}{18}\)

Explanation:
As per the given data,
Kyle drank 2/3 cup of apple juice
(2/3) x (2/2) = 4/6
(4/6) x (3/3) = 12/18
Equivalent fractions of 2/3 are 4/6 and 12/18

Common Core – Equivalent Fractions – Page No. 337

Write two equivalent fractions for each.

Question 1.
Go Math Grade 4 Answer Key Chapter 6 Fraction Equivalence and Comparison Common Core Equivalent Fractions img 5

Answer:
\(\frac{2}{6}\) and \(\frac{4}{12}\)

Explanation:
1/3
(1/3) x (2/2) = 2/6
(1/3) x (4/4) = 4/12
So, the equivalent fractions of 1/3 are 2/6 and 4/12

Question 2.
\(\frac{2}{3}\)
Type below:
_________

Answer:
\(\frac{4}{6}\) and \(\frac{8}{12}\)

Explanation:
2/3
(2/3) x (2/2) = 4/6
(2/3) x (4/4) = 8/12
Then, the equivalent fractions of 2/3 = 4/6 and 8/12

Question 3.
\(\frac{1}{2}\)
Type below:
_________

Answer:
\(\frac{2}{4}\) and \(\frac{4}{8}\)

Explanation:
1/2
(1/2) x (2/2) = 2/4
(1/2) x (4/4) = 4/8
Then, the equivalent fractions of 1/2 = 2/4, 4/8

Question 4.
\(\frac{4}{5}\)
Type below:
_________

Answer:
\(\frac{8}{10}\) and \(\frac{80}{100}\)

Explanation:
4/5
(4/5) x (2/2) = 8/10
(4/5) x (20/20) = 80/100
Then, the equivalent fractions of 4/5 = 8/10 and 80/100

Tell whether the fractions are equivalent. Write # or ≠.

Question 5.
\(\frac{1}{4}\) ______ \(\frac{3}{12}\)

Answer:
\(\frac{1}{4}\) = \(\frac{3}{12}\)

Explanation:
1/4
Multiply the numerator and denominator of 1/4 with 3
Then, (1/4) x (3/3) = 3/12
So, 1/4 = 3/12

Question 6.
\(\frac{4}{5}\) ______ \(\frac{5}{10}\)

Answer:
\(\frac{4}{5}\) ≠ \(\frac{5}{10}\)

Explanation:
4/5
Multiply numerator and denominator of 4/5 with 2
(4/5) x (2/2) = 8/10
Then 4/5 ≠ 5/10

Question 7.
\(\frac{3}{8}\) ______ \(\frac{2}{6}\)

Answer:
\(\frac{3}{8}\) ≠ \(\frac{2}{6}\)

Explanation:
3/8 ≠ 2/6

Question 8.
\(\frac{3}{4}\) ______ \(\frac{6}{8}\)

Answer:
\(\frac{3}{4}\) = \(\frac{6}{8}\)

Explanation:
3/4
Multiply the numerator and denominator of 3/4 with 2
Then, (3/4) x (2/2) = 6/8
So, 3/4 = 6/8

Question 9.
\(\frac{5}{6}\) ______ \(\frac{10}{12}\)

Answer:
\(\frac{5}{6}\) = \(\frac{10}{12}\)

Explanation:
5/6
Multiply the numerator and denominator with 2
(5/6) x (2/2) = 10/12
So, 5/6 = 10/12

Question 10.
\(\frac{6}{12}\) ______ \(\frac{5}{8}\)

Answer:
\(\frac{6}{12}\) ≠ \(\frac{5}{8}\)

Explanation:
6/12 ≠ 5/8

Question 11.
\(\frac{2}{5}\) ______ \(\frac{4}{10}\)

Answer:
\(\frac{2}{5}\) = \(\frac{4}{10}\)

Explanation:
2/5
Multiply the numerator and denominator of 2/5 with 2
(2/5) x (2/2) = 4/10
So, 2/5 = 4/10

Question 12.
\(\frac{2}{4}\) ______ \(\frac{3}{12}\)

Answer:
\(\frac{2}{4}\) ≠ \(\frac{3}{12}\)

Explanation:
2/4
Multiply the numerator and denominator of 2/4 with 3
(2/4) x (3/3) = 6/12
So, 2/4 ≠ 3/ 12

Question 13.
Jan has a 12-ounce milkshake. Four ounces in the milkshake are vanilla, and the rest is chocolate. What are two equivalent fractions that represent the fraction of the milkshake that is vanilla?
Type below:
_________

Answer:
\(\frac{1}{3}\) and \(\frac{2}{6}\)

Explanation:
As per the given data,
Jan has a 12-ounce milkshake
Four ounces in the milkshake are vanilla = 4/12 = 1/3
Then, 8-ounces in milkshake are chocolate = 8/12 = 2/3
4/12 = 1/3
By multiplying 1/3 with 2
(1/3) x (2/2) = 2/6
So, the equivalent fractions of vanilla milkshake are 1/3 and 2/6

Question 14.
Kareem lives \(\frac{4}{10}\) of a mile from the mall. Write two equivalent fractions that show what fraction of a mile Kareem lives from the mall.
Type below:
_________

Answer:
\(\frac{2}{5}\) and \(\frac{8}{20}\)

Explanation:
As per the given data,
Kareem lives 4/10 of a mile from the mall
To find the equivalent fractions of 4/10
Simplify the 4/10 = 2/5
Multiply the numerator and denominator of 2/5 with 4
(2/5) x (4/4) = 8/20
Then, the equivalent fraction of a mile Kareem lives from the mall = 2/5 and 8/20

Common Core – Equivalent Fractions – Page No. 338

Question 1.
Jessie colored a poster. She colored \(\frac{2}{5}\) of the poster red. Which fraction is equivalent to \(\frac{2}{5}\)?
Options:
a. \(\frac{4}{10}\)
b. \(\frac{7}{10}\)
c. \(\frac{4}{5}\)
d. \(\frac{2}{2}\)

Answer:
a. \(\frac{4}{10}\)

Explanation:
As per the given data,
Jessie colored a poster
She colored 2/5th of the poster red
Multiply the numerator and denominator of 2/5 with 2
Then, (2/5) x (2/2) = 4 /10
So, the equivalent fraction of 2/5 is 4/10

Question 2.
Marcus makes a punch that is \(\frac{1}{4}\) cranberry juice. Which two fractions are equivalent to \(\frac{1}{4}\)?
Options:
a. \(\frac{2}{5}, \frac{3}{12}\)
b. \(\frac{2}{8}, \frac{4}{12}\)
c. \(\frac{3}{4}, \frac{6}{8}\)
d. \(\frac{2}{8}, \frac{3}{12}\)

Answer:
d. \(\frac{2}{8}, \frac{3}{12}\)

Explanation:
As per the given data,
Marcus makes a punch that is 1/4th of cranberry juice
Multiply the numerator and denominator of 1/4 with 2
Then, (1/4) x (2/2) = 2/8
Multiply the numerator and denominator of 1/4 with 3
Then, (1/4) x (3/3) = 3/12
Equivalent fractions of 1/4 are 2/8 and 3/12

Question 3.
An electronics store sells a large flat screen television for $1,699. Last month, the store sold 8 of these television sets. About how much money did the store make on the television sets?
Options:
a. $160,000
b. $16,000
c. $8,000
d. $1,600

Answer:
b. $16,000

Explanation:
As per the given data,
An electronics store sells a large flat-screen television for $1,699
Last month, the store sold 8 of these television sets = 8 x $1,699 = $13,952. The money is about to $16,000.

Question 4.
Matthew has 18 sets of baseball cards. Each set has 12 cards. About how many baseball cards does Matthew have in all?
Options:
a. 300
b. 200
c. 150
d. 100

Answer:
b. 200

Explanation:
From the given data,
Matthew has 18 sets of basketball cards
Each set has 12 cards = 12 x 18
= 216
Total number of basketball cards with Matthew = 216. So, it is near to 200.

Question 5.
Diana had 41 stickers. She put them in 7 equal groups. She put as many as possible in each group. She gave the leftover stickers to her sister. How many stickers did Diana give to her sister?
Options:
a. 3
b. 4
c. 5
d. 6

Answer:
d. 6

Explanation:
As per the given data,
Diana has 41 stickers
She put them in 7 equal groups = 41/7
= 5 (remaining 6)
She gave the leftover stickers to her sister
The number of stickers Diana give to her sister = 6

Question 6.
Christopher wrote the number pattern below. The first term is 8.
8, 6, 9, 7, 10, …
Which is a rule for the pattern?
Options:
a. Add 2, add 3.
b. Add 6, subtract 3.
c. Subtract 6, add 3.
d. Subtract 2, add 3

Answer:
d. Subtract 2, add 3

Explanation:
From the given data,
Christopher wrote the number pattern = 8, 6, 9, 7, 10, …..
The first number in the pattern = 8
8 – 2 = 6 + 3 = 9 – 2 = 7 +3 = 10 ….
So, the rule for the above pattern is to subtract 2, add 3

Page No. 341

Question 1.
Write \(\frac{8}{10}\) in simplest form.
\(\frac{8}{10}\) = \(\frac { 8÷□ }{ 10÷□ } \) = \(\frac{□}{□}\)
\(\frac{□}{□}\)

Answer:
\(\frac{4}{5}\)

Explanation:
8/10 in simplest form
Divide the 8/10 with 2
(8/2)/(10/2) = 4/5
So, the simplest form of 8/10 is 4/5

Write the fraction in simplest form.

Question 2.
\(\frac{6}{12}\)
\(\frac{□}{□}\)

Answer:
\(\frac{1}{2}\)

Explanation:
6/12 in simplest form
Divide the 6/12 with 6
(6/6)/(12/6) = 1/2
So, the simplest form of 6/12 is 1/2

Question 3.
\(\frac{2}{10}\)
\(\frac{□}{□}\)

Answer:
\(\frac{1}{5}\)

Explanation:
2/10 in simplest form
Divide the 2/10 with 2
(2/2)/(10/2) = 1/5
So, the simplest form of 2/10 is 1/5

Question 4.
\(\frac{6}{8}\)
\(\frac{□}{□}\)

Answer:
\(\frac{3}{4}\)

Explanation:
6/8 in simplest form
Divide the 6/8 with 2
(6/2)/(8/2) = 3/4
So, the simplest form of 6/8 is 3/4

Question 5.
\(\frac{4}{6}\)
\(\frac{□}{□}\)

Answer:
\(\frac{2}{3}\)

Explanation:
4/6 in simplest form
Divide the 4/6 with 2
(4/2)/(6/2) = 2/3
So, the simplest form of 4/6 is 2/3

Write the fraction in simplest form.

Question 6.
\(\frac{9}{12}\)
\(\frac{□}{□}\)

Answer:
\(\frac{3}{4}\)

Explanation:
9/12in simplest form
Divide the 9/12 with 3
(9/3)/(12/3) = 3/4
So, the simplest form of 9/12 is 3/4

Question 7.
\(\frac{4}{8}\)
\(\frac{□}{□}\)

Answer:
\(\frac{1}{2}\)

Explanation:
4/8in simplest form
Divide the 4/8 with 4
(4/4)/(8/4) = 1/2
So, the simplest form of 4/8 is 1/2

Question 8.
\(\frac{10}{12}\)
\(\frac{□}{□}\)

Answer:
\(\frac{5}{6}\)

Explanation:
10/12 in simplest form
Divide the 10/12 with 2
(10/2)/(12/2) = 5/6
So, the simplest form of 10/12 is 5/6

Question 9.
\(\frac{20}{100}\)
\(\frac{□}{□}\)

Answer:
\(\frac{1}{5}\)

Explanation:
20 /100 in simplest form
Divide the 20/100 with 20
(20/20)/(100/20) = 1/5
So, the simplest form of 20/100 is 1/5

Tell whether the fraction is in simplest form. Write yes or no.

Question 10.
\(\frac{2}{8}\)
______

Answer:
No

Explanation:
2/8 in simplest form
Divide the 2/8 with 2
(2/2)/(8/2) = 1/4
The simplest form of 2/8 is 1/4
So, 2/8 is not the simplest form

Question 11.
\(\frac{9}{12}\)
______

Answer:
No

Explanation:
9/12 in simplest form
Divide the 9/12 with 3
(9/3)/(12/3) = 3/4
The simplest form of 9/12 is 3/4
So, 9/12 is not the simplest form

Question 12.
\(\frac{5}{6}\)
______

Answer:
Yes

Explanation:
5/6 is not divided by any number
Yes, 5/6 is the simplest form

Question 13.
\(\frac{4}{10}\)
______

Answer:
No

Explanation:
4/10 in simplest form
Divide the 4/10 with 2
(4/2)/(10/2) = 2/5
So, 4/10 is not the simplest form

Question 14.
There are 18 students in Jacob’s homeroom. Six students bring their lunch to school. The rest eat lunch in the cafeteria. In simplest form, what fraction of students eat lunch in the cafeteria?
\(\frac{□}{□}\) of students

Answer:
\(\frac{2}{3}\) of students

Explanation:
As per the given data,
There are 18 students in Jacob’s homeroom
6 students bring their lunch to school = 6/18 = 1/3
The rest eat lunch in the cafeteria = 18 – 6 = 12/18
Divide the numerator and denominator of 12/18 with 6
(12/6) x (18/6) = 2/3
So, 2/3 of students eat lunch in the cafeteria

Page No. 342

Use the map for 15−16.
Go Math Grade 4 Answer Key Chapter 6 Fraction Equivalence and Comparison img 6

Question 15.
Identify Relationships What fraction of the states in the southwest region share a border with Mexico? Is this fraction in simplest form?
\(\frac{□}{□}\)

Answer:
Yes, \(\frac{3}{4}\)

Explanation:
As per the given data,
Southwest region states = 4
Number of states in the southwest region shares a border with Mexico out of total southwest region states = 3/4
Yes, 3/4 is the simplest form

Question 16.
What’s the Question? \(\frac{1}{3}\) of the states in this region are on the Gulf of Mexico.
Type below:
_________

Answer:
In the simplest form, what fraction of the states in the southeast area on the Gulf of Mexico.

Question 17.
Pete says that to write \(\frac{4}{6}\) as \(\frac{2}{3}\), you combine pieces, but to write \(\frac{4}{6}\) as \(\frac{8}{12}\), you break apart pieces. Does this make sense? Explain.
______

Answer:
As per the given data,
Yes, it makes sense,
To write 4/6 as 2/3 combine sixth size pieces into equal groups of 2
Then (4/2)/(6/2) = 2/3
To write 4/6 as 8/12, break each sixth piece into 2 pieces
Then, 4/6 = (4 x 2)/(6 x 2) = 8/12

Question 18.
In Michelle’s homeroom, \(\frac{9}{15}\) of the students ride the bus to school, \(\frac{4}{12}\) get a car ride, and \(\frac{2}{30}\) walk to school.
For numbers 18a–18c, select True or False for each statement.
a. In simplest form, \(\frac{3}{5}\) of the students ride the bus to school.
i. True
ii. False

Answer:
i. True

Explanation:
9/15 of the students ride the bus to school
By dividing the numerator and denominator of 9/15 with 3
(9/3)/(15/3) =3/5
So, 3/5 of the students ride the bus to school
True

Question 18.
b. In simplest form, \(\frac{1}{4}\) of the students get a car ride to school.
i. True
ii. False

Answer:
ii. False

Explanation:
a. 4/12 of the students get a car ride
The simplest form of 4/12 = 1/3
So, 1/4 of the students get a car ride to school is a False statement

Question 18.
c. In simplest form, \(\frac{1}{15}\) of the students walk to school.
i. True
ii. False

Answer:
i. True

Explanation:
a. 2/30 of the students walk to school
By dividing the 2/30 with 2
(2/2)/(30/2) = 1/15
So, 1/15 of the students walk to school is a true statement

Common Core – Simplest Form – Page No. 343

Write the fraction in simplest form.

Question 1.
Go Math Grade 4 Answer Key Chapter 6 Fraction Equivalence and Comparison Common Core Simplest Form img 7

Answer:
\(\frac{3}{5}\)

Explanation:
To write the 6/10 in a simplest form
Divide the numerator and denominator of 6/10 with 2
(6 ÷2)/(10 ÷2) = 3/5
So, the simplest form of 6/10 = 3/5

Question 2.
\(\frac{6}{8}\) = \(\frac{□}{□}\)

Answer:
\(\frac{3}{4}\)

Explanation:
To write the 6/8in a simplest form
Divide the numerator and denominator of 6/8 with 2
(6 ÷2)/(8 ÷2) = 3/4
So, the simplest form of 6/8 = 3/4

Question 3.
\(\frac{5}{5}\) = \(\frac{□}{□}\)

Answer:
\(\frac{1}{1}\) = 1

Explanation:
To write the 5/5in a simplest form
Divide the numerator and denominator of 5/5 with 5
(5 ÷5)/(5 ÷5) = 1/1
So, the simplest form of 5/5 = 1

Question 4.
\(\frac{8}{12}\) = \(\frac{□}{□}\)

Answer:
\(\frac{2}{3}\)

Explanation:
To write the 8/12in a simplest form
Divide the numerator and denominator of 8/12 with 4
(8 ÷4)/(12 ÷4) = 2/3
So, the simplest form of 8/12 = 2/3

Question 5.
\(\frac{100}{100}\) = \(\frac{□}{□}\)

Answer:
\(\frac{1}{1}\) = 1

Explanation:
The simplest form of 100/100 = 1

Question 6.
\(\frac{2}{6}\) = \(\frac{□}{□}\)

Answer:
\(\frac{1}{3}\)

Explanation:
To write the 2/6in a simplest form
Divide the numerator and denominator of 2/6 with 2
(2 ÷2)/(6 ÷2) = 1/3
So, the simplest form of 2/6 = 1/3

Question 7.
\(\frac{2}{8}\) = \(\frac{□}{□}\)

Answer:
\(\frac{1}{4}\)

Explanation:
To write the 2/8in a simplest form
Divide the numerator and denominator of 2/8 with 2
(2 ÷2)/(8 ÷2) = 1/4
So, the simplest form of 2/8 = 1/4

Question 8.
\(\frac{4}{10}\) = \(\frac{□}{□}\)

Answer:
\(\frac{2}{5}\)

Explanation:
To write the 4/10 in a simplest form
Divide the numerator and denominator of 4 /10 with 2
(4 ÷2)/(10 ÷2) = 2/5
So, the simplest form of 4/10 = 2/5

Tell whether the fractions are equivalent. Write = or ≠. (if you dont have ≠on your keybord, copy and paste this one: ≠ )

Question 9.
\(\frac{6}{12}\) _______ \(\frac{1}{12}\)

Answer:
\(\frac{6}{12}\) ≠ \(\frac{1}{12}\)

Explanation:
6/12 ≠ 1/12

Question 10.
\(\frac{3}{4}\) _______ \(\frac{5}{6}\)

Answer:
\(\frac{3}{4}\) ≠ \(\frac{5}{6}\)

Explanation:
3/4 ≠ 5/6

Question 11.
\(\frac{6}{10}\) _______ \(\frac{3}{5}\)

Answer:
\(\frac{6}{10}\) = \(\frac{3}{5}\)

Explanation:
6/10
Divide the numerator and denominator of 6/10 with 2
(6 ÷ 2)/( 10 ÷ 2) = 3/5
So, 6/10 = 3/5

Question 12.
\(\frac{3}{12}\) _______ \(\frac{1}{3}\)

Answer:
\(\frac{3}{12}\) ≠ \(\frac{1}{3}\)

Explanation:
3/12 ≠ 1/3

Question 13.
\(\frac{6}{10}\) _______ \(\frac{60}{100}\)

Answer:
\(\frac{6}{10}\) = \(\frac{60}{100}\)

Explanation:
6/10
Multiply the numerator and denominator of 6/10 with 10
(6 x 10)/(10 x 10) = 60/100
So, 6/10 = 60/100

Question 14.
\(\frac{11}{12}\) _______ \(\frac{9}{10}\)

Answer:
\(\frac{11}{12}\) ≠ \(\frac{9}{10}\)

Explanation:
11/12 ≠ 9/10

Question 15.
\(\frac{2}{5}\) _______ \(\frac{8}{20}\)

Answer:
\(\frac{2}{5}\) = \(\frac{8}{20}\)

Explanation:
2/5
Multiply the numerator and denominator of 2/5 with 4
(2 x 4)/(5 x 4) = 8/20
So, 2/5 = 8/20

Question 16.
\(\frac{4}{8}\) _______ \(\frac{1}{2}\)

Answer:
\(\frac{4}{8}\) = \(\frac{1}{2}\)

Explanation:
4/8
Divide the numerator and denominator of 4/8 with 4
(4 x 4)/(8 x 4) = 1/2
So, 4/8 = 1/2

Question 17.
At Memorial Hospital, 9 of the 12 babies born on Tuesday were boys. In simplest form, what fraction of the babies born on Tuesday were boys?
_______

Answer:
\(\frac{3}{4}\)

Explanation:
As per the given data,
At memorial hospital, 9 of the 12 babies born on Tuesday were boys = 9/12
Divide the numerator and denominator of 9/12 with 3
(9 ÷ 3)/(12 ÷ 3) = 3/4
So, in the simplest form
3/4 of the babies born on Tuesday were boys

Question 18.
Cristina uses a ruler to measure the length of her math textbook. She says that the book is \(\frac{4}{10}\) meter long. Is her measurement in simplest form? If not, what is the length of the book in simplest form?
\(\frac{□}{□}\)

Answer:
\(\frac{2}{5}\)

Explanation:
As per the given data,
Cristiana uses a ruler to measure the length of her math textbook
She says that the book is 4/10meter long
It is not in simplest form
Divide the numerator and denominator of 4/10 with 2
(4÷ 2)/( 10 ÷ 2) = 2/5
The length of the book in the simplest form = 2/5

Common Core – Simplest Form – Page No. 344

Question 1.
Six out of the 12 members of the school choir are boys. In simplest form, what fraction of the choir is boys?
Options:
a. \(\frac{1}{6}\)
b. \(\frac{6}{12}\)
c. \(\frac{1}{2}\)
d. \(\frac{12}{6}\)

Answer:
c. \(\frac{1}{2}\)

Explanation:
As per the given data,
Six out of the 12 members of the school choir are boys = 6/12
To write the simplest form of 6/12, divide the numerator and denominator with 6
Then, (6 ÷ 6)/(12 ÷ 6) = 1/2
In simplest form, 1/2 of the choir is boys

Question 2.
Which of the following fractions is in simplest form?
Options:
a. \(\frac{5}{6}\)
b. \(\frac{6}{8}\)
c. \(\frac{8}{10}\)
d. \(\frac{2}{12}\)

Answer:
a. \(\frac{5}{6}\)

Explanation:
5/6 is in the simplest form
6/8 simplest form = 3/4
8/10 simplest form = 4/5
2/12 simplest form = 1/6

Question 3.
Each of the 23 students in Ms. Evans’ class raised $45 for the school by selling coupon books. How much money did the class raise in all?
Options:
a. $207
b. $225
c. $1,025
d. $1,035

Answer:
d. $1,035

Explanation:
As per the given data,
Each of the 23 students in Ms. Evan’s class raised $45 for the school by selling coupon books
= 23 x $45
= $1,035

Question 4.
Which pair of numbers below have 4 and 6 as common factors?
Options:
a. 12, 18
b. 20, 24
c. 28, 30
d. 36, 48

Answer:
d. 36, 48

Explanation:
36, 48
Here, 36 = 4 x 9
= 2 x 2 x 3 x 3
48 = 6 x 8
= 2 x 3 x 4 x 2

Question 5.
Bart uses \(\frac{3}{12}\) cup milk to make muffins. Which fraction is equivalent to \(\frac{3}{12}\)?
Options:
a. \(\frac{1}{4}\)
b. \(\frac{1}{3}\)
c. \(\frac{1}{2}\)
d. \(\frac{2}{3}\)

Answer:
a. \(\frac{1}{4}\)

Explanation:
As per the given data,
Bart uses 3/12 cup milk to make muffins
Divide the fraction with 3
(3 ÷ 3)/(12 ÷ 3) = 1/4
So, the equivalent fraction for 3/12 = 1/4

Question 6.
Ashley bought 4 packages of juice boxes. There are 6 juice boxes in each package. She gave 2 juice boxes to each of 3 friends. How many juice boxes does Ashley have left?
Options:
a. 24
b. 22
c. 18
d. 12

Answer:
c. 18

Explanation:
As per the given data,
Ashley bought 4 packages of juice boxes
There are 6 juice boxes in each package = 6 x 4 = 24
She gave 2 juice boxes to each of 3 friends = 2 x 3 = 6 juice boxes
So, 24 – 6 = 18
Total number of juice boxes left with Ashley = 18

Page No. 347

Question 1.
Find a common denominator for \(\frac{1}{3}\) and \(\frac{1}{12}\) by dividing each whole into the same number of equal parts. Use the models to help.
Go Math Grade 4 Answer Key Chapter 6 Fraction Equivalence and Comparison img 8
common denominator:

Answer:
common denominator: 12

Explanation:
List the multiples of 3 = 3, 6, 9, 12, 15, 18, 21, ….
List the multiples of 12 = 12, 24, 36, 48, ….
So, common denominators of 1/3 and 1/ 12 is 12

Write the pair of fractions as a pair of fractions with a common denominator.

Question 2.
\(\frac{1}{2}\) and \(\frac{1}{4}\)
Type below:
_________

Answer:
\(\frac{4}{8}\) and \(\frac{2}{8}\)

Explanation:
Common denominator of 1/2 and 1/4
List the multiples of 2 = 2, 4, 6, 8, 10, …
List the multiples of 4 = 4, 8, 12, 16, 20, 24, . . .
Then, the common denominator of 1/2 and 1/4 is 4
For the Common pair of fractions, multiply the common denominator with fractions
That is, (1 x 4) ÷( 2 x 4) and ( 1 x 4 ) ÷ ( 4 x 4)
So, the common pair of fractions = 4/8 and 2/8

Question 3.
\(\frac{3}{4}\) and \(\frac{5}{8}\)
Type below:
_________

Answer:
\(\frac{6}{8}\) and \(\frac{5}{8}\)

Explanation:
Common denominator of 3/4 and 5/8
List the multiples of 4 = 4, 8, 12, 16, 20, 24, . . .
List the multiples of 8 = 8, 16, 24, 32, . . . .
Then, the common denominator of 3/4 and 5/8 is 8
For the Common pair of fractions, multiply the common denominator with fractions
That is, (3 x 8) ÷( 4 x 8) and ( 5 x 8 ) ÷ ( 8 x 8)
So, the common pair of fractions = 6/8 and 5/8

Question 4.
\(\frac{1}{3}\) and \(\frac{1}{4}\)
Type below:
_________

Answer:
\(\frac{4}{12}\) and \(\frac{3}{12}\)

Explanation:
The common denominator of 1/3 and 1/4
List the multiples of 3 = 3, 6, 9, 12, 15, 18, 21, 24, ….
List the multiples of 4 = 4, 8, 12, 16, 20, 24, . . .
Then, the common denominator of 1 /3 and 1/4 is 12
For the Common pair of fractions, multiply the common denominator with fractions
That is, (1 x 12) ÷( 3 x 12) and ( 1 x 12 ) ÷ ( 4 x 12)
So, the common pair of fractions = 4/12 and 3/12

Question 5.
\(\frac{4}{12}\) and \(\frac{5}{8}\)
Type below:
_________

Answer:
\(\frac{8}{24}\) and \(\frac{15}{24}\)

Explanation:
Common denominator of 4/12 and 5/8
List the multiples of 12 = 12, 24, 36, 48, 60, …..
List the multiples of 8 = 8, 16, 24, 32, 40, 48, …
Then, the common denominator of 4/12 and 5/8 is 24
For the Common pair of fractions, multiply the common denominator with fractions
That is, (4 x 24) ÷( 12 x 24) and ( 5 x 24 ) ÷ ( 8 x 24)
So, the common pair of fractions = 8/24 and 15/24

Write the pair of fractions as a pair of fractions with a common denominator.

Question 6.
\(\frac{1}{4}\) and \(\frac{5}{6}\)
Type below:
_________

Answer:
\(\frac{3}{12}\) and \(\frac{10}{12}\)

Explanation:
The common denominator of 1/4 and 5/6
List the multiples of 4 = 4, 8, 12, 16, 20, 24, . . .
List the multiples of 6 = 6, 12, 18, 24, 30, 36, ….
Then, the common denominator of 1/4 and 5/6 is 12
For the Common pair of fractions, multiply the common denominator with fractions
That is, (1 x 12) ÷( 4 x 12) and ( 5 x 12 ) ÷ ( 6 x 12)
So, common pair of fractions = 3/12 and 10/12

Question 7.
\(\frac{3}{5}\) and \(\frac{4}{10}\)
Type below:
_________

Answer:
\(\frac{6}{10}\) and \(\frac{4}{10}\)

Explanation:
Common denominator of 3/5 and 4/10
List the multiples of 5 = 5, 10, 15, 20, 25, 30, …..
List the multiples of 10 = 10, 20, 30, 40, 50 ….
Then, the common denominator of 3/5 and 4/10 is 10
For the Common pair of fractions, multiply the common denominator with fractions
That is, (3 x 10) ÷( 5 x 10) and ( 4 x 10 ) ÷ ( 10 x 10)
So, the common pair of fractions = 6/10 and 4/10

Tell whether the fractions are equivalent. Write = or ≠.

Question 8.
\(\frac{3}{4}\) ______ \(\frac{1}{2}\)

Answer:
\(\frac{3}{4}\) ≠ \(\frac{1}{2}\)

Explanation:
3/4 ≠ 1/2

Question 9.
\(\frac{3}{4}\) ______ \(\frac{6}{8}\)

Answer:
\(\frac{3}{4}\) = \(\frac{6}{8}\)

Explanation:
3/4
Multiply the numerator and denominator of 3/4 with 2
(3 x 2) ÷ ( 4 x 2 ) = 6/8
So, 3/4 = 6/8

Question 10.
\(\frac{1}{2}\) ______ \(\frac{4}{8}\)

Answer:
\(\frac{1}{2}\) = \(\frac{4}{8}\)

Explanation:
1/2
Multiply the numerator and denominator of 1/2 with 4
(1 x 4) ÷ ( 2 x 4 ) = 4/8
So, 1/2 = 4/8

Question 11.
\(\frac{6}{8}\) ______ \(\frac{4}{8}\)

Answer:
\(\frac{6}{8}\) ≠ \(\frac{4}{8}\)

Explanation:
6/8 ≠ 4/8

Question 12.
Jerry has two same-size circles divided into the same number of equal parts. One circle has \(\frac{3}{4}\) of the parts shaded, and the other has \(\frac{2}{3}\) of the parts shaded. His sister says the least number of pieces each circle could be divided into is 7. Is his sister correct? Explain.
______

Answer:
As per the given data,
Jerry has two same size circles divided into the same number of equal parts
One circle has 3/4 of the parts shaded
So, non- shaded parts of one circle = 1 – 3/4 = 1/4
Another circle has 2/3 of the parts shaded
Non – shaded parts = 1 – 2/3 = 1/3
We can’t draw a conclusion that in how many parts or pieces a circle can be divided
So, his sister is incorrect

Page No. 348

Question 13.
Carrie has a red streamer that is \(\frac{3}{4}\) yard long and a blue streamer that is \(\frac{5}{6}\) yard long. She says the streamers are the same length. Does this make sense? Explain.
Go Math Grade 4 Answer Key Chapter 6 Fraction Equivalence and Comparison img 9
______

Answer:
Carrie has a red streamer that is 3/4 yard long
The blue streamer that is 5/6 yard long
3/4 ≠ 5/6
She says the streamers are the same length, it doesn’t make any sense.

Question 14.
Leah has two same-size rectangles divided into the same number of equal parts. One rectangle has \(\frac{1}{3}\) of the parts shaded, and the other has \(\frac{2}{5}\) of the parts shaded. What is the least number of parts into which both rectangles could be divided?
______ parts

Answer:
15 parts

Explanation:
As per the given data,
Leah has two same size rectangles divided into the same number of equal parts
One rectangle has 1/3 of the parts shaded
Other rectangle has 2/5 of the parts shaded
15 parts

Question 15.
Julian says a common denominator for \(\frac{3}{4}\) and \(\frac{2}{5}\) is 9. What is Julian’s error? Explain.
Type below:
___________

Answer:
As per the given data,
Julian says a common denominator for 3/4 and 2/5 is 9
To find the common denominator for 3/4 and 2/5
List the multiples of 4 = 4, 8, 12, 16, 20, 24, 28, 32, …..
List the multiples of 5 = 5, 10, 15, 20, 25, 30, ….
So, the common denominator for 3/4 and 2/5 is 20
Julian says 9 in place of 20 and it is wrong.

Question 16.
Miguel has two same-size rectangles divided into the same number of equal parts. One rectangle has \(\frac{3}{4}\) of the parts shaded, and the other has \(\frac{5}{8}\) of the parts shaded.
Into how many parts could each rectangle be divided? Show your work by sketching the rectangles.
Go Math Grade 4 Answer Key Chapter 6 Fraction Equivalence and Comparison img 10
______ parts

Answer:
chapter 6 - simplest form - image 1. jpg
8 parts

Explanation:
As per the given data,
Miguel has two same – size rectangles divided into the same number of equal parts.
One rectangle has 3/4 of the parts shaded.
Another has 5/8 of the parts shaded.
The possible parts are 8.

Common Core – Common Denominators – Page No. 349

Write the pair of fractions as a pair of fractions with a common denominator.

Question 1.
\(\frac{2}{3} \text { and } \frac{3}{4}\)
Go Math Grade 4 Answer Key Chapter 6 Fraction Equivalence and Comparison Common Core Common Denominators img 11

Answer:
\(\frac{8}{12} \text { and } \frac{9}{12}\)

Explanation:
2/3 and 3/4
List the multiples of 3 = 3, 6, 9, 12, 15, 18, 21, 24, …
List the multiples of 4 = 4, 8, 12, 16, 20, …
Common multiple of 3 and 4 is 12
For the Common pair of fractions, multiply the common denominator with fractions
That is, (2 x 12) ÷( 3 x 12) and ( 3 x 12 ) ÷ ( 4 x 12)
So, common pair of fractions = 8/12 and 9/12

Question 2.
\(\frac{1}{4} \text { and } \frac{2}{3}\)
Type below:
_________

Answer:
\(\frac{3}{12} \text { and } \frac{8}{12}\)

Explanation:
1/4 and 2/3
List the multiples of 4 = 4, 8, 12, 16, 20, …
List the multiples of 3 = 3, 6, 9, 12, 15, 18, …
Common multiple of 4 and 3 is 12
For the Common pair of fractions, multiply the common denominator with fractions
That is, (1 x 12) ÷( 4 x 12) and ( 2 x 12 ) ÷ ( 3 x 12)
So, common pair of fractions = 3/12 and 8/12

Question 3.
\(\frac{3}{10} \text { and } \frac{1}{2}\)
Type below:
_________

Answer:
\(\frac{3}{10} \text { and } \frac{5}{10}\)

Explanation:
3/10 and 1/2
List the multiples of 10 = 10, 20, 30, 40, 50, ….
List the multiples of 2 = 2, 4, 6, 8, 10, 12, 14, ….
Common multiple of 10 and 2 is 10
For the Common pair of fractions, multiply the common denominator with fractions
That is, (3 x 10) ÷( 10 x 10) and ( 1 x 10 ) ÷ ( 2 x 10)
So, common pair of fractions = 3/10 and 5/10

Question 4.
\(\frac{3}{5} \text { and } \frac{3}{4}\)
Type below:
_________

Answer:
\(\frac{12}{20} \text { and } \frac{15}{20}\)

Explanation:
3/5 and 3/4
List the multiples of 5 = 5, 10, 15, 20, 25, 30, ….
List the multiples of 4 = 4, 8, 12, 16, 20, 24, …
Common multiple of 5 and 4 is 20
For the Common pair of fractions, multiply the common denominator with fractions
That is, (3 x 20) ÷( 5 x 20) and ( 3 x 20 ) ÷ ( 4 x 20)
So, common pair of fractions = 12/20 and 15/20

Question 5.
\(\frac{2}{4} \text { and } \frac{7}{8}\)
Type below:
_________

Answer:
\(\frac{4}{8} \text { and } \frac{7}{8}\)

Explanation:
2/4 and 7/8
List the multiples of 4 = 4, 8, 12, 16, 20, 24, …
List the multiples of 8 = 8, 16, 24, 32, 40, ….
Common multiple of 4 and 8 is 8
For the Common pair of fractions, multiply the common denominator with fractions
That is, (2 x 8) ÷( 4 x 8) and ( 7 x 8 ) ÷ ( 8 x 8)
So, common pair of fractions = 4/8 and 7/8

Question 6.
\(\frac{2}{3} \text { and } \frac{5}{12}\)
Type below:
_________

Answer:
\(\frac{8}{12} \text { and } \frac{5}{12}\)

Explanation:
2/3 and 5/12
List the multiples of 3 = 3, 6, 9, 12, 15, 18, …
List the multiples of 12 = 12, 24, 36, 48, 60, …
Common multiple of 3 and 12 is 12
For the Common pair of fractions, multiply the common denominator with fractions
That is, (2 x 12) ÷( 3 x 12) and ( 5 x 12 ) ÷ ( 12 x 12)
So, common pair of fractions = 8/12 and 5/12

Question 7.
\(\frac{1}{4} \text { and } \frac{1}{6}\)
Type below:
_________

Answer:
\(\frac{3}{12} \text { and } \frac{2}{12}\)

Explanation:
1/4 and 1/6
List the multiples of 4 = 4, 8, 12, 16, 20, 24, …
List the multiples of 6 = 6, 12, 18, 24, 30, …
Common multiple of 4 and 6 is 12
For the Common pair of fractions, multiply the common denominator with fractions
That is, (1 x 12) ÷( 4 x 12) and ( 1 x 12 ) ÷ ( 6 x 12)
So, common pair of fractions = 3/12 and 2/12

Tell whether the fractions are equivalent. Write = or ≠.

Question 8.
\(\frac{1}{2}\) ______ \(\frac{2}{5}\)

Answer:
\(\frac{1}{2}\) ≠ \(\frac{2}{5}\)

Explanation:
Multiply the numerator and denominator of 1/2 with 2
(1 x 2) ÷ (2 x 2) = 2/4
So, 1/2 ≠ 2/5

Question 9.
\(\frac{1}{2}\) ______ \(\frac{3}{6}\)

Answer:
\(\frac{1}{2}\) = \(\frac{3}{6}\)

Explanation:
1/2
Multiply the numerator and denominator of 1/2 with 3
(1 x 3) ÷ (2 x 3) = 3/6
So, 1/2 = 3/6

Question 10.
\(\frac{3}{4}\) ______ \(\frac{5}{6}\)

Answer:
\(\frac{3}{4}\) ≠ \(\frac{5}{6}\)

Explanation:
3/4 ≠ 5/6

Question 11.
\(\frac{6}{10}\) ______ \(\frac{3}{5}\)

Answer:
\(\frac{6}{10}\) = \(\frac{3}{5}\)

Explanation:
6/10
Divide the numerator and denominator of 6/10 with 2
(6 ÷ 2)/(10 ÷2) = 3/5
So, 6/10 = 3/5

Question 12.
\(\frac{6}{8}\) ______ \(\frac{3}{4}\)

Answer:
\(\frac{6}{8}\) = \(\frac{3}{4}\)

Explanation:
6/8
Divide the numerator and denominator of 6/8 with 2
(6 ÷2)/(8 ÷2) = 3/4
So, 6/8 = 3/4

Question 13.
\(\frac{3}{4}\) ______ \(\frac{2}{3}\)

Answer:
\(\frac{3}{4}\) ≠ \(\frac{2}{3}\)

Explanation:
3/4 ≠ 2/3

Question 14.
\(\frac{2}{10}\) ______ \(\frac{4}{5}\)

Answer:
\(\frac{2}{10}\) ≠ \(\frac{4}{5}\)

Explanation:
2/10
Divide the numerator and denominator of 2/10 with 2
(2 ÷ 2)/(10 ÷ 2) = 1/5
So, 2/10 ≠ 1/5

Question 15.
\(\frac{1}{4}\) ______ \(\frac{3}{12}\)

Answer:
\(\frac{1}{4}\) = \(\frac{3}{12}\)

Explanation:
1/4
Multiply the numerator and denominator of 1/4 with 3
(1 x 3)/(4 x 3) = 3/12
So, 1/4 = 3/12

Question 16.
Adam drew two same size rectangles and divided them into the same number of equal parts. He shaded \(\frac{1}{3}\) of one rectangle and \(\frac{1}{4}\) of other rectangle. What is the least number of parts into which both rectangles could be divided?
_________

Answer:
12 parts

Explanation:
As per the given data,
Adam drew two same size rectangles and divided them into the same number of equal parts
He shaded 1/3 of one rectangle
1/4 of another rectangle
List the multiples of 3 = 3, 6, 9, 12, 15, 18, …
List the multiples of 4 = 4, 8, 12, 16, 20, …
A common multiple of 3 and 4 is 12
So, the least number of parts which rectangles could be divided = 12 parts

Question 17.
Mera painted equal sections of her bedroom wall to make a pattern. She painted \(\frac{2}{5}\) of the wall white and \(\frac{1}{2}\) of the wall lavender. Write an equivalent fraction for each using a common denominator.
Type below:
_________

Answer:
1/2 are 4/10 and 5/10

Explanation:
As per the given data,
Mera painted equal sections of her bedroom wall to make a pattern
She painted 2/5 of the wall white and 1/2 of the wall lavender
List the multiples of 5 = 5, 10, 15, 20, 25, 30, …
List the multiples of 2 = 2 ,4, 6, 8, 10, 12, 14, …
The common denominator of 2/5 and 1/2 = 10
Multiply the 2/5 and 1/2 with 10
(2 x 10)/(5 x 10) and (1 x 10)/(2 x 10)
4/10 and 5/10
So, common fractions of 2/5 and 1/2 are 4/10 and 5/10

Common Core – Common Denominators – Page No. 350

Question 1.
Which of the following is a common denominator of \(\frac{1}{4}\) and \(\frac{5}{6}\)?
Options:
a. 8
b. 9
c. 12
d. 15

Answer:
c. 12

Explanation:
Common denominator of 1/4 and 5/6
List the multiples of 4 = 4, 8, 12, 16, 20, 24, …
List the multiples of 6 = 6, 12, 18, 24, 30, ….
So, the common denominator of 1/4 and 5/6 is 12

Question 2.
Two fractions have a common denominator of 8. Which of the following could be the two fractions?
Options:
a. \(\frac{1}{2} \text { and } \frac{2}{3}\)
b. \(\frac{1}{4} \text { and } \frac{1}{2}\)
c. \(\frac{3}{4} \text { and } \frac{1}{6}\)
d. \(\frac{1}{2} \text { and } \frac{4}{5}\)

Answer:
b. \(\frac{1}{4} \text { and } \frac{1}{2}\)

Explanation:
As per the given data,
Two fractions have a common denominator of 8
a. 1/2 and 2/3
List the multiples of 2 = 2, 4, 6, 8,10, ….
List the multiples of 3 = 3, 6, 9, 12, …
There is no common denominator of 8 for 1/2 and 2/3
b. 1/4 and 1 /2
List the multiples of 2 = 2, 4, 6, 8,10, ….
List the multiples of 4 = 4, 8, 12, 16, …
Here, the common denominator of 1 /4 and 1 /2 is 8
So, the answer is 1/4 and 1/2

Question 3.
Which number is 100,000 more than seven hundred two thousand, eighty-three?
Options:
a. 703,083
b. 712,083
c. 730,083
d. 802,083

Answer:
d. 802,083

Explanation:
802,083

Question 4.
Aiden baked 8 dozen muffins. How many total muffins did he bake?
Options:
a. 64
b. 80
c. 96
d. 104

Answer:
c. 96

Explanation:
As per the given data,
Aiden baked 8 dozen muffins
1 dozen = 12
then, 8 dozens = 12 x 8 = 96
So, Aiden baked totally 96 muffins

Question 5.
On a bulletin board, the principal, Ms. Gomez, put 115 photos of the fourthgrade students in her school. She put the photos in 5 equal rows. How many photos did she put in each row?
Options:
a. 21
b. 23
c. 25
d. 32

Answer:
b. 23

Explanation:
As per the given data,
On a bulletin board, the principal, Ms. Gomez, put 115 photos of the fourth-grade students in her school
She put the photos in 5 equal rows
Then, number of photos in each row = 115/5 = 23
So, Ms. Gomez put photos in each row = 23

Question 6.
Judy uses 12 tiles to make a mosaic. Eight of the tiles are blue. What fraction, in simplest form, represents the tiles that are blue?
Options:
a. \(\frac{2}{3}\)
b. \(\frac{2}{5}\)
c. \(\frac{3}{4}\)
d. \(\frac{12}{18}\)

Answer:
a. \(\frac{2}{3}\)

Explanation:
As per the given data,
Judy uses 12 tiles to make a mosaic
Eight of the tiles are blue = 8/12
Divide the numerator and denominator of 8/12 with 4
(8 ÷ 4)/(12 ÷ 4) = 2/3
The simplest form of 8/12 is 2/3

Page No. 353

Question 1.
Keisha is helping plan a race route for a 10-kilometer charity run. The committee wants to set up the following things along the course.
Viewing areas: At the end of each half of the course
Water stations: At the end of each fifth of the course
Distance markers: At the end of each tenth of the course
Which locations have more than one thing located there?
First, make a table to organize the information.
Go Math Grade 4 Answer Key Chapter 6 Fraction Equivalence and Comparison img 12
Next, identify a relationship. Use a common denominator, and find equivalent fractions.
Finally, identify the locations at which more than one thing will be set up. Circle the locations.
Type below:
___________

Answer:
Keisha is helping plan a race route for a 10-kilometer charity run.

Question 2.
What if distance markers will also be placed at the end of every fourth of the course? Will any of those markers be set up at the same location as another distance marker, a water station, or a viewing area? Explain.
Type below:
___________

Answer:
It really depends on where you place the other markers.

Question 3.
Fifty-six students signed up to volunteer for the race. There were 4 equal groups of students, and each group had a different task.
How many students were in each group?
_____ students

Answer:
14 students

Explanation:
As per the given data,
Fifty-six students signed up to volunteer for the race
There are four groups of students
Number of students in each group = 56/4 = 14
Total number of students in each group = 14

Page No. 354

Question 4.
A baker cut a pie in half. He cut each half into 3 equal pieces and each piece into 2 equal slices. He sold 6 slices. What fraction of the pie did the baker sell?
\(\frac{□}{□}\)

Answer:
\(\frac{1}{2}\)

Explanation:
A baker cut a pie in half. He cut each half into 3 equal pieces and each piece into 2 equal slices. He sold 6 slices. So, the remaining part is 1/2 of the pie.

Question 5.
Andy cut a tuna sandwich and a chicken sandwich into a total of 15 same-size pieces. He cut the tuna sandwich into 9 more pieces than the chicken sandwich. Andy ate 8 pieces of the tuna sandwich. What fraction of the tuna sandwich did he eat?
\(\frac{□}{□}\)

Answer:
\(\frac{2}{3}\)

Explanation:
Let x be the number of pieces of the chicken sandwich so that x + 9 is the number of pieces of a tuna sandwich.
There is a total of 15 same-size pieces. So, we can write as
x + (x + 9) = 15
2x + 9 = 15
2x = 6
x = 3.
This means that there ate 3 + 9 = 12 pieces of a tuna sandwich. Since Andy ate 8, then this corresponds to a fraction of 8/12 = 2/3.

Question 6.
Luke threw balls into these buckets at a carnival. The number on the bucket gives the number of points for each throw. What is the least number of throws needed to score exactly 100 points? Explain.
Go Math Grade 4 Answer Key Chapter 6 Fraction Equivalence and Comparison img 13
_____ throws

Answer:
13 throws

Explanation:
Take the maximum number to get the minimum throws = 9 X 10 = 90.
6 X 1 = 6; 2 X 2 = 4.
Add 90 + 6 + 4 = 100;
So, the least number of throws needed to score exactly 100 points = 10 + 1 + 2 = 13.

Question 7.
Victoria arranges flowers in vases at her restaurant. In each arrangement, \(\frac{2}{3}\) of the flowers are yellow. What other fractions can represent the part of the flowers that are yellow? Shade the models to show your work.
Go Math Grade 4 Answer Key Chapter 6 Fraction Equivalence and Comparison img 14
\(\frac{□}{□}\)

Answer:
chapter 6
\(\frac{2}{3}\), \(\frac{8}{12}\), \(\frac{40}{60}\)

Explanation:
Basically, any fraction obtained by multiplying both the numerator and denominator by the same value would be an equivalent fraction:
2/3 = 2/3 * 4/4 = 8/12
8/12 = 8/12 * 5/5 = 40/60
etc.

Common Core – Find Equivalent Fractions – Page No. 355

Question 1.
Miranda is braiding her hair. Then she will attach beads to the braid. She wants \(\frac{1}{3}\) of the beads to be red. If the greatest number of beads that will fit on the braid is 12, what other fractions could represent the part of the beads that are red?
Go Math Grade 4 Answer Key Chapter 6 Fraction Equivalence and Comparison Common Core Find Equivalent Fractions img 15

Answer:
\(\frac{2}{6}\), \(\frac{3}{9}\), \(\frac{4}{12}\)

Explanation:
Miranda is braiding her hair. Then she will attach beads to the braid. She wants \(\frac{1}{3}\) of the beads to be red. If the greatest number of beads that will fit on the braid is 12.
\(\frac{1}{3}\) X \(\frac{2}{2}\) = \(\frac{2}{6}\)
\(\frac{1}{3}\) X \(\frac{3}{3}\) = \(\frac{3}{9}\)
\(\frac{1}{3}\) X \(\frac{4}{4}\) = \(\frac{4}{12}\)

Question 2.
Ms. Groves has trays of paints for students in her art class. Each tray has 5 colors. One of the colors is purple. What fraction of the colors in 20 trays is purple?
\(\frac{□}{□}\)

Answer:
\(\frac{20}{100}\) or \(\frac{1}{5}\)

Explanation:
If you have 20 trays that are 100 colors with 20 being purple. 20/ 100 is 1/5

Question 3.
Miguel is making an obstacle course for field day. At the end of every sixth of the course, there is a tire. At the end of every third of the course, there is a cone. At the end of every half of the course, there is a hurdle. At which locations of the course will people need to go through more than one obstacle?
Type below:
_________

Answer:
\(\frac{1}{3}\), \(\frac{1}{2}\), \(\frac{2}{3}\) and final locations

Explanation:
We have three fractions with different denominators: sixths, thirds, and halves.
The first step is to make all the denominators equal for 1/6, 1/3, 1/2.
In this case, we want sixths since LCM(2, 3, 6) = 6
since 1/3 = 2/6, and 1/2 = 3/6. Now we can start solving.
1. There are six tires at the following: 1/6, 2/6, 3/6, 4/6, 5/6, and 6/6.
2. There are three cones at the following (G.C.F.): 2/6 (or 1/3), 4/6 (or 2/3), and 6/6 (or 3/3).
3. There are two hurdles at the following (G.C.F.): 3/6 (or 1/2) and 6/6 (or 2/2).
We look for common numbers.
1. On 2/6, there are two obstacles: a tire and a cone.
2. On 3/6, there are two obstacles: a tire and a hurdle.
3. On 4/6, there are two obstacles: a tire and a cone.
4. At 6/6, there are three obstacles: a tire, cone, and a hurdle.
2/6 = 1/3
3/6 = 1/2
4/6 = 2/3
6/6 = 1
The answers are 1/3, 1/2, 2/3, and 1.

Question 4.
Preston works in a bakery where he puts muffins in boxes. He makes the following table to remind himself how many blueberry muffins should go in each box.
Go Math Grade 4 Answer Key Chapter 6 Fraction Equivalence and Comparison Common Core Find Equivalent Fractions img 16
How many blueberry muffins should Preston put in a box with 36 muffins?
_________

Answer:
12 blueberry muffins

Explanation:
Preston works in a bakery where he puts muffins in boxes. He makes the following table to remind himself how many blueberry muffins should go in each box.
So, he had 2 blueberry muffins out of 6 muffins.
2/6 X 2/2 = 4/12. 4 blueberry muffins out of 12 muffins.
2/6 X 4/4 = 8/24. 8 blueberry muffins out of 24 muffins.
2/6 X 6/6 = 12/36. 12 blueberry muffins out of 36 muffins.

Common Core – Find Equivalent Fractions – Page No. 356

Question 1.
A used bookstore will trade 2 of its books for 3 of yours. If Val brings in 18 books to trade, how many books can she get from the store?
Options:
a. 9
b. 12
c. 18
d. 27

Answer:
b. 12

Explanation:
A used bookstore will trade 2 of its books for 3 of yours. If Val brings in 18 books to trade 2/3 X 6/6 = 12/18, she get 12 books

Question 2.
Every \(\frac{1}{2}\) hour Naomi stretches her neck; every \(\frac{1}{3}\) hour she stretches her legs; and every \(\frac{1}{6}\) hour she stretches her arms. Which parts of her body will Naomi stretch when \(\frac{2}{3}\) of an hour has passed?
Options:
a. neck and legs
b. neck and arms
c. legs and arms
d. none

Answer:
c. legs and arms

Explanation:
Summing \(\frac{1}{2}\)‘s only gives integer values giving 1, 2, 3, 4…or
integer values +\(\frac{1}{2}\) and 0 + \(\frac{1}{2}\) = \(\frac{1}{2}\), 1 \(\frac{1}{2}\), 2 \(\frac{1}{2}\)…
So neck is excluded
Every \(\frac{1}{3}\): \(\frac{1}{3}\) + \(\frac{1}{2}\) = \(\frac{2}{3}\)
Legs will be stretched at \(\frac{2}{3}\) hour
Every \(\frac{1}{6}\): \(\frac{1}{6}\) + \(\frac{1}{6}\) + \(\frac{1}{6}\) + \(\frac{1}{6}\) = \(\frac{4}{6}\)
Divide top and bottom by 2 giving:
(4 ÷ 2)/(6 ÷ 2) = \(\frac{2}{3}\)
Arms will be stretched at latex]\frac{2}{3}[/latex] hour

Question 3.
At the beginning of the year, the Wong family car had been driven 14,539 miles. At the end of the year, their car had been driven 21,844 miles. How many miles did the Wong family drive their car during that year?
Options:
a. 6,315 miles
b. 7,295 miles
c. 7,305 miles
d. 36,383 miles

Answer:
c. 7,305 miles

Explanation:
If at the beginning of the year, the Wong family’s car had driven 14539 miles and at the end of the year, it had driven 21844 miles, then subtract 14539 from 21844 to determine the difference between the two values, which will tell you how many miles the Wong family drove their car for during the year.
21844 – 14539 = 7305 miles

Question 4.
Widget Company made 3,600 widgets in 4 hours. They made the same number of widgets each hour. How many widgets did the company make in one hour?
Options:
a. 80
b. 90
c. 800
d. 900

Answer:
d. 900

Explanation:

3,600 widgets in 4 hours therefore 3,600 / 4 for one hour = 900 widgets 900 widgets in one hour.

Question 5.
Tyler is thinking of a number that is divisible by 2 and by 3. By which of the following numbers must Tyler’s number also be divisible?
Options:
a. 6
b. 8
c. 9
d. 12

Answer:
a. 6

Explanation:
The number 6 is divisible by 2 and by 3.

Question 6.
Jessica drew a circle divided into 8 equal parts. She shaded 6 of the parts. Which fraction is equivalent to the part of the circle that is shaded?
Options:
a. \(\frac{2}{3}\)
b. \(\frac{3}{4}\)
c. \(\frac{10}{16}\)
d. \(\frac{12}{18}\)

Answer:
b. \(\frac{3}{4}\)

Explanation:
Jessica drew a circle divided into 8 equal parts. She shaded 6 of the parts.
6/8 = 3/4

Page No. 357

Choose the best term from the box.
Go Math Grade 4 Answer Key Chapter 6 Fraction Equivalence and Comparison img 17

Question 1.
________ name the same amount.
________

Answer:
Equivalent Fractions

Question 2.
A _________ is a common multiple of two or more denominators
________

Answer:
Common Denominator

Write two equivalent fractions.

Question 3.
\(\frac{2}{5}\)
Type below:
________

Answer:
\(\frac{4}{10}\) and \(\frac{6}{15}\)

Explanation:
Two equivalent fractions of 2/5
Multiply the 2/5 with 2
(2 x 2)/(5 x 2) = 4/10
Multiply the 2/5 with 3
(2 x 3)/(5 x 3) = 6/15
So, the equivalent fractions of 2/5 are 4/10 and 6/15

Question 4.
\(\frac{1}{3}\)
Type below:
________

Answer:
\(\frac{2}{6}\) and \(\frac{3}{9}\)

Explanation:
Two equivalent fractions of 1/3
Multiply the 1/3 with 2
(1 x 2)/(3 x 2) = 2/6
Multiply the 1/3 with 3
(1 x 3)/(3 x 3) = 3/9
So, the equivalent fractions of 1/3 are 2/6 and 3/9

Question 5.
\(\frac{3}{4}\)
Type below:
________

Answer:
\(\frac{6}{8}\) and \(\frac{9}{12}\)

Explanation:
Two equivalent fractions of 3/4
Multiply the 3/4 with 2
(3 x 2)/(4 x 2) = 6/8
Multiply the 3/4 with 3
(3 x 3)/(4 x 3) = 9/12
So, the equivalent fractions of 3/4 are 6/8 and 9/12

Tell whether the fractions are equivalent. Write = or ≠.

Question 6.
\(\frac{2}{3}\) ______ \(\frac{4}{12}\)

Answer:
\(\frac{2}{3}\) ≠ \(\frac{4}{12}\)

Explanation:
2/ 3
Multiply the numerator and denominator of 2/3 with 2
(2 x 2)/(3 x 2) = 4/6
So, 2/3 ≠ 4/12

Question 7.
\(\frac{5}{6}\) ______ \(\frac{10}{12}\)

Answer:
\(\frac{5}{6}\) =_ \(\frac{10}{12}\)

Explanation:
5/6
Multiply the 5/6 with 2
(5 x 2)/(6 x 2) = 10/12
So, 5/6 = 10/12

Question 8.
\(\frac{1}{4}\) ______ \(\frac{4}{8}\)

Answer:
\(\frac{1}{4}\) ≠ \(\frac{4}{8}\)

Explanation:
1/4
Multiply the numerator and denominator of 1/4 with 4
(1 x 4)/(4 x 4) = 4/16
So, 1/4 ≠ 4/8

Write the fraction in simplest form.

Question 9.
\(\frac{6}{8}\)
\(\frac{□}{□}\)

Answer:
\(\frac{3}{4}\)

Explanation:
6/8
Divide the numerator and denominator of 6/8 with 2
(6 ÷ 2)/( 8 ÷ 2) = 3/4
The simplest form of 6/8 is 3/4

Question 10.
\(\frac{25}{100}\)
\(\frac{□}{□}\)

Answer:
\(\frac{1}{4}\)

Explanation:
25/100
Divide the numerator and denominator of 25/100 with 25
(25 ÷ 25)/( 100 ÷ 25) = 1/4
The simplest form of 25/100 is 1/4

Question 11.
\(\frac{8}{10}\)
\(\frac{□}{□}\)

Answer:
\(\frac{4}{5}\)

Explanation:
8/10
Divide the numerator and denominator of 8/10 with 2
(8 ÷ 2)/( 10 ÷ 2) = 4/5
The simplest form of 8/10 is 4/5

Write the pair of fractions as a pair of fractions with a common denominator.

Question 12.
\(\frac{3}{10} \text { and } \frac{2}{5}\)
Type below:
_________

Answer:
\(\frac{3}{10} \text { and } \frac{4}{10}\)

Explanation:
3/ 10 and 2/5
List the multiples of 10 = 10, 20, 30, 40, 50, …
List the multiples of 5 = 5, 10, 15, 20, 25, 30, …
Common denominator of 3/10 and 2/5 = 10
Multiply the 3/10 and 2/5 with 10
(3 x 10)/(10 x 10) and (2 x 10)/(5 x 10)
3/ 10 and 4/10
Pair of fractions of 3/10 and 2/5 are 3/10 and 4/10

Question 13.
\(\frac{1}{3} \text { and } \frac{3}{4}\)
Type below:
_________

Answer:
\(\frac{3}{12} \text { and } \frac{9}{12}\)

Explanation:
1/3 and 3/4
List the multiples of 3 = 3, 6, 9, 12, 15, 18, …
List the multiples of 4 = 4, 8, 12, 16, 20, ….
The common denominator of 1/3 and 3/4 are 12
Multiply the 1/3 and 3/4 with 12
(1 x 12)/(3 x 12) and (3 x 12)/(4 x 12)
3/ 12 and 9/12.
Pair of fractions of 1/3 and 3/4 are 3/12 and 9/12

Page No. 358

Question 14.
Sam needs \(\frac{5}{6}\) cup mashed bananas and \(\frac{3}{4}\) cup mashed strawberries for a recipe. He wants to find whether he needs more bananas or more strawberries. How can he write \(\frac{5}{6}\) and \(\frac{3}{4}\) as a pair of fractions with a common denominator?
Type below:
_________

Answer:
\(\frac{10}{12}\) and \(\frac{9}{12}\)

Explanation:
Sam needs 5/6 cup mashed bananas and 3/4 cup mashed strawberries for a recipe
He wants to find whether he needs more bananas or strawberries
List the multiples of 6 = 6, 12, 18, 24, 30, 36, 42,…..
List the multiples of 4 = 4, 8, 12, 16, 20, 24, ….
The common denominator of 6 and 4 is 12
Multiply the numerator and denominator of 5/6 and 3/4 with 12
(5 x 12)/(6 x 12) and (3 x 12)/(4 x 12)
10/12 and 9/12
Pair of fractions with a common denominator for 5/6 and 3/4 are 10/12 and 9/12

Question 15.
Karen will divide her garden into equal parts. She will plant corn in \(\frac{8}{12}\) of the garden. What is the fewest number of parts she can divide her garden into?
______ parts

Answer:
\(\frac{2}{3}\) parts

Explanation:
As per the given data,
Keren will divide her garden into equal parts
She will plant corn in 8/12 of the garden
To get the least number of parts she can divide her garden, simplify the 8/12
Divide the numerator and denominator of 8/12 with 4
(8 ÷ 4)/(12 ÷ 4) = 2/3
So, Karen can divide her garden into 2/3 of parts

Question 16.
Olivia is making scarves. Each scarf will have 5 rectangles, and \(\frac{2}{5}\) of the rectangles will be purple. How many purple rectangles does she need for 3 scarves?
______ purple rectangles

Answer:
6 purple rectangles

Explanation:
As per the given data,
Olivia is making scarves
Each scarf will have 5 rectangles and 2/5 of the rectangles will be purple = 5 x 2/5 = 2
That means each scarf will have 2 purple rectangles
For 3 scarves = 3 x 2 = 6
So, she needs 6 purple rectangles.

Question 17.
Paul needs to buy \(\frac{5}{8}\) pound of peanuts. The scale at the store measures parts of a pound in sixteenths. What measure is equivalent to \(\frac{5}{8}\) pound?
\(\frac{□}{□}\) pound of peanuts

Answer:
\(\frac{10}{16}\) pound of peanuts

Explanation:
As per the given data,
Paul needs to buy 5/8 pounds of peanuts
The scale at the store measures parts of a pound in sixteenths = 16 x 5/8 = 10
To find Equivalent fraction of 5/8
Multiply the numerator and denominator of 5/8 with 2
(5 x 2)/( 8 x 2) = 10/16
So, the equivalent fraction of 5/8 is 10/16

Page No. 361

Question 1.
Compare \(\frac{2}{5}\) and \(\frac{1}{8}\). Write < or >.
Go Math Grade 4 Answer Key Chapter 6 Fraction Equivalence and Comparison img 18
\(\frac{2}{5}\) _____ \(\frac{1}{8}\)

Answer:
\(\frac{2}{5}\) > \(\frac{1}{8}\)

Explanation:
Least common denominator of 5 and 8 = 40
Multiply the numerator and denominator of 2/5 and 1/8 with 40
2/ 5 = (2 x 8)/(5 x 8) = 16/40
1/8 = (1 x 5)/(8 x 5) = 5/40
The denominators are same now
So, compare the numerator to find the greater number
16/40 > 5/40
So, 2/5 > 1/8

Compare. Write < or >.

Question 2.
\(\frac{1}{2}\) _____ \(\frac{4}{6}\)

Answer:
\(\frac{1}{2}\) < \(\frac{4}{6}\)

Explanation:
1/2 and 4/6
Least common denominator of 2 and 6 = 6
Multiply the numerator and denominator of 1/2 and 4/6 with 6
1/ 2 = (1 x 6)/(2 x 6) = 6/12
4/ 6 = (4x 2)/(6 x 2) = 8/12
The denominators are same now
So, compare the numerator to find the greater number.
6/12 < 8/12
So, 1/2 < 4/6

Question 3.
\(\frac{3}{10}\) _____ \(\frac{1}{2}\)

Answer:
\(\frac{3}{10}\) > \(\frac{1}{2}\)

Explanation:
1 / 10 and 1/2
Least common denominator of 10 and 2 = 10
Multiply the numerator and denominator of 3/10 and 1/2 with 10
3/ 10 = (3 x 2)/(10 x 2) = 6/20
1/2 = (1 x 10)/(2 x 10) = 10/20
The denominators are same now
So, compare the numerator to find the greater number.
6/20 < 10/20
So, 3/10 > 1/2

Question 4.
\(\frac{11}{12}\) _____ \(\frac{4}{8}\)

Answer:
\(\frac{11}{12}\) > \(\frac{4}{8}\)

Explanation:
11/12 and 4/8
Least common denominator of 12 and 8 = 24
Multiply the numerator and denominator of 11/12 and 4/8 with 24
11/ 12 = (11 x 8)/(12 x 8) = 88/96
4/8 = (4 x 12)/(8 x 12) = 48/96
The denominators are same now
So, compare the numerator to find the greater number
88/96 > 48/96
So, 11/12 > 4/8

Question 5.
\(\frac{5}{8}\) _____ \(\frac{2}{5}\)

Answer:
\(\frac{5}{8}\) > \(\frac{2}{5}\)

Explanation:
5/ 8 and 2/5
Least common denominator of 5 and 8 = 40
Multiply the numerator and denominator of 5/8 and 2/8 with 40
5/ 8 = (5 x 5)/(8 x 5) = 25/40
2/5 = (2 x 8)/(5 x 8) = 16/40
The denominators are same now
So, compare the numerator to find the greater number
25/ 40 > 16/40
So, 5/8 > 2/5

Question 6.
\(\frac{8}{10}\) _____ \(\frac{3}{8}\)

Answer:
\(\frac{8}{10}\) > \(\frac{3}{8}\)

Explanation:
8/10 and 3/8
Least common denominator of 10 and 8 = 40
Multiply the numerator and denominator of 8/10 and 3/8 with 40
8/ 10 = (8 x 8)/(10 x 8) = 64/80
3/8 = (3 x 10)/(8 x 10) = 30/80
The denominators are same now
So, compare the numerator to find the greater number
64/80 > 30/80
So, 8/10 > 3/8

Question 7.
\(\frac{1}{3}\) _____ \(\frac{7}{12}\)

Answer:
\(\frac{1}{3}\) < \(\frac{7}{12}\)

Explanation:
1/3 and 7/12
Least common denominator of 3 and 12 = 12
Multiply the numerator and denominator of 1/3 and 7/12 with 40.
1/ 3 = (1 x 12)/(3 x 12) = 12/36
7/12 = (7 x 3)/(12 x 3) = 21/36
The denominators are same now
So, compare the numerator to find the greater number
12/36 < 21/36
So, 1/3 < 7/12

Question 8.
\(\frac{2}{6}\) _____ \(\frac{7}{8}\)

Answer:
\(\frac{2}{6}\) < \(\frac{7}{8}\)

Explanation:
2/6 and 7/8
Least common denominator of 6 and 8 = 24
Multiply the numerator and denominator of 2/6 and 7/8 with 40
2/ 6 = (2 x 8)/(6 x 8) = 16/48
7/8 = (7 x 6)/(8 x 6) = 42/48
The denominators are same now
So, compare the numerator to find the greater number
16/48<42/48
So, 2/6 < 7/8

Question 9.
\(\frac{4}{8}\) _____ \(\frac{2}{10}\)

Answer:
\(\frac{4}{8}\) > \(\frac{2}{10}\)

Explanation:
4/8 and 2/10
Least common denominator of 8 and 10 = 40
Multiply the numerator and denominator of 4/8 and 2/10 with 40
4/ 8 = (4 x 10)/(8 x 10) = 40/80
2/10 = (2 x 8)/(10 x 8) = 16/80
The denominators are same now
So, compare the numerator to find the greater number
40/80 > 16/80
So, 4/8 > 2/10

Reason Quantitatively Algebra Find a numerator that makes the statement true.

Question 10.
\(\frac{2}{4}<\frac { □ }{ 6 } \)
□ = _____

Answer:
4

Explanation:
2/4 < x/6
Least common denominator of 4 and 6 = 12
Multiply the numerator and denominator of 2/4 < x/6 with 40
2/4 = (2 x 6)/(4 x 6) = 12/24
x/6 = (x x 4)/(6 x 4) = 4 x/24
The denominators are same now
So, compare the numerator to find the greater number
12/24 < 4 X 4/24

Question 11.
\(\frac{8}{10}>\frac { □ }{ 8 } \)
□ = _____

Answer:
1

Explanation:
8/10 < x/8
Least common denominator of 10 and 8 = 40
8/10 = (8 x 4)/(10 x 4) = 32/40
x/8 = (x X 5)/(8 x 5) = 5x/40
The denominators are same now
So, compare the numerator to find the greater number
8/10 < 5x/40. X will be 1

Question 12.
\(\frac{10}{12}>\frac { □ }{ 4 } \)
□ = _____

Answer:
1

Explanation:
10/12 < x/4
Least common denominator of 12 and 4 = 12
10/12 = (10 x 1)/(12 x 1) = 10/12
x/4 = (x X 3)/(4 x 3) = 3x/12
The denominators are same now
So, compare the numerator to find the greater number
10/12 < 3/12. X will be 1.

Question 13.
\(\frac{2}{5}<\frac { □ }{ 10 } \)
□ = _____

Answer:
5

Explanation:
2/5 < x/10
Least common denominator of 5 and 10 = 10
2/5 = (2x 2)/(5 x 2) = 4/10
x/10 = (x X 1)/(10 x 1) = x/10
The denominators are same now
So, compare the numerator to find the greater number
2/5 < 5/10. X will be 5.

Question 14.
When two fractions are between 0 and \(\frac{1}{2}\), how do you know which fraction is greater? Explain.
Type below:
_______

Answer:
When two fractions are between 0 and \(\frac{1}{2}\). \(\frac{1}{2}\) is greater. As the tenths place of 5 is greater than 0. \(\frac{1}{2}\) is greater.

Question 15.
If you know that \(\frac{2}{6}<\frac{1}{2}\) and \(\frac{3}{4}<\frac{1}{2}\), what do you know about \(\frac{2}{6} \text { and } \frac{3}{4}\)?
Type below:
_______

Answer:

Explanation:
As per the given data,
2/6 < 1/2 and 3/4 < 1/2
Then, 2/6 and 3/4 is
The least common denominator of 6 and 4 is 12
(2 x 4)/(6 x 4) and (3 x 6)/(4 x 6)
8/24 and 18/24
Now, the denominators are same, then compare the numerators
8/24 > 18/24
So, 2/6 > 3/4

Question 16.
Sandra has ribbons that are \(\frac{3}{4}\) yard, \(\frac{2}{6}\) yard, \(\frac{1}{5}\) yard, and \(\frac{4}{7}\) yard long. She needs to use the ribbon longer than \(\frac{2}{3}\) yard to make a bow. Which length of ribbon could she use for the bow?
\(\frac{□}{□}\) yard

Answer:

Explanation:

Page No. 362

Question 17.
Saundra ran \(\frac{7}{12}\) of a mile. Lamar ran \(\frac{3}{4}\) of a mile. Who ran farther? Explain.
_______

Answer:
As per the given data,
Saundra ran 7/12 of a mile
Lamar ran 3/4 of a mile
The least common denominator of 7/12 and 3/4 is 12
(7x 1)/( 12 x 1) and ( 3 x 3 )/( 4 x 3)
7/12 and 9/12
So, 7/12 < 9/12
So, 7/12 < 3/4
Lamar ran greater distance than Saundra

Question 18.
What’s the Question? Selena ran farther than Manny.
Type below:
_______

Answer:
Who ran farther? Selena or Manny

Question 19.
Chloe made a small pan of ziti and a small pan of lasagna. She cut the ziti into 8 equal parts and the lasagna into 9 equal parts. Her family ate \(\frac{2}{3}\) of the lasagna. If her family ate more lasagna than ziti, what fraction of the ziti could have been eaten?
Type below:
_______

Answer:
\(\frac{1}{4}\)

Explanation:
As per the given data,
Chloe made a small pan of ziti and a small pan of lasagna
She cut the ziti into 8 equal parts and the lasagna into 9 equal parts
Her family ate 2/3 of the lasagna = (2/3) x 9 = 6 parts
If her family ate more lasagna than ziti, then that is less than 6 parts
So, 1/4 of the ziti = (1/4) x 8 = 2 parts
So, 1/4 of ziti eaten by Chloe family

Question 20.
James, Ella, and Ryan biked around Eagle Lake. James biked \(\frac{2}{10}\) of the distance in an hour. Ella biked \(\frac{4}{8}\) of the distance in an hour. Ryan biked \(\frac{2}{5}\) of the distance in an hour. Compare the distances biked by each person by matching the statements to the correct symbol. Each symbol may be used more than once or not at all.
Go Math Grade 4 Answer Key Chapter 6 Fraction Equivalence and Comparison img 19
Type below:
_______

Answer:
2/10 < 4/8
1 / 8 > 2/5
2/10 < 2/5

Explanation:
As per the given data,
James, Ella, and Ryan biked around eagle lake
James biked 2/10 of the distance in an hour
Ella biked 4/8 of the distance in an hour
Ryan biked 2/5 of the distance in an hour
Least common denominator of 2 /10, 4/8, and 2/5 is 40
(2x 4)/(10 x 4), (4 x 5)/(8 x 5), and (2 x 8)/(5 x 8)
8/40, 20/ 40, and 16/ 40
8/40 < 16/40 < 20/40
2/10 < 2/5 < 4/8
So, 2/10 < 4/8
1 / 8 > 2/5
2/10 < 2/5

Common Core – Compare Fractions Using Benchmarks – Page No. 363

Compare. Write < or > .

Question 1.
Go Math Grade 4 Answer Key Chapter 6 Fraction Equivalence and Comparison Common Core Compare Fractions Using Benchmarks img 20

Answer:
\(\frac{1}{8}\) < \(\frac{6}{10}\)

Explanation:
Go Math Grade 4 Answer Key Chapter 6 Fraction Equivalence and Comparison Common Core Compare Fractions Using Benchmarks img 20

Question 2.
\(\frac{4}{12}\) _______ \(\frac{4}{6}\)

Answer:
\(\frac{4}{12}\) < \(\frac{4}{6}\)

Explanation:
4/12 and 4/6
4/12 is less than 1/2
4/6 is greater than 1/2
So, 4/12 < 4/6

Question 3.
\(\frac{2}{8}\) _______ \(\frac{1}{2}\)

Answer:
\(\frac{2}{8}\) < \(\frac{1}{2}\)

Explanation:
2/8 and 1/2
2/8 is less than 1/2
1/2 is equal to 1/2
So, 2/8 < 1/2

Question 4.
\(\frac{3}{5}\) _______ \(\frac{3}{3}\)

Answer:
\(\frac{3}{5}\) < \(\frac{3}{3}\)

Explanation:
3/5 and 3/3
3/5 is greater than 1/2
3/3 is equal to 1
So, 3/5 < 3/3

Question 5.
\(\frac{7}{8}\) _______ \(\frac{5}{10}\)

Answer:
\(\frac{7}{8}\) > \(\frac{5}{10}\)

Explanation:
7/8 and 5/10
7/8 is greater than 1/2
5/10 is equal to 1/2
So, 5/10 < 7/8

Question 6.
\(\frac{9}{12}\) _______ \(\frac{1}{3}\)

Answer:
\(\frac{9}{12}\) > \(\frac{1}{3}\)

Explanation:
9/12 and 1/3
9/ 12 is greater than 1/2
1/3 is less than 1/2
1/3 < 9/12

Question 7.
\(\frac{4}{6}\) _______ \(\frac{7}{8}\)

Answer:
\(\frac{4}{6}\) < \(\frac{7}{8}\)

Explanation:
4/6 and 7/8
4/6 is greater than 1/2
7/8 is closer to 1
So, 4/6 < 7/8

Question 8.
\(\frac{2}{4}\) _______ \(\frac{2}{3}\)

Answer:
\(\frac{2}{4}\) < \(\frac{2}{3}\)

Explanation:
2/4 and 2/3
2/4 is equal to 1/2
2/3 is greater than 1/2
So, 2/4 < 2/3

Question 9.
\(\frac{3}{5}\) _______ \(\frac{1}{4}\)

Answer:
\(\frac{3}{5}\) > \(\frac{1}{4}\)

Explanation:
3/5 and 1/4
3/5 is greater than 1/2
1/4 is less than 1/2
So, 1/4 < 3/5

Question 10.
\(\frac{6}{10}\) _______ \(\frac{2}{5}\)

Answer:
\(\frac{6}{10}\) > \(\frac{2}{5}\)

Explanation:
6/10 and 2/5
6/10 is greater than 1/2
2/5 is less than 1/2
So, 2/5 < 6/10

Question 11.
\(\frac{1}{8}\) _______ \(\frac{2}{10}\)

Answer:
\(\frac{1}{8}\) < \(\frac{2}{10}\)

Explanation:
1/8 and 2/10
1/8 is less than 1/2
2/10 is less than 1/2 but greater than 1/8
So, 1/8 < 2/10

Question 12.
\(\frac{2}{3}\) _______ \(\frac{5}{12}\)

Answer:
\(\frac{2}{3}\) > \(\frac{5}{12}\)

Explanation:
2/3 and 5/12
2/3 is greater than 1/2
5/12 is less than 1/2
So, 5/12 < 2/3

Question 13.
\(\frac{4}{5}\) _______ \(\frac{5}{6}\)

Answer:
\(\frac{4}{5}\)< \(\frac{5}{6}\)

Explanation:
4/5 and 5/6
4/5 is greater than 1/2
5/6 is greater than 1/2
Common denominator is 30
(4×6)/(5×6) and (5×5)/(6×5)
24/30 and 25/30
24/30 < 25/30
So, 4/5 < 5/6

Question 14.
\(\frac{3}{5}\) _______ \(\frac{5}{8}\)

Answer:
\(\frac{3}{5}\) < \(\frac{5}{8}\)

Explanation:
3/5 and 5/8
3/5 is greater than 1/2
5/8 is greater than 1/2
Common denominator is 40
(3×8)/(5×8) and (5×5)/(8×5)
24/40 and 25/ 40
24/40 < 25/40
3/5 < 5/8

Question 15.
\(\frac{8}{8}\) _______ \(\frac{3}{4}\)

Answer:
\(\frac{8}{8}\) > \(\frac{3}{4}\)

Explanation:
8/8 and 3/4
8/8 is equal to 1
3/4 is less than 1
3/4 < 8/8

Question 16.
Erika ran \(\frac{3}{8}\) mile. Maria ran \(\frac{3}{4}\) mile. Who ran farther?
_________

Answer:
Maria

Explanation:
As per the data,
Erika ran 3/8 mile
Maria ran 3/4 mile
Multiply the numerator and denominator of 3/4 with 2
(3×2)/(4×2) = 6/8
3/8 < 6/8
So, 3/8 < 3/4
So, Maria ran faster than Erika

Question 17.
Carlos finished \(\frac{1}{3}\) of his art project on Monday. Tyler finished \(\frac{1}{2}\) of his art project on Monday. Who finished more of his art project on Monday?
_________

Answer:
Tyler

Explanation:
From the given data,
Carlos finished 1/3 of his art project on Monday
Tyler finished ½ of his art project on Monday
1/3 is less than 1/2
1/2 is equal to 1/2
So, 1/3 < 1/2
Then, Tyler finished more of his work on Monday

Common Core – Compare Fractions Using Benchmarks – Page No. 364

Question 1.
Which symbol makes the statement true?
Go Math Grade 4 Answer Key Chapter 6 Fraction Equivalence and Comparison Common Core Compare Fractions Using Benchmarks img 21
Options:
a. >
b.<
c. =
d. none

Answer:
a. >

Explanation:
4/6 ? 3/8
By comparing 4/6 with 1/2, 4/6 > 1/2
By comparing 3/8 with 1/2, 3/8 < 1/2
So, 4/6 > 3/8

Question 2.
Which of the following fractions is greater than \(\frac{3}{4}\)?
Options:
a. \(\frac{1}{4}\)
b. \(\frac{5}{6}\)
c. \(\frac{3}{8}\)
d. \(\frac{2}{3}\)

Answer:
b. \(\frac{5}{6}\)

Explanation:
From the given data,
By comparing the 3/4 with 1/2, 3/4 > 1/2
Same as above, compare the options with ½
a. 1/4 < 1/2
b. 5/6 > 1/2
c. 3/8 < 1/2
d. 2/3 > 1/2
5/6 and 2/3 are greater than the 1/2
So, compare the 5/6 with 2/3
Then, 5/6 > 2/3
So, 5/6 > 3/4

Question 3.
Abigail is putting tiles on a table top. She needs 48 tiles for each of 8 rows. Each row will have 6 white tiles. The rest of the tiles will be purple. How many purple tiles will she need?
Options:
a. 432
b. 384
c. 336
d. 48

Answer:
c. 336

Explanation:
As per the given data
Abigail is putting tiles on a table top
Number of rows = 8
She needs 48 tiles for each of row = 48×8 = 384
Number of white tiles per row = 6×8 = 48
Rest of the tiles will be purple = 384 – 48 =336
So, the total number of purple color tiles = 336

Question 4.
Each school bus going on the field trip holds 36 students and 4 adults. There are 6 filled buses on the field trip. How many people are going on the field trip?
Options:
a. 216
b. 240
c. 256
d. 360

Answer:
b. 240

Explanation:
From the given data,
Each school bus going on the field trip holds 36 students and 4 adults
There are 6 filled buses on the field trip
6 x (36 + 4) = 6 x 40 = 240
So, the total number of people on the field trip = 240

Question 5.
Noah wants to display his 72 collector’s flags. He is going to put 6 flags in each row. How many rows of flags will he have in his display?
Options:
a. 12
b. 15
c. 18
d. 21

Answer:
a. 12

Explanation:
As mentioned in the data,
Noah wants to display his 72 collector’s flag
He is going to put 6 flags in each row = 6x = 72
X = 12
So, total 12 number of rows of flags will have in his display

Question 6.
Julian wrote this number pattern on the board:
3, 10, 17, 24, 31, 38.
Which of the numbers in Julian’s pattern are composite numbers?
Options:
a. 3, 17, 31
b. 10, 24, 38
c. 10, 17, 38
d. 17, 24, 38

Answer:
b. 10, 24, 38

Explanation:
As per the given information
Julian wrote his number pattern on the board =3, 10, 17, 24, 31, 38
Factors of 3 = 1,3
Factors of 10 = 1,2,5,10
Factors of 17 = 1, 17
Factors of 24 = 1, 2, 3, 4, 6
Factors of 31 = 1, 31
Factors of 38 = 1, 2, 19, 38
So, the composite number is 10, 24, and 38, which numbers have more than 2 factors

Page No. 367

Question 1.
Compare \(\frac{2}{5}\) and \(\frac{1}{10}\).
Think: Use ______ as a common denominator.
\(\frac{2}{5}=\frac { □×□ }{ □×□ } \) = \(\frac{□}{□}\)
\(\frac{1}{10}\)
Think: 4 tenth-size parts Go Math Grade 4 Answer Key Chapter 6 Fraction Equivalence and Comparison img 22 1 tenth-size part.
\(\frac{2}{5}\) _____ \(\frac{1}{10}\)

Answer:
\(\frac{2}{5}\) > \(\frac{1}{10}\)

Explanation:
Compare 2/5 and 1/10
Think: 10 as common denominator
Multiply the numerator and denominator of 2/5 with 2
Then, (2×2) ÷ (5×2) = 4/10
Now, compare the 4/10 with 1/10
4/10 > 1/10
So, 2/5 > 1/10

Question 2.
Compare \(\frac{6}{10}\) and \(\frac{3}{4}\).
Think: Use ______ as a common denominator.
\(\frac{6}{10}\)
\(\frac{3}{4}=\frac { □×□ }{ □×□ } \) = \(\frac{□}{□}\)
Think: A tenth-size part Go Math Grade 4 Answer Key Chapter 6 Fraction Equivalence and Comparison img 23 an eighth-size part.
\(\frac{6}{10}\) _____ \(\frac{3}{4}\)

Answer:
\(\frac{6}{10}\) < \(\frac{3}{4}\)

Explanation:
Compare 6/10 and 3/4
Think: Use 40 as a common denominator
So, multiply the denominator and numerator of 3/4 with 10
That is, (3×10) ÷ (4×10) = 30/40
Multiply the numerator and denominator of 6/10 with 4
That is, (6×4) ÷ (10×4) = 24/40
Denominators are same, compare the numerator values of 24/40 and 30/40
So, 24/40 < 30/40
Then, 6/10 < 3/4

Compare. Write <, >, or =.

Question 3.
\(\frac{7}{8}\) _____ \(\frac{2}{8}\)

Answer:
\(\frac{7}{8}\) > \(\frac{2}{8}\)

Explanation:
Compare 7/8 and 2/8
Denominator values are same but numerator values are different
Now, compare the numerator values of 7/8 and 2/8
Then, 7/8 > 2/8

Question 4.
\(\frac{5}{12}\) _____ \(\frac{3}{6}\)

Answer:
\(\frac{5}{12}\) < \(\frac{3}{6}\)

Explanation:
Compare 5/12 and 3/6
Multiply the numerator and denominator of 3/6 with 2
(3×2) ÷ (6×2) = 6/12
So, 5/12 < 6/12

Question 5.
\(\frac{4}{10}\) _____ \(\frac{4}{6}\)

Answer:
\(\frac{4}{10}\) < \(\frac{4}{6}\)

Explanation:
Compare 4/10 and 4/6
Multiply the numerator and denominator of 4/6 with 10
(4×10) ÷ (6×10) = 40/60
Multiply the numerator and denominator of 4/10 with 6
(4×6) ÷ (10×6) = 24/60
So, 24/60 < 40/60
Then, 4/10 < 4/6

Question 6.
\(\frac{6}{12}\) _____ \(\frac{2}{4}\)

Answer:
\(\frac{6}{12}\) = \(\frac{2}{4}\)

Explanation:
Compare 6/12 and 2/4
Multiply the numerator and denominator of 2/4 with 3
(2×3) ÷ (4×3) = 6/12
So, 6/12 = 6/12
Then, 6/12 = 2/4

Question 7.
\(\frac{1}{3}\) _____ \(\frac{1}{4}\)

Answer:
\(\frac{1}{3}\) < \(\frac{1}{4}\)

Explanation:
Compare 1/3 and 1/4
Multiply the numerator and denominator of 1/3 with 4
(1×4) ÷ (3×4) = 4/12
Multiply the numerator and denominator of 1/4 with 3
(1×3) ÷ (4×3) = 3/12
So, 4/12 < 3/12
Then, 1/3 < 1/4

Question 8.
\(\frac{4}{5}\) _____ \(\frac{8}{10}\)

Answer:
\(\frac{4}{5}\) = \(\frac{8}{10}\)

Explanation:
Compare 4/5 and 8/10
Multiply the numerator and denominator of 4/5 with 2
(4×2) ÷ (5×2) = 8/10
So, 8/10 = 8/10
Then, 4/5 = 8/10

Question 9.
\(\frac{3}{4}\) _____ \(\frac{2}{6}\)

Answer:
\(\frac{3}{4}\) < \(\frac{2}{6}\)

Explanation:
Compare 3/4 and 2/6
Multiply the numerator and denominator of 3/4 with 6
(3×6) ÷ (4×6) = 18/24
Multiply the numerator and denominator of 2/6 with 4
(2×4) ÷ (6×4) = 8/24
So, 18/24 < 8/24
Then, 3/4 < 2/6

Question 10.
\(\frac{1}{2}\) _____ \(\frac{5}{8}\)

Answer:
\(\frac{1}{2}\) < \(\frac{5}{8}\)

Explanation:
Compare 1/2 and 5/8
Multiply the numerator and denominator of 1/2 with 4
(1×4) ÷ (2×4) = 4/8
So, 4/8 < 5/8
Then, 1/2 < 5/8

Reason Quantitatively Algebra Find a number that makes the statement true.

Question 11.
\(\frac{1}{2}>\frac { □ }{ 3 } \)
□ = ______

Answer:
1

Explanation:
1/2 > x/3
Multiply the numerator and denominator of 1/2 with 3
(1×3) ÷ (2×3) = 3/6
Multiply the numerator and denominator of x/3 with 2
(Xx2) ÷ (3×2) = 2x/6
3/6 > 2x/6
So, x= 1
Then, 3/6 > 2/6
1/2 > 1/3

Question 12.
\(\frac{3}{10}>\frac { □ }{ 5 } \)
□ = ______

Answer:
1

Explanation:
3/10 > x/5
Multiply the numerator and denominator of x/5 with 2
(Xx2) ÷ (5×2) =2x/10
3/10 > 2x/10
So, x=1
3/10 > 2/10
3/10 > 1/5

Question 13.
\(\frac{5}{12}>\frac { □ }{ 3 } \)
□ = ______

Answer:
1

Explanation:
5/12 > x/3
Multiply numerator and denominator of x/3 with 4
(Xx4) ÷(3×4) = 4x/12
5/12 > 4x/12
So, x = 1
Then, 5/12 > 4/12
5/12 > 1/3

Question 14.
\(\frac{2}{3}>\frac { 4 }{ □ } \)
□ = ______

Answer:

Explanation:

Question 15.
Students cut a pepperoni pizza into 12 equal slices and ate 5 slices. They cut a veggie pizza into 6 equal slices and ate 4 slices. Use fractions to compare the amounts of each pizza that were eaten.
Type below:
_________

Answer:
\(\frac{5}{12}\) < \(\frac{4}{6}\)

Explanation:
As per the given data,
Students cut a pepperoni pizza into 12 equal slices and ate 5 slices
=5/12
They cut veggie pizza into 6 equal slices and ate 4 slices = 4/6
Compare 5/12 and 4/6
Multiply the numerator and denominator of 4/6 with 2
(4×2) ÷ (6×2) = 8/12
So, 5/12 < 8/12
Then, 5/12 < 4/6

Page No. 368

Question 16.
Jerry is making a strawberry smoothie. Which measure is greatest, the amount of milk, cottage cheese, or strawberries?
Go Math Grade 4 Answer Key Chapter 6 Fraction Equivalence and Comparison img 24
a. What do you need to find?
Type below:
_________

Answer:
I need to find the greatest measure from milk, cottage cheese, or strawberries

Question 16.
b. How will you find the answer?
Type below:
_________

Answer:
Equal the denominators of 3/4, 2/6, and 8/12
Multiply the numerator and denominator of 3/4 with 3
(3×3) ÷ (4×3) = 9/12
Multiply the numerator and denominator of 2/6 with 2
(2×2) ÷ (6×2) = 4/12
Compare 4/12 < 8/12 < 9/12
So, 2/6 < 8/12 <3/4

Question 16.
c. Show your work.
Type below:
_________

Answer:
2/6 < 8/12 < 3/4

Question 16.
d. Jerry needs more ________ than the other two ingredients.
________

Answer:
Jerry needs more strawberries than the other two ingredients

Question 17.
Angie, Blake, Carlos, and Daisy went running. Angie ran \(\frac{1}{3}\) mile, Blake ran \(\frac{3}{5}\) mile, Carlos ran \(\frac{7}{10}\) mile, and Daisy ran \(\frac{1}{2}\) mile. Which runner ran the shortest distance? Who ran the greatest distance?
The shortest distance: ________
The greatest distance: ________

Answer:
The shortest distance: \(\frac{1}{3}\)
The greatest distance: \(\frac{7}{10}\)

Explanation:
As per the given data,
Angie, Blake, Carlos, and Daisy went running
Angie ran 1/3 mile, Blake ran 3/5 mile, Carlos ran 7/10 mile, and Daisy ran 1/2 mile
Least common denominator of 1/3, 3/5, 7/10, and 1/2 =30
(1x 10)/(3×10), (3×6)/(5×6), (7×3)/(10×3), (1×15)/(2×15)
10/30, 18/30, 21/30, 15/30
10/30 < 15/30 < 18/30 < 21/30
1/3 < 1/2 < 3/5 < 7/10
The shortest distance ran by Angie and that is 1/ 3
The greatest distance ran by Carlos and that is 7/10

Question 18.
Elaine bought \(\frac{5}{8}\) pound of potato salad and \(\frac{4}{6}\) pound of macaroni salad for a picnic. Use the numbers to compare the amounts of potato salad and macaroni salad Elaine bought.
Go Math Grade 4 Answer Key Chapter 6 Fraction Equivalence and Comparison img 25
Type below:
_________

Answer:
As per the given data,
Elaine bought 5/8 pound of potato salad and 4/6 pound of macaroni salad for a picnic
Multiply the numerator and denominator of 5/8 with 6
(5×6) / (8×6) = 30/48
Multiply the numerator and denominator of 4/6 with 8
(4×8) / (6×8) = 32/48
30/48 < 32/48
So, 5/8 < 4/6
Elaine bought more macaroni salad than potato salad

Common Core – Compare Fractions – Page No. 369

Compare. Write <, >, or =

Question 1.
Go Math Grade 4 Answer Key Chapter 6 Fraction Equivalence and Comparison Common Core Compare Fractions img 26

Answer:
\(\frac{1}{5}\) < \(\frac{2}{10}\)

Explanation:
Go Math Grade 4 Answer Key Chapter 6 Fraction Equivalence and Comparison Common Core Compare Fractions img 26

Question 2.
\(\frac{1}{5}\) _____ \(\frac{2}{10}\)

Answer:
\(\frac{1}{5}\) = \(\frac{2}{10}\)

Explanation:
1/5 and 2/10
Think: 10 is a common denominator
1/5 = (1×2) / (5×2) = 2/10
2/10 = 2/10
So, 1/5 = 2/10

Question 3.
\(\frac{2}{4}\) _____ \(\frac{2}{5}\)

Answer:
\(\frac{2}{4}\) > \(\frac{2}{5}\)

Explanation:
2/4 and 2/5
20 is a common denominator
2/4 = (2×5)/(4×5) = 10/20
2/5 = (2×4)/(5×4) = 8/20
10/20 > 8/20
So, 2/4 > 2/5

Question 4.
\(\frac{3}{5}\) _____ \(\frac{7}{10}\)

Answer:
\(\frac{3}{5}\) < \(\frac{7}{10}\)

Explanation:
3/5 and 7/10
10 is a common denominator
3/5 = (3×2)/(5×2) = 6/10
7/10
6/10 < 7/10
So, 3/5 < 7/10

Question 5.
\(\frac{4}{12}\) _____ \(\frac{1}{6}\)

Answer:
\(\frac{4}{12}\) > \(\frac{1}{6}\)

Explanation:
4/12 and 1/6
12 is a common denominator
4/12
1/6 = (1×2)/(6×2) = 2/12
4/12 > 2/12
So, 4/12 > 1/6

Question 6.
\(\frac{2}{6}\) _____ \(\frac{1}{3}\)

Answer:
\(\frac{2}{6}\) = \(\frac{1}{3}\)

Explanation:
2/6 and 1/3
6 is a common denominator
2/6
1/3 = (1×2)/(3×2) = 2/6
So, 2/6 =2/6
So, 2/6 = 1/3

Question 7.
\(\frac{1}{3}\) _____ \(\frac{2}{4}\)

Answer:
\(\frac{1}{3}\) < \(\frac{2}{4}\)

Explanation:
1/3 and 2/4
12 is a common denominator
1/3 = (1×4)/(3×4) = 4/12
2/4 = (2×3)/(4×3) = 6/12
4/12 < 6/12
So, 1/3 < 2/4

Question 8.
\(\frac{2}{5}\) _____ \(\frac{1}{2}\)

Answer:
\(\frac{2}{5}\) < \(\frac{1}{2}\)

Explanation:
2/5 and 1/2
10 is a common denominator
2/5 = (2×2)/(5×2) = 4/10
1/2 = (1×5)/(2×5) = 5/10
4/10 < 5/10
So, 2/5 < 1/2

Question 9.
\(\frac{4}{8}\) _____ \(\frac{2}{4}\)

Answer:
\(\frac{4}{8}\) = \(\frac{2}{4}\)

Explanation:
4/8 and 2/4
8 is a common denominator
4/8
2/4 = (2×2)/(4×2) = 4/8
2/4 = 4/8
So, 4/8 = 2/4

Question 10.
\(\frac{7}{12}\) _____ \(\frac{2}{4}\)

Answer:
\(\frac{7}{12}\) < \(\frac{2}{4}\)

Explanation:
7/12 and 2/4
12 is a common denominator
7/12
2/4 = (2×3)/(4×3) = 6/12
7/12 < 6/12
So, 7/12 < 2/4

Question 11.
\(\frac{1}{8}\) _____ \(\frac{3}{4}\)

Answer:
\(\frac{1}{8}\) <  \(\frac{3}{4}\)

Explanation:
1/8 and 3/4
8 is a common denominator
1/8
3/4 = (3×2)/(4×2) = 6/8
1/8 < 6/8
So, 1/8 < 3/4

Question 12.
A recipe uses \(\frac{2}{3}\) of flour and \(\frac{5}{8}\) cup of blueberries. Is there more flour or more blueberries in the recipe?
more _____

Answer:
flour

Explanation:
From the given data,
A recipe uses 2/3 of flour and 5/8 cup of blueberries
Common denominator is 24
2/3 = (2×8)/(3×8) = 16/24
5/8 = (5×3)/(8×3) = 15/24
16/24 > 15/24
So, 2/3 > 5/8
So, flour is more in the recipe

Question 13.
Peggy completed \(\frac{5}{6}\) of the math homework and Al completed \(\frac{4}{5}\) of the math homework. Did Peggy or Al complete more of the math homework?
_________

Answer:
Peggy completed more work than Al

Explanation:
As per the given data,
Peggy completed 5/6 of the math homework
A1 completed 4/5 of the math homework
30 is a common denominator
5/6 = (5×5)/(6×5) = 25/30
4/5 = (4×6)/(5×6) =24/30
25/30 > 24/30
So, 5/6 > 4/5
So, Peggy completed more work than Al

Common Core – Compare Fractions – Page No. 370

Question 1.
Pedro fills a glass \(\frac{2}{4}\) full with orange juice. Which of the following fractions is greater than \(\frac{2}{4}\)?
Options:
a. \(\frac{3}{8}\)
b. \(\frac{4}{6}\)
c. \(\frac{5}{12}\)
d. \(\frac{1}{3}\)

Answer:
b. \(\frac{4}{6}\)

Explanation:
\(\frac{4}{6}\) > \(\frac{2}{4}\)

Question 2.
Today Ian wants to run less than \(\frac{7}{12}\) mile. Which of the following distances is less than \(\frac{7}{12}\) mile?
Options:
a. \(\frac{3}{4}\) mile
b. \(\frac{2}{3}\) mile
c. \(\frac{5}{6}\) mile
d. \(\frac{2}{4}\) mile

Answer:
d. \(\frac{2}{4}\) mile

Explanation:
\(\frac{2}{4}\) is less than \(\frac{7}{12}\)

Question 3.
Ms. Davis traveled 372,645 miles last year on business. What is the value of 6 in 372,645?
Options:
a. 6
b. 60
c. 600
d. 6,000

Answer:
c. 600

Explanation:
Ms. Davis traveled 372, 645 miles last year on business
The value of 6 in 372,645 is 600

Question 4.
One section of an auditorium has 12 rows of seats. Each row has 13 seats. What is the total number of seats in that section?
Options:
a. 25
b. 144
c. 156
d. 169

Answer:
c. 156

Explanation:
From the given information
One section of an auditorium has 12 rows of seats
Each row has 13 seats = 13×12 = 156 seats
So, the total number of seats in the auditorium = 156 seats

Question 5.
Sam has 12 black-and-white photos and 18 color photos. He wants to put the photos in equal rows so each row has either black-and-white photos only or color photos only. In how many rows can Sam arrange the photos?
Options:
a. 1, 2, 3, or 6 rows
b. 1, 3, 6, or 9 rows
c. 1, 2, or 4 rows
d. 1, 2, 3, 4, 6, or 9 rows

Answer:
a. 1, 2, 3, or 6 rows

Explanation:
As per the given information
Sam has 12 black and white photos 18 color photos
He wants to put the photos in equal rows
So each row has either black and white photos only or color photos only
H.C.F of 12 and 18 is 6
Rows of 6.
2 rows of black equal 12.
3 rows of white equals 18.

Question 6.
The teacher writes \(\frac{10}{12}\) on the board. He asks students to write the fraction in simplest form. Who writes the correct answer?
Options:
a. JoAnn writes \(\frac{10}{12}\)
b. Karen writes \(\frac{5}{12}\)
c. Lynn writes \(\frac{6}{5}\)
d. Mark writes \(\frac{5}{6}\)

Answer:
d. Mark writes \(\frac{5}{6}\)

Explanation:
As per the given data,
The teacher writes 10/12 on the board
He asks students to write the fraction in simplest form
For the simplest form of 10/12, divide the 10/12 with 2
(10÷2)/(12÷2) = 5/6
5/6 is the simplest form of 10/12
So, Mark writes the correct answer

Page No. 373

Question 1.
Locate and label points on the number line to help you write \(\frac{3}{10}, \frac{11}{12}, \text { and } \frac{5}{8}\) in order from least to greatest.
Go Math Grade 4 Answer Key Chapter 6 Fraction Equivalence and Comparison img 27
Type below:
___________

Answer:
chapter 6 - compare fractions and order fractions- image1

Explanation:
3/10, 11/12, 5/8
3/10 is closer to 0
11/12 is closer to 1
5/8 is closer to 1/2
So, 3/10 < 5/8 < 11/12

Write the fraction with the greatest value.

Question 2.
\(\frac{7}{10}, \frac{1}{5}, \frac{9}{10}\)
\(\frac{□}{□}\)

Answer:
\(\frac{9}{10}\)

Explanation:
7/10, 1/5, and 9/10
7/10 is closer to 1/2
1/5 is closer to 0
9/10 is closer to 1
So, 9/10 > 7/10 > 1/5
Greatest value is 9/10

Question 3.
\(\frac{5}{6}, \frac{7}{12}, \frac{7}{10}\)
\(\frac{□}{□}\)

Answer:
\(\frac{5}{6}\)

Explanation:
7/12 is less than 1/2
7/10 and 5/6 are greater than 1/2
Compare 5/6 and 7/12
Multiply the numerator and denominator of 5/6 with 2
(5×2)/(6×2) = 10/12 > 7/12
So, 5/6 > 7/12
Compare 5/6 and 7/10
Multiply the 5/6 with 10
(5×10)/(6×10) = 50/60
Multiply the 7/10 with 6
(7×6)/(10×6) = 42/60
So, 5/6> 7/10
So, 7/12 <7/10<5/6

Question 4.
\(\frac{2}{8}, \frac{1}{8}, \frac{2}{4}, \frac{2}{6}\)
\(\frac{□}{□}\)

Answer:
\(\frac{2}{4}\)

Explanation:
2/8, 1/8, 2/4, 2/6
Common denominator of 4,6,8 = 24
(2×3)/(8×3), (1×3)/(8×3), (2×6)/(4×6), (2×4)/(6×4)
6/24, 3/24, 12/24, 8/24
Compare the numerator values
12/24 > 8/24 > 6/24 > 3/24
So, 2/4 > 2/6 > 2/8 >1/8

Write the fractions in order from least to greatest.

Question 5.
\(\frac{1}{4}, \frac{3}{6}, \frac{1}{8}\)
\(\frac{□}{□}\)
Type below:
________

Answer:
\(\frac{1}{8}, \frac{3}{6}, \frac{1}{4}\)

Explanation:
1/4, 3/6, 1/8
1/ 4 is closer to 1/2
3/6 is equal to 1/2
1/8 is closer to 0
So, 1/8 < 3/6 < 1/4

Question 6.
\(\frac{3}{5}, \frac{2}{3}, \frac{3}{10}, \frac{4}{5}\)
\(\frac{□}{□}\)
Type below:
________

Answer:
\(\frac{4}{5}, \frac{3}{10}, \frac{3}{5}, \frac{2}{3}\)

Explanation:
3/5, 2/3, 3/10, 4/5
3/5 is closer to 1/2
2/3 is greater than 1/2
3/10 is less than 1/2
4/5 is closer to 0
So, 4/5 < 3/10 < 3/5 < 2/3

Question 7.
\(\frac{3}{4}, \frac{7}{12}, \frac{5}{12}\)
\(\frac{□}{□}\)
Type below:
________

Answer:
\(\frac{5}{12}, \frac{7}{12}, \frac{3}{4}\)

Explanation:
3/4, 7/12, 5/12
3/ 4 is closer to 1
7/12 is greater than 1/2
5/ 12 is closer to 1/2
So, 5/12 < 7/12 < 3/4

Write the fractions in order from least to greatest.

Question 8.
\(\frac{2}{5}, \frac{1}{3}, \frac{5}{6}\)
\(\frac{□}{□}\)
Type below:
________

Answer:
\(\frac{1}{3}, \frac{2}{5}, \frac{5}{6}\)

Explanation:
2/5, 1/3, 5/6
2/5 is closer to 1/2
1/3 is closer to 0
5/6 is closer to 1
So, 1/3 < 2/5 < 5/6

Question 9.
\(\frac{4}{8}, \frac{5}{12}, \frac{1}{6}\)
\(\frac{□}{□}\)
Type below:
________

Answer:
\(\frac{1}{6}, \frac{5}{12}, \frac{4}{8}\)

Explanation:
4/8, 5/12, 1/6
4/8 is equal to1/2
5/12 is closer to 1/2
1/6 is closer to 0
So, 1/6 < 5/12 < 4/ 8

Question 10.
\(\frac{7}{100}, \frac{9}{10}, \frac{4}{5}\)
\(\frac{□}{□}\)
Type below:
________

Answer:
\(\frac{7}{100}, \frac{4}{5}, \frac{9}{10}\)

Explanation:
7/100, 9/10, 4/5
7/100 is closer to 0
9/10 is closer to 1
4/5 is greater than 1/2
So, 7/100 < 4/5 < 9/10

Reason Quantitatively Algebra Write a numerator that makes the statement true.

Question 11.
\(\frac{1}{2}<\frac { □ }{ 10 } <\frac{4}{5}\)
□ = _____

Answer:
6 or 7

Explanation:
1/2 < x/10 < 4/5
Common denominator is 10
(1×5)/(2×5) < x/10 < (4×2)/(5×2)
5/10 < x/10 < 8/10
Then, x = 6 or 7

Question 12.
\(\frac{1}{4}<\frac{5}{12}<\frac { □ }{ 6 } \)
□ = _____

Answer:
6

Explanation:
1/4 < 5/12 < x/6
Common denominator is 24
(1×6)/(4×6) < (5×2)/(12×2) < 4x/(6×4)
6/24 < 10/24 < 4x/24
If x = 6, then 4x = 24
So, 6/24 < 10/24 < 24/24

Question 13.
\(\frac { □ }{ 8 } <\frac{3}{4}<\frac{7}{8}\)
□ = _____

Answer:
1,2,3,4,5

Explanation:
x/8 < 3/4 < 7/8
Common denominator is 8
x/8 < (3×2)/(4×2) < 7/8
x/8 < 6/8 < 7/8
so x = 1,2,3,4,5

Page No. 374

Question 14.
Nancy, Lionel, and Mavis ran in a 5-kilometer race. The table shows their finish times. In what order did Nancy, Lionel, and Mavis finish the race?
Go Math Grade 4 Answer Key Chapter 6 Fraction Equivalence and Comparison img 28
a. What do you need to find?

Answer:
In which Nancy, Lionel, and Mavis finished the race?

Question 14.
b. What information do you need to solve the problem?
Type below:
_________

Answer:
the amount of time it took each runner to finish the race

Question 14.
c. What information is not necessary?
Type below:
_________

Answer:
the distance of the race

Question 14.
d. How will you solve the problem?
Type below:
_________

Answer:
By using the running race time of Nancy, Lionel, and Mavis

Question 14.
e. Show the steps to solve the problem.
Type below:
_________

Answer:
Common denominator of 2/3, 7/12, 3/4 is 12
(2×4)/(3×4), (7/12), (3×3)/(4×3)
8/12, 7/12, 9/12
7/12 < 8/12 < 9/12
7/12 < 2/3 < 3/4
Lionel < Nancy < Mavis

Question 14.
f. Complete the sentences.
The runner who finished first is _______.
The runner who finished second is _______.
The runner who finished third is _______.
The first: _______
The second: _______
The third: _______

Answer:
Lionel finished the race first
Nancy finished the race second
Mavis finished the race third
Lionel
Nancy
Mavis

Common Core – Compare and Order Fractions – Page No. 375

Write the fractions in order from least to greatest.

Question 1.
\(\frac{5}{8}, \frac{2}{12}, \frac{8}{10}\)
Go Math Grade 4 Answer Key Chapter 6 Fraction Equivalence and Comparison Common Core Compare and Order Fractions img 29

Answer:
\(\frac{2}{12}, \frac{5}{8}, \frac{8}{10}\)

Explanation:
Go Math Grade 4 Answer Key Chapter 6 Fraction Equivalence and Comparison Common Core Compare and Order Fractions img 29

Question 2.
\(\frac{1}{5}, \frac{2}{3}, \frac{5}{8}\)
Type below:
_________

Answer:
\(\frac{1}{5}, \frac{5}{8}, \frac{2}{3}\)

Explanation:
chapter 6 - compare fractions and order fractions- image4
1/5, 2/3, 5/8
1/5 is closer to 0
2/3 is greater than 1/2
5/8 greater than 1/2
1/5 < 5/8 < 2/3

Question 3.
\(\frac{1}{2}, \frac{2}{5}, \frac{6}{10}\)
Type below:
_________

Answer:
\(\frac{2}{5}, \frac{1}{2}, \frac{6}{10}\)

Explanation:
chapter 6 - compare fractions and order fractions- image5
1/2, 2/5, 6/10
1/2 is equal to 1/2
2/5 is less than 1/2
6/10 is greater than 1/2

Question 4.
\(\frac{4}{6}, \frac{7}{12}, \frac{5}{10}\)
Type below:
_________

Answer:
\(\frac{5}{10}\) < \(\frac{7}{12}\) < \(\frac{4}{6}\)

Explanation:
chapter 6 - compare fractions and order fractions- image6
4/6, 7/12, 5/10
4/6 is closer to 1
7/12 is greater than 1/2
5/10 is equal to 1/2

Question 5.
\(\frac{1}{4}, \frac{3}{6}, \frac{1}{8}\)
Type below:
_________

Answer:
\(\frac{1}{8}\) < \(\frac{1}{4}\) < \(\frac{3}{6}\)

Explanation:
chapter 6 - compare fractions and order fractions- image7
1/4, 3/6, 1/8
1/4 is less than 1/2
3/6 is equal to 1/2
1/8 is closer to 0

Question 6.
\(\frac{1}{8}, \frac{3}{6}, \frac{7}{12}\)
Type below:
_________

Answer:
\(\frac{1}{8}\) < \(\frac{7}{12}\) < \(\frac{3}{6}\)

Explanation:
chapter 6 - compare fractions and order fractions- image8
1/8, 3/6, 7/12
1/8 is closer to 0
3/6 is equal to 1/2
7/12 is greater than 1/2

Question 7.
\(\frac{8}{100}, \frac{3}{5}, \frac{7}{10}\)
Type below:
_________

Answer:
\(\frac{8}{100}\) < \(\frac{3}{5}\) < \(\frac{7}{10}\)

Explanation:
chapter 6 - compare fractions and order fractions- image9
8/100, 3/5, 7/10
8/100 is closer to 0
3/5 is greater than 1/2
7/10 is closer to 1

Question 8.
\(\frac{3}{4}, \frac{7}{8}, \frac{1}{5}\)
Type below:
_________

Answer:
\(\frac{1}{5}\) < \(\frac{3}{4}\) < \(\frac{7}{8}\)

Explanation:
chapter 6 - compare fractions and order fractions- image10
3/4, 7/8, 1/5
3/4 is greater than 1/2
7/8 is closer to 1
1/5 is closer to 0

Question 9.
Amy’s math notebook weighs \(\frac{1}{2}\) pound, her science notebook weighs \(\frac{7}{8}\) pound, and her history notebook weighs \(\frac{3}{4}\) pound. What are the weights in order from lightest to heaviest?
Type below:
_________

Answer:
\(\frac{1}{2}\) pound, \(\frac{3}{4}\) pound, \(\frac{7}{8}\) pound

Explanation:
From the given data,
Amy’s math notebook weighs 1/2 pound
Science notebook weighs 7/8 pound
History notebook weighs 3/4 pound
7/8 is closer to 1
3/4 is greater than 1/2
1/2 < 3/4 < 7/8
So, Amy’s math notebook weight < history notebook weight < science notebook

Question 10.
Carl has three picture frames. The thicknesses of the frames are \(\frac{4}{5}\) inch, \(\frac{3}{12}\) inch, and \(\frac{5}{6}\) inch. What are the thicknesses in order from least to greatest?
Type below:
_________

Answer:
\(\frac{3}{12}\) inch, \(\frac{4}{5}\) inch, \(\frac{5}{6}\) inch

Explanation:
As per the given data,
Carl has three picture frames
The thickness of the frames are 4/5 inch, 3/12 inch, 5/6 inch
4/5 is greater than 1/2
3/12 is less than 1/2
5/6 is closer to 1
3/12 < 4/5 < 5/6

Common Core – Compare and Order Fractions – Page No. 376

Question 1.
Juan’s three math quizzes this week took him \(\frac{1}{3}\) hour, \(\frac{4}{6}\) hour, and \(\frac{1}{5}\) hour to complete. Which list shows the lengths of time in order from least to greatest?
Options:
a. \(\frac{1}{3}\) hour, \(\frac{4}{6}\) hour, \(\frac{1}{5}\) hour
b. \(\frac{1}{5}\) hour, \(\frac{1}{3}\) hour, \(\frac{4}{6}\) hour
c. \(\frac{1}{3}\) hour, \(\frac{1}{5}\) hour, \(\frac{4}{6}\) hour
d. \(\frac{4}{6}\) hour, \(\frac{1}{3}\) hour, \(\frac{1}{5}\) hour

Answer:
b. \(\frac{1}{5}\) hour, \(\frac{1}{3}\) hour, \(\frac{4}{6}\) hour

Explanation:
From the given information
Juan’s three math quizzes this week took him 1/3 hour, 4/6 hour, and 1/5 hour
Compare 1/3 and 1/2
1/3 is less than 1/2
4/6 is greater than 1/2
1/5 is closer to 0
1/5 < 1/3 < 4/6
So, Juan’s math quizzes times from least to greatest is 1/5, 1/3, 4/6

Question 2.
On three days last week, Maria ran \(\frac{3}{4}\) mile, \(\frac{7}{8}\) mile, and \(\frac{3}{5}\) mile. What are the distances in order from least to greatest?
Options:
a. \(\frac{3}{4}\) mile, \(\frac{7}{8}\) mile, \(\frac{3}{5}\) mile
b. \(\frac{3}{5}\) mile, \(\frac{3}{4}\) mile, \(\frac{7}{8}\) mile
c. \(\frac{7}{8}\) mile, \(\frac{3}{4}\) mile, \(\frac{3}{5}\) mile
d. \(\frac{7}{8}\) mile, \(\frac{3}{5}\) mile, \(\frac{3}{4}\) mile

Answer:
b. \(\frac{3}{5}\) mile, \(\frac{3}{4}\) mile, \(\frac{7}{8}\) mile

Explanation:
As per the information
On three days last week, Maria ran 3/4 mile, 7/8 mile, and 3/5 mile
3/4 is greater than 1/2
7/8 is closer to 1
3/5 is greater than 1/2
Compare 3/5 and 3/4
3/4 is greater than 3/5
So, 3/5 < 3/4 < 7/8
Distance from least to greatest is 3/5, 3/4 , 7/8

Question 3.
Santiago collects 435 cents in nickels. How many nickels does he collect?
Options:
a. 58
b. 78
c. 85
d. 87

Answer:
d. 87

Explanation:
As per the given data,
Santiago collects 435 cents in nickels
1 nickel worth is 5 cents
Then, nickels per 435 cents = 435/5 = 87
So, Santiago collects 87 nickels

Question 4.
Lisa has three classes that each last 50 minutes. What is the total number of minutes the three classes last?
Options:
a. 15 minutes
b. 150 minutes
c. 153 minutes
d. 156 minutes

Answer:
b. 150 minutes

Explanation:
From the given data,
Lisa has three classes that each last 50 minutes
The total number of minutes the three classes last = 3×50 =150 minutes

Question 5.
Some students were asked to write a composite number. Which student did NOT write a composite number?
Options:
a. Alicia wrote 2.
b. Bob wrote 9.
c. Arianna wrote 15.
d. Daniel wrote 21.

Answer:
a. Alicia wrote 2.

Explanation:
As per the information
Some students were asked to write a composite number
a. Alicia wrote 2
Factors of 2 is 1 and 2
b. Bob wrote 9
Factors of 9 is 1, 3, 9
c. Arianna wrote 15
Factors of 15 is 1, 3, 5, 15
d. Daniel wrote 21
Factors of 21 is 1,3,7,21
So, Alicia did not write a composite number

Question 6.
Mrs. Carmel serves \(\frac{6}{8}\) of a loaf of bread with dinner. Which fraction is equivalent to \(\frac{6}{8}\)?
Options:
a. \(\frac{2}{4}\)
b. \(\frac{9}{16}\)
c. \(\frac{2}{3}\)
d. \(\frac{3}{4}\)

Answer:
d. \(\frac{3}{4}\)

Explanation:
As per the given information
Mrs. Carmel serves 6/8 of a loaf of bread with dinner
To find the equivalent fraction of 6/8, simplify the 6/8 by dividing with the 2
(6÷2)/(8÷2) = ¾
So, the equivalent fraction of 6/8 is 3/4

Page No. 377

Question 1.
For numbers 1a–1d, tell whether the fractions are equivalent by selecting the correct symbol.
a. \(\frac{4}{16}\) _____ \(\frac{1}{4}\)

Answer:
\(\frac{4}{16}\) = \(\frac{1}{4}\)

Explanation:
4/16 and 1/4
Divide the numerator and denominator of 4/16 with 4
(4÷4)/(16÷4) = 1/4
So, 4/16 = 1/4

Question 1.
b. \(\frac{3}{5}\) _____ \(\frac{12}{15}\)

Answer:
\(\frac{3}{5}\) ≠ \(\frac{12}{15}\)

Explanation:
3/5 and 12/15
Multiply the numerator and denominator of 3/5 with 3
(3×3)/(5×3) = 9/15
So, 3/5 ≠ 12/15

Question 1.
c. \(\frac{5}{6}\) _____ \(\frac{25}{30}\)

Answer:
\(\frac{5}{6}\) = \(\frac{25}{30}\)

Explanation:
c. 5/6 and 25/30
Multiply the numerator and denominator of 5/6 with 5
(5×5)/(6×5) = 25/30
So, 5/6 = 25/30

Question 1.
d. \(\frac{6}{10}\) _____ \(\frac{5}{8}\)

Answer:
\(\frac{6}{10}\) ≠ \(\frac{5}{8}\)

Explanation:
6/10 and 5/8
Divide the numerator and denominator of 6/10 with 2
(6÷2)/(10÷2) = 3/5
6/10 ≠5/8

Question 2.
Juan’s mother gave him a recipe for trail mix.
\(\frac{3}{4}\) cup cereal \(\frac{2}{3}\) cup almonds
\(\frac{1}{4}\) cup peanuts \(\frac{1}{2}\) cup raisins
Order the ingredients used in the recipe from least to greatest.
Type below:
_________

Answer:
As per the given data,
Juan’s mother gave him a recipe for trail mix
3/4 cup cereal and 2/3 cup almonds
1/4 cup peanuts and 1/2 cup raisins
3/4 is closer to 1
2/3 is greater than 1/2
1/4 is less than 1/2
1/2 is equal to 1/2
So, 1/4 < 1/2 <2/3 < 3/4
So, Jaun’s mother gave him a recipe for trail mix in order
1/4 cup of peanuts < 1/2 cup of raisins < 2/3 cup almonds < 3/4 cup of cereals

Question 3.
Taylor cuts \(\frac{1}{5}\) sheet of construction paper for an arts and crafts project. Write \(\frac{1}{5}\) as an equivalent fraction with the denominators shown.
Go Math Grade 4 Answer Key Chapter 6 Fraction Equivalence and Comparison img 30
Type below:
_________

Answer:
From the given data,
Taylor cuts 1/5 sheet of construction paper for an arts and crafts project
So, the equivalent fractions of 1/5
Multiply the numerator and denominator of 1/5 with 2
(1×2)/(5×2) = 2/10
Multiply the numerator and denominator of 1/5 with 3
(1×3)/(5×3) = 3/15
Multiply the numerator and denominator of 1/5 with 5
(1×5)/(5×5) = 5/25
Multiply the numerator and denominator of 1/5 with 8
(1×8)/(5×8) = 8/40
So, the equivalent fractions of 1/5 are 2/10, 3/15, 5/25, 8/40

Question 4.
A mechanic has sockets with the sizes shown below. Write each fraction in the correct box.
\(\frac{7}{8} in. \frac{3}{16} in. \frac{1}{4} in. \frac{3}{8} in. \frac{4}{8} in. \frac{11}{16} in.\)
Go Math Grade 4 Answer Key Chapter 6 Fraction Equivalence and Comparison img 31
Type below:
_________

Answer:
chapter 6 - compare fractions and order fractions- image11

Explanation:
As per the given data,
A mechanic has sockets with the sizes 7/8 inch, 3/16 inch, 1/4 inch, 3/8 inch, 4/8 inch, 11/16 inch
7/8 is greater than 1/2
3/16 is less than 1/2
1/4 is less than 1/2
3/8 is less than 1/2
4/8 is equal to 1/2
11/16 is greater than 1/2

Page No. 378

Question 5.
Darcy bought \(\frac{1}{2}\) pound of cheese and \(\frac{3}{4}\) pound of hamburger for a barbecue. Use the numbers to compare the amounts of cheese and hamburger Darcy bought.
Go Math Grade 4 Answer Key Chapter 6 Fraction Equivalence and Comparison img 32

Answer:
grade 4 chapter 6 image 3

Explanation:
From the given data,
Darcy bought 1/2 pound of cheese and 3/4 pound of hamburger for a barbecue
3/4 is greater than 1/2

Question 6.
Brad is practicing the piano. He spends \(\frac{1}{4}\) hour practicing scales and \(\frac{1}{3}\) hour practicing the song for his recital. For numbers 6a–6c, select Yes or No to tell whether each of the following is a true statement.
a. 12 is a common denominator of \(\frac{1}{4}\) and \(\frac{1}{3}\).
i. yes
ii. no

Answer:
i. yes

Explanation:
12 is a common denominator of 1/3 and 1/4

Question 6.
b. The amount of time spent practicing scales can be rewritten as \(\frac{3}{12}\).
i. yes
ii. no

Answer:
i. yes

Explanation:
b. The amount of time spent practicing scales can be rewritten as 3/12
Multiply the numerator and denominator of 1/4 with 3
(1×3)/(4×3) = 3/12
Yes, amount of time spent practicing scales can be rewritten as 3/12

Question 6.
c. The amount of time spent practicing the song for the recital can be rewritten as \(\frac{6}{12}\).
i. yes
ii. no

Answer:
ii. no

Explanation:
c. The amount of time spent practicing the song for the recital can be rewritten as 6/12
The amount of time spent practicing for the song for his recital = 1/3
Multiply the numerator and denominator of 1/3 with 4
(1×4)/(3×4) = 4/12
No, time spent practicing the song for the recital can not be written as 6/12

Question 7.
In the school chorus, \(\frac{4}{24}\) of the students are fourth graders. In simplest form, what fraction of the students in the school chorus are fourth graders?
\(\frac{□}{□}\)

Answer:
\(\frac{1}{6}\)

Explanation:
As per the given information,
In the school chorus,
4/24 of the students are fourth graders
For the simplest form of 4/24
Divide the numerator and denominator of 4/24 with 4
(4÷4)/(24÷4) =1/6
The simplest form of 4/24 is 1/6

Question 8.
Which pairs of fractions are equivalent? Mark all that apply.
a. \(\frac{8}{12} \text { and } \frac{2}{3}\)
b. \(\frac{3}{4} \text { and } \frac{20}{24}\)
c. \(\frac{4}{5} \text { and } \frac{12}{16}\)
d. \(\frac{7}{10} \text { and } \frac{21}{30}\)

Answer:
a. \(\frac{8}{12} \text { and } \frac{2}{3}\)

Explanation:
a. 8/12 and 2/3
Multiply the numerator and denominator of 2/3 with 4
(2×4)/(3×4) = 8/12
So, 8/12 = 2/3
b. 3/4 and 20/24
Multiply the numerator and denominator of 3/4 with 6
(3×6)/(4×6) = 18/24
c. 4/5 and 12/16
4/5 ≠ 12/16
d. 7/10 and 21/30
Multiply the numerator and denominator of 7/10 with 3
(7×3)/(10×3) =21/30
So, 7/10 = 21/30

Question 9.
Sam worked on his science fair project for \(\frac{1}{4}\) hour on Friday and \(\frac{1}{2}\) hour on Saturday. What are four common denominators for the fractions? Explain your reasoning.

Answer:
From the given data,
Sam worked on his science fair project for 1/4 hour on Friday and 1/2 hour on Saturday
4,8,12,16 are all common denominators because they all multiples of 2 and 4

Page No. 379

Question 10.
Morita works in a florist shop and makes flower arrangements. She puts 10 flowers in each vase, and \(\frac{2}{10}\) of the flowers are daisies.
Part A
If Morita makes 4 arrangements, how many daisies does she need? Show how you can check your answer.
_____ daisies

Answer:
8 daisies

Explanation:
If Morita makes 4 arrangements, 4 X 2 = 8.

Question 10.
Part B
Last weekend, Morita used 10 daisies to make flower arrangements. How many flowers other than daisies did she use to make the arrangements? Explain your reasoning.
_____ other flowers

Answer:
40 other flowers

Explanation:
If she used 10 daises, she must have made 5 arrangements. In each vase, she put \(\frac{2}{10}\) of the flowers are daisies. So, remaining flowers for each vase = 10 – 2 = 8. If she made 5 arrangements, 8 X 5 = 40 other flowers.

Question 11.
In Mary’s homeroom, \(\frac{10}{28}\) of the students have a cat, \(\frac{6}{12}\) have a dog, and \(\frac{2}{14}\) have a pet bird. For numbers 11a–11c, select True or False for each statement.
a. In simplest form, \(\frac{5}{14}\) of the students have a cat.
i. True
ii. False

Answer:
i. True

Explanation:
In simplest form 5/14 of the students have a cat
From the above, 10/28 of the students have a cat
Divide the numerator and denominator of 10/28 with 2
(10÷2)/(28÷2) = 5/14
True

Question 11.
b. In simplest form, \(\frac{2}{4}\) of the students have a dog.
i. True
ii. False

Answer:
i. True

Explanation:
In simplest form, 2/4 of the students have a dog
From the above, 6/12 of the students have a dog
Divide the 6/12 with 3
(6 = 2/4
True

Question 11.
c. In simplest form, \(\frac{1}{7}\) of the students have a pet bird.
i. True
ii. False

Answer:
i. True

Explanation:
In the simplest form, 1/7 of the students have a pet bird
From the data, 2/14 of the students have a pet bird
Divide the numerator and denominator of 2/14 with 2
(2÷2)/(14÷2) = 1/7
True

Page No. 380

Question 12.
Regina, Courtney, and Ellen hiked around Bear Pond. Regina hiked \(\frac{7}{10}\) of the distance in an hour. Courtney hiked \(\frac{3}{6}\) of the distance in an hour. Ellen hiked 38 of the distance in an hour. Compare the distances hiked by each person by matching the statements to the correct symbol. Each symbol may be used more
than once or not at all.
Go Math Grade 4 Answer Key Chapter 6 Fraction Equivalence and Comparison img 33
Type below:
_________

Answer:
chapter 6 - compare fractions and order fractions- image13

Explanation:
From the given information
Regina, Courtney, and Ellen hiked around Bear Pond
Regina hiked 7/10 of the distance in an hour
Courtney hiked 3/6 of the distance in an hour
Ellen hiked 3 /8 of the distance in an hour
Compare 7/10 and 3/6
The common denominator of 7/10 and 3/6 is 30
(7×3)/(10×3) and (3×5)/(6×5)
21/30 and 15/30
So, 21/30 > 15/30
So, 7/10 > 15/30
Compare 3/8 and 3/6
The common denominator of 3/8 and 3/6 is 24
(3×3)/(8×3) and (3×4)/(6×4)
9/24 and 12/24 = 9/24 < 12/24 = 3/8 < 3/6
Compare 7/10 and 3/8
The common denominator of 7/10 and 3/8 is 40
(7×4)/(10×4) and (3×5)/(8×5)
28/40 >15/40 = 7/10 > 3/8

Question 13.
Ramon is having some friends over after a baseball game. Ramon’s job is to make a vegetable dip. The ingredients for the recipe are given.
Go Math Grade 4 Answer Key Chapter 6 Fraction Equivalence and Comparison img 34
Part A
Which ingredient does Ramon use the greater amount of, buttermilk or cream cheese? Explain how you found your answer.
Type below:
_________

Answer:
Ramon use 5/8 cup of buttermilk and 1/2 cup cream cheese
By comparing these two ingredients
The common denominator of 5/8 and 1/2 are 8
(1×4)/(2×4) =4/8
So, 5/8 > 4/8
So, 5/8 cup buttermilk is > ½ cup cream cheese

Question 13.
Part B
Ramon says that he needs the same amount of two different ingredients. Is he correct? Support your answer with information from the problem.
______

Answer:
Ramon says that he needs the same amount of two ingredients
Yes, Ramon uses 3/4 cup parsley and 6/8 cup scallions
Multiply the 3/4 with 2
(3×2)/(4×2) = 6/8
So, Ramon uses the same amount that is 3/4 cup for parsley and scallions

Page No. 381

Question 14.
Sandy is ordering bread rolls for her party. She wants \(\frac{3}{5}\) of the rolls to be whole wheat. What other fractions can represent the part of the rolls that will be whole wheat? Shade the models to show your work.
Go Math Grade 4 Answer Key Chapter 6 Fraction Equivalence and Comparison img 35
Type below:
_________

Answer:
chapter 6 - compare fractions and order fractions- image15

Explanation:
As per the information,
Sandy is ordering bread rolls for her party
She wants 3/5 of the rolls to be whole wheat
For an equivalent fraction of 3/5, multiply with 5
(3×5)/(5×5) = 15/25
Again multiply the 15/25 with 4
(15×4)/(25×4) = 60/100

Question 15.
Angel has \(\frac{4}{8}\) yard of ribbon and Lynn has \(\frac{3}{4}\) yard of ribbon. Do Angel and Lynn have the same amount of ribbon? Shade the model to show how you found your answer. Explain your reasoning.
Go Math Grade 4 Answer Key Chapter 6 Fraction Equivalence and Comparison img 36
Type below:
_________

Answer:
grade 4 chapter 6 image 4
Angel and Lynn didn’t have the same amount of ribbon. 4/8 is a greater fraction compared to 3/4. So, Angel’s ribbon is long compared to Lynn’s ribbon.

Question 16.
Ella used \(\frac{1}{4}\) yard of red ribbon. Fill in each box with a number from the list to show equivalent fractions for \(\frac{1}{4}\). Not all numbers will be used.
Go Math Grade 4 Answer Key Chapter 6 Fraction Equivalence and Comparison img 37
Type below:
_________

Answer:
grade 4 chapter 6 image 3

Explanation:
1/4 = 2/8 = 4/16 = 3/12

Page No. 382

Question 17.
Frank has two same-size rectangles divided into the same number of equal parts. One rectangle has \(\frac{3}{4}\) of the parts shaded, and the other has \(\frac{1}{3}\) of the parts shaded.
Part A
Into how many parts could each rectangle be divided? Show your work by drawing the parts of each rectangle.
Go Math Grade 4 Answer Key Chapter 6 Fraction Equivalence and Comparison img 38
_____ parts

Answer:
grade 4 chapter 6 image 2
12 parts

Question 17.
Part B
Is there more than one possible answer to Part A? If so, did you find the least number of parts into which both rectangles could be divided? Explain your reasoning.
Type below:
_________

Answer:
Yes, as long it is a multiple of 12.
And yes,12 is the least in order to have 1 rectangle have 3/4 shaded and the other 1/3 shaded.

Question 18.
Suki rode her bike \(\frac{4}{5}\) mile. Claire rode her bike \(\frac{1}{3}\) mile. They want to compare how far they each rode their bikes using the benchmark \(\frac{1}{2}\). For numbers 18a–18c, select the correct answers to describe how to solve the problem.
a. Compare Suki’s distance to the benchmark:
\(\frac{4}{5}\) _____ \(\frac{1}{2}\)

Answer:
\(\frac{4}{5}\) ≠ \(\frac{1}{2}\)

Explanation:
The fraction \(\frac{4}{5}\) is not equal to \(\frac{1}{2}\).

Question 18.
b. Compare Claire’s distance to the benchmark:
\(\frac{1}{3}\) _____ \(\frac{1}{2}\)

Answer:
\(\frac{1}{3}\) ≠ \(\frac{1}{2}\)

Explanation:
The fraction \(\frac{1}{3}\) is not equal to \(\frac{1}{2}\)

Question 18.
c. Suki rode her bike _____ Claire.

Answer:
Suki rode her bike faster than Claire.

Page No. 387

Use the model to write an equation.

Question 1.
Go Math Grade 4 Answer Key Chapter 6 Fraction Equivalence and Comparison img 39
Type below:
_________

Answer:
\(\frac{3}{5}\) + \(\frac{1}{5}\) = \(\frac{4}{5}\)

Question 2.
Go Math Grade 4 Answer Key Chapter 6 Fraction Equivalence and Comparison img 40
Type below:
_________

Answer:
\(\frac{2}{3}\) – \(\frac{1}{3}\) = \(\frac{1}{3}\)

Question 3.
Go Math Grade 4 Answer Key Chapter 6 Fraction Equivalence and Comparison img 41
Type below:
_________

Answer:
\(\frac{1}{4}\) + \(\frac{1}{4}\) = \(\frac{2}{4}\)

Question 4.
Go Math Grade 4 Answer Key Chapter 6 Fraction Equivalence and Comparison img 42
Type below:
_________

Answer:
1 – \(\frac{5}{8}\) = \(\frac{8}{8}\) – \(\frac{5}{8}\) = \(\frac{3}{8}\)

Use the model to solve the equation.

Question 5.
Go Math Grade 4 Answer Key Chapter 6 Fraction Equivalence and Comparison img 43
\(\frac{3}{4}-\frac{1}{4}\) = \(\frac{□}{□}\)

Answer:
\(\frac{2}{4}\)

Question 6.
Go Math Grade 4 Answer Key Chapter 6 Fraction Equivalence and Comparison img 44
\(\frac{5}{6}+\frac{1}{6}\) = \(\frac{□}{□}\)

Answer:
\(\frac{6}{6}\) = 1

Question 7.
Reason Abstractly Sean has \(\frac{1}{5}\) of a cupcake and \(\frac{1}{5}\) of a large cake.
a. Are the wholes the same? Explain.
______

Answer:
Yes; From the given information, the fraction of the cupcake and large cake are the same.

Explanation:

Question 7.
Does the sum \(\frac{1}{5}+\frac{1}{5}=\frac{2}{5}\) make sense in this situation? Explain.
______

Answer:
Yes; it makes sense. From the given data, 1 part is out of 5 parts. So, adding two fractions (1 part is out of 5 parts), the complete fraction becomes 2/5.

Question 8.
Carrie’s dance class learned \(\frac{1}{5}\) of a new dance on Monday, and \(\frac{2}{5}\) of the dance on Tuesday. What fraction of the dance is left for the class to learn on Wednesday?
\(\frac{□}{□}\)

Answer:
\(\frac{3}{5}\)

Explanation:
The fraction of left for the class to learn on Wednesday is \(\frac{3}{5}\).

Page No. 388

Question 9.
Samantha and Kim used different models to help find \(\frac{1}{3}+\frac{1}{6}\). Whose model makes sense? Whose model is nonsense? Explain your reasoning below each model.
Go Math Grade 4 Answer Key Chapter 6 Fraction Equivalence and Comparison img 45

Answer:
Both Samantha and Kim’s statements make sense. Because both models have an equal number of fractions for each diagram.

Question 10.
Draw a model you could use to add \(\frac{1}{4}+\frac{1}{2}\).
Type below:
___________

Answer:
grade 4 chapter 6 image 1

Question 11.
Cindy has two jars of paint. One jar is \(\frac{3}{8}\) full. The other jar is \(\frac{2}{8}\) full. Use the fractions to write an equation that shows the amount of paint Cindy has.
Go Math Grade 4 Answer Key Chapter 6 Fraction Equivalence and Comparison img 46
Go Math Grade 4 Answer Key Chapter 6 Fraction Equivalence and Comparison img 47
Type below:
___________

Answer:
\(\frac{3}{8}+\frac{2}{8}\) = \(\frac{5}{8}\)

Conclusion:

By downloading the Go Math Grade 4 Answer Key Chapter 6 Fraction Equivalence and Comparison PDF, students of grade 4 will aid you to understand different topics in Chapter 6 easily. Prepare well with the help of Go Math Grade 4 Answer Key PDFand solve each and every question properly. For more help utilize this Go Math Grade 4 Solution Key Chapter 6 Fraction Equivalence and Comparison PDF and gain what you require.

Go Math Grade 5 Answer Key Chapter 2 Divide Whole Numbers

go-math-grade-5-chapter-2-divide-whole-numbers-answer-key

Trying to Score Better Scores in Grade 5 Maths? Utilize Go Math Grade 5 Answer Key Chapter 2 Divide Whole Numbers and make the most out of them. Begin your practice right away using the Go Math Grade 5 Answer Key and understand the concepts. HMH Go Math Grade 5 Practice Tests along with questions and answers exist here. Attain more knowledge using the Go Math Grade 5 Chapter 2 Divide Whole Numbers Solution Key and cross-check the Solutions from it after your practice sessions.

Go Math Grade 5 Chapter 2 Divide Whole Numbers Answer Key

Apply Maths in your Real-Time and get the Tips & Tricks to Solve Various Problems using  HMH Go Math Grade 5 Answer Key. We have provided shortcuts to solve all the Problems in Go Math Chapter 2 Divide Whole Numbers. Try to utilize the resource available Go Math Grade 5 Chapter 2 Divide Whole Numbers Answer Key and make your preparation way more effective.

Lesson 1: Place the First Digit

Lesson 2: Divide by 1-Digit Divisors

Lesson 3: Investigate • Division with 2-Digit Divisors

Lesson 4: Partial Quotients

Mid-Chapter Checkpoint

Lesson 5: Estimate with 2-Digit Divisors

Lesson 6: Divide by 2-Digit Divisors

Lesson 7: Interpret the Remainder

Lesson 8: Adjust Quotients

Lesson 9: Problem Solving • Division

Review/Test

Place the First Digit – Share and Show – Page No. 63

Divide.

Question 1.
3)\(\overline { 579 } \)
_____

Answer:
193

Explanation:
Divide integers 57/3 = 19
Multiply 19 x 3 = 57; Subtract 57 – 57 = 0
Write down 9 and divide integers 9/3 = 3.
Multiply 3 x 3 = 9. Subtract 9 – 9 = 0.
The remainder is 0.
So, 193 is the answer.

Question 2.
5)\(\overline { 1,035 } \)
_____

Answer:
207

Explanation:
Divide integers 10/5 = 2
Multiply 2 x 5 = 10; Subtract 10 – 10 = 0
Write down 35 and divide integers 35/5 = 7.
Multiply 7 x 5 = 35. Subtract 35 – 35 = 0.
The remainder is 0.
So, 207 is the answer.

Question 3.
8)\(\overline { 1,766 } \)
_____ R _____

Answer:
220 R 6

Explanation:
Divide integers 17/8 = 2
Multiply 2 x 8 = 16; Subtract 17 – 16 = 1
Write down 16 and divide integers 16/8 = 2.
Multiply 2 x 8 = 16. Subtract 16 – 16 = 0.
Write down 6; 6 < 8. There are not enough tens
So, the remainder is 6
So, 220 is the answer.

On Your Own

Divide.

Question 4.
8)\(\overline { 275 } \)
_____ R _____

Answer:
43 R 3

Explanation:
Divide integers 27/8 = 3
Multiply 8 x 3 = 24; Subtract 27 – 24= 3
Write down 3 and divide integers 35/8 = 4.
Multiply 8 x 4 = 32. Subtract 35 – 32 = 3.
The remainder is 3.
So, 43 is the answer.

Question 5.
3)\(\overline { 468 } \)
_____

Answer:
155 R 3

Explanation:
Divide integers 46/3 = 15
Multiply 3 x 15 = 45; Subtract 46 – 45= 1
Write down 18 and divide integers 18/3 = 5.
Multiply 3 x 5 = 15. Subtract 18 – 15 = 3.
The remainder is 3.
So, 155 is the answer.

Question 6.
4)\(\overline { 3,220 } \)
_____

Answer:
805

Explanation:
Divide integers 32/4 = 8
Multiply 4 x 8 = 32; Subtract 32 – 32 = 0
Write down 20 and divide integers 20/4 = 5.
Multiply 4 x 5 = 20. Subtract 20 – 20= 0.
The remainder is 0.
So, 805 is the answer.

Question 7.
6)\(\overline { 618 } \)
_____

Answer:
103

Explanation:
Divide integers 61/6 = 10
Multiply 6 x 10 = 60; Subtract 61 – 60 = 1
Write down 18 and divide integers 18/6 = 3.
Multiply 6 x 3 = 18. Subtract 18 – 18 = 0.
The remainder is 0.
So, 103 is the answer.

Question 8.
4)\(\overline { 716 } \)
_____

Answer:
179

Explanation:
Divide integers 71/4 = 17
Multiply 4 x 17 = 68; Subtract 71 – 68 = 3
Write down 36 and divide integers 36/4 = 9.
Multiply 4 x 9 = 36. Subtract 36 – 36 = 0.
The remainder is 0.
So, 179 is the answer.

Question 9.
9)\(\overline { 1,157 } \)
_____ R _____

Answer:
128 R 5

Explanation:
Divide integers 11/9 = 1
Multiply 9 x 1 = 9; Subtract 11 – 9 = 2
Write down 25 and divide integers 25/9 = 2.
Multiply 9 x 2 = 18. Subtract 25 – 18 = 7.
Write down 77 and divide integers 77/9 = 8.
Multiply 9 x 8 = 72. Subtract 77 – 72= 5.
The remainder is 5.
So, 128 is the answer.

Question 10.
6)\(\overline { 6,827 } \)
_____ R _____

Answer:
1,137 R 5

Explanation:
Divide integers 6/6 = 1
Multiply 6 x 1 = 6; Subtract 6 – 6 = 0
Write down 82 and divide integers 82/6 = 13.
Multiply 6 x 13 = 78. Subtract 82 – 78 = 4.
Write down 47 and divide integers 47/6 = 7.
Multiply 6 x 7 = 42. Subtract 47 – 42= 5.
The remainder is 5.
So, 1,137 is the answer.

Question 11.
7)\(\overline { 8,523 } \)
_____ R _____

Answer:
1,217 R 4

Explanation:
Divide integers 8/7 = 1
Multiply 7 x 1 = 7; Subtract 8 – 7 = 1
Write down 15 and divide integers 15/7 = 2.
Multiply 7 x 2 = 14. Subtract 15 – 14 = 1.
Write down 12 and divide integers 12/7 = 1.
Multiply 7 x 1 = 7. Subtract 12 – 7= 5.
Write down 53 and divide integers 53/7 = 7.
Multiply 7 x 7 = 49. Subtract 53 – 49= 4.
The remainder is 4.
So, 1,217 is the answer.

Practice: Copy and Solve Divide.

Question 12.
645 ÷ 8 = _____ R _____

Answer:
645 ÷ 8 = 80 R 5

Explanation:
Divide integers 64/8 = 8
Multiply 8 x 8 = 64; Subtract 64 – 64 = 0
Write down 05; 5 < 8; There are not enough tens
The remainder is 5.
So, 80 is the answer.

Question 13.
942 ÷ 6 = _____

Answer:
157

Explanation:
Divide integers 9/6 = 1
Multiply 6 x 1 = 6; Subtract 9 – 6 = 3
Write down 34 and divide integers 34/6 = 5.
Multiply 6 x 5 = 30. Subtract 34 – 30 = 4.
Write down 42 and divide integers 42/6 = 7.
Multiply 6 x 7 = 42. Subtract 42 – 42 = 0.
The remainder is 0.
So, 157 is the answer.

Question 14.
723 ÷ 7 = _____ R _____

Answer:
103 R 2

Explanation:
Divide integers 7/7 = 1
Multiply 7 x 1 = 7; Subtract 7 – 7 = 0
Write down 23 and divide integers 23/7 = 3.
Multiply 7 x 3 = 21. Subtract 23 – 21 = 2.
The remainder is 2.
So, 103 is the answer.

Question 15.
3,478 ÷ 9 = _____ R _____

Answer:
386 R 4

Explanation:
Divide integers 34/9 = 3
Multiply 9 x 3 = 27; Subtract 34 – 27 = 7
Write down 77 and divide integers 77/9 = 8.
Multiply 9 x 8 = 72. Subtract 77 – 72 = 5.
Write down 58 and divide integers 58/9 = 6.
Multiply 9 x 6 = 54. Subtract 58 – 54= 4.
The remainder is 4.
So, 386 is the answer.

Question 16.
3,214 ÷ 5 = _____ R _____

Answer:
642 R 4

Explanation:
Divide integers 32/5 = 6
Multiply 5 x 6 = 30; Subtract 32 – 30 = 2
Write down 21 and divide integers 21/5 = 4.
Multiply 5 x 4 = 20. Subtract 21 – 20 = 1.
Write down 14 and divide integers 14/5 = 2.
Multiply 5 x 2 = 10. Subtract 14 – 10 = 4.
The remainder is 4.
So, 642 is the answer.

Question 17.
492 ÷ 4 = _____

Answer:
123

Explanation:
Divide integers 4/4 = 1
Multiply 4 x 1 = 4; Subtract 4 – 4 = 0
Write down 9 and divide integers 9/4 = 2.
Multiply 4 x 2 = 8. Subtract 9 – 8 = 1.
Write down 12 and divide integers 12/4 = 3.
Multiply 4 x 3 = 12. Subtract 12 – 12 = 0.
The remainder is 0.
So, 123 is the answer.

Question 18.
2,403 ÷ 9 = _____

Answer:
267

Explanation:
Divide integers 24/9 = 2
Multiply 9 x 2 = 18; Subtract 24 – 18 = 6
Write down 60 and divide integers 60/9 = 6.
Multiply 9 x 6 = 54. Subtract 60 – 54 = 6.
Write down 63 and divide integers 63/9 = 7.
Multiply 9 x 7 = 63. Subtract 63 – 63 = 0.
The remainder is 0.
So, 267 is the answer.

Question 19.
2,205 ÷ 6 = _____ R _____

Answer:
367 R 3

Explanation:
Divide integers 22/6 = 3
Multiply 6 x 3 = 18; Subtract 22 – 18 = 4
Write down 40 and divide integers 40/6 = 6.
Multiply 6 x 6 = 36; Subtract 40 – 36 = 4
Write down 45 and divide integers 45/6 = 7.
Multiply 6 x 7 = 42; Subtract 45 – 42 = 3
The remainder is 3.
So, 367 is the answer.

Question 20.
2,426 ÷ 3 = _____ R _____

Answer:
808 R 2

Explanation:
Divide integers 24/3 = 8
Multiply 3 x 8 = 24; Subtract 24 – 24 = 0
Write down 26 and divide integers 26/3 = 8.
Multiply 3 x 8 = 24. Subtract 26 – 24 = 2.
The remainder is 2.
So, 808 is the answer.

Question 21.
1,592 ÷ 8 = _____ R _____

Answer:
199

Explanation:
Divide integers 15/8 = 1
Multiply 8 x 1 = 8; Subtract 15 – 8 = 7
Write down 79 and divide integers 79/8 = 9.
Multiply 8 x 9 = 72; Subtract 79 – 72 = 7
Write down 72 and divide integers 72/8 = 9.
Multiply 8 x 9 = 72; Subtract 72 – 72 = 0
The remainder is 0.
So, 199 is the answer.

Question 22.
926 ÷ 4 = _____ R _____

Answer:
231 R 2

Explanation:
Divide integers 9/4 = 2
Multiply 4 x 2 = 8; Subtract 9 – 8 = 1
Write down 12 and divide integers 12/4 = 3.
Multiply 4 x 3 = 12; Subtract 12 – 12 = 0
Write down 6 and divide integers 6/4 = 1.
Multiply 4 x 1 = 4; Subtract 6 – 4 = 2
The remainder is 2.
So, 231 is the answer.

Question 23.
6,033 ÷ 5 = _____ R _____

Answer:
1,206 R 3

Explanation:
Divide integers 6/5 = 1
Multiply 5 x 1 = 5; Subtract 6 – 5 = 1
Write down 10 and divide integers 10/5 = 2.
Multiply 5 x 2 = 10; Subtract 10 – 10 = 0
Write down 33 and divide integers 33/5 = 6.
Multiply 5 x 6 = 30; Subtract 33 – 30 = 3
The remainder is 3.
So, 1206 is the answer.

Place the First Digit – UNLOCK the Problem – Page No. 64

Question 24.
Rosa has a garden divided into sections. She has 125 daisy plants. If she plants an equal number of the daisy plants in each section of daisies, will she have any left over? If so, how many daisy plants will be left over?
Go Math Grade 5 Answer Key Chapter 2 Divide Whole Numbers Divide Whole Numbers; Place the First Digit img 1
a. What information will you use to solve the problem?
Type below:
__________

Answer:
We can use the fact that she has 125 daisy plants and she plants an equal number of the daisy plants in each of 3 sections.

Question 24.
b. How will you use division to find the number of daisy plants left over?
Type below:
__________

Answer:
We have to do 125/3
Divide integers 12/3 = 4
Multiply 3 x 4 = 12; Subtract 12 – 12 = 0
Write down 5 and divide integers 5/3 = 1.
Multiply 3 x 1 = 3; Subtract 5 – 3 = 2
The remainder is 2.
41 daisy plants in each section.
2 daisy plants left over

Question 24.
c. Show the steps you use to solve the problem. Estimate: 120 ÷ 3 = _____
Type below:
__________

Answer:
Divide integers 12/3 = 4
Multiply 3 x 4 = 12; Subtract 12 – 12 = 0
The remainder is 0.
So, 40 is the answer.

Question 24.
d. Complete the sentences:
Rosa has _____ daisy plants.
She puts an equal number in each of _____ sections.
Each section has _____ plants.
Rosa has _____ daisy plants left over.
Type below:
__________

Answer:
Rose has 125 daisy planes.
She puts an equal number in each of 3 sections.
Each section has 41 plants.
Rosa has 2 daisy plants left over.

Question 25.
One case can hold 3 boxes. Each box can hold 3 binders. How many cases are needed to hold 126 binders?
_____ cases

Answer:
14 cases

Explanation:
One case can hold 3 boxes. Each box can hold 3 binders. 3 x 3 = 9.
For 12 binders,
126/ (3 x 3) = 126/9 = 14

Question 26.
Test Prep In which place is the first digit in the quotient 1,497 ÷ 5?
Options:
a. thousands
b. hundreds
c. tens
d. ones

Answer:
b. hundreds

Explanation:
1,497 ÷ 5 = 499. the first digit 4 is in hundreds place.

Divide by 1-Digit Divisors – Share and Show – Page No. 67

Divide. Check your answer.

Question 1.
8)\(\overline { 624 } \)
Check
_____

Answer:
78

Explanation:
Divide integers 62/8 = 7
Multiply 8 x 7 = 56; Subtract 62 – 56 = 6
Write down 64 and divide integers 64/8 = 8.
Multiply 8 x 8 = 64. Subtract 64 – 64 = 0.
The remainder is 0.
So, 78 is the answer.
Check:
78 x 8 = 624;
624 = 624

Question 2.
4)\(\overline { 3,220 } \)
Check
_____

Answer:
805

Explanation:
Divide integers 32/4 = 8
Multiply 4 x 8 = 32; Subtract 32 – 32 = 0
Write down 20 and divide integers 20/4 = 5.
Multiply 4 x 5 = 20. Subtract 20 – 20 = 0.
The remainder is 0.
So, 805 is the answer.
Check:
805 x 4 = 3,220;
3,220 = 3,220.

Question 3.
4)\(\overline { 1,027 } \)
Check
_____ R _____

Answer:
256 R 3

Explanation:
Divide integers 10/4 = 2
Multiply 4 x 2 = 8; Subtract 10 – 8 = 2
Write down 22 and divide integers 22/4 = 5.
Multiply 4 x 5 = 20. Subtract 22 – 20= 2.
Write down 27 and divide integers 27/4 = 6.
Multiply 4 x 6 = 24. Subtract 27 – 24 = 3.
The remainder is 3.
So, 256 R 3.
Check:
256 x 4 = 1,024;
1,024 + 3 = 1,027.
1,027 = 1,027

On Your Own

Divide.

Question 4.
6)\(\overline { 938 } \)
_____ R _____

Answer:
156 R 2

Explanation:
Divide integers 9/6 = 1
Multiply 6 x 1 = 6; Subtract 9 – 6 = 3
Write down 33 and divide integers 33/6 = 5.
Multiply 6 x 5 = 30. Subtract 33 – 30 = 3.
Write down 38 and divide integers 38/6 = 6.
Multiply 6 x 6 = 36. Subtract 38 – 36 = 2.
The remainder is 2.
So, 156 R 2.

Question 5.
4)\(\overline { 762 } \)
_____ R _____

Answer:
190 R 2

Explanation:
Divide integers 7/4 = 1
Multiply 4 x 1 = 4; Subtract 7 – 4 = 3
Write down 36 and divide integers 36/4 = 9.
Multiply 4 x 9 = 36. Subtract 36 – 36 = 0.
Write down 2. 2 < 4; There are not enough tens
The remainder is 2.
So, 190 R 2.

Question 6.
3)\(\overline { 5,654 } \)
_____ R _____

Answer:
1884 R 2

Explanation:
Divide integers 5/3 = 1
Multiply 3 x 1 = 3; Subtract 5 – 3 = 2
Write down 26 and divide integers 26/3 = 8.
Multiply 3 x 8 = 24. Subtract 26 – 24 = 2.
Write down 25 and divide integers 25/3 = 8.
Multiply 3 x 8 = 24. Subtract 25 – 24 = 1.
Write down 14 and divide integers 14/3 = 4.
Multiply 3 x 4 = 12. Subtract 14 – 12 = 2.
The remainder is 2.
So, 1884 R 2.

Question 7.
8)\(\overline { 475 } \)
_____ R _____

Answer:
59 R 3

Explanation:
Divide integers 47/8 = 5
Multiply 8 x 5 = 40; Subtract 47 – 40 = 7
Write down 75 and divide integers 75/8 = 9.
Multiply 9 x 8 = 72. Subtract 75 – 72 = 3.
The remainder is 3.
So, 59 R 3.

Practice: Copy and Solve Divide.

Question 8.
4)\(\overline { 671 } \)
_____ R _____

Answer:
167 R 3

Explanation:
Divide integers 6/4 = 1
Multiply 4 x 1 = 4; Subtract 6 – 4 = 2
Write down 27 and divide integers 27/4 = 6.
Multiply 4 x 6 = 24. Subtract 27 – 24 = 3.
Write down 31 and divide integers 31/4 = 7.
Multiply 4 x 7 = 28. Subtract 31 – 28 = 3.
The remainder is 3.
So, 167 R 3.

Question 9.
9)\(\overline { 2,023 } \)
_____ R _____

Answer:
224 R 7

Explanation:
Divide integers 20/9 = 2
Multiply 9 x 2 = 18; Subtract 20 – 18 = 2
Write down 22 and divide integers 22/9 = 2.
Multiply 9 x 2 = 18. Subtract 22 – 18 = 4.
Write down 43 and divide integers 43/9 = 4.
Multiply 9 x 4 = 36. Subtract 43 – 36 = 7.
The remainder is 7.
So, 224 R 7.

Question 10.
3)\(\overline { 4,685 } \)
_____ R _____

Answer:
1,561 R 2

Explanation:
Divide integers 4/3 = 1
Multiply 3 x 1 = 3; Subtract 4 – 3 = 1
Write down 16 and divide integers 16/3 = 5.
Multiply 3 x 5 = 15. Subtract 16 – 15 = 1.
Write down 18 and divide integers 18/3 = 6.
Multiply 3 x 6 = 18. Subtract 18 – 18 = 0.
Write down 5 and divide integers 5/3 = 1.
Multiply 3 x 1 = 3. Subtract 5 – 3 = 2.
The remainder is 2.
So, 1,561 R 2.

Question 11.
8)\(\overline { 948 } \)
_____ R _____

Answer:
118 R 4

Explanation:
Divide integers 9/8 = 1
Multiply 8 x 1 = 8; Subtract 9 – 8 = 1
Write down 14 and divide integers 14/8 = 1.
Multiply 8 x 1 = 8. Subtract 14 – 8 = 6.
Write down 68 and divide integers 68/8 = 8.
Multiply 8 x 8 = 64. Subtract 68 – 64 = 4.
The remainder is 4.
So, 118 R 4.

Question 12.
1,326 ÷ 4 = _____ R _____

Answer:
331 R 2

Explanation:
Divide integers 13/4 = 3
Multiply 4 x 3 = 12; Subtract 13 – 12 = 1
Write down 12 and divide integers 12/4 = 3.
Multiply 4 x 3 = 12. Subtract 12 – 12 = 0.
Write down 6 and divide integers 6/4 = 1.
Multiply 4 x 1 = 4. Subtract 6 – 4 = 2.
The remainder is 2.
So, 331 R 2.

Question 13.
5,868 ÷ 6 = _____

Answer:
978

Explanation:
Divide integers 58/6 = 9
Multiply 6 x 9 = 54; Subtract 58 – 54 = 4
Write down 46 and divide integers 46/6 = 7.
Multiply 6 x 7 = 42. Subtract 46 – 42 = 4.
Write down 48 and divide integers 48/6 = 8.
Multiply 6 x 8 = 48. Subtract 48 – 48 = 0.
The remainder is 0.
So, 978.

Question 14.
566 ÷ 3 = _____ R _____

Answer:
188 R 2

Explanation:
Divide integers 5/3 = 1
Multiply 3 x 1 = 3; Subtract 5 – 3 = 2
Write down 26 and divide integers 26/3 = 8.
Multiply 3 x 8 = 24. Subtract 26 – 24 = 2.
Write down 26 and divide integers 26/3 = 8.
Multiply 3 x 8 = 24. Subtract 26 – 24 = 2.
The remainder is 2.
So, 188 R 2.

Question 15.
3,283 ÷ 9 = _____ R _____

Answer:
364 R 7

Explanation:
Divide integers 32/9 = 3
Multiply 9 x 3 = 27; Subtract 32 – 27 = 5
Write down 58 and divide integers 58/9 = 6.
Multiply 9 x 6 = 54. Subtract 58 – 54 = 4.
Write down 43 and divide integers 43/9 = 4.
Multiply 9 x 4 = 36. Subtract 43 – 36 = 7.
The remainder is 7.
So, 364 R 7.

Algebra Find the value of n in each equation. Write what n represents in the related division problem.

Question 16.
n = 4 × 58
Value of n = _______
Represents: _______

Answer:
Value of n = 232
Represents: dividend

Explanation:
n = 4 × 58;
232 = 4 x 58;
n is the dividend

Question 17.
589 = 7 × 84 + n
Value of n = _______
Represents: _______

Answer:
Value of n = 1
Represents: remainder

Explanation:
589 = 7 × 84 + n
589 = 588 + n;
589 – 588 = n;
1 = n
n is the remainder

Question 18.
n = 5 × 67 + 3
Value of n = _______
Represents: _______

Answer:
Value of n = 338
Represents: dividend

Explanation:
n = 5 × 67 + 3
n = 335 + 3
n = 338
n is the dividend

Divide by 1-Digit Divisors – Problem Solving – Page No. 68

Use the table to solve 19–20.
Go Math Grade 5 Answer Key Chapter 2 Divide Whole Numbers Divide Whole Numbers; Divide by 1-Digit Divisors img 2

Question 19.
If the Welcome gold nugget were turned into 3 equal-sized gold bricks, how many troy ounces would each brick weigh?
_____ troy ounces

Answer:
739 troy ounces

Explanation:
Welcome gold nugget = 2,217 troy ounces.
If it turned into 3 equal-sized gold bricks, 2,217/3 = 739.
739 troy ounces

Question 20.
Pose a Problem Look back at Problem 19. Write a similar problem by changing the nugget and the number of bricks. Then solve the problem.
Type below:
__________

Answer:
571 troy ounces

Explanation:
If Welcome Stranger nugget were turned into 4 equal-sized gold bricks, how many troy ounces would each brick weigh?
Welcome Stranger nugget = 2,284.
If it turned into 4 equal-sized gold bricks, 2,217/3 = 571.
571 troy ounces

Question 21.
There are 246 students going on a field trip to pan for gold. If they are going in vans that hold 9 students each, how many vans are needed? How many students will ride in the van that isn’t full?
The number of vans: _________
_________ students in the van that isn’t full

Answer:
The number of vans: 27
3 students will ride in the van that isn’t full

Explanation:
There are 246 students going on a field trip to pan for gold. If they are going in vans that hold 9 students each, 246/9 = 27 R 3
The number of vans: 27
3 students will ride in the van that isn’t full
Question 22.
One crate can hold 8 cases of trading cards. How many crates are needed to hold 128 cases of trading cards?
_____ crates

Answer:
16 crates

Explanation:
One crate can hold 8 cases of trading cards. To hold 128 cases of trading cards, 128/8 = 16 crates needed.

Question 23.
Test Prep At a bake sale, a fifth-grade class sold 324 cupcakes in packages of 6. How many packages of cupcakes did the class sell?
Options:
a. 1,944
b. 108
c. 64
d. 54

Answer:
d. 54

Explanation:
At a bake sale, a fifth-grade class sold 324 cupcakes in packages of 6. 324/6 = 54

Division with 2-Digit Divisors – Share and Show – Page No. 71

Use the quick picture to divide.

Question 1.
Go Math Grade 5 Answer Key Chapter 2 Divide Whole Numbers Divide Whole Numbers; Division with 2-Digit Divisors img 3
143 ÷ 13 = _____

Answer:
grade 5 chapter 2 Division with 2-Digit Divisors image 1
143 ÷ 13 = 11

Explanation:
143 = 100 + 40 + 3
Model the first partial quotient by making a rectangle with the hundred and 3 tens. In the Record section, cross out the hundred and tens you use.
(10 x 10) + (10 + 10 + 10) = 100 + 30 = 130.
The rectangle shows 10 groups of 13.
Model the second partial quotient by making a line with the ten and 3 ones. In the Record section, cross out the ten and ones you use.
10 + (1 + 1 + 1) = 10 + 3 = 13
130 + 13 = 143;
So, the answer is 10 + 1 = 11

Divide. Use base-ten blocks.

Question 2.
168 ÷ 12 = _____

Answer:
grade 5 chapter 2 Division with 2-Digit Divisors image 2
168 ÷ 12 = 14

Explanation:
168 ÷ 12
Model the first partial quotient by making a rectangle with the hundred and 2 tens. In the Record section, cross out the hundred and tens you use.
(10 x 10) + (10 + 10) = 100 + 20 = 120.
The rectangle shows 10 groups of 12.
Model the second partial quotient by making a line with the ten and 2 ones. In the Record section, cross out the ten and ones you use.
10 + (1 + 1) = 10 + 2 = 12.
Repeat the above step more three times to get
120 + 12 + 12 + 12 + 12 = 168;
So, the answer is 10 + 1 + 1 + 1 + 1 = 14

Question 3.
154 ÷ 14 = _____

Answer:
grade 5 chapter 2 Division with 2-Digit Divisors image 3
154 ÷ 14 = 11

Explanation:
154 ÷ 14
Model the first partial quotient by making a rectangle with the hundred and 4 tens. In the Record section, cross out the hundred and tens you use.
(10 x 10) + (10 + 10 + 10 + 10) = 100 + 40 = 140.
The rectangle shows 10 groups of 14.
Model the second partial quotient by making a line with the ten and 4 ones. In the Record section, cross out the ten and ones you use.
10 + (1 + 1 + 1 + 1) = 10 + 4 = 14.
Repeat the above step more three times to get
140 + 14 = 154;
So, the answer is 10 + 1 = 11

Question 4.
187 ÷ 11 = _____

Answer:
grade 5 chapter 2 Division with 2-Digit Divisors image 4
187 ÷ 11 = 17

Explanation:
187 ÷ 11 =
Model the first partial quotient by making a rectangle with the hundred and 1 tens. In the Record section, cross out the hundred and tens you use.
(10 x 10) + (10) = 100 + 10 = 110.
The rectangle shows 10 groups of 11.
Model the second partial quotient by making a line with the ten and 1 ones. In the Record section, cross out the ten and ones you use.
10 + (1) = 10 + 1 = 11.
Repeat the above step more six times to get
110 + 11 + 11 + 11 + 11 + 11 + 11 + 11 = 187;
So, the answer is 10 + 1 + 1 + 1 + 1 + 1 + 1 + 1 = 17

Divide. Draw a quick picture.

Question 5.
165 ÷ 11 = _____

Answer:
grade 5 chapter 2 Division with 2-Digit Divisors image 5
165 ÷ 11 = 15

Explanation:
165 ÷ 11
Model the first partial quotient by making a rectangle with the hundred and 1 tens. In the Record section, cross out the hundred and tens you use.
(10 x 10) + (10) = 100 + 10 = 110.
The rectangle shows 10 groups of 11.
Model the second partial quotient by making a line with the ten and 1 ones. In the Record section, cross out the ten and ones you use.
10 + (1) = 10 + 1 = 11.
Repeat the above step more four times to get
110 + 11 + 11 + 11 + 11 + 11 = 165;
So, the answer is 10 + 1 + 1 + 1 + 1 + 1 = 15

Question 6.
216 ÷ 18 = _____

Answer:
grade 5 chapter 2 Division with 2-Digit Divisors image 6
216 ÷ 18 = 12

Explanation:
216 ÷ 18
Model the first partial quotient by making a rectangle with the hundred and 8 tens. In the Record section, cross out the hundred and tens you use.
(10 x 10) + (10 + 10 + 10 + 10 + 10 + 10 + 10 + 10) = 100 + 80 = 180.
The rectangle shows 10 groups of 18.
Model the second partial quotient by making a line with the ten and 8 ones. In the Record section, cross out the ten and ones you use.
10 + (1 + 1 + 1 + 1 + 1 + 1 + 1 + 1) = 10 + 8 = 18.
Repeat the above step to get
180 + 18 + 18  = 216;
So, the answer is 10 + 1 + 1 = 12

Question 7.
196 ÷ 14 = _____

Answer:
grade 5 chapter 2 Division with 2-Digit Divisors image 7
196 ÷ 14 = 14

Explanation:
196 ÷ 14
Model the first partial quotient by making a rectangle with the hundred and 4 tens. In the Record section, cross out the hundred and tens you use.
(10 x 10) + (10 + 10 + 10 + 10) = 100 + 40 = 140.
The rectangle shows 10 groups of 14.
Model the second partial quotient by making a line with the ten and 4 ones. In the Record section, cross out the ten and ones you use.
10 + (1 + 1 + 1 + 1) = 10 + 4 = 14.
Repeat the above step more three times to get
140 + 14 + 14 + 14 + 14  = 196;
So, the answer is 10 + 1 + 1 + 1 + 1 = 14

Question 8.
195 ÷ 15 = _____

Answer:
grade 5 chapter 2 Division with 2-Digit Divisors image 8
195 ÷ 15 = 13

Explanation:
195 ÷ 15
Model the first partial quotient by making a rectangle with the hundred and 5 tens. In the Record section, cross out the hundred and tens you use.
(10 x 10) + (10 + 10 + 10 + 10 + 10) = 100 + 50 = 150.
The rectangle shows 10 groups of 15.
Model the second partial quotient by making a line with the ten and 5 ones. In the Record section, cross out the ten and ones you use.
10 + (1 + 1 + 1 + 1 + 1) = 10 + 5 = 15.
Repeat the above step more three times to get
150 + 15 + 15 + 15  = 195;
So, the answer is 10 + 1 + 1 + 1 = 13

Question 9.
182 ÷ 13 = _____

Answer:
grade 5 chapter 2 Division with 2-Digit Divisors image 9
182 ÷ 13 = 14

Explanation:
182 ÷ 13
Model the first partial quotient by making a rectangle with the hundred and 3 tens. In the Record section, cross out the hundred and tens you use.
(10 x 10) + (10 + 10 + 10 ) = 100 + 30 = 130.
The rectangle shows 10 groups of 13.
Model the second partial quotient by making a line with the ten and 3 ones. In the Record section, cross out the ten and ones you use.
10 + (1 + 1 + 1) = 10 + 3 = 13.
Repeat the above step more four times to get
130 + 13 + 13 + 13 + 13 = 182;
So, the answer is 10 + 1 + 1 + 1 + 1 = 14

Question 10.
228 ÷ 12 = _____

Answer:
grade 5 chapter 2 Division with 2-Digit Divisors image 10
228 ÷ 12 = 19

Explanation:
228 ÷ 12
Model the first partial quotient by making a rectangle with the hundred and 2 tens. In the Record section, cross out the hundred and tens you use.
(10 x 10) + (10 + 10) = 100 + 20 = 120.
The rectangle shows 10 groups of 12.
Model the second partial quotient by making a line with the ten and 2 ones. In the Record section, cross out the ten and ones you use.
10 + (1 + 1) = 10 + 2 = 12.
Repeat the above step more eight times to get
120 + 12 + 12 + 12 + 12 + 12 + 12 + 12 + 12 + 12 = 228;
So, the answer is 10 + 1 + 1 + 1 + 1 + 1 + 1 + 1 + 1 + 1 = 19

Division with 2-Digit Divisors – Connect to Social Studies – Page No. 72

Pony Express

The Pony Express used men riding horses to deliver mail between St. Joseph, Missouri, and Sacramento, California, from April, 1860 to October, 1861. The trail between the cities was approximately 2,000 miles long. The first trip from St. Joseph to Sacramento took 9 days 23 hours. The first trip from Sacramento to St. Joseph took 11 days 12 hours.
Go Math Grade 5 Answer Key Chapter 2 Divide Whole Numbers Divide Whole Numbers; Division with 2-Digit Divisors img 4

Solve.

Question 11.
Suppose two Pony Express riders rode a total of 165 miles. If they replaced each horse with a fresh horse every 11 miles, how many horses would they have used?
_____ horses

Answer:
16 horses

Explanation:
Suppose two Pony Express riders rode a total of 165 miles. If they replaced each horse with a fresh horse every 11 miles. Then, 16 horses used.

Question 12.
Suppose a Pony Express rider was paid $192 for 12 weeks of work. If he was paid the same amount each week, how much was he paid for each week of work?
$ _____

Answer:
$16

Explanation:
Suppose a Pony Express rider was paid $192 for 12 weeks of work.
For each week. $192/12 = $16.

Question 13.
Suppose three riders rode a total of 240 miles. If they used a total of 16 horses, and rode each horse the same number of miles, how many miles did they ride before replacing each horse?
_____ miles

Answer:
15 miles

Explanation:
Assuming each horse was only ridden once then a total of 16 horses were ridden for a total of 240 miles
240 miles/16 horses = 15 miles/horse
if each horse was ridden more than once before being replaced the distance between replacements could be reduced.
The fact that there were 3 riders is irrelevant.

Question 14.
Suppose it took 19 riders a total of 11 days 21 hours to ride from St. Joseph to Sacramento. If they all rode the same number of hours, how many hours did each rider ride?
_____ hours

Answer:
15 hours

Explanation:
Suppose it took 19 riders a total of 11 days 21 hours to ride from St. Joseph to Sacramento.
(11 x 24 + 21)/19 = (264 + 21)/19 = 285/19 = 15 hours.

Partial Quotients – Share and Show – Page No. 75

Divide. Use partial quotients.

Question 1.
18)\(\overline { 648 } \)
_____

Answer:
36

Explanation:
Multiply 18 x 10 = 180; Subtract: 648 – 180 = 468.
partial quotient = 10
Multiply 18 x 10 = 180; Subtract: 468 – 180= 288.
partial quotient = 10
Multiply 18 x 10 = 180; Subtract: 288- 180= 108.
partial quotient = 10
Multiply 18 x 6 = 108; Subtract: 108 – 108 = 0.
partial quotient = 6;
The remainder is 0;
Add partial quotient to find the wholenumber quotient;
10 + 10 + 10 + 6 = 36 R 0

Question 2.
62)\(\overline { 3,186 } \)
_____ R _____

Answer:

Explanation:
Multiply 62 x 10 = 620; Subtract: 3,186 – 620 = 2,566.
partial quotient = 10
Multiply 62 x 10 = 620; Subtract: 2,566 – 620 = 1,946.
partial quotient = 10
Multiply 62 x 10 = 620; Subtract: 1,946 – 620 = 1,326.
partial quotient = 10
Multiply 62 x 10 = 620; Subtract: 1,326 – 620 = 706.
partial quotient = 10
Multiply 62 x 10 = 620; Subtract: 706 – 620 = 86.
partial quotient = 10
Multiply 62 x 1 = 62; Subtract: 86 – 62 = 24.
partial quotient = 1
The remainder is 24;
Add partial quotient to find the wholenumber quotient;
10 + 10 + 10  + 10 + 1 = 51 R 24

Question 3.
858 ÷ 57
_____ R _____

Answer:

Explanation:
Multiply 57 x 10 = 570; Subtract: 858 – 570 = 288.
partial quotient = 10
Multiply 57 x 5 = 285; Subtract: 288 – 285 = 3.
partial quotient = 5
The remainder is 3;
Add partial quotient to find the wholenumber quotient;
10 + 5 = 15 R 3

On Your Own

Divide. Use partial quotients.

Question 4.
73)\(\overline { 584 } \)
_____

Answer:
8

Explanation:
Multiply 73 x 8 = 584; Subtract: 584 – 584 = 0.
partial quotient = 8
The remainder is 0;

Question 5.
51)\(\overline { 1,831 } \)
_____ R _____

Answer:
35 R 46

Explanation:
Multiply 51 x 10 = 510; Subtract: 1,831 – 510 = 1,321.
partial quotient = 10
Multiply 51 x 10 = 510; Subtract: 1,321 – 510 = 811.
partial quotient = 10
Multiply 51 x 10 = 510; Subtract: 811 – 510 = 301.
partial quotient = 10
Multiply 51 x 5 = 255; Subtract: 301 – 255 = 46.
partial quotient = 5
The remainder is 46;
Add partial quotient to find the wholenumber quotient;
10 + 10 + 10 + 5 = 35 R 46

Question 6.
82)\(\overline { 2,964 } \)
_____ R _____

Answer:
36 R 12

Explanation:
Multiply 82 x 10 = 820; Subtract: 2,964 – 820 = 2,144.
partial quotient = 10
Multiply 82 x 10 = 820; Subtract: 2,144 – 820 = 1,324.
partial quotient = 10
Multiply 82 x 10 = 820; Subtract: 1,324 – 820 = 504.
partial quotient = 10
Multiply 82 x 6 = 492; Subtract: 504 – 492= 12.
partial quotient = 6
The remainder is 12;
Add partial quotient to find the wholenumber quotient;
10 + 10 + 10 + 6 = 36 R 12

Question 7.
892 ÷ 26
_____ R _____

Answer:
34 R 8

Explanation:
Multiply 26 x 10 = 260; Subtract: 892 – 260 = 632.
partial quotient = 10
Multiply 26 x 10 = 260; Subtract: 632 – 260 = 372.
partial quotient = 10
Multiply 26 x 10 = 260; Subtract: 372 – 260 = 112.
partial quotient = 10
Multiply 26 x 4 = 104; Subtract: 112 – 104 = 8.
partial quotient = 4
The remainder is 8;
Add partial quotient to find the wholenumber quotient;
10 + 10 + 10 + 4 = 34 R 8

Question 8.
1,056 ÷ 48
_____

Answer:
22

Explanation:
Multiply 48 x 10 = 480; Subtract: 1,056 – 480 = 576.
partial quotient = 10
Multiply 48 x 10 = 480; Subtract: 576 – 480 = 96.
partial quotient = 10
Multiply 48 x 2 = 96; Subtract: 96 – 96 = 0.
partial quotient = 2
The remainder is 0;
Add partial quotient to find the wholenumber quotient;
10 + 10 + 2 = 22

Question 9.
2,950 ÷ 67
_____ R _____

Answer:
44 R 2

Explanation:
Multiply 67 x 10 = 670; Subtract: 2,950 – 670 = 2,280.
partial quotient = 10
Multiply 67 x 10 = 670; Subtract: 2,280 – 670 = 1,610.
partial quotient = 10
Multiply 67 x 10 = 670; Subtract: 1,610 – 670 = 940.
partial quotient = 10
Multiply 67 x 10 = 670; Subtract: 940 – 670 = 270.
partial quotient = 10
Multiply 67 x 4= 268; Subtract: 270 – 268 = 2.
partial quotient = 4
The remainder is 2;
Add partial quotient to find the wholenumber quotient;
10 + 10 + 10 + 10 + 4 = 44 R 2

Practice: Copy and Solve Divide. Use partial quotients.

Question 10.
653 ÷ 42
_____ R _____

Answer:
15 R 23

Explanation:
Multiply 42 x 10 = 420; Subtract: 653 – 420 = 233.
partial quotient = 10
Multiply 42 x 5 = 210; Subtract: 233 – 210 = 23.
partial quotient = 5
The remainder is 23;
Add partial quotient to find the wholenumber quotient;
10 + 5 = 15 R 23

Question 11.
946 ÷ 78
_____ R _____

Answer:
12 R 10

Explanation:
Multiply 78 x 10 = 780; Subtract: 946 – 780 = 166.
partial quotient = 10
Multiply 78 x 2 = 156; Subtract: 166 – 156 = 10.
partial quotient = 2
The remainder is 10;
Add partial quotient to find the wholenumber quotient;
10 + 2 = 12 R 10

Question 12.
412 ÷ 18
_____ R _____

Answer:
22 R 16

Explanation:
Multiply 18 x 10 = 180; Subtract: 412 – 180 = 232.
partial quotient = 10
Multiply 18 x 10 = 180; Subtract: 232 – 180 = 52.
partial quotient = 10
Multiply 18 x 2 = 36; Subtract: 52 – 36 = 16.
partial quotient = 2
The remainder is 16;
Add partial quotient to find the wholenumber quotient;
10 + 10 + 2 = 22 R 16

Question 13.
871 ÷ 87
_____ R _____

Answer:
10 R 1

Explanation:
Multiply 87 x 10 = 870; Subtract: 871 – 870 = 1.
partial quotient = 10
The remainder is 1;
10 R 1

Question 14.
1,544 ÷ 34
_____ R _____

Answer:
45 R 14

Explanation:
Multiply 34 x 10 = 340; Subtract: 1,544 – 340 = 1,204.
partial quotient = 10
Multiply 34 x 10 = 340; Subtract: 1,204 – 340 = 864.
partial quotient = 10
Multiply 34 x 10 = 340; Subtract: 864 – 340 = 524.
partial quotient = 10
Multiply 34 x 10 = 340; Subtract: 524 – 340 = 184.
partial quotient = 10
Multiply 34 x 5 = 170; Subtract: 184 – 170 = 14.
partial quotient = 5
The remainder is 14;
Add partial quotient to find the wholenumber quotient;
10 + 10 + 10 + 10 + 5 = 45 R 14

Question 15.
2,548 ÷ 52
_____ R _____

Answer:
47 R 14

Explanation:
Multiply 52 x 10 = 520; Subtract: 2,548 – 520 = 2028.
partial quotient = 10
Multiply 52 x 10 = 520; Subtract: 2028- 520 = 1508.
partial quotient = 10
Multiply 52 x 10 = 520; Subtract: 1508- 520 = 988.
partial quotient = 10
Multiply 52 x 10 = 520; Subtract: 988 – 520 = 468.
partial quotient = 10
Multiply 52 x 9 = 468; Subtract: 468 – 468= 0.
partial quotient = 9
The remainder is 0;
Add partial quotient to find the wholenumber quotient;
10 + 10 + 10 + 10 + 9 = 49 R 0

Question 16.
2,740 ÷ 83
_____ R _____

Answer:
33 R 1

Explanation:
Multiply 83 x 10 = 830; Subtract: 2,740 – 830= 1910.
partial quotient = 10
Multiply 83 x 10 = 830; Subtract: 1910 – 830= 1080.
partial quotient = 10
Multiply 83 x 10 = 830; Subtract: 1080 – 830= 250.
partial quotient = 10
Multiply 83 x 3 = 249; Subtract: 250 – 249 = 1.
partial quotient = 3
The remainder is 1;
Add partial quotient to find the wholenumber quotient;
10 + 10 + 10 + 3 = 33 R 1

Question 17.
4,135 ÷ 66
_____ R _____

Answer:
62 R 43

Explanation:
Multiply 66 x 10 = 660; Subtract: 4,135 – 660 = 3475.
partial quotient = 10
Multiply 66 x 10 = 660; Subtract: 3475 – 660 = 2815.
partial quotient = 10
Multiply 66 x 10 = 660; Subtract: 2815 – 660 = 2155.
partial quotient = 10
Multiply 66 x 10 = 660; Subtract: 2155 – 660 = 1495.
partial quotient = 10
Multiply 66 x 10 = 660; Subtract: 1495 – 660 = 835.
partial quotient = 10
Multiply 66 x 10 = 660; Subtract: 835 – 660 = 175.
partial quotient = 10
Multiply 66 x 2 = 132; Subtract: 175 – 132 = 43.
partial quotient = 2
The remainder is 43;
Add partial quotient to find the wholenumber quotient;
10 + 10 + 10 + 10 + 10 + 10 + 2 = 62 R 43

Partial Quotients – Problem Solving – Page No. 76

Use the table to solve 18–20 and 22.
Go Math Grade 5 Answer Key Chapter 2 Divide Whole Numbers Divide Whole Numbers; Partial Quotients img 5

Question 18.
How many years would it take for a person in the United States to eat 855 pounds of apples?
_____ years

Answer:
45 years

Explanation:
Each year a person eats 19 pounds of apples. So, to eat 855 pounds of apples, it takes 855/19 = 45 years.

Question 19.
How many years would it take for a person in the United States to eat 1,120 pounds of turkey?
_____ years

Answer:
80 years

Explanation:
Each year a person eats 14 pounds of turkey. So, to eat 1,120 pounds of turkey, it takes 1,120/14 = 80 years.

Question 20.
If 6 people in the United States each eat the average amount of popcorn for 5 years, how many quarts of popcorn will they eat?
_____ quarts

Answer:
2,040 quarts

Explanation:
1 person eats 68 quarts of popcorn each year. 6 people = 6 x 68 quarts of popcorn = 408 quarts of popcorn for each year.
For 5 years, they will eat popcorn = 5 x 408 = 2,040 quarts

Question 21.
In a study, 9 people ate a total of 1,566 pounds of potatoes in 2 years. If each person ate the same amount each year, how many pounds of potatoes did each person eat in 1 year?
_____ pounds

Answer:
87 pounds

Explanation:
9 people ate a total of 1,566 pounds of potatoes in 2 years. If each person ate the same amount each year, 1,566/2 = 783.
To calculate how many pounds of potatoes did each person eat in 1 year, 783/9 = 87 pounds.

Question 22.
Sense or Nonsense? In the United States, a person eats more than 40,000 pounds of bread in a lifetime if he or she lives to be 80 years old. Does this statement make sense, or is it nonsense? Explain.
__________

Answer:
nonsense; 40,000 pounds / 80 years = 4,000 pounds / 8 years = 2,000 pounds (1 ton) / 4 years = 1,000 pounds / 2 years = 1,000 pounds / 2 years = 500 pounds per year = almost 1 and 1/2 pounds of bread every day of your life.

Question 23.
Test Prep The school auditorium has 448 seats arranged in 32 equal rows. How many seats are in each row?
Options:
a. 14,336
b. 480
c. 416
d. 14

Answer:
d. 14

Explanation:
The school auditorium has 448 seats arranged in 32 equal rows.
Each row = 448/32 = 14

Mid-Chapter Checkpoint – Page No. 77

Concepts and Skills

Question 1.
Explain how estimating the quotient helps you place the first digit in the quotient of a division problem.
Type below:
__________

Answer:
Let’s do 5980 divided by 347
Estimate: 6000/300 = 20
So, I now know my first digit will go into the 10’s place
or 57890 divided by 34
that is 60,000 divided by 30 = 2000
my first digit goes into the thousands place.

Question 2.
Explain how to use multiplication to check the answer to a division problem.
Type below:
__________

Answer:
Take 739/9 = 82 R 1.
Check: 9 x 82 + 1 = 739.
divisor x quotient + remainder = dividend.

Divide.

Question 3.
633 ÷ 3 = _____

Answer:
211

Explanation:
Divide integers 6/3 = 2
Multiply 3 x 2 = 6; Subtract 6 – 6 = 0
Write down 3 and divide integers 3/3 = 1.
Multiply 3 x 1 = 3. Subtract 3 – 3 = 0.
Write down 3 and divide integers 3/3 = 1.
Multiply 3 x 1 = 3. Subtract 3 – 3 = 0.
The remainder is 0.
So, 211 is the answer.

Question 4.
487 ÷ 8 = _____ R _____

Answer:
60 R 7

Explanation:
Divide integers 48/8 = 6
Multiply 8 x 6 = 48; Subtract 48 – 48 = 0
Write down 7;7 < 8.
The remainder is 7.
So, 60 R 7.

Question 5.
1,641 ÷ 4 = _____ R _____

Answer:
410 R 1

Explanation:
Divide integers 16/4 = 4
Multiply 4 x 4 = 16; Subtract 16 – 16 = 0
Write down 4 and divide integers 4/4 = 1.
Multiply 4 x 1 = 4; Subtract 4 – 4 = 0
Write down 1; 1<4
The remainder is 1.
So, 410 R 1.

Question 6.
2,765 ÷ 9 = _____ R _____

Answer:
307 R 2

Explanation:
Divide integers 27/9 = 3
Multiply 9 x 3 = 27; Subtract 27 – 27 = 0
Write down 65 and divide integers 65/9 = 7.
Multiply 9 x 7 = 63. Subtract 65 – 63 = 2.
The remainder is 2.
So, 307 R 2.

Divide. Use partial quotients.

Question 7.
156 ÷ 13 = _____

Answer:
12

Explanation:
Multiply 13 x 10 = 130; Subtract: 156 – 130 = 26.
partial quotient = 10
Multiply 13 x 2 = 26; Subtract: 26 – 26 = 0.
partial quotient = 2
The remainder is 0;
Add partial quotient to find the wholenumber quotient;
10 +2 = 12 R 0

Question 8.
318 ÷ 53 = _____

Answer:
6

Explanation:
Multiply 53 x 6 = 318; Subtract: 318 – 318= 0.
partial quotient = 6
The remainder is 0;
quotient = 6

Question 9.
1,562 ÷ 34 =
_____ r _____

Answer:
45 R 32

Explanation:
Multiply 34  x 10 = 340; Subtract: 1,562 – 340 = 1,222.
partial quotient = 10
Multiply 34  x 10 = 340; Subtract: 1,222 – 340 = 882.
partial quotient = 10
Multiply 34  x 10 = 340; Subtract: 882 – 340 = 542.
partial quotient = 10
Multiply 34  x 10 = 340; Subtract: 542 – 340 = 202.
partial quotient = 10
Multiply 34  x 5 = 170; Subtract: 202 – 170 = 32.
partial quotient = 5
The remainder is 32;
Add partial quotient to find the wholenumber quotient;
10 + 10 + 10 + 10 + 5 = 45 R 32

Question 10.
4,024 ÷ 68 =
_____ r _____

Answer:
59 R 12

Explanation:
Multiply 68 x 10 = 680; Subtract: 4,024 – 680 = 3,344.
partial quotient = 10
Multiply 68 x 10 = 680; Subtract: 3,344 – 680= 2664.
partial quotient = 10
Multiply 68 x 10 = 680; Subtract: 2664 – 680 = 1984.
partial quotient = 10
Multiply 68 x 10 = 680; Subtract: 1984 – 680= 1304.
partial quotient = 10
Multiply 68 x 10 = 680; Subtract: 1304 – 680 = 624.
partial quotient = 10
Multiply 68 x 9 = 612; Subtract: 624 – 612 = 12.
partial quotient = 9
The remainder is 12;
Add partial quotient to find the wholenumber quotient;
10 + 10 + 10 + 10 + 10 + 9 = 59 R 12

Mid-Chapter Checkpoint – Page No. 78

Question 11.
Emma is planning a party for 128 guests. If 8 guests can be seated at each table, how many tables will be needed for seating at the party?
_____ tables

Answer:
16 tables

Explanation:
Emma is planning a party for 128 guests. If 8 guests can be seated at each table 128/8 = 16.

Question 12.
Tickets for the basketball game cost $14 each. If the sale of the tickets brought in $2,212, how many tickets were sold?
_____ tickets

Answer:
158 tickets

Explanation:
Tickets for the basketball game cost $14 each. If the sale of the tickets brought in $2,212, $2,212/$14 = 158

Question 13.
Margo used 864 beads to make necklaces for the art club. She made 24 necklaces with the beads. If each necklace has the same number of beads, how many beads did Margo use for each necklace?
_____

Answer:
36 beads

Explanation:
Margo used 864 beads to make necklaces for the art club. She made 24 necklaces with the beads. If each necklace has the same number of beads, 864/24 = 36 beads

Question 14.
Angie needs to buy 156 candles for a party. Each package has 8 candles. How many packages should Angie buy?
_____ packages

Answer:
20 packages

Explanation:
Angie needs to buy 156 candles for a party. Each package has 8 candles.
156/8 = 20

Question 15.
Max delivers 8,520 pieces of mail in one year. About how many pieces of mail does he deliver in 2 months? Explain your steps.
_____ pieces

Answer:
1,420 pieces

Explanation:
Max delivers 8,520 pieces of mail in one year.
So, for 12 months, 8,520/12 = 710.
To deliver in 2 months, 710 x 2 = 1,420

Share and Show – Page No. 81

Use compatible numbers to find two estimates.

Question 1.
22)\(\overline { 154 } \)
140 ÷ 20 = _____
160 ÷ 20 = _____
Estimate: _____ ; _____

Answer:
140 ÷ 20 = 7
160 ÷ 20 = 8
Estimate: 7 ; 8

Explanation:
Two sets of compatible numbers to find two different estimates are
140 ÷ 20
14 ÷ 2 = 7
140 ÷ 20 = 7
160 ÷ 20
16 ÷ 2 = 8
160 ÷ 20  = 8

Question 2.
68)\(\overline { 503 } \)
Estimate: _____ ; _____

Answer:
476 ÷ 68= 7
544 ÷ 68 = 8
Estimate: 7 ; 8

Explanation:
Multiples of 68:
68 136 204 272 340 408 476 544
Find multiples that are close to the dividend. Use either or both numbers to estimate the quotient.
476/68 = 7
544/68 = 8
The quotient is between 7 and 8.

Question 3.
81)\(\overline { 7,052 } \)
Estimate: _____ ; _____

Answer:
6400 ÷ 80 = 80
7200 ÷ 80 = 90
Estimate: 80 ; 90

Explanation:
6400/80
64/8 = 8
640 / 80 = 8
6400/80 = 800
7200/80
72/8 = 9
720/80 =9
7200/80 = 90
Estimate: 80, 90

Question 4.
33)\(\overline { 291 } \)
Estimate: _____ ; _____

Answer:
240 ÷ 30= 8
270 ÷ 30 = 9
Estimate: 8 ; 9

Explanation:
240/30
24/3 = 8
240/30 = 8
270/30
27/3 = 9
270/30 = 9
Estimate: 8, 9

Question 5.
58)\(\overline { 2,365 } \)
Estimate: _____ ;

Answer:
2400 ÷ 60= 40
3000 ÷ 60 = 50
Estimate: 40 ; 50

Explanation:
2400/60
24/6 = 4
240/60 = 4
2400/60 = 40
3000/60
30/6 = 5
300/60 = 5
3000/60 = 50
Estimate: 40, 50

Question 6.
19)\(\overline { 5,312 } \)
Estimate: _____ ; _____

Answer:
5300 ÷ 20= 7
5320 ÷ 20 = 8
Estimate: 265 ; 266

Explanation:
5300/20
5300/20 = 265
5320/20
5320/20 = 266

On Your Own

Use compatible numbers to find two estimates.

Question 7.
42)\(\overline { 396 } \)
Estimate: _____ ; _____

Answer:
360 ÷ 40 = 9
400 ÷ 40 = 10
Estimate: 9 ; 10

Explanation:
360/40 = 9
400/40 = 10
Estimate: 9,10

Question 8.
59)\(\overline { 413 } \)
Estimate: _____ ; _____

Answer:
420 ÷ 60= 7
480 ÷ 60 = 8
Estimate: 7 ; 8

Explanation:
420/60 = 7
480/60 = 8

Question 9.
28)\(\overline { 232 } \)
Estimate: _____ ; _____

Answer:
240 ÷ 30 = 8
270÷ 30 = 9
Estimate: 8 ; 9

Explanation:
240/30 = 8
270/30 = 9
Estimate: 8 ; 9

Question 10.
22)\(\overline { 368 } \)
Estimate: _____ ; _____

Answer:
320 ÷ 20= 16
340 ÷ 20 = 17
Estimate: 16 ; 17

Explanation:
320/20 = 16
340/20 = 17
Estimate: 16 ; 17

Question 11.
78)\(\overline { 375 } \)
Estimate: _____ ; _____

Answer:
320 ÷ 80 = 4
400 ÷ 80 = 5
Estimate: 4 ; 5

Explanation:
320/80 = 4
400/80 = 5
Estimate: 16 ; 17

Question 12.
88)\(\overline { 6,080 } \)
Estimate: _____ ; _____

Answer:
6210÷ 90= 69
6300 ÷ 90 = 70
Estimate: 69 ; 70

Explanation:
6210/90 = 69
6300/90 = 70

Question 13.
5,821 ÷ 71
Estimate: _____ ; _____

Answer:
5180 ÷ 70 = 74
5250÷ 70 = 75
Estimate: 74 ; 75

Explanation:
5180/70 = 74
5250/70 = 75
Estimate: 74 ; 75

Question 14.
1,565 ÷ 67
Estimate: _____ ; _____

Answer:
1610 ÷ 70 = 23
1680 ÷ 70 = 24
Estimate: 23 ; 24

Explanation:
1610/70 = 23
1680/70 = 24
Estimate: 23 ; 24

Question 15.
7,973 ÷ 91
Estimate: _____ ; _____

Answer:
476 ÷ 90 = 87
544 ÷ 90 = 88
Estimate: 87 ; 88

Explanation:
6960/90 = 87
7920/90 = 88
Estimate: 87 ; 88

Use compatible numbers to estimate the quotient.

Question 16.
19)\(\overline { 228 } \)
Estimate: _____

Answer:
240 ÷ 20 = 12
260 ÷ 20 = 13
Estimate: 12 ; 13

Explanation:
240/20 = 12
260/20 = 13
Estimate: 12 ; 13

Question 17.
25)\(\overline { $595 } \)
Estimate: $ _____

Answer:
575 ÷ 25 = 23
600 ÷ 25 = 24
Estimate: 23 ; 24

Explanation:
575/25 = 23
600/25 = 24
Estimate: 23 ; 24

Question 18.
86)\(\overline { 7,130 } \)
Estimate: _____

Answer:
7380 ÷ 90 = 82
7470 ÷ 90 = 83
Estimate: 82 ; 83

Explanation:
7380/90 = 82
7470/90 = 83
Estimate: 82 ; 83

Question 19.
83)\(\overline { 462 } \)
Estimate: _____

Answer:
400 ÷ 80 = 5
480 ÷ 80 = 6
Estimate: 5 ; 6

Explanation:
400/80 = 5
480/80 = 6
Estimate: 5 ; 6

Question 20.
27)\(\overline { 9,144 } \)
Estimate: _____

Answer:
10,140 ÷ 30 = 338
10,170 ÷ 30 = 339
Estimate: 338 ; 339

Explanation:
10,140/30 = 338
10,170/30 = 339
Estimate: 338 ; 339

Question 21.
68)\(\overline { 710 } \)
Estimate: _____

Answer:
700 ÷ 70 = 10
770 ÷ 70 = 11
Estimate: 10 ; 11

Explanation:
700/70 = 10
770/70 = 11
Estimate: 10 ; 11

Question 22.
707 ÷ 36
Estimate: _____

Answer:
760 ÷ 40 = 19
800 ÷ 40 = 20
Estimate: 19 ; 20

Explanation:
760/40 = 19
800/40 = 20
Estimate: 19 ; 20

Question 23.
1,198 ÷ 41
Estimate: _____

Answer:
1160 ÷ 40 = 29
1200 ÷ 40 = 30
Estimate: 29 ; 30

Explanation:
1160/40 = 29
1200/40 = 30
Estimate: 29 ; 30

Question 24.
5,581 ÷ 72
Estimate: _____

Answer:
5390 ÷ 70 = 77
5460 ÷ 70 = 78
Estimate: 77 ; 78

Explanation:
5390/70 = 77
5460/70 = 78
Estimate: 77 ; 78

Problem Solving – Page No. 82

Use the picture to solve 25–26.
Go Math Grade 5 Answer Key Chapter 2 Divide Whole Numbers img 6

Question 25.
About how many meters tall is each floor of the Williams Tower?
_____ m

Answer:
4.29 meters

Explanation:
Williams Tower has 275 meters and 64 floors.
275/64 = 4.29 meters

Question 26.
About how many meters tall is each floor of the Chrysler Building?
_____ m

Answer:
4.142 m

Explanation:
Chrysler Building has 319 meters and 77 floors
319/77 = 4.142

Question 27.
Eli needs to save $235. To earn money, he plans to mow lawns and charge $21 for each. Write two estimates Eli could use to determine the number of lawns he needs to mow. Decide which estimate you think is the better one for Eli to use. Explain your reasoning.
Type below:
__________

Answer:
220/20 = 11

Explanation:
Calculate $235/$21
210/21 = 10
220/20 = 11
number 220 is closer to 235. So, the better estimate is 220/20 = 11.

Question 28.
Explain how you know whether the quotient of 298 ÷ 31 is closer to 9 or to 10.
Type below:
__________

Answer:
270/30 = 9
310/31 = 10
298 is closer to 270. So, the quotient is closer to 9 than 10.

Question 29.
Test Prep Anik built a tower of cubes. It was 594 millimeters tall. The height of each cube was 17 millimeters. About how many cubes did Anik use?
Options:
a. 10
b. 16
c. 30
d. 300

Answer:
c. 30

Explanation:
594/17
540/18 = 30
600/15 = 40
So, Anik use 30 cubes

Share and Show – Page No. 85

Divide. Check your answer.

Question 1.
28)\(\overline { 620 } \)
_____ R _____

Answer:
22 R 4

Explanation:
Divide integers 62/28 = 2
Multiply 28 x 2 = 56; Subtract 62 – 56 = 6
Write down 60 and divide integers 60/28 = 2.
Multiply 28 x 2 = 56. Subtract 60 – 56 = 4.
The remainder is 4.
So, 22 R 4.
Check:
22 x 28 = 616;
616 + 4 = 620
620 = 620

Question 2.
64)\(\overline { 842 } \)
_____ R _____

Answer:
13 R 10

Explanation:
Divide integers 84/64 = 1
Multiply 64 x 1 = 64; Subtract 84 – 64 = 20
Write down 202 and divide integers 202/64 = 3.
Multiply 64 x 3 = 192. Subtract 202 – 192  = 10.
The remainder is 10.
So, 13 R 10.
Check:
64 x 13 = 832;
832 + 10 = 842
842 = 842

Question 3.
53)\(\overline { 2,340 } \)
_____ R _____

Answer:
44 R 8

Explanation:
Divide integers 234/53 = 4
Multiply 53 x 4 = 212; Subtract 234 – 212 = 22
Write down 220 and divide integers 220/53 = 4.
Multiply 53 x 4 = 212. Subtract 220 – 212 = 8.
The remainder is 8.
So, 44 R 8.
Check:
53 x 44 = 2332;
2332 + 8 = 2340
2340 = 2340

Question 4.
723 ÷ 31
_____ R _____

Answer:
23 R 10

Explanation:
Divide integers 72/31 = 2
Multiply 31 x 2 = 62; Subtract 72 – 62 = 10
Write down 103 and divide integers 103/31 = 3.
Multiply 31 x 3 = 93. Subtract 103 – 93 = 10.
The remainder is 10.
So, 23 R 10.
Check:
31 x 23 = 713;
713 + 10 = 723
723 = 723

Question 5.
1,359 ÷ 45
_____ R _____

Answer:
30 R 9

Explanation:
Divide integers 135/45 = 3
Multiply 45 x 3 = 62; Subtract 135 – 135 = 0
Write down 9; 9<45
The remainder is 9.
So, 30 R 9.
Check:
45 x 30 = 1350;
1350 + 9 = 1359
1359 = 1359

Question 6.
7,925 ÷ 72
_____ R _____

Answer:
110 R 5

Explanation:
Divide integers 79/72 = 1
Multiply 72 x 1 = 72; Subtract 79 – 72 = 7
Write down 72 and divide integers 72/72= 1.
Multiply 72 x 1 = 72; Subtract 72 – 72 = 0
Write down 5; 5<72
The remainder is 5.
So, 110 R 5.
Check:
72 x 110 = 7920;
7920 + 5 = 7925
7925 = 7925

On Your Own

Divide. Check your answer.

Question 7.
16)\(\overline { 346 } \)
_____ R _____

Answer:

Explanation:
Divide integers 34/16 = 2
Multiply 16 x 2 = 32; Subtract 34 – 32 = 2
Write down 26 and divide integers 26/16= 1.
Multiply 16 x 1 = 16; Subtract 26 – 16 = 10
The remainder is 10.
So, 21 R 10.
Check:
16 x 21 = 336;
336 + 10 = 346
346 = 346

Question 8.
34)\(\overline { 241 } \)
_____ R _____

Answer:
7 R 3

Explanation:
Divide integers 241/34 = 7
Multiply 34 x 7 = 238; Subtract 241 – 238= 3
The remainder is 3.
So, 7 R 3
Check:
34  x 7 = 238;
238 + 3 = 241
241 = 241

Question 9.
77)\(\overline { 851 } \)
_____ R _____

Answer:
11 R 4

Explanation:
Divide integers 85/77 = 1
Multiply 77 x 1 = 77; Subtract 85  – 77 = 8
Write down 81 and divide integers 81/77= 1.
Multiply 77 x 1 = 77; Subtract 81 – 77 = 4
The remainder is 4.
So, 11 R 4.
Check:
77 x 11 = 847;
847 + 4 = 851
851 = 851

Question 10.
21)\(\overline { 1,098 } \)
_____ R _____

Answer:
52 R 6

Explanation:
Divide integers 109/21 = 5
Multiply 21 x 5 = 105; Subtract 109 – 105= 4
Write down 48 and divide integers 48/21 = 2.
Multiply 21 x 2 = 42; Subtract 48 – 42 = 6
The remainder is 6.
So, 52 R 6.
Check:
21 x 52 = 1092;
1092 + 6 = 1098
1098 = 1098

Question 11.
32)\(\overline { 6,466 } \)
_____ R _____

Answer:
202 R 2

Explanation:
Divide integers 64/32= 2
Multiply 32 x 2 = 64; Subtract 64 – 64 = 0
Write down 66 and divide integers 66/32 = 2.
Multiply 32 x 2 = 64; Subtract 66 – 64 = 2
The remainder is 2.
So, 202 R 2.
Check:
32 x 202 = 6464;
6464 + 2 = 6466
6466 = 6466

Question 12.
45)\(\overline { 9,500 } \)
_____ R _____

Answer:
211 R 5

Explanation:
Divide integers 95/45 = 2
Multiply 45 x 2 = 90; Subtract 95 – 90 = 5
Write down 50 and divide integers 50/45 = 1.
Multiply 45 x 1 = 45; Subtract 50 – 45 = 5
Write down 50 and divide integers 50/45 = 1.
Multiply 45 x 1 = 45; Subtract 50 – 45 = 5
The remainder is 5.
So, 211 R 5.
Check:
45 x 211 = 9495;
9495 + 5 = 9500
9500 = 9500

Question 13.
483 ÷ 21
_____

Answer:
23

Explanation:
Divide integers 48/21 = 2
Multiply 21 x 2 = 42; Subtract 48 – 42 = 6
Write down 63 and divide integers 63/21 = 3.
Multiply 21 x 3 = 63; Subtract 63 – 63 = 0
The remainder is 0.
So, 23 R 0.
Check:
23 x 21 = 483;
483 = 483

Question 14.
2,292 ÷ 19
_____ R _____

Answer:
120 R 12

Explanation:
Divide integers 22/19 = 1
Multiply 19 x 1 = 19; Subtract 22  – 19 = 3
Write down 39 and divide integers 39/19 = 2.
Multiply 19 x 2 = 38; Subtract 39 – 38 = 1
Write down 12; 12<19
The remainder is 12.
So, 120 R 12.
Check:
19 x 120 = 2280;
2280 + 12 = 2,292
2,292 = 2,292

Question 15.
4,255 ÷ 30
_____ R _____

Answer:
141 R 25

Explanation:
Divide integers 42/30 = 1
Multiply 30 x 1 = 30; Subtract 42 – 30 = 12
Write down 125 and divide integers 125/30 = 4.
Multiply 30 x 4 = 120; Subtract 125 – 120 = 5
Write down 55 and divide integers 55/30 = 1.
Multiply 30 x 1 = 30; Subtract 55 – 30 = 25
The remainder is 25.
So, 141 R 25.
Check:
30 x 141 = 4230;
4230 + 25 = 4,255
4,255 = 4,255

Practice: Copy and Solve Divide. Check your answer.

Question 16.
775 ÷ 35
_____ R _____

Answer:
22 R 5

Explanation:
Divide integers 77/35 = 2
Multiply 35 x 2 = 70; Subtract 77 – 70 = 7
Write down 75 and divide integers 75/35 = 2.
Multiply 35 x 2 = 70; Subtract 75 – 70 = 5
The remainder is 5.
So, 22 R 5.
Check:
22 x 35 = 770;
770 + 5 = 775
775 = 775

Question 17.
820 ÷ 41
_____

Answer:
20

Explanation:
Divide integers 82/41 = 2
Multiply 41 x 2 = 82; Subtract 82 – 82= 0
The remainder is 0.
So, 20 R 0.
Check:
41 x 20 = 820;
820 = 820

Question 18.
805 ÷ 24
_____ R _____

Answer:
33 R 13

Explanation:
Divide integers 80/24 = 3
Multiply 24 x 3 = 72; Subtract 80 – 72 = 8
Write down 85 and divide integers 85/24 = 3.
Multiply 24 x 3 = 72; Subtract 85 – 72 = 13
The remainder is 13.
So, 33 R 13.
Check:
24 x 33 = 792;
792 + 13 = 805
805 = 805

Question 19.
1,166 ÷ 53
_____ R _____

Answer:
22 R 0

Explanation:
Divide integers 116/53 = 2
Multiply 53 x 2 = 106; Subtract 116 – 106= 10
Write down 106 and divide integers 106/53 = 2.
Multiply 53 x 2 = 106; Subtract 106 – 106= 0
The remainder is 0.
So, 22 R 0.
Check:
53 x 22 = 1166;
1166 = 1166

Question 20.
1,989 ÷ 15
_____ R _____

Answer:
132 R 9

Explanation:
Divide integers 19/15 = 1
Multiply 15 x 1 = 15; Subtract 19 – 15 = 4
Write down 48 and divide integers 48/15 = 3.
Multiply 15 x 3 = 45; Subtract 48 – 45 = 3
Write down 39 and divide integers 39/15 = 2.
Multiply 15 x 2 = 30; Subtract 39 – 30= 9
The remainder is 9.
So, 132 R 9.
Check:
15 x 132 = 1980;
1980 + 9 = 1989
1989 = 1989

Question 21.
3,927 ÷ 35
_____ R _____

Answer:
112 R 7

Explanation:
Divide integers 39/35 = 1
Multiply 35 x 1 = 35; Subtract 39 – 35 = 4
Write down 42 and divide integers 42/35 = 1.
Multiply 35 x 1 = 35; Subtract 42 – 35 = 7
Write down 77 and divide integers 77/35 = 2.
Multiply 35 x 2 = 70; Subtract 77 – 70 = 7
The remainder is 7.
So, 112 R 7.
Check:
35 x 112 = 3920;
3920 + 7 = 3927
3927 = 3927

Problem Solving – Page No. 86

Use the list at the right to solve 22–24.
Go Math Grade 5 Answer Key Chapter 2 Divide Whole Numbers img 7

Question 22.
A smoothie shop receives a delivery of 980 ounces of grape juice. How many Royal Purple smoothies can be made with the grape juice?
_____ smoothies

Answer:
45 smoothies

Explanation:
A smoothie shop receives a delivery of 980 ounces of grape juice.
980 ounces of grape juice/22 ounces of grape juice = 45

Question 23.
The shop has 1,260 ounces of cranberry juice and 650 ounces of passion fruit juice. If the juices are used to make Crazy Cranberry smoothies, which juice will run out first? How much of the other juice will be left over?
Type below:
_________

Answer:
The shop has 1,260 ounces of cranberry juice and 650 ounces of passion fruit juice. If the juices are used to make Crazy Cranberry smoothies, passion fruit juice will run out first.
Because 650<1,260. So, passion fruit juice will run out first.
1,260 – 650 =610 Crazy Cranberry juice will be left over.

Question 24.
In the refrigerator, there are 680 ounces of orange juice and 410 ounces of mango juice. How many Orange Tango smoothies can be made? Explain your reasoning.
_____ smoothies

Answer:
In the refrigerator, there are 680 ounces of orange juice and 410 ounces of mango juice.
So, 410 Orange Tango smoothies can be made. Because there are 410 ounces of mango juices available.

Question 25.
Test Prep James has 870 action figures. He decides to divide them equally among 23 boxes. How many action figures will James have left over?
Options:
a. 19
b. 23
c. 31
d. 37

Answer:
d. 37

Explanation:
James has 870 action figures. He decides to divide them equally among 23 boxes.
870/23 = 37

Share and Show – Page No. 89

Interpret the remainder to solve.

Question 1.
Erika and Bradley want to hike the Big Cypress Trail. They will hike a total of 75 miles. If Erika and Bradley plan to hike for 12 days, how many miles will they hike each day?
Go Math Grade 5 Answer Key Chapter 2 Divide Whole Numbers img 8
a. Divide to find the quotient and remainder.
_____ R _____

Answer:
6 R 3

Explanation:
75/12 = 6
The remainder is 3
6 R 3

Question 1.
b. Decide how to use the quotient and remainder to answer the question.
Type below:
_________

Answer:
75/12 = 6  1/4
So, Each day they will hike 6\(\frac{1}{4}\) miles.

Question 2.
What if Erika and Bradley want to hike 14 miles each day? How many days will they hike exactly 14 miles?
_____ days

Answer:
196 days

Explanation:
If Erika and Bradley want to hike 14 miles each day, 14 x 14 = 196 days

Question 3.
Dylan’s hiking club is planning to stay overnight at a camping lodge. Each large room can hold 15 hikers. There are 154 hikers. How many rooms will they need?
_____ rooms

Answer:
11 rooms

Explanation:
Dylan’s hiking club is planning to stay overnight at a camping lodge. Each large room can hold 15 hikers. There are 154 hikers.
So, 154/15 = 10 and the remainder is 4.
Dylan’s hiking club require 10 rooms for 150 hikers and other room for 4 hikers. So, in total they need 10 + 1 = 11 rooms.

On Your Own

Interpret the remainder to solve.

Question 4.
The students in a class of 24 share 84 cookies equally among them. How many cookies did each student eat?
_____ \(\frac{□}{□}\) cookies

Answer:
3\(\frac{1}{2}\) cookies

Explanation:
The students in a class of 24 share 84 cookies equally among them.
So, 84/24 = 3\(\frac{12}{24}\) = 3\(\frac{1}{2}\)

Question 5.
A campground has cabins that can each hold 28 campers. There are 148 campers visiting the campground. How many cabins are full if 28 campers are in each cabin?
_____ cabins

Answer:
5\(\frac{1}{7}\) cabins

Explanation:
A campground has cabins that can each hold 28 campers. There are 148 campers visiting the campground.
184/28 = 5\(\frac{1}{7}\)

Question 6.
A total of 123 fifth-grade students are going to Fort Verde State Historic Park. Each bus holds 38 students. All of the buses are full except one. How many students will be in the bus that is not full?
_____ students

Answer:
9 students

Explanation:
A total of 123 fifth-grade students are going to Fort Verde State Historic Park. Each bus holds 38 students.
123/38 = 3 and the remainder is 9.
3 x 38 = 114 students. 1 bus is not full.
So, 9 students will be in the bus that is not full

Question 7.
What’s the Error? Sheila is going to divide a 36-inch piece of ribbon into 5 equal pieces. She says each piece will be 7 inches long.
Type below:
_________

Answer:
Sheila is going to divide a 36-inch piece of ribbon into 5 equal pieces.
36/5 = 7\(\frac{1}{5}\). She said each piece will be 7 inches long and forgot about \(\frac{1}{5}\) part.

UNLOCK the Problem – Page No. 90

Question 8.
Maureen has 243 ounces of trail mix. She puts an equal number of ounces in each of 15 bags. How many ounces of trail mix does Maureen have left over?
a. What do you need to find?

Answer:
We need to find how many ounces of trail mix does Maureen have left over?

Question 8.
b. How will you use division to find how many ounces of trail mix are left over?
Type below:
_________

Answer:
The division is 243/15

Question 8.
c. Show the steps you use to solve the problem.
Type below:
_________

Answer:
243/15
Divide integers 24/15 = 1
Multiply 15 x 1 = 15; Subtract 24 – 15 = 9
Write down 93 and divide integers 93/3 = 6.
Multiply 15 x 6 = 90. Subtract 93 – 90 = 3.
The remainder is 3.
So, 16 R 3.

Question 8.
d. Complete the sentences.
Maureen has _______ ounces of trail mix.
She puts an equal number in each of _______ bags.
Each bag has _______ ounces.
Maureen has _______ ounces of trail mix left over.
Type below:
_________

Answer:
Maureen has 243 ounces of trail mix.
She puts an equal number in each of 15 bags.
Each bag has 16 ounces.
Maureen has 3 ounces of trail mix left over.

Question 8.
e. Fill in the bubble completely to show your answer.
Options:
a. 3 ounces
b. 15 ounces
c. 16 ounces
d. 17 ounces

Answer:
c. 16 ounces

Question 9.
Mr. Field wants to give each of his 72 campers a certificate for completing an obstacle course. If there are 16 certificates in one package, how many packages will Mr. Field need?
Options:
a. 4
b. 5
c. 16
d. 17

Answer:
b. 5

Explanation:
Mr. Field wants to give each of his 72 campers a certificate for completing an obstacle course. If there are 16 certificates in one package, 72/16 = 4.5

Question 10.
James has 884 feet of rope. There are 12 teams of hikers. If James gives an equal amount of rope to each team, how much rope will each team receive?
Options:
a. 12
b. 73
c. 73 \(\frac{2}{3}\)
d. 74

Answer:
b. 73

Explanation:
James has 884 feet of rope. There are 12 teams of hikers. If James gives an equal amount of rope to each team, 884/12 = 73

Share and Show – Page No. 92

Adjust the estimated digit in the quotient, if needed. Then divide.

Question 1.
4
41)\(\overline { 1,546 }\)
_____ R _____

Answer:
37 R 29

Explanation:
41 x 4 = 164; Subtract: 154 – 164
the estimate too high.
Change the quotient to 3
41 x 3 = 123; Subtract: 154 – 123 = 31
Write down 316 and divide integers 316/41
41 x 7 = 287; Subtract: 316 – 287 = 29
37 R 29

Question 2.
2
16)\(\overline { 416 }\)
_____

Answer:
26

Explanation:
16 x 2 = 32; Subtract: 41 – 32 = 9
Write down 96 and divide integers 96/16
16 x 6 = 96; Subtract: 96 – 96 = 0
26

Question 3.
9
34)\(\overline { 2,831 }\)
_____ R _____

Answer:
83 R 9

Explanation:
34 x 9 = 306; Subtract: 283 – 306
the estimate too high.
Change the quotient to 8
34 x 8 = 272; Subtract: 283 – 272 = 11
Write down 111 and divide integers 111/34
34 x 3 = 102; Subtract: 111 – 102 = 9
83 R 9

Divide.

Question 4.
19)\(\overline { 915 }\)
_____ R _____

Answer:
48 R 3

Explanation:
900/18 = 50
19 x  5 = 95; Subtract: 91 – 95
the estimate too high.
Change the quotient to 4
19 x  4 = 76; Subtract: 91 – 76 = 15
Write down 155 and divide integers 155/19
19 x 7 = 133; Subtract: 155 – 133 = 22
22 > 19;
So Change the quotient to 8
19 x 8 = 152; Subtract: 155 – 152 = 3
48 R 3

Question 5.
28)\(\overline { 1,825 }\)
_____ R _____

Answer:

Explanation:
1800/30 = 60
28 x 6 = 168; Subtract: 182 – 168 = 14
Write down 145 and divide integers 145/28
28 x 5 = 140; Subtract: 145 – 140 = 5
65 R 5

Question 6.
45)\(\overline { 3,518 }\)
_____ R _____

Answer:

Explanation:
3600/40 = 90
45 x 9 = 405; Subtract: 351 – 405
the estimate too high.
Change the quotient to 7
45 x 7 = 315; Subtract: 351 – 315 = 36
Write down 368 and divide integers 368/45
45 x 8 = 360; Subtract: 368 – 315 = 8
78 R 8

On Your Own – Page No. 93

Adjust the estimated digit in the quotient, if needed. Then divide.

Question 7.
2
26)\(\overline { 541 }\)
_____ R _____

Answer:
20 R 21

Explanation:
500/25 = 2
26 x 2 = 52; Subtract: 54 – 52 = 2
Write down 21 and divide integers 21/26
20 R 21

Question 8.
1
43)\(\overline { 688 }\)
_____

Answer:
16

Explanation:
800/40 = 20
43 x 2 = 86; Subtract: 68 – 86
the estimate is too high.
Change the quotient to 1
43 x 1 = 43; Subtract: 68 – 43 = 25
Write down 258 and divide integers 258/43
43 x 6 = 258; Subtract: 258 – 258 = 0
So, 16

Question 9.
6
67)\(\overline { 4,873 }\)
_____ R _____

Answer:
72 R 49

Explanation:
4800/70 = 60
67 x 6 = 402; Subtract: 487 – 402 = 85
the estimate is too low.
Change the quotient to 7
67 x 7 = 469; Subtract: 487 – 469 = 18
Write down 183 and divide integers 183/67
67 x 2 = 134; Subtract: 183 – 134 = 49
72 R 49

Question 10.
15)\(\overline { 975 }\)
_____

Answer:
65

Explanation:
15 x 6 = 90; Subtract 97 – 90 = 7
Write down 75 and divide integers 75/15
15 x 5 = 75; Subtract: 75 – 75 = 0
So, 65

Question 11.
37)\(\overline { 264 }\)
_____ R _____

Answer:
7 R 5

Explanation:
240/40 = 6
37 x 6 = 222; Subtract: 264 – 222 = 42
the estimate is too low.
Change the quotient to 7
37 x 7 = 259; Subtract: 264 – 259 = 5
7 R 5

Question 12.
22)\(\overline { 6,837 }\)
_____ R _____

Answer:
310 R 17

Explanation:
6300/20 = 325
22 x 3 = 66; Subtract: 68 – 66 = 2
Write down 23 and divide integers 23/22
22 x 1 = 22; Subtract: 23 – 22 = 1
Write down 17; 17 < 22
310 R 17

Practice: Copy and Solve Divide.

Question 13.
452 ÷ 31
_____ \(\frac{□}{□}\)

Answer:
14\(\frac{18}{31}\)

Explanation:
Divide integers 45/31 = 1
Multiply 31 x 1 = 31; Subtract 45 – 31 = 14
Write down 142 and divide integers 142/31 = 4.
Multiply 31 x 4 = 124. Subtract 142 – 124 = 18.
The remainder is 18.
So, 14 R 18.
14\(\frac{18}{31}\)

Question 14.
592 ÷ 74
_____

Answer:
8

Explanation:
Divide integers 592/74 = 8
So, 8.

Question 15.
785 ÷ 14
_____ R _____

Answer:
56\(\frac{1}{14}\)

Explanation:
Divide integers 78/14 = 5
Multiply 14 x 5 = 70; Subtract 78 – 70 = 8
Write down 85 and divide integers 85/14 = 6.
Multiply 14 x 6 = 84. Subtract 85 – 84 = 1.
The remainder is 1.
So, 56 R 1.
56\(\frac{1}{14}\)

Question 16.
601 ÷ 66
_____ R _____

Answer:
9 R 7

Explanation:
Divide integers 601/66 = 9
Multiply 66 x 9 = 594 ; Subtract 601 – 594= 7
The remainder is 7.
So, 9 R 7.
9\(\frac{7}{66}\)

Question 17.
1,067 ÷ 97
_____

Answer:
11

Explanation:
Divide integers 106/97 = 1
Multiply 97 x 1 = 97; Subtract 106 – 97 = 9
Write down 97 and divide integers 97/97 = 1
Multiply 97 x 1 = 97; Subtract 97 – 97 = 0
The remainder is 0.
So, 11 is the answer.

Question 18.
2,693 ÷ 56
_____ R _____

Answer:
48 R 5

Explanation:
Divide integers 269/56 = 4
Multiply 56 x 4 = 224; Subtract 269 – 224 = 45
Write down 453 and divide integers 453/56 = 8
Multiply 56 x 8 = 448. Subtract 453 – 448 = 5.
The remainder is 5.
So, 48 R 5.

Question 19.
1,488 ÷ 78
_____ R _____

Answer:
19 R 6

Explanation:
Divide integers 148/78 = 1
Multiply 78 x 1 = 78; Subtract 148 – 78 = 70
Write down 708 and divide integers 708/78 = 9.
Multiply 78 x 9 = 702. Subtract 708 – 702 = 6.
The remainder is 6.
So, 19 R 6.

Question 20.
2,230 ÷ 42
_____ R _____

Answer:
53 R 4

Explanation:
Divide integers 223/42 = 5
Multiply 42 x 5 = 210; Subtract 223 – 210 = 13
Write down 130 and divide integers 130/42 = 3.
Multiply 42 x 3 = 126. Subtract 130 – 126 = 4.
The remainder is 4.
So, 53 R 4.

Question 21.
4,295 ÷ 66
_____ R _____

Answer:
65 R 5

Explanation:
Divide integers 429/66 = 6
Multiply 66 x 6 = 396; Subtract 429 – 396 = 33
Write down 335 and divide integers 335/66 = 5.
Multiply 66 x 5 = 330. Subtract 335 – 330 = 5.
The remainder is 5
So, 65 R 5

Algebra Write the unknown number for each ■.

Question 22.
■ ÷ 33 = 11
■ = _____

Answer:
363

Explanation:
n ÷ 33 = 11
n = 11 x 33 = 363

Question 23.
1,092 ÷ 52 = ■
■ = _____

Answer:
21

Explanation:
1,092 ÷ 52 = 21

Question 24.
429 ÷ ■ = 33
■ = _____

Answer:
13

Explanation:
429 ÷ n = 33
n = 429 ÷ 33
n = 13

UNLOCK the Problem – Page No. 94

Question 25.
A banquet hall serves 2,394 pounds of turkey during a 3-week period. If the same amount is served each day, how many pounds of turkey does the banquet hall serve each day?
Go Math Grade 5 Answer Key Chapter 2 Divide Whole Numbers img 9
a. What do you need to find?
Type below:
_________

Answer:
How many Lbs at turkey they serve each day?

Question 25.
b. What information are you given?
Type below:
_________

Answer:
Every 3 weeks, serves 2,394 lbs.

Question 25.
c. What other information will you use?
Type below:
_________

Answer:
Same each day, 3 weeks = 21 days

Question 25.
d. Find how many days there are in 3 weeks. There are ______ days in 3 weeks.
Type below:
_________

Answer:
There are ______ days in 3 weeks

Explanation:
1 week = 7 days.
3 x 7 = 21 days

Question 25.
e. Divide to solve the problem.
Type below:
_________

Answer:
2394/3 = t
t = 798
798/7 = 114 pounds

Question 25.
f. Fill in the bubble for the correct answer choice.
Options:
a. 50,274 pounds
b. 798 pounds
c. 342 pounds
d. 114 pounds

Answer:
d. 114 pounds

Question 26.
Marcos mixes 624 ounces of lemonade. He wants to fill the 52 cups he has with equal amounts of lemonade. How much lemonade should he put in each cup?
Options:
a. 8 ounces
b. 12 ounces
c. 18 ounces
d. 20 ounces

Answer:
b. 12 ounces

Explanation:
Marcos mixes 624 ounces of lemonade. He wants to fill the 52 cups he has with equal amounts of lemonade. 624/52 = 12 ounces.
12 ounces should he put in each cup

Question 27.
The Box of Sox company packs 18 pairs of socks in a box. How many boxes will the company need to pack 810 pairs of socks?
Options:
a. 40
b. 45
c. 55
d. 56

Answer:
b. 45

Explanation:
The Box of Sox company packs 18 pairs of socks in a box. So, for 810 pairs of socks, 810/18 = 45

Share and Show – Page No. 97

Question 1.
Paula caught a tarpon with a weight that was 10 times as great as the weight of a permit fish she caught. The total weight of the two fish was 132 pounds. How much did each fish weigh?
First, draw one box to represent the weight of the permit fish and ten boxes to represent the weight of the tarpon.
Next, divide the total weight of the two fish by the total number of boxes you drew. Place the quotient in each box.
Last, find the weight of each fish.
The permit fish weighed _____ pounds.
The tarpon weighed _____ pounds.
Type below:
_________

Answer:
The permit fish weighed 12 pounds.
The tarpon weighed 120 pounds.

Explanation:
Let S be the weight of a permit fish Paula caught.
The weight of the tarpon is 10 times as great as the weight of a permit fish she caught = 10 S
The total weight of the two fish was 132 pounds.
S + 10S = 132
11S = 132
S = 132/11 = 12
So, Paula caught a fish with the weight of 12 pounds.
The tarpon weighted 120 pounds.

Question 2.
What if the weight of the tarpon was 11 times the weight of the permit fish, and the total weight of the two fish was 132 pounds? How much would each fish weigh?
permit fish: _________ pounds
tarpon: _________ pounds

Answer:
permit fish: 11 pounds
tarpon: 11 x 11 = 121 pounds

Explanation:
Let S be the weight of a permit fish Paula caught.
The weight of the tarpon is 11 times as great as the weight of a permit fish she caught = 11S
Total weight is 132
11S + S = 132
12S = 132
S = 132/12 = 11.
permit fish: 11 pounds
tarpon: 11 x 11 = 121 pounds

Question 3.
Jon caught four fish that weighed a total of 252 pounds. The kingfish weighed twice as much as the amberjack and the white marlin weighed twice as much as the kingfish.
The weight of the tarpon was 5 times the weight of the amberjack. How much did each fish weigh?
amberjack: _________ pounds
kingfish: _________ pounds
marlin: _________ pounds
tarpon: _________ pounds

Answer:
amberjack: 21 pounds
kingfish: 42 pounds
marlin: 84 pounds
tarpon: 105 pounds

Explanation:
Let S be the weight of the amberjack.
The kingfish weighed twice as much as the amberjack = 2S
The white marlin weighed twice as much as the kingfish = 2 X 2S = 4S
The weight of the tarpon was 5 times the weight of the amberjack = 5S
Total weight = 252 pounds.
2S + 4S + 5S + S = 252
12S = 252
S = 252/12
S = 21.
The kingfish weighed twice as much as the amberjack = 2S = 2 x 21 = 42 pounds.
The white marlin weighed twice as much as the kingfish = 2 X 2S = 4S = 4 x 21 = 84 pounds.
The weight of the tarpon was 5 times the weight of the amberjack = 5S = 5 x 21 = 105 pounds.

On Your Own – Page No. 98

Use the table to solve 4–7.
Go Math Grade 5 Answer Key Chapter 2 Divide Whole Numbers img 10

Question 4.
Kevin is starting a saltwater aquarium with 36 fish. He wants to start with 11 times as many damselfish as clown fish. How many of each fish will Kevin buy? How much will he pay for the fish?
Type below:
_________

Answer:
Kevin is starting a saltwater aquarium with 36 fish. He uses 1 damselfish and 11 clown fish. So, three groups form for 36 fishes.
1 damselfish and 11 clown fish = $7 + (11 x $20) = 7 + 220 = 227
He buys 3 damselfish and 33 clown fish.
3 x 7 = 21$ for damselfish and 33 x 20 = 660 for clown fish.
Each fish = 681/36 = 19

Question 5.
Kevin used a store coupon to buy a 40-gallon tank, an aquarium light, and a filtration system. He paid a total of $240. How much money did Kevin save by using the coupon?
$ _____

Answer:
$25

Explanation:
40-gallon tank = $170
aquarium light = $30
filtration system = $65
170 + 30 + 65 = 265
He paid a total of $240; 265 -240 = $25
Kevin save by using the coupon $25.

Question 6.
Kevin bought 3 bags of gravel to cover the bottom of his fish tank. He has 8 pounds of gravel left over. How much gravel did Kevin use to cover the bottom of the tank?
_____ pounds

Answer:
37 pounds.

Explanation:
15lb bag of gravel = $13.
3 bags of gravel = 45lb.
He has 8 pounds of gravel left over = 45 – 8 = 37
Kevin use 37 pounds of gravel to cover the bottom of the tank

Question 7.
Pose a Problem Look back at Problem 6. Write a similar problem by changing the number of bags of gravel and the amount of gravel left.
Type below:
_________

Answer:
If he bought 5 bags of gravel to cover the bottom of his fish tank. He has 10 pounds of gravel left over.
5 bags of gravel = 15 x 5 = 75lbs
He has 10 pounds of gravel left over = 75 – 10 lbs = 65 lbs
65lbs

Explanation:

Question 8.
Test Prep Captain James offers a deep-sea fishing tour. He charges $2,940 for a 14-hour trip. How much does each hour of the tour cost?
Options:
a. $138
b. $201
c. $210
d. $294

Answer:
c. $210

Explanation:
Captain James offers a deep-sea fishing tour. He charges $2,940 for a 14-hour trip.
Each hour = $2940/14 = 210

Chapter Review/Test – Vocabulary – Page No. 99

Choose the best term from the box.
Go Math Grade 5 Answer Key Chapter 2 Divide Whole Numbers Chapter Review/Test img 11

Question 1.
You can to estimate quotients because they are easy
use _________ to compute with mentally

Answer:
Compatible Numbers

Question 2.
To decide where to place the first digit in the
quotient, you can estimate or use _________

Answer:
Place Value

Concepts and Skills

Use compatible numbers to estimate the quotient.

Question 3.
522 ÷ 6 = _____

Answer:
90

Explanation:
522 is close to 540. 540 ÷ 6 = 90
The quotient is about 90

Question 4.
1,285 ÷ 32 = _____

Answer:
40

Explanation:
1,280 ÷ 32 = 40
The quotient is about 40

Question 5.
6,285 ÷ 89 = _____

Answer:
70

Explanation:
6,300 ÷ 90 = 70
The quotient is about 70

Divide. Check your answer.

Question 6.
2)\(\overline { 554 } \) = _____

Answer:
277

Explanation:
Divide integers 5/2 = 2
Multiply 2 x 2 = 4; Subtract 5 – 4 = 1
Write down 15 and divide integers 15/2 = 7.
Multiply 2 x 7 = 14. Subtract 15 – 14 = 1
Write down 14 and divide integers 14/2 = 7.
Multiply 2 x 7 = 14. Subtract 14 – 14 = 0
The remainder is 0.
So, 277 is the answer.

Question 7.
8)\(\overline { 680 } \) = _____

Answer:
85

Explanation:
Divide integers 68/8 = 8
Multiply 8 x 8 = 64; Subtract 68 – 64 = 4
Write down 40 and divide integers 40/8 = 5.
Multiply 8 x 5 = 40. Subtract 40 – 40 = 0
The remainder is 0.
So, 85 is the answer.

Question 8.
5)\(\overline { 462 } \) = _____ R _____

Answer:
92 R 2

Explanation:
Divide integers 46/5 = 9
Multiply 5 x 9 = 45; Subtract 46 – 45 = 1
Write down 12 and divide integers 12/5 = 2.
Multiply 5 x 2 = 10. Subtract 12 – 10 = 2
The remainder is 2.
So, 92 R 2
Check:
(92 x 5) + 2 = 460 + 2 = 462

Question 9.
522 ÷ 18 = _____

Answer:
29

Explanation:
Divide integers 52/18 = 2
Multiply 18 x 2 = 36; Subtract 52 – 36 = 16
Write down 162 and divide integers 162/8 = 9.
Multiply 8 x 9 = 162. Subtract 162 – 162 = 0
The remainder is 0
So, 29 is the answer.

Question 10.
529 ÷ 37 = _____ R _____

Answer:
14 R 11

Explanation:
Divide integers 52/37 = 1
Multiply 37 x 1 = 37; Subtract 52 – 37 = 15
Write down 159 and divide integers 159/37 = 4.
Multiply 37 x 4 = 148. Subtract 159 – 148 = 11
The remainder is 11.
So, 14 R 11
Check:
(14 x 37) + 11 = 518 + 11 = 529

Question 11.
987 ÷ 15 = _____ R _____

Answer:
65 R 12

Explanation:
Divide integers 98/15 = 6
Multiply 15 x 6 = 90; Subtract 98 – 90 = 8
Write down 87 and divide integers 87/15 = 5.
Multiply 15 x 5 = 75. Subtract 87 – 75 = 12
The remainder is 12.
So, 65 R 12
Check:
(15 x 65) + 12 = 975 + 12 = 987

Question 12.
1,248 ÷ 24 = _____

Answer:
52

Explanation:
Divide integers 124/24 = 5
Multiply 24 x 5 = 120; Subtract 124 – 120 = 4
Write down 48 and divide integers 48/24 = 2.
Multiply 24 x 2 = 48. Subtract 48 – 48 = 0
The remainder is 0
So, 52 is the answer.

Question 13.
5,210 ÷ 17 = _____ R _____

Answer:
306 R 8

Explanation:
Divide integers 52/17 = 3
Multiply 17 x 3 = 51; Subtract 52 – 51 = 1
Write down 110 and divide integers 110/17 = 6.
Multiply 17 x 6 = 102. Subtract 110 – 102 = 8
The remainder is 8.
So, 306 R 8
Check:
(306 x 17) + 8 = 5202 + 8 = 5210

Question 14.
8,808 ÷ 42 = _____ R _____

Answer:
209 R 30

Explanation:
Divide integers 88/42 = 2
Multiply 42 x 2 = 84; Subtract 88 – 84 = 4
Write down 408 and divide integers 408/42 = 9.
Multiply 42 x 9 = 378. Subtract 408 – 378 = 30
The remainder is 30.
So, 209 R 30
Check:
(209 x 42) + 30 = 8778 + 30 = 8808

Chapter Review/Test – Page No. 100

Fill in the bubble completely to show your answer.

Question 15.
Samira bought 156 ounces of trail mix. She wants to divide the amount equally into 24 portions. How many ounces of trail mix will be in each portion?
Options:
A. 6 ounces
B. 6 \(\frac{1}{2}\) ounces
C. 7 ounces
D. 12 ounces

Answer:
B. 6 \(\frac{1}{2}\) ounces

Explanation:
Samira bought 156 ounces of trail mix. She wants to divide the amount equally into 24 portions.156/24 = 6.5 = 6 \(\frac{1}{2}\) ounces

Question 16.
A school band performed 6 concerts. Every seat for each performance was sold. If a total of 1,248 seats were sold for all 6 concerts, how many seats were sold for each performance?
Options:
A. 28
B. 200
C. 206
D. 208

Answer:
D. 208

Explanation:
A school band performed 6 concerts. Every seat for each performance was sold. If a total of 1,248 seats were sold for all 6 concerts, then 1,248/6 = 208.

Question 17.
Dylan’s dog weighs 12 times as much as his pet rabbit. The dog and rabbit weigh 104 pounds altogether. How much does Dylan’s dog weigh?
Options:
A. 104 pounds
B. 96 pounds
C. 88 pounds
D. 8 pounds

Answer:
D. 8 pounds

Explanation:
Dylan’s dog weighs 12 times as much as his pet rabbit. The dog and rabbit weigh 104 pounds altogether.
rabbit weight = S
dog weighs = 12S
S + 12S = 104; 13S = 104; S = 104/13 =8.

Question 18.
Jamie is sewing 14 identical costumes for the school play. She needs 210 buttons to complete all of the costumes. How many buttons will she sew onto each costume?
Options:
A. 15
B. 14
C. 11
D. 9

Answer:
A. 15

Explanation:
Jamie is sewing 14 identical costumes for the school play. She needs 210 buttons to complete all of the costumes. 210/14 = 15

Chapter Review/Test – Page No. 101

Fill in the bubble completely to show your answer.

Question 19.
A book publishing company is shipping an order of 300 books. The books are packaged in boxes that each can hold 24 books. How many boxes are needed to ship the order of books?
Options:
A. 10
B. 11
C. 12
D. 13

Answer:
D. 13

Explanation:
A book publishing company is shipping an order of 300 books. The books are packaged in boxes that each can hold 24 books.
300/24 = 12.5. That is 12 and above boxes. So, the answer is 13

Question 20.
Richard is planning a trip to Italy. He thinks he will need $2,750 for the trip. If the trip is 40 weeks away, which is the best estimate of how much money Richard needs to save each week?
Options:
A. $60
B. $70
C. $600
D. $700

Answer:
B. $70

Explanation:
Richard is planning a trip to Italy. He thinks he will need $2,750 for the trip. If the trip is 40 weeks away, $2,750/40 = $68.75. That is equal to $70

Question 21.
A school club raises $506 to spend on a field trip. There are 23 people going on the trip. How much money did the club raise for each person going?
Options:
A. $27
B. $22
C. $18
D. $12

Answer:
B. $22

Explanation:
A school club raises $506 to spend on a field trip. There are 23 people going on the trip. $506/23 = $22.

Question 22.
A local orange grower processes 2,330 oranges from his grove this year. The oranges are packaged in crates that each hold 96 oranges. All but one crate is full. How many oranges are in this last crate?
Options:
A. 24
B. 25
C. 26
D. 27

Answer:
C. 26

Explanation:
2330 oranges / 96 orange/crate = 24.2708333 crates the decimal portion is the fraction of 96 in the last crate= 96 x .2708333 = 26 oranges in the last crate.

Chapter Review/Test – Page No. 102

Constructed Response

Question 23.
On Monday, 1,900 bottles of perfume are delivered to a warehouse. The bottles are packed in boxes. Each box can hold 32 bottles. How many boxes were delivered? Explain how you found your answer.
_____ boxes

Answer:
I need to divide 1,900 by 32, which is 59 r12. That means the bottles will completely fill 59 boxes. But there will be 12 bottles left over. These would be packed in another box, which makes a total of 60 boxes.

Performance Task

Question 24.
Quincy needs 322 yards of ribbon to decorate quilts for a craft fair.The ribbon comes in rolls of 15 yards.
A. How many rolls of ribbon should Quincy buy? Explain your answer.
_____ rolls

Answer:
I need to divide 322 by 15. The answer is 21  R 7. Since he can’t buy a part of a roll, I need to add 1 to the quotient. So, the final answer is 22.

Question 24.
B. Alice needs twice as many yards of ribbon as Quincy. How many rolls of ribbon does Alice need? Explain your answer.
_____ rolls

Answer:
Twice the length of 322 yards is 644 yards. If I divide 644 by 15, the answer is 42 R 14. Alice needs to buy 43 rolls of ribbon. The remainder doubled is still less than the amount In 1 roll.

Question 24.
C Elena needs yellow, red, and blue ribbon. She needs 285 yards of the three colors combined. Suggest numbers of rolls of each color that would give her enough ribbon. (HINT: Break apart the 285 yards into any combination of 3 groups that total this amount.)
Type below:
_________

Answer:
Sample 1: If Elena wants the same amounts, she will need 95 yards of each color. 95 divided by 15 is 6 R 5, so she will need 7 rolls of each color.
Sample 2: If she wants 109 yards of yellow, 100 yards of red, and 85 yards of blue, she will need 7 rolls of yellow, 7 rolls of red, and 6 rolls of blue.

Conclusion

We wish Go Math Grade 5 Answer Key Chapter 2 Divide Whole Numbers has helped you in your preparation. We have provided step by step solutions to all the questions in Chapter 2 by which you can score well.  Bookmark our site to know more about Class-Specific Go Math Answer Keys.

Go Math Grade 8 Answer Key Chapter 11 Angle Relationships in Parallel Lines and Triangles

go-math-grade-8-chapter-11-angle-relationships-in-parallel-lines-and-triangles-answer-key

Are you browsing for Go Math Grade 8 Answer Key for Chapter 11 Angle Relationships in Parallel Lines and Triangles on the various websites? Don’t worry discover all the questions, answers, and explanations on Go Math Grade 8 Solution Key Ch 11 Angle Relationships in Parallel Lines and Triangles here. Get free access to Download Go Math Grade 8 Chapter 11 Angle Relationships in Parallel Lines and Triangles Solution Key PDF from this page. Finish your homework in time with the help of Go Math Grade 8 Answer Key.

Download Go Math Grade 8 Chapter 11 Angle Relationships in Parallel Lines and Triangles Answer Key PDF

Enhance your skills by using the Go Math Grade 8 Answer Key Chapter 11 Angle Relationships in Parallel Lines and Triangles. Download Go Math Grade 8 Chapter 11 Answer Key and go through Questions and Answers on our website. Follow the below given Go Math Grade 8 Chapter 11 Angle Relationships in Parallel Lines and Triangles Answer Key topic wise links and start your preparation. Make use of the links and secure a good percentage in the exam.

Lesson 1: Parallel Lines Cut by a Transversal

Lesson 2: Angle Theorems for Triangles

Lesson 3: Angle-Angle Similarity

Model Quiz

Review

Guided Practice – Parallel Lines Cut by a Transversal – Page No. 350

Use the figure for Exercises 1–4.
Go Math Grade 8 Answer Key Chapter 11 Angle Relationships in Parallel Lines and Triangles Lesson 1: Parallel Lines Cut by a Transversal img 1

Question 1.
∠UVY and ____ are a pair of corresponding angles.
∠ _________

Answer:
∠ VWZ

Explanation:
∠UVY and ∠ VWZ are a pair of corresponding angles.
When two lines are crossed by Transversal the angles in matching corners are called corresponding angles.

Question 2.
∠WVY and ∠VWT are _________ angles.
____________

Answer:
∠WVY and ∠VWT are alternate interior angles.
Alternate Interior Angles are a pair of angles on the inner side of each of those two lines but on opposite sides of the transversal.

Explanation:
∠WVY and ∠VWT are alternate interior angles.
Alternate Interior Angles are a pair of angles on the inner side of each of those two lines but on opposite sides of the transversal.

Question 3.
Find m∠SVW.
_________ °

Answer:
80º

Explanation:
∠SVW and ∠VWT are same sider interior angles. Therefore,
m∠SVW + m∠VWT = 180º
4xº +5xº = 180º
9x = 180º
x = 180/9
x = 20
m∠SVW = 4xº = (4.20)º = 80º

Question 4.
Find m∠VWT.
_________ °

Answer:
100º

Explanation:
∠SVW and ∠VWT are same sider interior angles. Therefore,
m∠SVW + m∠VWT = 180º
4xº +5xº = 180º
9x = 180º
x = 180/9
x = 20
m∠VWT = 5xº = (5.20)º = 100º

Question 5.
Vocabulary When two parallel lines are cut by a transversal, _______________ angles are supplementary.
____________

Answer:
If two parallel lines are cut by a transversal, then the pairs of alternate exterior angles are congruent. If two parallel lines are cut by a transversal, then the pairs of consecutive interior angles are supplementary.

ESSENTIAL QUESTION CHECK-IN

Question 6.
What can you conclude about the interior angles formed when two parallel lines are cut by a transversal?
Type below:
____________

Answer:
Alternate interior angles are congruent; same-side interior angles are supplementary.

Explanation:
When two parallel lines are cut by a transversal, the interior angles will be the angles between the two parallel lines. Alternate interior angles will be on opposite sides of the transversal; the measures of these angles are the same.
Same-side interior angles will be on the same side of the transversal; the measures of these angles will be supplementary, adding up to 180 degrees.

11.1 Independent Practice – Parallel Lines Cut by a Transversal – Page No. 351

Vocabulary Use the figure for Exercises 7–10.
Go Math Grade 8 Answer Key Chapter 11 Angle Relationships in Parallel Lines and Triangles Lesson 1: Parallel Lines Cut by a Transversal img 2

Question 7.
Name all pairs of corresponding angles.
Type below:
____________

Answer:
∠1 and ∠5
∠3 and ∠7
∠2 and ∠6
∠4 and ∠8

Explanation:
Corresponding angles are
∠1 and ∠5
∠3 and ∠7
∠2 and ∠6
∠4 and ∠8

Question 8.
Name both pairs of alternate exterior angles.
Type below:
____________

Answer:
∠1 and ∠8
∠2 and ∠7

Explanation:
Alternate exterior angles
∠1 and ∠8
∠2 and ∠7

Question 9.
Name the relationship between ∠ 3 and ∠6.
Type below:
____________

Answer:
alternate interior angles

Explanation:
∠3 and ∠6 are alternate interior angles.
Alternate Interior Angles are a pair of angles on the inner side of each of those two lines but on opposite sides of the transversal.

Question 10.
Name the relationship between ∠4 and ∠6.
Type below:
____________

Answer:
same-side interior angles

Explanation:
∠4 and ∠6 are same-side interior angles.

Find each angle measure.
Go Math Grade 8 Answer Key Chapter 11 Angle Relationships in Parallel Lines and Triangles Lesson 1: Parallel Lines Cut by a Transversal img 3

Question 11.
m∠AGE when m∠FHD = 30°
_________ °

Answer:
m∠AGE = 30°

Explanation:
∠AGE and ∠FHD are alternate exterior angles.
Therefore, m∠AGE = m∠FHD = 30°
m∠AGE = 30°

Question 12.
m∠AGH when m∠CHF = 150°
_________ °

Answer:
150°

Explanation:
∠AGH and ∠CHF are corresponding angles.
Therefore, m∠AGH = m∠CHF = 150°
m∠AGH = 150°

Question 13.
m∠CHF when m∠BGE = 110°
_________ °

Answer:
110°

Explanation:
∠CHF and ∠BGE are alternate exterior angles.
Therefore, m∠CHF = m∠BGE = 110°
m∠CHF = 110°

Question 14.
m∠CHG when m∠HGA = 120°
_________ °

Answer:
m∠CHG = 60º

Explanation:
∠CHF and ∠HGA are same-side interior angles.
m∠CHG + m∠HGA = 180°
m∠CHG + 120° = 180°
m∠CHG = 180 – 120 = 60
m∠CHG = 60º

Question 15.
m∠BGH
_________ °

Answer:
78º

Explanation:
∠BGH and ∠GHD are same-side interior angles.
So, ∠BGH + ∠GHD = 180º
3x + (2x + 50)º = 180º
5x = 180º – 50º = 130º
x = 130/5 = 26º
∠BGH = 3xº = 3 × 26º = 78º
∠GHD = (2x + 50) += (2 × 26 + 50) = 102º

Question 16.
m∠GHD
_________ °

Answer:
102º

Explanation:
∠BGH and ∠GHD are same-side interior angles.
So, ∠BGH + ∠GHD = 180º
3x + (2x + 50)º = 180º
5x = 180º – 50º = 130º
x = 130/5 = 26º
∠BGH = 3xº = 3 × 26º = 78º
∠GHD = (2x + 50) += (2 × 26 + 50) = 102º

Question 17.
The Cross Country Bike Trail follows a straight line where it crosses 350th and 360th Streets. The two streets are parallel to each other. What is the measure of the larger angle formed at the intersection of the bike trail and 360th Street? Explain.
Go Math Grade 8 Answer Key Chapter 11 Angle Relationships in Parallel Lines and Triangles Lesson 1: Parallel Lines Cut by a Transversal img 4
_________ °

Answer:
The larger angle formed at the intersection of the bike trail and 360th Street is 132º

Explanation:
grade 8 chapter 11 image 1
The larger angle formed at the intersection of the bike trail and 360th Street is the angle 5 in our schema. ∠5 and ∠3 are same-side interior angles. Therefore, m∠5 + m∠3 = 180º
m∠5 + 48º = 180º
m∠5 = 180º – 48º
m∠5 = 132º

Question 18.
Critical Thinking How many different angles would be formed by a transversal intersecting three parallel lines? How many different angle measures would there be?
_________ different angles
_________ different angle measures

Answer:
12 different angles
2 different angle measures

Explanation:
There are 12 different angles formed by a transversal intersecting three parallel lines.
There are 2 different angle measures:
m∠1 = m∠4 = m∠5 = m∠8 = m∠9 = m∠12
m∠2 = m∠3 = m∠6 = m∠7 = m∠10 = m∠11

Parallel Lines Cut by a Transversal – Page No. 352

Question 19.
Communicate Mathematical Ideas In the diagram at the right, suppose m∠6 = 125°. Explain how to find the measures of each of the other seven numbered angles.
Go Math Grade 8 Answer Key Chapter 11 Angle Relationships in Parallel Lines and Triangles Lesson 1: Parallel Lines Cut by a Transversal img 5
Type below:
____________

Answer:
m∠2 = m∠6 = 125º because ∠2 and ∠6 are corresponding angles.
m∠3 = m∠2 = 125º because ∠3 and ∠2 are vertical angles.
m∠7 = m∠3 = 125º because ∠7 and ∠3 are corresponding angles.
∠4 and ∠6 are same-side interior angles.
Therefore, m∠4 + m∠6 = 180º
m∠4 + 125º = 180º
m∠4 = 180º – 125º
m∠4 = 55º
m∠8 = m∠4 = 55º because ∠8 and ∠4 are corresponding angles.
m∠1 = m∠4 = 55º because ∠1 and ∠4 are vertical angles.
m∠5 = m∠1 = 55º because ∠5 and ∠1 are corresponding angles.

FOCUS ON HIGHER ORDER THINKING

Question 20.
Draw Conclusions In a diagram showing two parallel lines cut by a transversal, the measures of two same-side interior angles are both given as 3x°. Without writing and solving an equation, can you determine the measures of both angles? Explain. Then write and solve an equation to find the measures.

Answer:
m∠1 and m∠2 are same-side interior angles is 180º
Therefore, m∠1 + m∠2 = 180º
3x + 3x = 180º
6x = 180º
x = 180/6 = 30
m∠1 = m∠2 = 3x = 3(30) = 90º

Question 21.
Make a Conjecture Draw two parallel lines and a transversal. Choose one of the eight angles that are formed. How many of the other seven angles are congruent to the angle you selected? How many of the other seven angles are supplementary to your angle? Will your answer change if you select a different angle?
Type below:
____________

Answer:
grade 8 chapter 11 image 3
We have to select ∠a form of eight angles that are formed. There are two other angles that are congruent to the angle ∠a. Two other angles are ∠e and ∠g.
There are no supplementary to ∠a.
If we select a different angle then the answer will also change.

Question 22.
Critique Reasoning In the diagram at the right, ∠2, ∠3, ∠5, and∠8 are all congruent, and∠1, ∠4, ∠6, and ∠7 are all congruent. Aiden says that this is enough information to conclude that the diagram shows two parallel lines cut by a transversal. Is he correct? Justify your answer.
Go Math Grade 8 Answer Key Chapter 11 Angle Relationships in Parallel Lines and Triangles Lesson 1: Parallel Lines Cut by a Transversal img 6
____________

Answer:
This is not enough information to conclude that the diagram shows two parallel lines cut by a transversal. Because ∠2 and ∠3 are same-side interior angles. But ∠5 and ∠8 are not congruent with each other. And ∠6 and ∠7 are same-side interior angles. But ∠1 and ∠4 are not congruent with each other.

Guided Practice – Angle Theorems for Triangles – Page No. 358

Find each missing angle measure.

Question 1.
Go Math Grade 8 Answer Key Chapter 11 Angle Relationships in Parallel Lines and Triangles Lesson 2: Angle Theorems for Triangles img 7
m∠M = _________ °

Answer:
m∠M = 71º

Explanation:
From the Triangle Sum Theorem,
m∠L + m∠N + m∠M = 180º
78º + 31º + m∠M = 180º
109º + m∠M = 180º
m∠M = 180º – 109º
m∠M = 71º

Question 2.
Go Math Grade 8 Answer Key Chapter 11 Angle Relationships in Parallel Lines and Triangles Lesson 2: Angle Theorems for Triangles img 8
m∠Q = _________ °

Answer:
m∠Q = 30º

Explanation:
From the Triangle Sum Theorem,
m∠Q + m∠S + m∠R = 180º
m∠Q + 24º + 126º = 180º
m∠Q + 150º = 180º
m∠Q = 180º – 150º
m∠Q = 30º

Use the Triangle Sum Theorem to find the measure of each angle in degrees.

Question 3.
Go Math Grade 8 Answer Key Chapter 11 Angle Relationships in Parallel Lines and Triangles Lesson 2: Angle Theorems for Triangles img 9
m∠T = _________ °
m∠V = _________ °
m∠U = _________ °

Answer:
m∠T = 88°
m∠V = 63°
m∠U = 29°

Explanation:
From the Triangle Sum Theorem,
m∠U + m∠T + m∠V = 180º
(2x + 5)º + (7x + 4)º + (5x + 3)º = 180º
2xº + 5º + 7xº + 4º + 5xº + 3º = 180º
14xº + 12º = 180º
14xº = 168º
x = 168/14 = 12
Substitute x value to find the angles
m∠U = (2x + 5)º = ((2 . 12) + 5)º = 29º
m∠U = 29º
m∠T = (7x + 4)º = ((7 . 12) + 4)º = 88º
m∠T = 88º
m∠V = (5x + 3)º = ((5 . 12) + 3)º = 63º
m∠V = 63º

Question 4.
Go Math Grade 8 Answer Key Chapter 11 Angle Relationships in Parallel Lines and Triangles Lesson 2: Angle Theorems for Triangles img 10
m∠X = _________ °
m∠Y = _________ °
m∠Z = _________ °

Answer:
m∠X = 90°
m∠Y = 45 °
m∠Z = 45°

Explanation:
From the Triangle Sum Theorem,
m∠X + m∠Y + m∠Z = 180º
nº + (1/2 . n)º + (1/2 . n)º = 180º
2nº = 180º
n = 90
Substitute n values to find the angles
m∠X = nº = 90º
m∠X = 90º
m∠Y = (1/2 . n)º = (1/2 . 90)º = 45º
m∠Y = 45º
m∠Z = (1/2 . n)º = (1/2 . 90)º = 45º
m∠Z = 45º

Use the Exterior Angle Theorem to find the measure of each angle in degrees.

Question 5.
Go Math Grade 8 Answer Key Chapter 11 Angle Relationships in Parallel Lines and Triangles Lesson 2: Angle Theorems for Triangles img 11
m∠C = _________ °
m∠D = _________ °

Answer:
m∠C = 40°
m∠D = 76°

Explanation:
Given m∠C = 4y°, m∠D = (7y + 6)°, m∠E = 116°
By using exterior angle theorem,
∠DEC + ∠DEF = 180°
grade 8 chapter 11 image 4
∠DEC + 116° = 180°
∠E = ∠DEC = 180° – 116° = 64°
The sum of the angles of a traingle = 180°
∠C +∠D + ∠E = 180°
4y° + (7y + 6)°+ 64° = 180°
11y° + 70° = 180°
11y° = 180° – 70° = 110°
y = 10
∠C = 4y° = 4. 10 = 40°
∠D = (7y + 6)° = ((7 . 10)  + 6)° = (70 + 6)° = 76°

Question 6.
Go Math Grade 8 Answer Key Chapter 11 Angle Relationships in Parallel Lines and Triangles Lesson 2: Angle Theorems for Triangles img 12
m∠L = _________ °
m∠M = _________ °

Answer:
m∠L = 129°
m∠M = 32°

Explanation:
Given that m∠M = (5z – 3)°, m∠L = (18z + 3)°, m∠JKM = 161°
From the Exterior Angle Theorem,
m∠M + m∠L = m∠JKM
(5z – 3)° + (18z + 3)° = 161°
5z° – 3° + 18z° + 3° = 161°
23z° = 161°
z = 161/23 = 7
Substitute z values to find the angles
m∠M = (5z – 3)° = ((5 . 7) – 3)° = 32°
m∠L = (18z + 3)° = ((18 . 7) + 3)° = 129°
From the Triangle Sum Theorem,
m∠M + m∠L + m∠LKM = 180º
32º + 129º + m∠LKM = 180º
161º + m∠LKM = 180º
m∠LKM = 19º

ESSENTIAL QUESTION CHECK-IN

Question 7.
Describe the relationships among the measures of the angles of a triangle.
Type below:
______________

Answer:
The sum of all measures of the interior angles of a triangle is 180°. The measure of an exterior angle of a triangle is equal to the sum of its remote interior angles.

11.2 Independent Practice – Angle Theorems for Triangles – Page No. 359

Find the measure of each angle.

Question 8.
Go Math Grade 8 Answer Key Chapter 11 Angle Relationships in Parallel Lines and Triangles Lesson 2: Angle Theorems for Triangles img 13
m∠E = _________ °
m∠F = _________ °

Answer:
m∠E = 41°
m∠F = 41°

Explanation:
m∠E = x°, m∠F = x°,  m∠D = 98°
From the Triangle Sum Theorem, sum of the angles of the traingle is 180°
m∠E + m∠D + m∠F = 180°
x + 98 + x = 180°
2x + 98 = 180°
2x = 82°
x = 41°
So, m∠E = 41°
m∠F = 41°

Question 9.
Go Math Grade 8 Answer Key Chapter 11 Angle Relationships in Parallel Lines and Triangles Lesson 2: Angle Theorems for Triangles img 14
m∠T = _________ °
m∠V = _________ °

Answer:
m∠T = 60°
m∠V = 30°

Explanation:
m∠W = 90°, m∠T = 2x°,  m∠V = x°
From the Triangle Sum Theorem, sum of the angles of the traingle is 180°
m∠T + m∠V + m∠W = 180°
2x + x + 90 = 180°
3x = 90°
x = 30°
So, m∠T = 2x° = 2 . 30° = 60°
m∠V = x° = 30°

Question 10.
Go Math Grade 8 Answer Key Chapter 11 Angle Relationships in Parallel Lines and Triangles Lesson 2: Angle Theorems for Triangles img 15
m∠G = _________ °
m∠H = _________ °
m∠J = _________ °

Answer:
m∠G = 75°
m∠H = 60°
m∠J = 45°

Explanation:
m∠G = 5x°, m∠H = 4x°,  m∠J = 3x°
From the Triangle Sum Theorem, sum of the angles of the traingle is 180°
m∠G + m∠H + m∠J = 180°
5x + 4x + 3x = 180°
12x = 90°
x = 15°
So, m∠G = 5x° = 5 . 15° = 75°
m∠H = 4x° = 4. 15° = 60°
m∠J = 3x° = 3. 15° = 45°

Question 11.
Go Math Grade 8 Answer Key Chapter 11 Angle Relationships in Parallel Lines and Triangles Lesson 2: Angle Theorems for Triangles img 16
m∠Q = _________ °
m∠P = _________ °
m∠QRP = _________ °

Answer:
m∠Q = 98°
m∠P = 55°
m∠QRP = 27°

Explanation:
Given that m∠Q = (3y + 5)°, m∠P = (2y – 7)°, m∠QRS = 153°
From the exterior angle Theorem,
∠QRS + ∠QRP = 180°
153° + ∠QRP = 180°
grade 8 chapter 11 image 5
m∠R = m∠QRP = 180° – 153° = 27°
From the Triangle Sum Theorem, the sum of the angles of the triangle is 180°
m∠P + m∠Q + m∠R = 180°
(3y + 5)° + (2y – 7)°+ 27° = 180°
5y° + 25 = 180°
5y° = 155°
y = 31°
m∠Q = (3y + 5)° = ((3 . 31°) + 5)° = 98°
m∠P = (2y – 7)° = ((2. 31° – 7)° = 55°
m∠QRP = 27°

Question 12.
Go Math Grade 8 Answer Key Chapter 11 Angle Relationships in Parallel Lines and Triangles Lesson 2: Angle Theorems for Triangles img 17
m∠ACB = _________ °
m∠DCE = _________ °
m∠BCD = _________ °

Answer:
m∠ACB = 44°
m∠DCE = 35°
m∠BCD = 101°

Explanation:
In traingle ABC, m∠A = 78°, m∠B = 58°, m∠ACB = ?°
From the Triangle Sum Theorem, the sum of the angles of the triangle is 180°
m∠A + m∠B + m∠ACB = 180°
78° + 58° + m∠ACB = 180°
m∠ACB = 180° – 136°
m∠ACB = 44°
In traingle CDE, m∠D = 85°, m∠E = 60°, m∠CDE = ?°
From the Triangle Sum Theorem, the sum of the angles of the triangle is 180°
m∠D + m∠E + m∠CDE = 180°
85° + 60° + m∠CDE = 180°
m∠CDE = 180° – 145°
m∠CDE = 35°
From the Exterior Angle Theorem,
m∠ACB + m∠CDE + m∠BCD = 180°
44° + 35° + m∠BCD = 180°
m∠BCD = 180° – 79°
m∠BCD = 101°

Question 13.
Go Math Grade 8 Answer Key Chapter 11 Angle Relationships in Parallel Lines and Triangles Lesson 2: Angle Theorems for Triangles img 18
m∠K = _________ °
m∠L = _________ °
m∠KML = _________ °
m∠LMN = _________ °

Answer:

Explanation:
m∠K = 2x°, m∠L = 3x°, m∠KML = x°
So, From the Triangle Sum Theorem, the sum of the angles of the triangle is 180°.
m∠K + m∠L + m∠KML = 180°
2x° + 3x° + x° = 180°
6x° = 180°
x= 30°
∠KML = x = 30°
∠L = 3x = 3 . 30° = 90°
∠K = 2x = 2 . 30° = 60°
From the Exterior Angle Theorem,
∠KML + ∠LMN = 180°
∠LMN = 180° – 30° = 150°

Question 14.
Multistep The second angle in a triangle is five times as large as the first. The third angle is two-thirds as large as the first. Find the angle measures.
The measure of the first angle: _________ °
The measure of the second angle: _________ °
The measure of the third angle: _________ °

Answer:
The measure of the first angle: 27°
The measure of the second angle: 135°
The measure of the third angle: 18°

Explanation:
Let us name the angles of a triangle as ∠1, ∠2, ∠3.
Consider ∠1 as x.
∠2 is 5 times as large as the first.
∠2 = 5x
Also, ∠3 = 2/3 . x
So, From the Triangle Sum Theorem, the sum of the angles of the triangle is 180°.
x+ 5x + (2/3 . x) = 180°
20x = 540°
x = 27°
So, ∠1 = x = 27°
∠2 = 5x = 5 . 27° = 135°
∠3 = 2/3 . x = 2/3 . 27° = 18°
The measure of the first angle: 27°
The measure of the second angle: 135°
The measure of the third angle: 18°

Angle Theorems for Triangles – Page No. 360

Question 15.
Analyze Relationships Can a triangle have two obtuse angles? Explain.
___________

Answer:
No; a triangle can NOT have two obtuse angles

Explanation:
The measure of an obtuse angle is greater than 90°. Two obtuse angles and the third angle would have a sum greater than 180°

FOCUS ON HIGHER ORDER THINKING

Question 16.
Critical Thinking Explain how you can use the Triangle Sum Theorem to find the measures of the angles of an equilateral triangle.
Type below:
___________

Answer:
All angles have the same measure in an equilateral triangle

Explanation:
Using the Triangle Sum Theorem,
∠x + ∠x + ∠x = 180°
3∠x = 180°
∠x = 60°
All angles have the same measure in an equilateral triangle

Question 17.
a. Draw Conclusions Find the sum of the measures of the angles in quadrilateral ABCD. (Hint: Draw diagonal \(\overline { AC } \). How can you use the figures you have formed to find the sum?)
Go Math Grade 8 Answer Key Chapter 11 Angle Relationships in Parallel Lines and Triangles Lesson 2: Angle Theorems for Triangles img 19
Sum: _________ °

Answer:
Sum: 360°

Question 17.
b. Make a Conjecture Write a “Quadrilateral Sum Theorem.” Explain why you think it is true.
Type below:
___________

Answer:
The sum of the angle measures of a quadrilateral is 360°
Any quadrilateral can be divided into two triangles (180 + 180 = 360)

Question 18.
Communicate Mathematical Ideas Describe two ways that an exterior angle of a triangle is related to one or more of the interior angles.
Type below:
___________

Answer:
An exterior angle and it’s an adjacent interior angle equal 180°
An exterior angle equals the sum of the two remote interior angles.

Guided Practice – Angle-Angle Similarity – Page No. 366

Question 1.
Explain whether the triangles are similar. Label the angle measures in the figure.
Go Math Grade 8 Answer Key Chapter 11 Angle Relationships in Parallel Lines and Triangles Lesson 3: Angle-Angle Similarity img 20
Type below:
___________
△ABC has angle measures _______and △DEF has angle measures______. Because _______in one triangle are congruent to ______in the other triangle, the triangles are_____.

Answer:
△ABC has angle measures 40°, 30°, and 109° and △DEF has angle measures 41°, 109°, and 30°. Because 2∠s in one triangle are congruent to in the other triangle, the triangles similar.

Question 2.
A flagpole casts a shadow 23.5 feet long. At the same time of day, Mrs. Gilbert, who is 5.5 feet tall, casts a shadow that is 7.5 feet long. How tall in feet is the flagpole? Round your answer to the nearest tenth.
Go Math Grade 8 Answer Key Chapter 11 Angle Relationships in Parallel Lines and Triangles Lesson 3: Angle-Angle Similarity img 21
_________ ft

Answer:
17.2 ft

Explanation:
In similar triangles, corresponding side lengths are proportional.
5.5/7.5 = h/23.5
h (7.5) = 129.25
h = 129.25/7.5
h = 17.23
Rounding to the nearest tenth
h = 17.2 feet

Question 3.
Two transversals intersect two parallel lines as shown. Explain whether △ABC and △DEC are similar.
Go Math Grade 8 Answer Key Chapter 11 Angle Relationships in Parallel Lines and Triangles Lesson 3: Angle-Angle Similarity img 22
∠BAC and∠EDC are ___________ since they are ___________.
∠ABC and∠DEC are ___________ since they are ___________.
By ________, △ABC and△DEC are ___________.
Type below:
___________

Answer:
∠BAC and∠EDC are congruent since they are alt. interior ∠s
∠ABC and∠DEC are congruent since they are alt. interior ∠s.
By AA similarity, △ABC and△DEC are similar.

ESSENTIAL QUESTION CHECK-IN

Question 4.
How can you determine when two triangles are similar?
Type below:
___________

Answer:
If 2 angles of one triangle are congruent to 2 angles of another triangle, the triangles are similar by the Angle-Angle Similarity Postulate

11.3 Independent Practice – Angle-Angle Similarity – Page No. 367

Use the diagrams for Exercises 5–7.
Go Math Grade 8 Answer Key Chapter 11 Angle Relationships in Parallel Lines and Triangles Lesson 3: Angle-Angle Similarity img 23

Question 5.
Find the missing angle measures in the triangles.
Type below:
___________

Answer:
m∠B = 42°
m∠F = 69°
m∠H = 64°
m∠K = 53°

Explanation:
Using the Triangle Sum Theorem,
m∠A + m∠B + m∠C = 180°
85° + m∠B + 53° = 180°
138° + m∠B = 180°
m∠B = 180° – 138°
m∠B = 42°
Using the Triangle Sum Theorem,
m∠D + m∠E + m∠F = 180°
We substitute the given angle measures and we solve for m∠F
64° + 47° + m∠F = 180°
111° + m∠F = 180°
m∠F = 180° – 111°
m∠F = 69°
Using the Triangle Sum Theorem,
m∠G + m∠H + m∠J = 180°
We substitute the given angle measures and we solve for m∠H
47° + m∠H + 69° = 180°
116° + m∠H = 180°
m∠H = 180° – 116°
m∠H = 64°
Using the Triangle Sum Theorem,
m∠J + m∠K + m∠L = 180°
We substitute the given angle measures and we solve for m∠K
85° + m∠K + 42° = 180°
127° + m∠K = 180°
m∠K = 180° – 127°
m∠K = 53°

Question 6.
Which triangles are similar?
Type below:
___________

Answer:
△ABC and △JKL are similar because their corresponding angles are congruent. Also, △DEF and △GHJ are similar because their corresponding is congruent.

Question 7.
Analyze Relationships Determine which angles are congruent to the angles in △ABC.
∠A ≅ ∠ ________
∠B ≅ ∠ ________
∠C ≅ ∠ ________

Answer:
△JKL ≅ △ABC

Explanation:
△JKL has angle measures that are the same as those is △ABC
∠A ≅ ∠ J
∠B ≅ ∠ L
∠C ≅ ∠ K
Therefore, they are congruent.

Question 8.
Multistep A tree casts a shadow that is 20 feet long. Frank is 6 feet tall,and while standing next to the tree he casts a shadow that is 4 feet long.
Go Math Grade 8 Answer Key Chapter 11 Angle Relationships in Parallel Lines and Triangles Lesson 3: Angle-Angle Similarity img 24
a. How tall is the tree?
h = ________ ft

Answer:
h = 30 ft

Explanation:
In similar triangles, corresponding side lengths are proportional.
20/4 = h/6
5 = h/6
h = 30
The tree is 30 feet tall.

Question 8.
b. How much taller is the tree than Frank?
________ ft

Answer:
24 ft

Explanation:
30 – 6 = 24
The tree is 24 feet taller than Frank.

Question 9.
Represent Real-World Problems Sheila is climbing on a ladder that is attached against the side of a jungle gym wall. She is 5 feet off the ground and 3 feet from the base of the ladder, which is 15 feet from the wall. Draw a diagram to help you solve the problem. How high up the wall is the top of the ladder?
________ ft

Answer:
25 ft

Explanation:
grade 8 chapter 11 image 6
3/15 = 5/h
15 ×3 = 3h
75 = 3h
h = 75/3 = 25

Question 10.
Justify Reasoning Are two equilateral triangles always similar? Explain.
______________

Answer:
yes; two equilateral triangles are always similar.
Each angle of an equilateral triangle is 60°. Since both triangles are equilateral then they are similar.

Angle-Angle Similarity – Page No. 368

Question 11.
Critique Reasoning Ryan calculated the missing measure in the diagram shown. What was his mistake?
Go Math Grade 8 Answer Key Chapter 11 Angle Relationships in Parallel Lines and Triangles Lesson 3: Angle-Angle Similarity img 25
\(\frac{3.4}{6.5}=\frac{h}{19.5}\)
19.5 × \(\frac{3.4}{6.5}=\frac{h}{19.5}\) × 19.5
\(\frac{66.3}{6.5}\) = h
10.2cm = h
Type below:
___________

Answer:
In the first line, Ryan did not take the sum of 6.5 and 19.5 to get the denominator on the right.
The denominator on the right should be 26 instead of 19.5
the correct value for h
3.4/6.5 = h/26
h = (3.4/6.5) × 26
h = 13.6cm

FOCUS ON HIGHER ORDER THINKING

Question 12.
Communicate Mathematical Ideas For a pair of triangular earrings, how can you tell if they are similar? How can you tell if they are congruent?
Type below:
___________

Answer:
The earrings are similar if two angle measures of one are equal to two angle measures of the other.
The earrings are congruent if they are similar and if the side lengths of one are equal to the side lengths of the other.

Question 13.
Critical Thinking When does it make sense to use similar triangles to measure the height and length of objects in real life?
Type below:
___________

Answer:
If the item is too tall or the distance is too long to measure directly, similar triangles can help with measuring.

Question 14.
Justify Reasoning Two right triangles on a coordinate plane are similar but not congruent. Each of the legs of both triangles are extended by 1 unit, creating two new right triangles. Are the resulting triangles similar? Explain using an example.
___________

Answer:
Two triangles are similar if their corresponding angles are congruent and the lengths of their corresponding sides are proportional. If each of the legs of both triangles is extended by 1 unit, the ratio between proportional sides does not change. Therefore, the resulting triangles are similar.

Ready to Go On? – Model Quiz – Page No. 369

11.1 Parallel Lines Cut by a Transversal

In the figure, line p || line q. Find the measure of each angle if m∠8 = 115°.
Go Math Grade 8 Answer Key Chapter 11 Angle Relationships in Parallel Lines and Triangles Model Quiz img 26

Question 1.
m∠7 = _________ °

Answer:
m∠7 = 65°

Explanation:
According to the exterior angle theorem,
m∠7 + m∠8 = 180°
m∠7 + 115° = 180°
m∠7 = 180° – 115°
m∠7 = 65°

Question 2.
m∠6 = _________ °

Answer:
m∠6 = 115°

Explanation:
From the given figure, Line P is parallel to line Q. So, the angles given in line P is equal to the angles in line Q. They are corresponding angles.
So, m∠8 is parallel is m∠6 or m∠8 = m∠6 = 115°

Question 3.
m∠1 = _________ °

Answer:
m∠1 = 115°

Explanation:
∠1 and ∠6 are alternative exterior angles.
So, m∠1 = m∠6 = 115°

11.2 Angle Theorems for Triangles

Find the measure of each angle.
Go Math Grade 8 Answer Key Chapter 11 Angle Relationships in Parallel Lines and Triangles Model Quiz img 27

Question 4.
m∠A = _________ °

Answer:
m∠A = 48°

Explanation:
m∠A + m∠B + m∠C = 180°
4y° + (3y + 22)° + 74° = 180°
7y = 180 – 96 = 84
y = 12°
m∠A = 4y° = 4 (12°) = 48°
m∠B = (3y + 22)° = (3(12°) + 22)° = 58°

Question 5.
m∠B = _________ °

Answer:
m∠B = 58°

Explanation:
m∠A + m∠B + m∠C = 180°
4y° + (3y + 22)° + 74° = 180°
7y = 180 – 96 = 84
y = 12°
m∠A = 4y° = 4 (12°) = 48°
m∠B = (3y + 22)° = (3(12°) + 22)° = 58°

Question 6.
m∠BCA = _________ °

Answer:
m∠BCA = 74°

Explanation:
m∠BCD + m∠BCA = 180°
106° + m∠BCA = 180°
m∠BCA = 180° – 106°
m∠BCA = 74°
So, m∠BCA = 74°

11.3 Angle-Angle Similarity

Triangle FEG is similar to triangle IHJ. Find the missing values.
Go Math Grade 8 Answer Key Chapter 11 Angle Relationships in Parallel Lines and Triangles Model Quiz img 28

Question 7.
x = _________

Answer:
x = 16

Explanation:
In similar triangles, corresponding side lengths are proportional.
HJ/EG = IJ/FG
(x + 12)/42 = 40/60
(x + 12)/42 = 4/6
6x = 96
x = 16

Question 8.
y = _________

Answer:
y = 9

Explanation:
In similar triangles, corresponding side lengths are congruent.
m∠HJI = m∠EGF
(5y + 7)° = 52°
5y° + 7° = 52°
5y° = 45°
y = 9

Question 9.
m∠H = _________°

Answer:
m∠H = 92°

Explanation:
Using the Triangle Sum Theorem,
m∠E + m∠F + m∠G = 180°
We substitute the given angle measures and we solve for m∠E
m∠E + 36° + 52° = 180°
m∠E + 88° = 180°
m∠E = 92°
In similar angles, corresponding side lengths are congruent
m∠H = m∠E
m∠H = 92°

ESSENTIAL QUESTION

Question 10.
How can you use similar triangles to solve real-world problems?
Type below:
____________

Answer:
we know that if two triangles are similar, then their corresponding angles are congruent and the lengths of their corresponding sides are proportional. We can use this to determine values that we cannot measure directly. For example, we can calculate the length of the tree if we measure its shadow and our shadow on a sunny day.

Selected Response – Mixed Review – Page No. 370

Use the figure for Exercises 1 and 2.
Go Math Grade 8 Answer Key Chapter 11 Angle Relationships in Parallel Lines and Triangles Mixed Review img 29

Question 1.
Which angle pair is a pair of alternate exterior angles?
Options:
A. ∠5 and ∠6
B. ∠6 and∠7
C. ∠5 and ∠4
D. ∠5 and ∠2

Answer:
C. ∠5 and ∠4

Explanation:
∠5 and ∠4 are alternate exterior angles

Question 2.
Which of the following angles is not congruent to ∠3?
Options:
A. ∠1
B. ∠2
C. ∠6
D. ∠8

Answer:
B. ∠2

Explanation:
∠2 and ∠3 are same-side interior angles. They are not congruent instead their sum is equal to 180°

Question 3.
The measures, in degrees, of the three angles of a triangle are given by 2x + 1, 3x – 3, and 9x. What is the measure of the smallest angle?
Options:
A. 13°
B. 27°
C. 36°
D. 117°

Answer:
B. 27°

Explanation:
From the Triangle Sum Theorem, the sum of the angles of the triangle is 180°
m∠1 + m∠2 + m∠3 = 180°
(2x + 1)° + (3x – 3)° + (9x)° = 180°
2x° + 1° + 3x° – 3° + 9x° = 180°
14x° – 2° = 180°
14x° = 178°
x = 13
Substitute the value of x to find the m∠1, m∠2, and m∠3
m∠1 = (2x + 1)° = (2(13) + 1)° = 27°
m∠2 = (3x – 3)° = (3(13) – 3)° = 36°
m∠3 = (9x)° = (9(13))° = 117°
The smallest angle is 27°

Question 4.
Which is a possible measure of ∠DCA in the triangle below?
Go Math Grade 8 Answer Key Chapter 11 Angle Relationships in Parallel Lines and Triangles Mixed Review img 30
Options:
A. 36°
B. 38°
C. 40°
D 70°

Answer:
D 70°

Explanation:
Using the Exterior Angle Theorem
m∠A + m∠B = m∠DCA
m∠A + 40° = m∠DCA
m∠DCA will be greater than 40°. The only suitable option is D, 70°.

Question 5.
Kaylee wrote in her dinosaur report that the Jurassic period was 1.75 × 108 years ago. What is this number written in standard form?
Options:
A. 1,750,000
B. 17,500,000
C. 175,000,000
D. 17,500,000,000

Answer:
C. 175,000,000

Explanation:
1.75 × 108 standard form
Move the decimal point to 8 right places.
175,000,000

Question 6.
Given that y is proportional to x, what linear equation can you write if y is 16 when x is 20?
Options:
A. y = 20x
B. y = \(\frac{5}{4}\) x
C. y = \(\frac{4}{5}\)x
D. y = 0.6x

Answer:
C. y = \(\frac{4}{5}\)x

Explanation:
Y=4/5x
16=4/5(20)
4/5×20/1=80/5
80/5=16

Mini-Task

Question 7.
Two transversals intersect two parallel lines as shown.
Go Math Grade 8 Answer Key Chapter 11 Angle Relationships in Parallel Lines and Triangles Mixed Review img 31
a. What is the value of x?
x = ________

Answer:
x = 4

Explanation:
mJKL = mLNM
6x + 1 = 25
6x = 24
x = 4

Question 7.
b. What is the measure of ∠LMN?
_________°

Answer:
23°

Explanation:
m∠LMN = 3x + 11 = 3(4) + 11 = 12 + 11 = 23

Question 7.
c. What is the measure of ∠KLM?
∠KLM = _________°

Answer:
∠KLM = 48°

Explanation:
∠KLM exterior angle of the triangle LMN
m∠KLM = m∠LNM + m∠LMN
= 25 + 23 = 48

Question 7.
d. Which two triangles are similar? How do you know?
Type below:
_____________

Answer:
triangle JKL = triangle LNM
triangle KJL = triangle LMN

Explanation:
triangle JLK and triangle LNM are similar.
triangle JKL = triangle LNM
triangle KJL = triangle LMN

Summary:

The solutions provided in the Go Math Grade 8 Answer Key Chapter 11 Angle Relationships in Parallel Lines and Triangles are made by the professionals. Practice all the math questions available on the 8th Grade Text Book and learn how to solve the questions in a simple way. Hope the information provided in this article is beneficial for all the students of grade 8. Keep in touch with our website to get the pdfs of all the Go Math Grade 8 Answer Key Chapterwise.